Internal Med Topic List Rosh Questions

अब Quizwiz के साथ अपने होमवर्क और परीक्षाओं को एस करें!

The emergency department staff began treatment for a woman who presented with chest pain. The pain is described as retrosternal, worse with minimal activity, better with rest, sharp in character and 9/10 in intensity. You are paged to admit her to the intensive care unit under the working diagnosis of unstable angina. Her vitals have remained stable after beginning antiplatelet, antihypertensive and antithrombotic medications. Two hours after admission, a repeat history and physical and review of available test results offers the following information: serial electrocardiograms reveal increasing R wave amplitude; an echocardiogram calculates an ejection fraction of 50%; angina is reported as 9/10 in intensity; atrial natriuretic peptide levels are elevated. Which of the following historical facts would prompt you to immediately consult interventional cardiology for invasive coronary revascularization? 50% ejection fraction Continued chest pain Elevated atrial natriuretic peptide R wave progression

Continued chest pain Patients with unstable angina are mostly admitted to a critical care unit after initial presentation. There, an anti-ischemic regimen, if not already begun, is initiated. This typically includes oxygen if hypoxemic, nitrates if not hypotensive, analgesics only if pain is not controlled, and beta-blockers. Serial monitoring for new dysrhythmias, recurrent ischemia, dynamic electrocardiography, changing laboratory results and worsening angina is necessary to maximize patient outcomes. Further management includes risk stratification to determine if early invasive treatment is appropriate. High-risk indicators that favor early invasive treatment strategies include hemodynamic instability, elevated troponin levels, a history of CABG, a history of percutaneous coronary intervention (PCI) within the past 6 months, recurrent angina despite anti-ischemic therapy, symptoms of congestive heart failure (S3, pulmonary edema, crackles, mitral regurgitation) or an ejection fraction < 40%.

A 12-year-old girl with no significant medical history presents to her pediatrician complaining of two weeks of joint pain. Initially, the pain started at the same time as a fever, first in her knees, then her ankles, and now she feels like her elbows are bothering her as well. Ibuprofen helps a great deal, but the pain is still present after the medication wears off. Her only recent illness was strep throat which was treated with amoxicillin. On physical exam, mild erythema and swelling around her elbows is appreciated and her lower extremities appear normal but are painful to move. The rest of her physical exam is also normal, including ear, nose and throat, cardiac, and skin exams. Which of the following is the most appropriate diagnostic test? Antistreptolysin O titer Complete blood count with differential Echocardiogram X-ray of affected joints

Echocardiogram This patient presents with a migratory polyarthritis, primarily affecting her large joints. Her recent history of streptococcal pharyngitis should raise concern for acute rheumatic fever. The most severe subsequent complication is rheumatic heart disease, the most common acquired valvular heart disease globally. The Jones criteria describe manifestations of acute rheumatic fever and are used by the American Heart Association to guide diagnosis. The manifestations are divided into "major" and "minor" categories. In patients with a history of a preceding group A Streptococcus infection (GAS), a diagnosis of acute rheumatic fever can be made when two major manifestations are present, or one major and two minor manifestations are present. The five major manifestations are arthritis, carditis, Sydenham chorea, erythema marginatum, and subcutaneous nodules. Of these, arthritis and carditis are by far the most commonly seen. Typically, arthritis is the first symptom to appear, about 21 days after the associated GAS infection, and is characterized by joint pain which responds to nonsteroidal anti-inflammatory medications. Carditis may be more difficult to diagnose because it is often asymptomatic. Acute rheumatic fever causes a pancarditis, but the primary manifestation is a valvulitis, usually involving the mitral and aortic valves. In some patients, a mid-systolic murmur, heard loudest at the apex, can indicate the regurgitation associated with this valvulitis; however, it may not be appreciated in all patients. Thus, echocardiography is recommended in all cases of suspected and confirmed acute rheumatic fever. In addition to making the diagnosis of carditis, echocardiography also provides robust data essential to the long-term management of the most important sequelae of acute rheumatic fever.

Which of the following is most appropriate in the treatment plan for a patient with idiopathic pulmonary fibrosis? Albuterol Azathioprine Bosentan Pulmonary rehabilitation

Pulmonary rehabilitation The goal in the treatment of idiopathic pulmonary fibrosis is to reduce symptoms, prevent acute exacerbations, vaccinate, slow progression, and increase survival. There are several treatment options, however, most medication trials have shown no benefit. Sufficient clinical evidence that any treatment improves survival or quality of life for patients with idiopathic pulmonary fibrosis (IPF) is lacking. Supportive care, supplemental oxygen, and pulmonary rehabilitation appear to be the most beneficial options. In addition, medications such as nintedanib may be used. Lung transplantation may be an option in those with disease progression and minimal comorbidities. IPF prognosis is poor. Only 20-30% of patients survive five years after diagnosis.

Perforation is a complication that occurs in 10% of patients with acute cholecystitis and is characterized by which of the following? Coarse cough Hypoactive bowel sounds Jaundice Nausea and vomiting

Hypoactive bowel sounds Hypoactive bowel sounds are an indicator that a perforation has occurred. Other symptoms include high fever, systemic signs of toxicity (tachycardia and increased respiratory rate), and increased abdominal pain with rebound tenderness.

Which of the following statements is most accurate regarding health maintenance for a patient with chronic pancreatitis? Abstinence from alcohol will improve symptoms but will not decrease mortality Enteric coated pancreatic enzymes require coadministration with an H2 antagonist Long chain triglycerides can provide extra calories in patients with weight loss Malabsorption of fat soluble vitamins may occur but is rarely clinically symptomatic

Malabsorption of fat soluble vitamins may occur but is rarely clinically symptomatic Inflammation of the pancreas which is persistent and causes structural damage and eventual decline in pancreatic function is known as chronic pancreatitis. The cause of chronic pancreatitis may be obstructive or necrotic in nature. In obstructive pancreatitis, ductal stone formation leads to chronic inflammation which causes progressive pancreatic fibrosis and eventual atrophy. In necrotic pancreatitis, recurrent attacks of acute pancreatitis lead to necrotic tissue, which is eventually replaced by fibrotic tissue. Smoking and alcohol consumption are two main risk factors for pancreatitis. Other etiologies of pancreatitis include autoimmune disease, genetic disorders, and tumors. Symptoms of chronic pancreatitis are abdominal pain, nausea, vomiting and weight loss. Steatorrhea develops later in the disease course after the pancreas has lost much of its exocrine function. Diagnosis can be made via visualizations of calcifications on plain film or CT scan, magnetic resonance cholangiopancreatography, endoscopic ultrasound, or pancreatic function tests (such as stimulation of the pancreas and subsequent collection of duodenal fluid for analysis). Treatment of chronic pancreatitis involves lifestyles changes, pain management, and administration of pancreatic enzymes. Cessation of alcohol intake and smoking is imperative. Small, low-fat meals should be advised. Pain can be managed with tricyclic antidepressants, narcotics, or occasional hospitalizations to keep the patient nothing by mouth (NPO) for a short duration and allow pancreatic rest. Pancreatic enzymes are prescribed for patients with chronic pancreatitis to break the feedback loop of cholecystokinin release and pancreatic exocrine stimulation. Medium chain triglycerides are easily absorbed and can provide extra calories for patients suffering weight loss. Malabsorption of vitamins due to pancreatic exocrine malfunction may occur but is usually not clinically symptomatic. Replacement of the fat soluble vitamins and cyanocobalamin may be necessary for patients with substantial steatorrhea. Patients who fail medical therapy may be candidates for surgical procedures, such as denervation, decompression, and resection. Abstinence from alcohol (A) is essential for patients with chronic pancreatitis, although abstaining from alcohol may not improve the patient's symptoms. Studies have shown that patients with chronic pancreatitis who continue to consume alcohol have higher mortality rates. Smoking cessation is also essential, as smoking increases the risk of pancreatic cancer and may accelerate the progression of chronic pancreatitis. Several formulations of pancreatic enzyme supplements exist, and many of them are enteric coated (B). This helps to prevent inactivation of the enzymes by gastric acid. If non-enteric coated formulations are prescribed, the patient will also need to take an H2 antagonist or a proton pump inhibitor to suppress gastric acid and reduce inactivation of the enzymes. Medium chain triglycerides are prescribed for patients with chronic pancreatitis and weight loss, because they do not require bile acid to be absorbed. Thus, they can provide extra calories for cachectic patients. Long chain triglycerides (C) require bile acid to be degraded, and cannot be directly absorbed by the intestinal mucosa. Medium chain triglycerides can be directly absorbed from the intestinal mucosa and therefore cause less stimulation of pancreatic secretion.

A 28-year-old South Asian immigrant who is in her second trimester of her first pregnancy presents to the emergency department with worsening dyspnea, orthopnea, and lower extremity edema. She has never experienced anything like this before. She has no past medical history, but she reports frequent sore throats and ear infections as a child. Which of the following is most likely to be heard on auscultatory exam? A diastolic decrescendo murmur heard at the left lower sternal border A diastolic low-pitched decrescendo murmur best heard at the cardiac apex A holosystolic murmur heard best at the cardiac apex A systolic crescendo-decrescendo murmur best heard at the right upper sternal border

A diastolic low-pitched decrescendo murmur best heard at the cardiac apex A diastolic low-pitched decrescendo murmur best heard at the cardiac apex would be the most likely auscultatory finding on exam. This woman likely has mitral stenosis secondary to rheumatic heart disease. Mitral stenosis encountered in women of childbearing age is nearly always rheumatic in origin. Maternal and perinatal complications during pregnancy in women with mitral stenosis reflect the unfavorable interaction between the normal cardiovascular changes of pregnancy and the stenotic mitral valve. Pregnancy is a high flow state and blood volume, cardiac output, and heart rate are increased. In pregnant patients with mitral stenosis, this increases the pressure across the mitral valve and can lead to pulmonary edema and other signs and symptoms of heart failure. Although mitral stenosis is relatively uncommon in industrialized countries, it is a common condition in pregnant women with heart disease in areas where rheumatic heart disease is prevalent. Although rheumatic heart disease may affect other valves, including aortic and tricuspid, the mitral valve is the predominant lesion and repeated attacks of rheumatic fever over time causes mitral valve stenosis.

A 35-year-old woman comes to the clinic complaining of difficulty seeing, blurred vision, eye pain, and cough. She describes the cough as being dry and nonproductive. She has no past medical history and takes no medications. Ophthalmologic examination shows uveitis. Chest X-ray reveals bilateral hilar adenopathy. Which of the following laboratory findings would also most likely be associated with this patient's condition? Elevated C-reactive protein Elevated ESR Elevated serum angiotensin converting enzyme levels Increased sweat chloride

Elevated serum angiotensin converting enzyme levels Uveitis, bilateral hilar adenopathy, and a dry cough most likely indicate sarcoidosis. Sarcoidosis is granulomatous disease that can form nodules in multiple organs. Patients with sarcoidosis typically present with fatigue, weight loss, arthritis, dry eyes, blurry vision, and respiratory symptoms (eg, cough, dyspnea). Management usually involves the use of corticosteroids (eg, prednisone). Elevated levels of serum angiotensin converting enzyme (ACE) are typically elevated in about 75% of patients with sarcoidosis. However, elevated levels of ACE can also be seen in other conditions, such as, asbestosis, diabetes, and lung cancer. Therefore, it is not routinely used as a diagnostic test.

True or false: Bacterial infections are the most common cause of acute bronchitis?

False, viruses are the common cause (Adenovirus, Parainfluenza, Influenza, Coronavirus,Coxsackie, Rhinovirus, Respiratory syncytial virus)

A woman presents with nausea and vomiting, chest pain at rest and altered mental status. Her past medical history is significant for poorly controlled diabetes mellitus and hypertension, as well as tobacco abuse. Which of the following would you most expect to see on an electrocardiogram? Absent P waves Prolonged QT interval R wave greater than S wave in lead V1 ST segment elevation

ST segment elevation Myocardial ischemia is usually due to atherosclerosis, whereas myocardial infarction results from ruptured and dislodged atherosclerotic plaques. This is the underlying pathology in the acute coronary syndromes, which encompass unstable angina, non-ST-elevation myocardial infarction and ST-elevation myocardial infarction (STEMI). Symptoms indicative of myocardial infarction include severe and usually progressive substernal chest pain with radiation to the neck, jaw or left upper extremity, diaphoresis, dyspnea and nausea and vomiting. STEMI angina also commonly occurs during rest and lasts > 30 minutes. The presence of ST-elevations on ECG suggests a high likelihood of coronary artery occlusion. Causes of ST-elevation include acute MI, pulmonary embolism, coronary spasm and myopericarditis. Treatment necessitates prompt percutaneous coronary intervention and revascularization.

A 32-year-old man is admitted to the hospital with sepsis. His vital signs at the time of admission included a temperature of 100.2°F, heart rate of 135 bpm, and a blood pressure of 135/90 mm Hg. Physical examination at admission revealed nontender erythematous macules on the bilateral palms, track marks in the bilateral antecubital fossa, and crackles and diminished breath sounds in the bilateral lungs. Laboratory findings at admission include a white blood cell count of 24,000 cells/microliter and a lactic acid of 4 mmol/L. Urinalysis was negative for pyuria and bacteriuria. Chest radiograph showed multiple pulmonary infiltrates with central cavitation. You are reviewing the patient's preliminary results of blood cultures 24 hours after admission and notice they are positive for Staphylococcus aureus. Which of the following is considered a major criteria to diagnose the suspected condition? Blood cultures positive for Klebsiella pneumoniae Edematous hemorrhagic retinal lesions with pale centers Presence of a prosthetic heart valve Tender subcutaneous nodules on the pads of the fingers Vegetation of the tricuspid valve on echocardiogram

Vegetation of the tricuspid valve on echocardiogram The patient in this vignette has a presentation concerning for right-sided infective endocarditis. The track marks suggest intravenous drug use, which is an important risk factor for infective endocarditis, and the pulmonary infiltrates with central cavitation are consistent with septic pulmonary emboli from vegetations. Infective endocarditis is an infection of the endocardial surface of the heart involving either the heart valves or an intracardiac device. The common causative organisms are Staphylococcus aureus, viridans streptococci, Streptococcus gallolyticus, and HACEK group organisms (Haemophilus aphrophilus, Actinobacillus actinomycetemcomitans, Cardiobacterium hominis, Eikenella corrodens, and Kingella kingae). The risk factors for infective endocarditis include prior history of infective endocarditis, prosthetic heart valves, indwelling vascular lines, indwelling intracardiac devices, prior valvular damage, congenital heart disease, recent dental procedures or surgeries, hemodialysis, immunosuppression, and intravenous drug use. The presentation of infective endocarditis is variable with some patients presenting with acute and rapidly progressive symptoms and others presenting with subacute or chronic symptoms. Fever is the most common finding, and patients often have other systemic symptoms, such as chills, night sweats, weight loss, headaches, and myalgias. New heart murmur is another classic finding, although this is usually absent in right-sided infective endocarditis. In addition, there are several less common but specific findings that may be appreciate on exam: Osler nodes (tender subcutaneous nodules on the pads of the fingers, toes, thenar, or hypothenar eminence), Janeway lesions (painless erythematous macules on the palms or soles), and Roth spots (hemorrhagic lesions on the retina with pale centers). While there are many disease processes that can cause systemic symptoms, clinicians should have suspicion for infective endocarditis in patients with risk factors for infective endocarditis and in patients who have blood cultures that are positive for organisms known to cause infective endocarditis. The modified Duke criteria are used to stratify patients into three categories: definite, possible, and rejected (not) infective endocarditis based on major and minor criteria. Definite infective endocarditis is established according to the modified Duke criteria if a patient meets either pathologic or clinical criteria. The pathologic criteria include either histology confirming active endocarditis on a vegetation or intracardiac abscess or a positive culture from a vegetation or intracardiac abscess. Either of these pathologic criteria would require an invasive procedure to collect a specimen for pathology or culture. The clinical criteria of infective endocarditis are categorized as either major or minor criteria. The major criteria are a positive blood culture for an organism known to cause infective endocarditis and evidence of endocardial involvement, such as echocardiogram evidence of infective endocarditis (vegetation, abscess, or new partial dehiscence of prosthetic valve) or new valvular regurgitation. The minor criteria include predisposition for infective endocarditis (intravenous drug use or presence of a predisposing heart condition), fever, vascular phenomena (major arterial emboli, septic pulmonary infarcts, mycotic aneurysm, intracranial hemorrhages, conjunctival hemorrhages, or Janeway lesions), immunologic phenomena (glomerulonephritis, Osler nodes, Roth spots, or positive rheumatoid factor), and microbiologic evidence that does not meet the major criteria. The diagnostic criteria for definitive infective endocarditis include one of the following: two major clinical criteria, one major and three minor criteria, or five minor clinical criteria.The criteria for possible infective endocarditis according to the modified Duke criteria include one major and one minor criteria or three minor criteria. The criteria for rejected infective endocarditis include a firm alternative diagnosis, the resolution of clinical manifestation after fewer than 5 days of antibiotics, no pathologic evidence of infective endocarditis found at surgery, or the clinical criteria not being met for definite or possible infective endocarditis. Antibiotics are the main treatment for infective endocarditis. However, patients with complications, such as valvular dysfunction, may also need surgery. An important preventative measure for infective endocarditis is administering prophylactic antibiotics before dental procedures to individuals with prosthetic heart valves or known valvular disease.

You suspect a new patient has systemic lupus erythematosus based on symptomatology and physical exam. What laboratory tests in addition to an antinuclear antibody should be ordered to help confirm the diagnosis? Anti-Sjögren syndrome A antibody and anti-Sjögren syndrome B antibody Anti-Smith antibody and anti-double-stranded DNA Erythrocyte sedimentation rate and C-reactive protein Rheumatoid factor and anti-cyclic citrullinated peptide

Anti-Smith antibody and anti-double-stranded DNA Antibodies to an RNA-protein complex called anti-Smith and to double-stranded DNA are highly associated with systemic lupus erythematosus (SLE), both being 95% specific. Antinuclear antibody (ANA), anti-double-stranded DNA, and antiphospholipid antibody are all markers for SLE, but these test results may be misleading if not considered in clinical context. Between 2 and 5% of patients with SLE are ANA negative, whereas 5% of the general population and up to 20% of healthy young women are ANA positive. SLE is most common in women of reproductive age (15-40 years). The female to male ratio is approximately 2:1 before puberty and 4:1 after puberty. However, SLE is seen in all ages, including infants and older adults. In these two subpopulations, the female to male ratio is only 2:1. SLE affects approximately 1 in 1,000-2,500 in the general population, but disease incidence in women of Black and Latino ethnicity is much higher (up to 1 in 250 in women of Black ethnicity aged 18-65 years). The 5-year survival after diagnosis is 90%. Secondary to the variety of presentations, the American College of Rheumatology created a classification system to standardize the diagnosis of SLE. To confirm a diagnosis, patients must have at least 4 of the 11 criteria present, either serially or simultaneously. Constitutional symptoms found in patients with SLE include fatigue, malaise, fever, and weight loss. Thus, the clinician has a critical role to play in the diagnosis of this multisystem disease, which must be differentiated from disease processes such as HIV (often false positive in SLE), subacute bacterial endocarditis, and other connective tissue diseases, including vasculitis, rheumatoid arthritis, mixed connective tissue disease, and malignancies such as lymphomas. SLE is characterized by specific organ system abnormalities. The disease most often affects the skin, joints, kidneys, CNS, gastrointestinal tract, and lungs, with a varying spectrum of disease severity and unpredictable clinical course. SLE is truly systemic. Other asymptomatic organ involvement must be investigated during active periods of disease, regardless of the presenting sign or symptom.

A four-week-old infant presents in cardiac arrest. Cardiopulmonary resuscitation is initiated and the monitor shows pulseless electrical activity. What is the correct concentration and dose of intravenous epinephrine in this setting? 0.01 mL/kg of 1:1,000 concentration 0.01 mL/kg of 1:10,000 concentration 0.1 mL/kg of 1:1,000 concentration 0.1 mL/kg of 1:10,000 concentration

0.1 mL/kg of 1:10,000 concentration The mechanism of action of epinephrine includes both alpha- and beta-adrenergic stimulation. Alpha-adrenergic stimulation results in increased peripheral vasoconstriction with resultant increased circulation to the heart and brain and decreased splanchnic, renal, and dermal circulation. Increases in myocardial contractility and heart rate as well as relaxation of the smooth muscle of the coronary and cerebral arteries and bronchi is the result of beta-adrenergic stimulation. Epinephrine is indicated in the setting of cardiac arrest with pulseless electrical activity, ventricular fibrillation, or ventricular tachycardia, symptomatic bradycardia, refractory hypotension and anaphylaxis. In the setting of cardiac arrest, the correct dose of epinephrine is 0.1 mL/kg of 1:10,000 concentration given intravenously or via a functional intraosseous line. Repeated doses may be given every three to five minutes.

A 42-year-old woman presents to the office reporting pain in her fingertips that has worsened over the past year. She notices it when the weather is cold or when she is anxious. When it occurs, her fingertips become pale and numb, and after she warms her hands, the same fingers become bright red with painful numbness and tingling throughout that lasts for several minutes before resolving. She also reports having increased epigastric pain with a burning sensation and a bitter taste in her mouth, especially after meals. She has been taking over-the-counter antacids, which help. Which of the following antibodies are most likely to be found on a blood test? Anti-cyclic citrullinated peptide antibodies Anti-double stranded DNA antibodies Anti-Sjögren syndrome A antibodies and anti-Sjögren syndrome B antibodies Anti-Smith antibodies Anticentromere antibodies

Anticentromere antibodies Scleroderma or systemic sclerosis is a rare, chronic, connective tissue disease characterized by diffuse fibrosis of the skin and the internal organs. Symptoms appear in patients who are in their 30s to 50s, with women two or three times more likely to be affected than men. It is caused by the deposition of an abnormally high amount of collagen, causing abnormal thickening of the connective tissues involved. There are two types of scleroderma: limited (more common) and diffuse. Limited scleroderma is also called CREST syndrome (calcinosis cutis, Raynaud phenomenon, esophageal motility disorder, sclerodactyly, and telangiectasias), and fibrosis of the skin is limited to the face and hands. Patients with limited scleroderma are also susceptible to digital ischemia and life-threatening pulmonary hypertension. Diffuse scleroderma is a more severe form of the disease that leads to extensive hardening of the skin involving the trunk and extremities. These patients have poorer outcomes due to kidney, lung, and cardiac involvement. Raynaud phenomenon is present in almost all patients with both types of scleroderma and appears before any other signs or symptoms. Patients with diffuse scleroderma have a more rapid onset of symptoms and develop joint pain, weight loss, and malaise early on. These symptoms are not common in those with limited disease. Skin will become thickened with loss of normal folds over time and will develop telangiectasias, pigmentation, and depigmentation. Ulcerations of the fingertips and cutaneous calcifications of the affected skin are also signs of developing disease. Symptoms of multi-organ involvement include dysphagia and reflux symptoms due to esophageal dysfunction from hypomotility of the gastrointestinal tract, which can later develop into fibrosis. Lung involvement from diffuse pulmonary fibrosis resulting in restrictive lung disease and pulmonary hypertension can lead to right heart failure, pericarditis, myocardial fibrosis, and scleroderma renal crisis. Laboratory findings may reveal positive antinuclear antibody tests, positive anticentromere antibodies (highly specific for limited scleroderma), positive anti-topoisomerase I (anti-Scl-70) antibodies (highly specific for diffuse scleroderma and may indicate a poor prognosis due to high chance of serious internal organ involvement), and anti-RNA polymerase III antibodies, which are associated with rapidly developing skin disease and increased risk of scleroderma renal crisis. There is currently no effective treatment for the underlying disease process, and thus treatment is focused on the organ system involved. Patients with Raynaud phenomenon are advised to keep their hands warm and to avoid tobacco use. If these measures are ineffective, calcium channel blockers can be given for severe cases. Angiotensin-converting enzyme inhibitors can be used for kidney disease, proton pump inhibitors or H2-blockers for reflux symptoms, and nonsteroidal anti-inflammatory drugs for musculoskeletal pain. Systemic immunosuppressive agents may be given for diffuse skin and organ involvement. Prognosis tends to be worse in patients with diffuse scleroderma, in the older population, and in men. Lung disease in the form of either pulmonary fibrosis or pulmonary arterial hypertension is the most common cause of death. Other causes of death include heart failure or chronic kidney disease. Anti-cyclic citrullinated peptide antibodies (A) are specific to rheumatoid arthritis, not systemic sclerosis, and are therefore not the best answer. Anti-double stranded DNA antibodies (B) are highly specific for systemic lupus erythematosus but not for systemic sclerosis. Anti-Sjögren syndrome A antibodies (anti-SSA) and anti-Sjögren syndrome B antibodies (anti-SSB) (C) are associated with systemic lupus erythematosus, Sjӧgren syndrome, and complement deficiency but are uncommon in patients with systemic sclerosis. Anti-Smith antibodies (D) are highly specific for systemic lupus erythematosus but not for systemic sclerosis.

Which of the following is recommended for secondary stroke prevention for non-cardioembolic transient ischemic attack and ischemic stroke? Aspirin Aspirin plus clopidogrel Cilostazol Indomethacin

Aspirin Aspirin is recommended for secondary stroke prevention for non-cardioembolic transient ischemic attack (TIA) and ischemic stroke. Guidelines from the American Heart Association/American Stroke Association (AHA/ASA) recommend that patients with a non-cardioembolic atherothrombotic, lacunar, or cryptogenic stroke or TIA (and no contraindications) receive an antiplatelet agent to reduce the risk of recurrent stroke. These guidelines note that aspirin, clopidogrel, and the combination of aspirin-extended-release dipyridamole are all acceptable options for preventing recurrent noncardioembolic ischemic stroke or transient ischemic attack.

A 19-year-old man presents to the clinic with 10 days of worsening pharyngitis, fatigue, and fever. His exam is remarkable for significant posterior lymphadenopathy, a white-purple tonsillar exudate, and palpable hepatosplenomegaly. Which of the following laboratory results would be most expected in this patient? Positive KOH preparation Positive rapid antigen detection test for group A Streptococcus Significant lymphocytosis Throat culture confirming Corynebacterium diphtheriae

Significant lymphocytosis Significant lymphocytosis with large, atypical lymphocytes is the most likely laboratory finding expected in this patient who is exhibiting typical symptoms and physical exam findings of infectious mononucleosis caused by Epstein Barr virus (EBV). EBV infection should be suspected in patients who present with pharyngitis, marked posterior lymphadenopathy, conjunctival hemorrhages, fever, fatigue, and a shaggy white-purple tonsillar exudate which may extend into the nasopharynx. Hepatosplenomegaly may also be present, and many patients will also have transient upper eyelid edema, known as Hoagland sign. Young adults are the most common age group to be affected. Lymphocytosis in these patients may be impressive, with lymphocyte-to-white blood cell ratios greater than 35%. A positive heterophile agglutination test or anti-EBV titer is also supportive of the diagnosis. Most patients with infectious mononucleosis recover normally with simple supportive care measures, such as rest, acetaminophen for fever, and warm saltwater gargles. Corticosteroids are used but should be reserved for patients with impending airway obstruction from enlarging lymph nodes. There is no clinical benefit shown with antivirals. Patients should be counseled to avoid contact sports due to the risk of splenic rupture, which is an uncommon but life-threatening complication. Patients should also be cautioned against sharing beverages with others, as the virus is transmitted by saliva and can remain in secretion for 6 months or longer after symptom onset.

A 10-year-old boy presents to the pediatrics clinic for evaluation of his asthma. He is currently only taking a rescue inhaler as needed. He has daytime symptoms four days per week, nighttime symptoms once per month, and uses a rescue inhaler four days per week. Which of the following medication regimens is most appropriate for this patient? Albuterol Albuterol and fluticasone Albuterol, fluticasone, and salmeterol Albuterol, fluticasone, salmeterol, and a short course of oral prednisone

Albuterol and fluticasone Asthma is a reversible obstructive airway disease caused by hyperresponsiveness of the airways to allergens. Common triggers include pollen, house dust, mold, cold air, exercise, dogs, cats, smoke, and viral infections. The classic symptoms of asthma include chest tightness, dyspnea, cough, and wheezing. Asthma symptoms begin during childhood in the majority of cases, and asthma symptoms are usually intermittent, associated with triggers, and worse at night. Wheezing is the most common exam finding. It occurs during exhalation initially but may occur during inhalation and exhalation as severity worsens. Tachypnea, retractions, cyanosis, and inability to speak in complete sentences may occur in severe cases. The diagnosis of asthma is confirmed with spirometry showing a reversible obstructive pattern. Asthma is classified according to severity to help guide treatment and monitor symptoms. The classifications include intermittent asthma and mild, moderate, or severe persistent asthma. These classifications are made according to the frequency of daytime symptoms, frequency of nighttime symptoms, frequency of use of rescue inhaler, measurement on spirometry, and limitation on activities. Patients are classified according to their most severe measurement. Intermittent asthma is defined as having daytime symptoms two or fewer days per week, nighttime symptoms two or fewer days per month, use of rescue inhaler two or fewer days per week, and no interference with daily activities. The treatment for intermittent asthma is use of a short-acting beta-agonist (SABA), which is also known as a rescue inhaler. Albuterol is the most common example. Mild persistent asthma is defined as daytime symptoms more than two days per week but not daily, nighttime symptoms three or four times per month, use of rescue inhaler more than two days per week but not daily, or minor limitations of daily activities. The patient in this vignette has mild persistent asthma since he has daytime symptoms four days per week (more than two but less than daily) and uses his rescue inhaler four days per week. The treatment of mild persistent asthma is a SABA (used as needed) and a low-dose inhaled corticosteroid, such as fluticasone, taken daily. Moderate persistent asthma is defined as daily daytime symptoms, nighttime symptoms more than once per week, use of rescue inhaler daily, or moderate limitation of physical activities. The recommended treatment of moderate persistent asthma is a SABA (used as needed), a low- or medium-dose inhaled corticosteroid taken daily, and a long-acting beta-agonist (LABA) taken daily. Severe persistent asthma is defined as symptoms multiple times per day, nighttime symptoms nightly, use of rescue inhaler multiple times per day, or severe limitation in physical activities. The recommended treatment of severe persistent asthma includes a SABA (as needed), high-dose inhaled corticosteroid taken daily, and a LABA taken daily. In some cases, a short course of oral corticosteroids, such as prednisone, is needed. Patients with each severity of asthma should be educated to avoid asthma triggers. The treatment goals for patients with asthma include meeting the intermittent asthma criteria, having the ability to participate in physical activities without limitation, and avoiding oral corticosteroids, emergency department visits, hospitalization, and mechanical intubation. Patients with an acute asthma exacerbation are most often treated with bronchodilators and systemic steroids. Supplemental oxygen is recommended if oxygen saturation is below 92% on room air. In severe asthma exacerbations, intravenous magnesium sulfate or mechanical intubation may be indicated.

A 61-year-old man presents to the office reporting low back, pelvic, and bilateral upper thigh pain. He describes the pain as deep and aching and reports that it is worse at night. He does not have any radiating sharp pain, numbness, or tingling in his legs. His physical exam is unremarkable. A radiograph of the lumbar spine and pelvis shows focal osteolytic lesions with focal radiolucencies throughout the lumbar vertebrae, pelvis, and the bilateral proximal femurs. Which of the following laboratory findings will help to establish a diagnosis in this patient? 25-hydroxyvitamin D level Alkaline phosphatase Calcitonin Calcitriol Parathyroid hormone

Alkaline phosphatase Paget disease of bone (also known as osteitis deformans) is characterized by high bone turnover and disorganized bone formation. The disease may involve one bone (monostotic) or multiple bones (polyostotic), with the latter being more common. The affected bone will initially have increased osteoclast activity causing lytic lesions throughout, then increased osteoblastic activity occurs, where bone is formed in a disorganized fashion. The high bone turnover results in highly vascular, weak, and deformed bones. Paget disease of bone is the second most common bone disease after osteoporosis and is common in patients over the age of 55 years. The cause of this disease is unknown, but a significant number of cases are genetic and passed on as an autosomal dominant trait. Patients with this condition are often asymptomatic or have mild symptoms. The bones most commonly affected are the pelvis, vertebrae, femur, humerus, and skull. Patients often report an aching, deep pain that is worse at night as the first symptom. This disease typically affects the bones proximally and then advances distally. Patients may also report arthritic pain if the adjacent joints are involved. Due to the disorganized bone turnover, bones can be soft, leading to bowed tibias, kyphosis, and easy fractures with light trauma. Patients with skull involvement may report increased hat size, headaches, or dilated scalp veins. If the temporal bone is involved, the cochlea can be damaged, leading to mixed sensorineural and conductive hearing loss, tinnitus, and vertigo. Laboratory findings will reveal a markedly elevated serum alkaline phosphatase, although patients with monostotic disease may have a normal level. In patients with normal serum alkaline phosphatase levels, a bone-specific alkaline phosphatase should be ordered, which will be high to distinguish an elevated phosphatase from bone as opposed to the liver. Other markers for bone turnover are serum N-terminal propeptide of type I collagen and serum beta C-terminal propeptide of type I collagen. If the patient is on bed rest, serum calcium may be elevated. A serum 25-OH vitamin D level should also be obtained to rule out vitamin D deficiency, which can also present with increased alkaline phosphatase in the presence of bone pain. Radiographs will reveal osteolytic lesions of the affected bones with focal radiolucencies or advancing flame-shaped lytic lesions. Bones may become sclerotic as well. Technetium-99m pyrophosphate bone scans may be helpful to determine the activity of bone lesions even before they appear on radiographs. Since most patients have mild disease or are asymptomatic, they are typically diagnosed incidentally during radiologic imaging or because of an elevation of alkaline phosphatase on a routine blood test. Treatment depends on the symptomaticity and severity of the disease. Asymptomatic patients may only require routine follow-up without treatment. However, even those who are asymptomatic but have significant long bone, skull, or vertebral involvement should be treated, closely monitored for serial serum alkaline phosphatase levels, and receive regular physical examinations. The mainstay of treatment for Paget disease is bisphosphonates, with intravenous zoledronic acid as the treatment of choice, which has been shown to normalize serum alkaline phosphatase levels in most patients within the first 2 years. Prognosis of this disease is good, but relapse can occur. Therefore, patients must be monitored long-term with yearly serum alkaline phosphatase measurements.

A 30-year-old man with ankylosing spondylitis presents for a routine physical. This is the first encounter that you appreciate a murmur during cardiac auscultation. You appreciate a diastolic, high pitched, blowing murmur while listening with the diaphragm in the left sternal border. There is no palpable thrill. Which of the following is the most likely diagnosis? Aortic regurgitation Aortic stenosis Mitral regurgitation Tricuspid stenosis

Aortic regurgitation The murmur of aortic regurgitation is best heard with the diaphragm in the left sternal border. It can be accentuated when a patient sits and leans forward. This is a diastolic murmur, as compared to the systolic stenotic murmur of the same valve. It is often high pitched, blowing and decrescendo in nature. A widened pulse pressure is seen due to incompetence of this valve as it allows previously "pumped-out" blood to return back into the left ventricle. Common causes include rheumatic heart disease, trauma and endocarditis, and it can also be associated with Marfan's syndrome, syphilis and ankylosing spondylitis.

Which of the following studies is recommended in all patients who present with syncope? Blood glucose Computed tomography of the head Echocardiogram Electrocardiogram Stool guaiac test

Electrocardiogram Syncope is characterized by a transient loss of consciousness due to either global hypoperfusion of the brain or focal hypoperfusion of the reticular activating system. Patients with syncope exhibit a sudden loss of postural tone but return to baseline functioning spontaneously without the need for resuscitation. Syncope should be differentiated from other causes of altered mental status such as alcohol intoxication, stroke, seizures, and traumatic brain injury. Because the causes of syncope are so varied, it can be difficult to determine the etiology, especially in the emergency department. The goal is to determine which patients are at high risk for death and other adverse events. The most concerning causes of syncope are cardiac in nature and require early diagnosis and management. Therefore, all patients who present with a history of syncope require electrocardiogram upon initial presentation. Cardiac causes of syncope include structural abnormalities, dysrhythmias, cardiac tamponade, and pacemaker malfunction. Patients with cardiac causes may present with dyspnea, palpitations, chest pain, and lower extremity edema. These patients may also endorse a family history of sudden death or take medications that are known to prolong the QT interval, such as antipsychotics and certain antibiotics, however, cardiac testing should never be excluded due to lack of these symptoms or historical findings. Syncope that occurs suddenly and without a prodrome or that occurs while the patient is in a seated or supine position should raise suspicion for a dysrhythmia. Syncope that occurs during exertion is worrisome and should also raise suspicion for a dysrhythmia or outflow obstruction. Physical exam may reveal a difference between upper extremity pulses in patients with aortic dissection or subclavian steal syndrome. Cardiac auscultation may reveal an irregular heart rate, murmurs, and extra heart sounds. Pacemakers should be checked for malfunction. Findings indicative of syncope due to dysrhythmia include bradycardia < 40 beats per minute, Mobitz II second-degree heart block, and third-degree heart block. Other dysrhythmias that should raise concern include alternating right and left bundle branch blocks, ventricular tachycardia, and pauses in rhythm with a pacemaker. Other abnormalities include right bundle branch block with ST elevations in leads V1-V3 as well as a shortened PR interval and slurred QRS complex, as seen in Brugada syndrome and Wolf-Parkinson-White syndrome, respectively. Additionally, any electrocardiogram changes that reflect the presence of ischemia, infarction, or hypertrophy would also warrant admission and intensive investigation. If there is concern about the presence of structural heart disease (left ventricular dysfunction, mitral and aortic stenosis, hypertrophic cardiomyopathy, etc.) and pericardial effusions, transthoracic echocardiography is necessary. Other useful studies that can be considered on a case-by-case basis in patients presenting with syncope include cardiac monitoring (and ambulatory electrocardiogram monitoring on an outpatient basis), orthostatic blood pressure measurement, carotid sinus massage, exercise testing, blood glucose, electrolyte levels, natriuretic peptide levels, troponin levels, and certain neurological studies. Other life-threatening causes of syncope include hemorrhage (gastrointestinal, ruptured ectopic pregnancy, ruptured aortic aneurysm, ruptured ovarian cyst, and ruptured spleen), pulmonary embolism, and subarachnoid hemorrhage. Other common causes of syncope include neurocardiogenic syncope (vasovagal or vasodepressor syncope), carotid sinus hypersensitivity, orthostasis, and medication side effects (leading to orthostasis or cardiotoxicity). Patients with a clear cardiac or neurologic cause of syncope, or those with concerning symptoms, should be admitted to inpatient or observational units. Low-risk patients with a history that points to cardiogenic syncope who are asymptomatic can be discharged. Those without a clear etiology must be stratified according to risk for adverse outcomes based on patient history, physical exam, and results of electrocardiogram and other studies.

A 26-year old woman presents to urgent care with a fever of 101.4°F, dysuria and nausea for the past 24 hours. On physical exam, exquisite pain is elicited on palpation of the right flank. She does not appear toxic. She is able to eat and drink, despite her nausea. Her pregnancy test is negative and her urinalysis is pending. What is the best management and treatment for this patient? Inpatient management with intravenous ceftriaxone Inpatient management with intravenous vancomycin Outpatient management with oral ciprofloxacin Outpatient management with oral trimethoprim/sulfamethoxazole

Outpatient management with oral ciprofloxacin Acute pyelonephritis is a common bacterial infection of the renal pelvis and kidney most often seen in young adult women. Most patients have fever, although it may be absent early in the illness. Flank pain is nearly universal, and its absence should raise suspicion of an alternative diagnosis. Risk factors for acute pyelonephritis in nonpregnant women include sexual intercourse three or more times per week during the previous 30 days, urinary tract infections in the previous 12 months, diabetes, stress incontinence in the previous 30 days, a new sex partner in the previous year, recent spermicide use, and a history of urinary tract infections in the patient's mother. A positive urinalysis confirms the diagnosis in patients with a compatible history and physical examination. Urine culture should be obtained in all patients to guide antibiotic therapy if the patient does not respond to initial empiric antibiotic regimens. Escherichia coli is the most common pathogen in acute pyelonephritis, and in the past decade, there has been an increasing rate of E. coli resistance to extended-spectrum beta-lactam antibiotics. Uncomplicated acute pyelonephritis typically occurs in healthy, young women without structural or functional urinary tract abnormalities and without relevant comorbidities. Complicated acute pyelonephritis occurs in patients with a structurally or functionally abnormal genitourinary tract, or a predisposing medical condition. Most cases of uncomplicated acute pyelonephritis can be managed in the outpatient setting. However, patients who appear ill may have severe pyelonephritis or a complication of acute pyelonephritis and should be considered for hospitalization and further evaluation. For uncomplicated pyelonephritis, outpatient treatment with fluoroquinolones is the preferred empiric antimicrobial class in communities where the local prevalence of resistance of community-acquired E. coli is 10 percent or less. If the prevalence of fluoroquinolone resistance among relevant organisms does not exceed 10 percent, patients not requiring hospitalization can be treated with oral ciprofloxacin (500 mg twice per day for seven days), or a once-daily oral fluoroquinolone, such as ciprofloxacin (1,000 mg, extended-release, for seven days) or levofloxacin (750 mg for five days). If the resistance rate exceeds 10 percent, an initial intravenous dose of ceftriaxone or gentamicin should be given, followed by an oral fluoroquinolone regimen.

A 57-year-old man presents to the emergency room after awakening with severe pain in his left big toe. He ate at steakhouse and had several bottles of beer the night before. He has a past medical history of chronic kidney disease. His latest creatinine clearance was 25 mL/min. Joint aspiration reveals negatively birefringent crystals. Which of the following is the most appropriate treatment? Allopurinol Colchicine Indomethacin Prednisone

Prednisone The man in this case has gout and should be managed with prednisone. Acute management of gout includes nonsteroidal anti-inflammatory drugs (NSAIDs), colchicine, and corticosteroids. Gout is the most common crystal-induced arthropathy. Gout may be classified as either primary or secondary. Primary gout is caused by underexcretion or overproduction of uric acid. Secondary gout is related to medications or conditions that induce hyperuricemia. Risk factors for primary gout include genetic predisposition, being male, increasing age, hypertension, and diets high in purines, fructose, and alcohol. Gout most commonly presents as acute pain in the metatarsal-phalangeal joint of the great toe (podagra). The base of the great toe is the first manifestation in approximately half of gout cases. In the majority of cases, a single joint is affected. Involvement of other joints, such as the ankle or wrist can occur, but is more likely in recurrent flares. Classic symptoms of an acute gout flare are severe pain, redness, warmth, and swelling of the affected joint. Onset is commonly sudden and nocturnal. Tophi may be observed in patients with a history of chronic gout. Synovial fluid analysis will have characteristic needle-shaped, negatively birefringent crystals under a polarized light microscope. White blood count of the joint fluid ranges between 10,000-100,000 cells with the majority being neutrophils. Serum uric acid levels during an acute flare are unreliable; values can be low, normal, or high. Treatment of an acute flare involves reducing inflammation and pain. Treatment at the first sign of a flare reduces the duration of the attack. NSAIDs are generally the first line treatment options if there are no contraindications, such as renal failure or active peptic ulcer disease. While indomethacin is traditionally used, most NSAIDs can be used. Colchicine is used less commonly today due to narrow therapeutic window and many side effects. Colchicine should be used with caution in renal failure. Corticosteroids, either orally, intravenously, intramuscularly, or intra-articularly, are the treatment of choice when colchicine and NSAIDs cannot be used. Oral steroids should be tapered to avoid a rebound flare. Allopurinol (A) is used to treat chronic gout and is contraindicated for the treatment of an acute gout flare. Allopurinol blocks xanthine oxidase and thus decreases the production of uric acid. Colchicine (B) should be used with caution in patients with renal failure due to narrowed therapeutic window. Colchicine prevents activation and migration of neutrophils by inhibiting microtubule formation. Colchicine has many unpleasant side effects like diarrhea, nausea, and abdominal pain. NSAIDS, like indomethacin (C), should be avoided in this patient due to his renal insufficiency. NSAIDs reduce the inflammatory response by decreasing proinflammatory cytokine activity and inhibiting neutrophil aggregation.

A patient presents 90 minutes after acute severe headache, nausea, and vomiting. He states it began during sexual intercourse. He has no fever or focal neurologic signs. Which of the following abnormalities would you most likely find on a noncontrast CT of the brain? Bright, high-attenuation density within the subarachnoid space Bright, low-attenuation density within the brainstem Dark, high-attenuation density within the epidural space Dark, low-attenuation density within the ventricles

Bright, high-attenuation density within the subarachnoid space Aneurysmal rupture is the fourth most common cause of cerebrovascular disease and, if large enough, can be fatal. Aneurysms arise from a congenital defect of the vessel wall's internal elastic lamina and media. The majority of all aneurysms occur on the internal carotid artery or its branches. Other locations include the basilar or vertebral arteries or their branches. The most common site is the anterior half of the circle of Willis at a bifurcation of a branching distal artery off of the circle. Most brain aneurysms are recognized only when they rupture, commonly between the ages of 35-65 years, resulting in subarachnoid hemorrhage and presenting as acute onset of severe headache, nausea, vomiting, and meningeal signs without fever. Rupture is not necessarily related to chronic hypertension. Sometimes rupture follows a Valsalva maneuver, intense physical effort, or sexual intercourse. Diagnosis is confirmed with CT scanning (depending on the time of symptom onset) or the presence of blood on a lumbar puncture or an aneurysm on CT angiography in the setting of a normal noncontrast head CT. On a noncontrast brain CT, fresh subarachnoid hemorrhage appears as a bright, white, high-attenuating, amorphous substance within the normally dark CSF-filled subarachnoid spaces (noncontrast CT bright = bone, clotted blood; dark = air, fluid, fat; gray = brain). Complications include mass effect, vasospasm, and cerebral infarction but also hydrocephalus and the syndrome of inappropriate antidiuretic hormone secretion. Definitive treatment is built on early diagnosis, defining the vasculature with angiography, and neurosurgical clipping.

An 8-year-old boy presents with a 2-day history of severe, watery diarrhea. He is currently living in a Haitian refugee camp. On exam, the child is afebrile but appears dehydrated. He is actively passing a large amount of watery diarrhea that is flecked with mucous. Darkfield microscopy of the stool reveals mobile organisms. Which of the following is the most likely diagnosis? Cholera Rotavirus Salmonellosis Shigellosis

Cholera This patient most likely has cholera. Cholera is an infectious diarrheal illness caused by a toxin-producing strain of Vibrio cholerae. Cholera is most commonly seen in resource-limited settings with poor access to clean water sources. Cholera is endemic in approximately 50 countries, many of which are in Asia, Africa, and South America. V. cholerae is a gram-negative curved rod that is highly motile and halophilic. Cholera is transmitted through ingestion of contaminated food or water. The incubation period varies based on the size of inoculum and host immune status. Average incubation period is 1-2 days, although it can range from a few hours to 5 days. Diarrhea is the hallmark symptom. Other common symptoms include abdominal discomfort, borborygmi, and vomiting. Fever and tenesmus are uncommon findings. The diarrhea associated with cholera is classically described as rice-water stool and typically has a fishy odor. Signs of severe dehydration may develop within hours of diarrhea onset. Signs of severe hypovolemia include sunken eyes, dry mucous membranes, cold skin, decreased skin turgor, apathy, and lethargy. The most common laboratory abnormality is isonatremic dehydration. Diagnosis is based on clinical manifestations but can be confirmed with stool culture or darkfield microscopy. Aggressive fluid repletion is the main treatment of cholera. Mild to moderate cases can be treated with oral rehydration solution. Severe cases may require intravenous fluids. Antibiotics are recommended in moderate to severe cases. Choices of antibiotics include fluoroquinolones, macrolides, and tetracyclines. A cholera vaccine is available for endemic areas and in areas at risk for an outbreak.

A 16-year-old boy is brought to the emergency department after being hit by a car while riding his bike. He is somnolent, opens his eyes to pain, responds to questions with inappropriate words, and uses his left hand to stop the physician attempting to start an IV in his right arm. What is the patient's Glasgow coma score? 10 11 12 9

10 The Glasgow coma scale (GCS) is a grading of patient responsiveness based on three components: eye opening (1 to 4 points), verbal response (1 to 5 points), and motor response (1 to 6 points). The severity of traumatic brain injury (TBI) is defined using the GCS with mild TBI being defined as a GCS of 14-15, moderate TBI a GCS of 9-13, and severe TBI a GCS 3-8. While other grading tools for mental status exist, the GCS is commonly used due to its interobserver reliability and prognostic value. The Glasgow coma scale has been shown to have prognostic value in traumatic brain injury, spontaneous subarachnoid hemorrhage, and bacterial meningitis. Limitations of the scale include its decreased utility in the setting of drugs, alcohol, sedation, and intubation and its reliance on behavioral response without regard for the underlying pathophysiology. The patient above would receive a score of 2 for eye opening, 3 for verbal response, and 5 for motor response for a total of 10.

A 28-year-old man presents to your office for follow up on a positive HIV test. He has never taken antiretroviral medications and his CD4 count is 610 cells/mm3. Which of the following medication regimens do you begin? 1 nucleoside/nucleotide reverse transcriptase inhibitor and 1 non-nucleoside reverse transcriptase inhibitor 2 nucleoside/nucleotide reverse transcriptase inhibitors and 1 non-nucleoside reverse transcriptase inhibitor 2 protease inhibitors and 2 non-nucleoside reverse transcriptase inhibitors The patient does not need antiretroviral therapy at this time due to a high CD4 count

2 nucleoside/nucleotide reverse transcriptase inhibitors and 1 non-nucleoside reverse transcriptase inhibitor The United States Department of Health and Human Services recommends an initial antiretroviral regimen for HIV treatment-naïve patients that includes two nucleoside/nucleotide reverse transcriptase inhibitors (NRTIs) with a third agent from one of the following drug classes: a non-nucleoside reverse transcriptase inhibitor (NNRTI), a protease inhibitor boosted with ritonavir or an integrase inhibitor. Regimen selection is made on an individualized basis after consideration of pill burden, drug interactions, comorbid conditions, cost and resistance testing results.

A 68-year-old woman presents with difficulty walking. Her neurologic examination is notable for ataxia. An MRI of the brain does not show any acute pathology. On laboratory analysis, her sodium is 108 mEq/L. Which of the following is the most appropriate next step in management? 0.9% NaCl 500 mL bolus 3% NaCl 100 mL bolus Free water restriction Mannitol

3% NaCl 100 mL bolus Hyponatremia is defined as a serum sodium level less than 135 mEq/L. There are multiple causes of hyponatremia and when developing a management plan it is critical to determine the underlying etiology. In general, the volume status of the patient helps identify the cause. The list of causes is different in cases of hypovolemic hyponatremia, hypervolemic hyponatremia, and euvolemic hyponatremia. Treatment decisions are made based on the etiology, the patient's clinical presentation, and duration of illness. Sodium is typically corrected over 48 to 72 hours. Treatment typically consists of either fluid replacement or fluid restriction depending on the cause. In some cases of severe hypervolemia, loop diuretics may be added to the therapy. As the sodium drops, patients begin to develop symptoms including headache, nausea, vomiting, confusion, seizures, neurologic abnormalities, and coma. The central nervous system effects are likely caused by some degree of cerebral edema resulting from osmotic fluid shifts. Additionally, aggressive treatment with correction of the serum sodium values too quickly may cause neurologic damage and central pontine myelinolysis. Patients with neurologic symptoms require treatment with hypertonic (3%) saline. This is typically administered as a 100 ml bolus over 10 minutes. Severe hyponatremia is defined as a level below 120 mEq/L and treatment is initiated at this level even in asymptomatic patients.

According to the American Diabetes Association (ADA) glycemic control goals, which of the following scenarios would prompt a change in insulin dosing in a patient with type 1 diabetes mellitus? A 10-year-old boy whose fasting serum glucose is 130 mg/dL A 14-year-old girl whose hemoglobin-A1c is 6.8% A 4-year-old girl whose hemoglobin-A1c is 7.8% An 8-year-old boy whose hemoglobin-A1c is 5.9%

A 4-year-old girl whose hemoglobin-A1c is 7.8% Both forms of diabetes are properly treated with a multidisciplinary approach. Diet modification and education are necessary (dietitian). Disease education is helpful (primary or advance practice provider). Injection procedure and glucose monitoring education is also important (primary or advanced practice provider). Exercise modification may also be needed (physical therapist). There are several different classes of medications used in the treatment of diabetes. A special case is the child with type 1 diabetes mellitus. This child will likely be on insulin therapy for life. Therefore, monitoring and regimen modification becomes a large responsibility of primary care providers. The ADA recommends hemoglobin-A1c testing for monitoring and insulin adjustment. In 2014, the ADA released a position statement on the diagnosis and management of type 1 diabetes in all age groups. The statement includes a new pediatric glycemic control target of HbA1c of less than 7.5% (58 mmol/mol)across all pediatric age groups, replacing earlier guidelines that specified different glycemic control targets by age. The adult HbA1c target of less than 7% did not change. The 4-year-old girl whose hemoglobin-A1c is 7.8% needs a change (increase) in her insulin dosing as her hemoglobin-A1c is too high and the target A1c level is < 7.5%.

Which of the following patients most likely has dementia? A confused 65-year-old man brought from a house fire who exhibits good long-term recall, no recall of immediate events, normal vital signs, and carboxyhemoglobin level of 7% A confused 65-year-old man brought from a house fire who exhibits poor long-term recall, no recall of immediate events, normal vital signs, and carboxyhemoglobin level of 25% A confused 80-year-old man brought from home with fever, back pain, and urinary retention A confused 80-year-old man brought from home with fever, cough, and hypoxia A confused 80-year-old man brought from home with increased thirst, increased urination, and blood sugar monitor that reads high

A confused 65-year-old man brought from a house fire who exhibits good long-term recall, no recall of immediate events, normal vital signs, and carboxyhemoglobin level of 7% Dementia results from a gradual loss of mental capacity with the preservation of motor and speech. Dementia generally occurs in the elderly. Remote memories are often preserved. The most common types of dementia are Alzheimer disease and vascular dementia, both insidious in onset. Symptoms may worsen acutely because of a concurrent medical condition. Physical exam and laboratory workup are generally unrevealing. Of the choices, a confused 65-year-old man brought from a house fire who exhibits good long-term recall, no recall of immediate events, normal vital signs, and a carboxyhemoglobin level of 7% is most likely to have dementia.

A 25-year-old woman with a recent history of antibiotic use presents to your office with a complaint of vaginal discomfort. For the past week she has been experiencing intense vaginal pruritus and has noticed a white discharge. Which of the following is the most appropriate next step in her management? A single dose of 150 mg fluconazole Boric acid intravagina for seven days Metronidazole 500 mg twice/day for seven days Topical nystatin 100,000 units daily for seven days

A single dose of 150 mg fluconazole Vulvar or vaginal pruritus is the most common clinical symptom of vulvovaginal candidiasis. Risk factors include recent antibiotic use, poor glycemic control in women with diabetes mellitus, immunosuppression, increased estrogen levels and use of certain contraceptive devices such as diaphragms. Vaginal discharge is not always seen in women with vulvovaginal candidiasis, however when present it is generally thick and white with a clumpy, cottage cheese-like texture. First line treatment is a single dose of fluconazole, 150 mg in non pregnant women. Oral fluconazole should be avoided in pregnant patients and instead vaginal clotrimazole or miconazole should be used.

A 19-year-old man presents with a maculopapular rash. Three weeks ago, he noticed an ulcer on his penis, but he did not seek medical attention because it resolved spontaneously. He reports associated malaise and myalgias. He denies penile discharge and dysuria. Skin examination reveals a maculopapular rash which includes his palms and soles. His genital examination is normal. Which of the following is the most appropriate treatment regimen? A single dose of oral azithromycin A single dose of oral ciprofloxacin A single intramuscular injection of benzathine penicillin G Weekly intramuscular injections of benzathine penicillin G for 3 weeks

A single intramuscular injection of benzathine penicillin G This patient has secondary syphilis, an infection caused by the spirochete Treponema pallidum. Syphilis infection proceeds in different stages, each with different signs and symptoms, as well as recommended treatment regimens. Secondary syphilis is characterized by a maculopapular, copper-colored rash and associated fever, malaise, and myalgias. Primary syphilis precedes this stage and is characterized by a painless ulcer in the genital area without systemic signs or symptoms. Primary, secondary, and early latent syphilis (< 1 year from inoculation) are treated with a single intramuscular injection of benzathine penicillin G. Doxycycline, tetracycline, and erythromycin can be used in penicillin allergic patients.

A 32-year-old man presents with an acute onset of right-sided facial weakness that he first noticed upon waking this morning. You ask him to smile, as seen in the image above. Which of the following clinical features, if present, suggests a more severe diagnosis? A vesicle found on the right tympanic membrane Dysgeusia Hyperacusis Unilateral symptoms

A vesicle found on the right tympanic membrane The patient has idiopathic facial nerve paralysis (Bell palsy). Bell palsy is characterized by a rapid onset of unilateral facial paralysis. More than half of patients have a prodromal viral illness (rhinorrhea, myalgias, low grade fever). Bell palsy is associated with ear pain, decreased tearing, an overflow of tears onto the cheek (epiphora), hyperacute hearing (hyperacusis), and dysgeusia (an impairment of taste). Idiopathic facial nerve paralysis involves the lower motor neuron. Features that suggest an upper motor neuron etiology include a slow progression of symptoms, recurrent symptoms, and intact forehead movement. A herpetiform vesicular eruption on the tympanic membrane, pinna, external auditory canal, soft palate, oral cavity, face, or neck suggests a diagnosis of Ramsay Hunt syndrome (herpes zoster oticus). In Ramsay Hunt syndrome, the pain is more severe than in Bell palsy, and the prognosis is worse, with a lower incidence of complete facial recovery and the possibility of associated sensorineural hearing loss.

A 35-year-old-man with a long history of occasional bloody diarrhea and abdominal pain presents with acute-onset severe abdominal pain. Vital signs are significant for a temperature of 102.2°F (39°C), heart rate 140 bpm, and blood pressure 82/55 mm Hg. On physical exam, his abdomen is distended and tympanitic. Which of the following diagnostic studies is indicated first at this time? Abdominal radiograph Barium enema CT scan of the abdomen pelvis with IV contrast Sigmoidoscopy

Abdominal radiograph The patient's presentation is suspicious for toxic megacolon, a complication of inflammatory bowel disease. The hallmark of toxic megacolon is colonic dilatation in a patient with a known inflammatory condition of the colon who appears systemically toxic. Patients usually have experienced symptoms of colitis for several days before the onset of toxic megacolon. Diagnosis is made with an abdominal radiograph that reveals dilatation of the colon with a diameter > 6 cm. The presence of inflammation and toxicity differentiates toxic megacolon from other disorders that cause colonic dilatation (mechanical obstruction, pseudo-obstruction, and congenital megacolon).

A 12-year-old boy presents to the ER complaining of right testicular pain. Symptoms began 1 hour ago. He describes the pain as a 10/10 stabbing sensation radiating from the right testicle to the right groin. He also complains of 3 episodes of emesis and intermittent nausea. A bedside testicular ultrasound is performed which shows decreased blood flow to the right testicle. What is the most sensitive physical exam finding associated with this pathology? Absence of the cremasteric reflex in the affected testicle High-riding appearance of the affected testicle Relief of pain with elevation of the affected testicle Tenderness and firmness of the affected testicle

Absence of the cremasteric reflex in the affected testicle Testicular torsion is characterized by twisting of the spermatic cord, is most commonly due to an anatomical defect and can result in ischemia of the testicle. The anatomical defect involves abnormal anchoring of the testicle within the tunica vaginalis which allows it to move freely in the scrotum.This free movement of the testicle mimics the movement of a clapper inside a bell and is therefore referred to as a "bell clapper deformity." Risk factors for testicular torsion include history of cryptorchidism, increased length of the spermatic cord, and horizontal lie of the testis. Although testicular torsion can occur in any age group, peak incidence occurs during the first year of life and during puberty (due to increase in testicular volume). Minor trauma can also precipitate testicular torsion. The classic presentation involves acute onset, unilateral testicular or abdominal pain, often with associated nausea and vomiting. Physical exam findings include a firm, tender, and high-riding testicle with a transverse lie. Absence of the cremasteric reflex is the most sensitive physical exam finding. This reflex is elicited by stroking or pinching the medial thigh, causing contraction of the cremaster muscle, which elevates the testis. The cremasteric reflex is considered positive if the testicle moves at least 0.5 cm. However, lack of this finding does not rule out testicular torsion. Ultrasound imaging is used to confirm the diagnosis by demonstrating a diffusely hypoechoic testicle with decreased blood flow. Management includes emergent urology consult and surgery. Salvage rate of the testicle is highest if treatment is initiated within 6 hours from symptom onset.

A 58-year-old man who is admitted to the hospital for an infected diabetic foot ulcer suddenly develops shortness of breath and decreased consciousness. His history includes insulin-dependent diabetes mellitus, hypertension, and dyslipidemia. He is currently receiving intravenous antibiotics for an infected foot wound, insulin, oral antihypertensives, and an oral statin. Last night, he became febrile but had a negative chest X-ray. His blood pressure is 90/60 mm Hg, pulse is 120 bpm, respirations are 22 per minute, temperature is 101.9°F, oxygen saturation is 80% on room air, and blood glucose level is 190 mmol/L. Physical exam reveals a purulent foot wound, an obtunded patient, decreased breath sounds with crackles bilaterally, weak pulses, cool extremities, and perioral cyanosis. A complete blood count is normal except for leukocytosis. A chest X-ray is shown above ("bilateral fluffy infiltrates"). Which condition most likely led to this patient's clinical presentation? Accumulation of fluid in the alveolar spaces Bacterial infiltrate in the interstitium Diffuse alveolar hemorrhage Eosinophilic proliferation in the interstitium Malignant cells in the endobronchial tissue

Accumulation of fluid in the alveolar spaces Acute respiratory distress syndrome (ARDS) is an acute lung injury due to the accumulation of fluid in the alveolar spaces. This fluid inhibits gas exchange and leads to hypoxia. Sepsis is the most common cause of ARDS, where acute inflammation of the alveolar walls causes the release of cytokines and neutrophils that further destroy the alveolar endothelium. Other causes of ARDS include blunt lung trauma, aspiration of gastric contents, transfusion-related acute lung injury, and community-acquired or nosocomial pneumonia. Common symptoms of ARDS appear 6 hours to 3 days after the inciting event and can include dyspnea, cyanosis, diffuse crackles on auscultation, tachypnea, diaphoresis, cough, and chest pain. X-ray of the chest reveals bilateral fluffy infiltrates. Treatment of ARDS involves oxygenation with mechanical ventilation, fluid management, and addressing the inciting event. The patient above would need blood cultures, aggressive fluid replacement for hemodynamic stabilization, and appropriate antibiotic treatment, in addition to mechanical ventilation.

A 26-year-old woman presents to the clinic reporting intermittent right-sided throbbing headaches for the past month. She has had no recent illnesses. The patient states that she is 34 weeks pregnant and that she had similar headaches prior to her pregnancy that resolved with ibuprofen and rest. The headaches have a gradual onset and are exacerbated by light and sound. Vital signs include a heart rate of 92 bpm, blood pressure of 120/80 mm Hg, respiratory rate of 20/min, oxygen saturation of 98% on room air, and temperature of 98.6°F. Physical examination reveals the patient to be in moderate distress due to the pain but with no nuchal rigidity or lower extremity edema. The patient has intact sensory and motor function, a normal cranial nerve exam, a steady gait, and intact finger-to-nose testing. Which of the following is the recommended initial treatment for the patient's symptoms? Acetaminophen Acetaminophen-butalbital-caffeine Naproxen Oxycodone Sumatriptan

Acetaminophen Migraine headaches are a common type of primary headache characterized by a throbbing unilateral headache associated with nausea and vomiting. They are more common in women than men. Migraine headaches are diagnosed clinically based on the patient meeting criteria listed in the International Classification of Headache Disorders. They may be associated with known triggers, such as nitrates, fasting, visual changes, or stress. The recommended treatment is different in pregnant patients because of concern regarding adverse fetal drug effects. Acetaminophen is recommended initially because it is effective and has the best maternal-fetal safety profile. However, migraine headaches that are not relieved with acetaminophen alone can be treated with acetaminophen combination therapy, such as acetaminophen and metoclopramide, acetaminophen and codeine, and butalbital-acetaminophen-caffeine. Patients who do not respond to acetaminophen or these acetaminophen combinations should be treated with aspirin, naproxen, ibuprofen, or ketorolac. Nonsteroidal anti-inflammatory drugs are most safe in the second trimester before 20 weeks and should be avoided after 30 weeks due to concerns about prenatal constriction of the ductus arteriosus, persistent pulmonary hypertension of the newborn, oligohydramnios, necrotizing enterocolitis, kidney dysfunction or failure, and intracranial hemorrhage. Opioids are considered a third-line therapy for migraine headaches during pregnancy and should be used at the lowest effective dose for the shortest duration. Opioids can cause maternal addiction and neonatal withdrawal. Furthermore, some data has shown an increased risk of fetal nervous system malformations when exposed to opioids in utero. Triptans, such as sumatriptan, are another medication class that can be used in patients who do not respond to several other medications. However, there is a theoretical risk of vasoconstriction of uteroplacental vessels.

A ten-year-old girl presents with drooping eyelids for the past month. The patient reports some difficulty completing her schoolwork due to blurry vision and hand weakness. On initial observation, her exam is notable for bilateral ptosis. Which of the following is the most likely pathophysiology behind her symptoms? Acetylcholine receptor antibodies Acetylcholinesterase deficiency Anti-thyroid antibodies Toxin exposure

Acetylcholine receptor antibodies Antibodies to the acetylcholine receptor are the most common cause of myasthenia gravis. Myasthenia gravis is a chronic, autoimmune disease characterized by fatigue of striated muscle. The first clinical signs are ptosis or extraocular muscle weakness or both. Later symptoms include bulbar muscle weakness, limb-girdle weakness, and hand weakness. This disease is notable for the rapid fatigue of muscles, which may be demonstrated by sustaining an upward gaze for 30-90 seconds or repetitive opening and closing of the fists. Symptoms increase throughout the day. Myasthenia gravis is a progressive disease and may be fatal due to respiratory muscle compromise or aspiration. A clinical diagnosis may be made with the administration of edrophonium chloride (Tensilon test), which causes immediate improvement in the ptosis and ophthalmoplegia. In infants, Neostigmine should be used to make the diagnosis, as edrophonium has increased risk of cardiac dysrhythmias in infants. Cholinesterase inhibitors are the first line of therapy. Plasmapheresis and IVIG, as well as thymectomy, can be used in refractory cases.

A 34-year-old man presents complaining of an increase in the size of his gloves and shoes. Physical examination reveals prognathism, oily skin, macroglossia, and visual field deficits. Vital signs are temperature 98.6°F, respiratory rate 16 breaths/minute, blood pressure 123/65 mm Hg, heart rate 68 bpm, and oxygen saturation 98% on room air. Laboratory studies reveal elevated serum insulin-like growth factor 1 and growth hormone. Which of the following is the most likely diagnosis? Acromegaly Central diabetes insipidus Diabetes mellitus type 2 Gigantism

Acromegaly Acromegaly results from persistent hypersecretion of growth hormone, which in turn causes hepatic secretion of insulin-like growth factor 1 (IGF-1). The majority of clinical manifestations of acromegaly result from hypersecretion of insulin-like growth factor 1. Acromegaly is often caused by a pituitary macroadenoma (> 1 cm in size) and is usually associated with multiple endocrine neoplasia type 1 or 4, McCune-Albright syndrome, Carney syndrome, or ectopic secretion of growth hormone (by a pancreatic tumor, hypothalamic tumor, bronchial carcinoid, or lymphoma). The height of the patient is determined by the age at presentation. Children have unfused growth plates and, therefore, develop linear height and gigantism. Adults with acromegaly do not have an increase in height but may have an increase in the size of hands, feet, facial features, and heart. Patients typically present with increasing hat size, shoe size, or glove size. Physical examination may show enlarged and doughy hands and feet, moist handshake, coarse facial features, oily skin, prognathism, malocclusion, widened tooth spacing, macroglossia, visual field deficits, and hypertrophy of the pharyngeal and laryngeal tissue. Comorbidities, such as sleep apnea, diabetes, hypertension, cardiomegaly, insulin resistance, dilated left ventricle, and heart failure, may be present. The best single test for the diagnosis of acromegaly is a serum insulin-like growth factor 1. Unequivocally elevated IGF-1 in the presence of the typical clinical manifestations confirms the diagnosis of acromegaly. Normal serum concentration of IGF-1 excludes the diagnosis of acromegaly. If the serum IGF-1 is equivocal, serum growth hormone should be measured after oral glucose administration. Inadequate suppression of the growth hormone after a glucose load confirms the diagnosis of acromegaly. Other laboratory studies, including complete blood count, comprehensive metabolic panel, free T4, and thyroid-stimulating hormone, should be obtained to exclude other diagnoses. Once growth hormone hypersecretion has been confirmed, MRI of the pituitary should be obtained, which can detect a pituitary adenoma. Treatment is by transsphenoidal resection of the pituitary adenoma.

A 15-year-old boy comes to the emergency department due to yellowish discoloration of the eyes for two days. He is noted to have generalized body weakness, poor appetite, nausea, and vomiting. The physical examination reveals a febrile boy with a temperature of 37.9°C in no cardiorespiratory distress with icteric sclerae, yellow-tinged oral mucosa, clear breath sounds, a soft abdomen with mild tenderness at the right upper quadrant, full and equal pulses, and no edema noted. Which of the following laboratory results will point to an acute hepatitis A infection? Anti-Hepatitis A Virus IgG positive Anti-Hepatitis A Virus IgM positive Elevated serum alanine aminotransferase, aspartate aminotransferase, and bilirubin Elevated serum alkaline phosphatase and 5′-nucleotidase

Anti-Hepatitis A Virus IgM positive The child presents with common manifestations of viral hepatitis. There are 5 pathogenic hepatotropic viruses which include hepatitis A, B, C, D, and E viruses. Both hepatitis A and E can be transmitted via fecal-oral route while hepatitis B, C, and D are transmitted parenterally and sexually. Other than jaundice, hepatitis presents with nonspecific signs and symptoms which include malaise, anorexia, nausea and vomiting, diarrhea and abdominal pain. Depending on the pathogenic strain, viral hepatitis can lead to acute liver failure or to a chronic disease. Diagnosing the specific viral hepatitis strain requires testing for the presence of antigens and antibodies. In the case of hepatitis A, acute infection can be detected by the presence of anti-hepatitis A virus IgM. Patients usually undergo full recovery with no long-term complications noted. There is no specific medication needed for hepatitis A infection. Supportive treatment can be given such as intravenous hydration, antiemetics, and fat-soluble vitamins. Proper handwashing is recommended to avoid transmission of disease as it is contagious two weeks before until seven days after the onset of jaundice.

A 17-year-old girl is brought into the emergency department by a friend after being found lethargic with an empty pill bottle at her bedside. An initial arterial blood gas shows a pH of 7.28, pCO2 of 55 mm Hg, and a serum HCO3 of 24 mEq/L. Which one of the following acid-base disturbances is present? Acute (compensated) primary metabolic alkalosis Acute (uncompensated) primary metabolic acidosis Acute (uncompensated) primary respiratory acidosis Uncompensated mixed metabolic/respiratory acidosis

Acute (uncompensated) primary respiratory acidosis This patient's arterial blood gas (ABG) shows an abnormally low pH (acidosis) with an elevated pCO2 and a normal bicarbonate level, consistent with an uncompensated primary respiratory acidosis. Normal values are pH 7.36-7.44, pCO2 36-44 mm Hg, and HCO3 22-26 mEq/L. The primary disturbance will either be respiratory (change in pCO2) or metabolic (change in HCO3). The body will attempt to compensate by retaining or exhaling CO2 (if metabolic) or HCO3 (if respiratory). For example, in the above case, the patient's kidneys will increase levels of HCO3 to counteract the elevated CO2 in the blood. Since this is not seen (HCO3 is normal), it can be assumed that this disturbance is acute and uncompensated.

A 58-year-old woman presents to the emergency department with right upper quadrant abdominal pain that has progressed throughout the day. She currently rates the pain a 9 out of 10. Her vitals are heart rate of 102 bpm, respiratory rate of 18/minute, oxygen saturation on room air of 99%, temperature of 101.2°F, and blood pressure of 125/86 mm Hg. Her physical exam is significant for scleral icterus and significant right upper quadrant tenderness to palpation without any rebound. Laboratory results show leukocytosis with a white blood cell count of 17,000/µL, elevated alanine aminotransferase at 202 units/L, elevated aspartate aminotransferase at 220 units/L, and elevated total bilirubin at 7.2 mg/dL. Which of the following is the most likely diagnosis for this patient? Acute cholangitis Acute cholecystitis Choledocholithiasis Cholelithiasis Primary sclerosing cholangitis

Acute cholangitis This patient most likely has acute cholangitis, which is a biliary tract infection due to obstruction of the common bile duct. This condition is most commonly due to Escherichia coli. Patients typically present with a triad of symptoms, including fever or chills, right upper quadrant abdominal pain, and jaundice. This is known as the Charcot triad. If the patient also presents with altered mental status and hypotension, the diagnosis would be acute suppurative cholangitis. The combination of these five symptoms is known as the Reynolds pentad and is a medical emergency. Patients with acute cholangitis will have a transient increase in aspartate aminotransferase and alanine aminotransferase, elevated bilirubin, and leukocytosis. The gold standard for diagnosis is a cholangiography via ERCP. However, there are times where an ultrasound or CT scan is done, which would reveal a dilated common bile duct. Treatment consists of broad-spectrum antibiotics such as piperacillin-tazobactam and drainage of the biliary ducts via ERCP.

A 52-year-old man presents with right eye redness that is associated with severe pain and blurred vision. On examination, there is some cloudiness to the cornea. Pupils are 5 mm OD and 3 mm OS. Which of the following is the most likely diagnosis? Acute glaucoma Conjunctivitis Iritis Keratitis

Acute glaucoma Glaucoma is characterized by an increase in intraocular pressure leading to irreversible damage to the optic nerve. There are two forms of glaucoma: open-angle and angle-closure. Open-angle glaucoma is the more common form and is most often seen in the elderly population. Acute angle-closure glaucoma is an ophthalmologic emergency and is caused by closure of the anterior chamber. It presents with severe pain and profound vision loss. Symptoms are rapid in onset and usually unilateral. As intraocular pressure increases, patients may experience headache, nausea, and vomiting. Exam reveals a red, tender globe that may be firm to the touch. A steamy or hazy cornea may be seen with a nonreactive, mid-dilated pupil. The diagnosis can be supported with tonometry demonstrating an elevated pressure. Treatment should be initiated to reduce the intraocular pressure and include topical beta-blockers, topical alpha-agonists, and carbonic anhydrase inhibitors. Patients require emergent ophthalmologic consultation.

An 18-year-old woman presents to the ED with dark-colored urine and malaise for the past three days. Her vital signs are BP 155/85 mm Hg, HR 80 bpm, RR 16/min, and T 36.7°C. On exam, you note 1+ pretibial edema. Urinalysis reveals proteinuria, hematuria, and red blood cell casts. Which of the following is the most likely diagnosis? Acute glomerulonephritis Acute tubular necrosis Minimal change disease Nephrotic syndrome

Acute glomerulonephritis Patients with acute glomerulonephritis present with a spectrum of clinical signs and symptoms. The presence of hematuria, proteinuria, and red blood cell casts are highly suggestive of the diagnosis. In fact, the presence of red blood cell casts alone is highly specific for acute glomerulonephritis. Conversely, if hematuria, proteinuria, and red blood cell casts are absent, glomerulonephritis is highly unlikely. In addition to the urinary findings, patients may present with hypertension, edema, or congestive heart failure secondary to volume overload. Acute tubular necrosis (B) is an intrinsic cause of acute kidney injury that is associated with a variety of renal insults, including ischemia and nephrotoxins. Muddy brown casts are often seen on urinalysis. Minimal change disease (C) is more common in pediatric patients and presents with proteinuria, edema, and hypertension. This condition is usually very responsive to steroids. Nephrotic syndrome (D) causes edema and proteinuria but is not associated with hematuria or red blood cell casts

A previously healthy 63-year-old woman presents to your office with a complaint of a painful rash that started two days ago. Physical exam shows a grouping of vesicles on an erythematous base just distal to the right scapula. There are no other skin lesions present. Which of the following is the most appropriate therapy? Acyclovir Amitriptyline Cephalexin Ganciclovir

Acyclovir Varicella-zoster virus (VZV) is the virus responsible for chickenpox and herpes zoster, also called shingles. Chicken pox is the initial infection that occurs after exposure to the virus. Herpes zoster occurs when the dormant virus is reactivated. Herpes zoster can occur at any age, but is more common in individuals older than 50 years. Clinical presentation of herpes zoster is a painful, unilateral, vesicular rash that occurs in a dermatomal distribution. In patients who are immunocompetent, the vesicles crust in 7-10 days and are no longer considered to be contagious at that point. Diagnosis is determined based on clinical presentation. Treatment decisions are based on duration of symptoms, clinical presentation, patient's age and immune state. Immunocompetent patients older than age 50 years who present within 72 hours of the onset of symptoms should be treated with an antiviral medication such as acyclovir (or valacyclovir) to help decrease symptoms and duration of the infection. The herpes zoster vaccine is recommended for individuals aged 50 years and older to help prevent herpes zoster infection.

Which one of the following is the most likely cause of acute kidney injury in a patient with eosinophiluria? Acute interstitial nephritis Ethylene glycol poisoning Poststreptococcal glomerulonephritis Rhabdomyolysis

Acute interstitial nephritis Acute interstitial nephritis is an important cause of acute renal failure resulting from immune-mediated tubulointerstitial injury. The presence of eosinophiluria in a patient with acute kidney injury suggests acute interstitial nephritis, which is typically an allergic reaction to medications such as penicillins, sulfa-containing antibiotics and diuretics, NSAIDs, and proton pump inhibitors. Patients with acute interstitial nephritis may also present with a rash, fever, eosinophilia, and other constitutional symptoms. The combination of elevated levels of creatine kinase or myoglobin, a dipstick positive for blood but negative for RBCs, and a history of muscle trauma would suggest rhabdomyolysis (D). Poisoning with ethylene glycol (B) or methanol should be suspected in a patient with acute kidney injury and altered mental status with an increased anion gap and osmolar gap. Ethylene glycol poisoning is associated with calcium oxalate crystals in the urine. An elevated antistreptolysin O titer when combined with the presentation of a full-blown nephritic syndrome with oliguric acute renal failure suggests poststreptococcal glomerulonephritis (C).

A 15-year-old boy presents to the emergency department with lower back pain. He reports heavy lifting 3 weeks ago, with gradual onset of pain and no relief with symptomatic care, including over-the-counter medication. The pain is localized to the paraspinal muscles over his lumbar back. He reports normal urine output. Baseline laboratory tests are notable for a serum sodium of 140 mEq/L, potassium of 5.2 mEq/L, chloride of 110 mEq/L, bicarbonate of 25 mEq/L, BUN of 20 mg/dL, and creatinine of 2.3 mg/dL. A urinalysis shows hyaline casts, 5 white blood cells/hpf, and 1+ protein. Which of the following is the most likely pathophysiology of his laboratory findings? Acute interstitial nephritis Acute tubular necrosis Hypovolemia Obstructive nephropathy

Acute interstitial nephritis This patient has acute kidney injury (AKI), exhibited by his elevated creatinine level. The common causes of AKI can be subdivided into three categories: prerenal, intrinsic renal, and postrenal. Intrinsic renal pathology can be further subdivided into glomerular disease, interstitial disease, vascular disease and tubular disease. This patient has intrinsic renal disease, as his BUN to creatinine ratio is less than 15 and his urine output is normal. The most likely etiology of his intrinsic renal disease is acute interstitial nephritis (AIN) secondary to overuse of nonsteroidal anti-inflammatory drugs for back pain. Many drugs and toxins can cause AIN. It typically presents 1 to 2 weeks following onset of the inciting agent, and may be associated with fever, rash, arthralgias, and eosinophilia. Urine output is typically preserved. Urinalysis may show white blood cells and hyaline or granular casts, as well as mild proteinuria. Removal of the offending agent often leads to recovery.

A 38-year-old woman was admitted to the intensive care unit four days ago after a suicide attempt from an ibuprofen overdose. Today she exhibits a sudden onset of decreased urine volume, altered mental status and generalized edema. Laboratory findings show increased levels of blood urea nitrogen and creatinine. Which of the following is the most likely cause? Acute kidney injury Diabetic ketoacidosis Heart failure Urinary tract infection

Acute kidney injury Acute kidney injury (AKI), previously referred to as acute renal failure, is an abrupt decline in renal filtration function that is generally reversible. AKI has many different possible causes and is categorized by pre-renal, intrinsic and post-renal etiologies. Pre-renal causes are the most common and can lead to intrinsic injury if not addressed promptly. In hospitalized patients AKI is most commonly seen when patients develop acute tubular necrosis due to sepsis, ischemia or nephrotoxic exposure, such as with a drug overdose. Diagnostic testing for AKI includes complete blood count, kidney function studies, urinalysis and renal ultrasound. Increased levels of blood urea nitrogen and creatinine are the classic laboratory findings in AKI. Initial management is to correct the life-threatening electrolyte and fluid abnormalities along with determination and correction of the original cause of the injury.

A 25-year-old woman comes to the office with diffuse pain, multiple tender points, and generalized fatigue. A full workup revealed no connective tissue disease, but you suspect fibromyalgia. Which of the following can be used for the initial treatment of fibromyalgia? Aerobic exercise High-dose opioids Physical therapy Trigger point injections

Aerobic exercise Aerobic exercise increases cardiovascular fitness and reduces pain and other fibromyalgia symptoms. Fibromyalgia is characterized by high levels of pain, sleep disturbance, and fatigue combined with a general increase in medical symptoms, including problems of memory or thinking and often psychological distress. Many issues that surround fibromyalgia are not scientific ones. It is widely agreed that pain and suffering are real. Instead, the primary issues are often social, political, and financial. Fibromyalgia lies at the end of a continuum of polysymptomatic distress rather than being a discrete disorder. Fibromyalgia may be diagnosed with American College of Rheumatology (ACR) 2010 or 1990 criteria, but clinical care does not require a diagnosis.

A 5-year-old boy is brought by his parents to the clinic for a limp. He has been having fever and bone pain for 2 weeks. For the past few days, the parents noted that he was having difficulty walking. Physical exam reveals multiple inguinal nodes that are nontender, firm, rubbery, and 20 mm in diameter. There is no evidence of point tenderness on palpation. Laboratory testing shows anemia, thrombocytopenia, and neutropenia. There are lymphoblasts on peripheral smear. Which of the following is the most likely diagnosis? Acute lymphocytic leukemia Epstein-Barr virus infection Immune thrombocytopenia Osteomyelitis

Acute lymphocytic leukemia The boy presents with limping, bone pain, fever, lymphadenopathy and lymphoblasts on peripheral smear. These findings are suspicious for acute lymphocytic leukemia (ALL), also known as acute lymphoblastic leukemia. Acute leukemia comprises approximately 30 percent of all childhood malignancies. Each year in the United States, approximately 2500 to 3500 new cases of ALL are diagnosed in children. The peak incidence occurs between 2 and 5 years of age. Also, it occurs more commonly among boys than girls. The most common presenting symptoms of ALL are nonspecific, which may include fever, bleeding, bone pain, and lymphadenopathy. Unexplained persistence of any of these common signs or symptoms should prompt consideration of malignancy as a possible cause. Most children with ALL have anemia, thrombocytopenia, and lymphoblasts on peripheral smear. Approximately 50 percent of children have WBC counts < 10,000/µL, and 20 percent have an initial leukocyte count > 50,000/µL. Approximately one-half of children with ALL present with bleeding, and three-quarters have a platelet count < 100,000/µL at the time of diagnosis.

A 52-year-old man presents with 2 weeks of worsening fatigue and generalized weakness. Over the past week, his gums have been bleeding when he brushes his teeth. Laboratory studies show normocytic, normochromic anemia with Auer rods on peripheral smear. The platelet count is 63 x 109/L. What is the most likely diagnosis? Acute lymphocytic leukemia Acute myeloid leukemia Chronic lymphocytic leukemia Chronic myeloid leukemia

Acute myeloid leukemia Acute myeloid leukemia (AML) is the most common leukemia in adults and accounts for 15-20% of childhood leukemias. The median age of diagnosis in adults is approximately 65 years, while the incidence peaks in the first 2 years of life in children. It is characterized by a clonal proliferation of myeloid precursors cells resulting in a variable reduction in the production of other cells, including red blood cells, platelets, and mature granulocytes. Patients present with symptoms related to pancytopenia, including pallor, fatigue, easy bruising, gingival bleeding, petechiae, and infection. Laboratory studies will show normocytic normochromic anemia with Auer rods (pink rod-like granular structures in the cytoplasm that are pathognomonic for myeloblasts) on peripheral smear. The majority of patients will have a platelet count < 100 x 109/L, while the leukocyte count may be low, normal, or elevated. Diagnosis is confirmed by bone marrow biopsy.

An 85-year-old man presents to the emergency department with altered mental status. Which finding would suggest a diagnosis of delirium rather than dementia? Acute onset Depressed mood Memory impairment Pupillary dilation

Acute onset Delirium is a common mental disorder seen in patients with a medical illness, especially older adults. Delirium is characterized by an acute onset (usually hours to days) disturbance in attention and cognition that is caused by a medical condition, substance withdrawal or intoxication, or a side effect from medication. The changes in mental status cannot be explained by a preexisting condition and are not related to a reduction in level of arousal. Patients with delirium may also exhibit psychomotor behavioral and emotional disturbances. The differential diagnosis for delirium is extensive, and diagnostic evaluation involves both identifying that delirium is present as well as determining the underlying medical cause. Obtaining a history and performing a physical exam on a patient with delirium is challenging. History should be obtained from family members or friends and physical exam should include a focused assessment with review of vital signs, skin condition, level of hydration and clues to an infectious etiology. Treatment of delirium involves managing the behavior disturbance along with finding and treating the underlying medical cause.

A urine microscopy of a patient with an elevated creatinine reveals muddy brown cellular casts. What is the most likely diagnosis? Acute interstitial nephritis Acute tubular necrosis Nephrolithiasis Pyelonephritis

Acute tubular necrosis Acute tubular necrosis (ATN) is responsible for many cases of acute kidney injury in the hospital setting. ATN occurs when there is decreased blood flow to the kidney usually secondary to a drop in blood pressure or secondary to nephrotoxins. The lack of perfusion causes ischemia to the renal tubules, and therefore necrosis. The most significant finding on urine microscopy is muddy brown cellular casts, but additionally, there may be fine granular casts as well as renal tubular epithelial cells. Other urinary labs will reveal a BUN to creatinine ratio of less than 20 as well as a fractional excretion of sodium (FeNa) greater than 2%, both indicative of renal injury. ATN can also be caused by medications, most commonly NSAIDs, ACE inhibitors, and angiotensin-receptor blocks (ARBs), which also act to decrease renal blood flow. Acute interstitial nephritis (AIN) (A) is the second most common diagnosis of AKI following ATN. AIN is an acute inflammation of the interstitial space of the kidney most commonly caused by drugs such as antibiotics, antivirals, NSAIDs, PPI's, and diuretics. Urinalysis will reveal hematuria and mild proteinuria. Treatment entails finding and discontinuing the insulting agent, and in some cases, short periods of steroid treatment. Nephrolithiasis (C), or kidney stones, can cause kidney injury by obstructing urine outflow. Patients with this condition are likely to have flank pain, dysuria, and hematuria. Pyelonephritis (D) is an infection of the kidney, which presents with white blood cell casts. This condition can be distinguished from a urinary tract infection by the presence of white cell casts, which are not usually present in an isolated UTI.

A 65-year-old woman with a history of mixed hyperlipidemia presents to your office with her daughter for concerns of memory loss and changes in mood. The daughter explains that for the past 5 years she has noticed that her mother has had progressively worsening memory impairment. At first, the patient mainly forgot recent events and had a hard time with recall. She then began to notice her mother having a hard time completing simple tasks at home. In the past 6 months, she states her mother has been very irritable and gets agitated very easily. On exam, the patient is calm with reassuring vital signs. HR is 80 bpm, RR is 18/min, BP is 120/82 mm Hg, and oxygen saturation is 98% SpO2 room air. She is able to answer your questions and recognizes that she sometimes has a hard time remembering certain words when talking, but she does not feel she has any significant loss of memory. You perform a Mini-Mental State Exam, and the patient is unable to recall three objects and cannot draw a clock correctly. What is the most likely diagnosis of the patient? Alzheimer disease Dementia with Lewy bodies Frontotemporal dementia Parkinson disease dementia Primary progressive aphasia

Alzheimer disease Dementia is defined as a decline in various areas of mental function that causes difficulty in performing activities of daily living. Alzheimer disease is the most common cause of dementia. It is a neurodegenerative disorder that most commonly affects people 60 years old or older. When symptoms occur prior to this age, it is considered early onset. The cause of Alzheimer disease is not known but may result from genetic or acquired risk factors. Acquired risk factors include hypertension, dyslipidemia, cerebrovascular disease, obesity, type 2 diabetes, obesity, and being less physically active. The most common early symptom of Alzheimer disease is loss of memory or memory impairment. Memory, which allows recall of events, is usually the first change, with vocabulary recall and motor learning being affected later in the disease. People with Alzheimer disease will have variable impairment in executive functioning in the early stages, but they commonly have poor insight into their disease progression. Insight from a friend or family member is very important, and they are often the ones to have concerns. As the disease progresses, the patient can develop neuropsychiatric symptoms such as irritability, apathy, aggression, and psychosis, especially late in the disease. If suspected, a mental exam should be performed, commonly the Mini-Mental State Exam or Montreal Cognitive Assessment, to determine if disease is present and at what stage. When deficits are present, such as loss of recall or inability to draw a clock, dementia should be considered. Brain imaging, including an MRI, is often performed to rule out other causes and confirm the diagnosis. Generalized and focal atrophy will be seen as well as a reduced hippocampal volume. Laboratory testing is widely known to not be helpful in the diagnosis or treatment, unless looking for other causes. Treatment includes use of cholinesterase inhibitors, such as donepezil, rivastigmine, and galantamine. They have been shown to have some symptomatic relief, but not all patients benefit from them and they do not appear to alter the course of the disease. Memantine, a N-methyl-D-aspartate (NMDA) receptor antagonist, has been found to have modest benefits in patients with moderate to severe Alzheimer disease and is recommended to be used in addition to a cholinesterase inhibitor.

A 23-year-old man with a history of type 1 diabetes presents to the emergency department with abdominal pain, vomiting, and fatigue. He has not been taking his insulin. He is found to have diabetic ketoacidosis. His vital signs and mental status are normal except for RR of 26/min and HR of 112 bpm. He has a normal electrocardiogram. Treatment is initiated with intravenous normal saline and insulin. After 1 hour of insulin therapy, his pH is 7.21, glucose is 190 mg/dL, anion gap is 22 mEq/L, bicarbonate is 13 mmol/L, and potassium is 5.5 mEq/L. Which of the following is the most appropriate next step? Add dextrose to the intravenous fluids Add sodium bicarbonate to the intravenous fluids Stop the intravenous insulin Treat the hyperkalemia

Add dextrose to the intravenous fluids Diabetic ketoacidosis is an acute, life-threatening complication of diabetes. It is most common in type 1 diabetics due to their lack of intrinsic insulin production. Without available insulin, cellular metabolism becomes glucose starved, resulting in protein breakdown and lipolysis. This metabolic cascade ultimately leads to the release of counterregulatory hormones, ketonemia, and acidosis. The result is often hypovolemia, hyperglycemia, acidemia, and hyperkalemia (but with a total body potassium deficit). Treatment of diabetic ketoacidosis involves fluid repletion, insulin administration, and treatment of electrolyte derangements. Since ongoing insulin therapy is needed to restore normal cellular metabolism, when the blood glucose is < 200 mg/dL, treatment involves adding dextrose to the intravenous fluids. This prevents hypoglycemia while still allowing the administration of insulin for the treatment of acidosis and ketonemia.

A 68-year-old man presents to the clinic reporting depressed mood every day of the week, for the majority of each day, for more than 3 months. He has a loss of interest in activities that used to give him pleasure and has feelings of worthlessness but reports no suicidal thoughts or intentions. His medical history includes Parkinson disease and urinary incontinence, for which he receives levodopa-carbidopa daily and mirabegron daily. He reports fair control of muscle rigidity and tremor and urinary symptoms on his current drug regimen. Which of the following is the best clinical intervention for this patient? Add amantadine Add quetiapine Add sertraline Discontinue levodopa-carbidopa Switch from levodopa-carbidopa to selegiline

Add sertraline Parkinson disease is a neurodegenerative disorder caused by decreased dopamine in the substantia nigra of the basal ganglia, and its hallmark characteristics include resting tremor, bradykinesia, muscular rigidity, and gait disturbance or imbalance. The tremor of Parkinson disease is most pronounced at rest and usually begins unilaterally. Bradykinesia refers to a generalized slowing of voluntary movement, causing the patient to have difficulty initiating tasks. Muscular rigidity is noted on physical exam when the examiner puts patients through passive range of motion exercises. Cogwheel rigidity is a stop and start movement that may be noted in some patients with Parkinson disease, while others demonstrate lead pipe rigidity, in which the limb is resistant to passive movement throughout the entire examination. Gait disturbances in Parkinson disease include shuffling gait, freezing, imbalance, and festination (an irresistible urge to take fast, small steps). The diagnosis of Parkinson disease is clinical, with gradual progression, postural instability, asymmetric motor symptoms, and bradykinesia being the hallmarks of diagnosis. Motor response to levodopa is also a clinical indicator of a correct Parkinson diagnosis. First-line therapy for motor symptoms of Parkinson disease is levodopa-carbidopa. The side-effect profile is low, and the drug is very effective at managing tremor, bradykinesia, and rigidity. However, levodopa-carbidopa administration does cause dyskinesias, including choreiform movement, over time. In fact, 100% of patients taking levodopa-carbidopa for 20 years will experience dyskinesias. For this reason, levodopa-carbidopa is best initiated when the patient is over 65 or has significant motor symptoms. Patients with Parkinson disease also have nonmotor symptoms, such as depression, constipation, somnolence, dementia, drooling, erectile dysfunction, insomnia, orthostasis, psychosis, rapid eye movement sleep disorder, and urinary incontinence. The patient in the above vignette is experiencing depression. First-line therapy for patients with Parkinson disease and concomitant depression is with selective serotonin reuptake inhibitors, such as sertraline. Other classes of antidepressants that may also be beneficial include tricyclic antidepressants and serotonin and norepinephrine reuptake inhibitors.

You are caring for a 60 kg patient who has been seizing for 30 minutes. You have already administered 4 mg of IV lorazepam and 1,200 mg of phenytoin without termination of seizure activity. Which of the following should most likely be your next step? Administer another bolus of IV phenytoin Administer IV fosphenytoin Administer IV pentobarbital Administer IV sodium bicarbonate

Administer IV pentobarbital Seizing patients become apneic and may vomit, compromising their airway and putting them at significant risk for hypoxia and aspiration. Failure to respond to optimal benzodiazepine and phenytoin loading functionally defines refractory status epilepticus. At this point, a third-line agent such as pentobarbital (15 mg/kg IV) or phenobarbital should be administered. Use caution when adding barbiturates to benzodiazepines because their coadministration may potentiate respiratory depression. Propofol is an alternative to pentobarbital and has the advantages of a short half-life and rapid clearance. These agents are given by IV drip and titrated to a burst-suppression pattern in the EEG tracing. Consider also other causes of benzodiazepine-resistant seizures such as isoniazid toxicity (treated with pyridoxine) and hyponatremia.

A 33-year-man presents for a skunk bite that occurred while was trying to free a skunk trapped in his basement, who subsequently ran free after biting him. The wound has already been cleaned with soap and water and is no longer bleeding. He says he received the human rabies immune globulin and human diploid cell rabies vaccine eight years ago after a raccoon bite. Which of the following treatments does this patient require today? Administer human diploid cell rabies vaccine only Administer human rabies immune globulin and human diploid cell rabies vaccine Administer human rabies immune globulin only No immunization is required

Administer human diploid cell rabies vaccine only Administering the human diploid cell rabies vaccine alone is still necessary in this patient who has experienced an animal bite from an animal for whom capture and assessment is unlikely to be possible. He has received the human rabies immune globulin and would not need a second dose. Prophylaxis for rabies, or rhabdovirus, encephalitis should be considered in any patient who presents with an animal bite in whom rabies status cannot be confirmed or patients with exposure to bats in which a bite goes unrecognized. This virus enters humans by saliva from infected animals or even other humans. These vaccinations are typically well-tolerated with allergic reaction occurring rarely. The most common side effects include local pruritus and tenderness; some people also experience systemic effects such as headache, fever, or nausea. The human rabies immune globulin should be given, as much as possible, at the locale of the bite. Injection in fingers and toe spaces is permissible without a large risk of compartment syndrome. The remaining immune globulin can be given in another intramuscular site. The human diploid cell rabies vaccine should be administered in a separate syringe and given intramuscularly as far from the site of immune globulin injection as possible. While the immune globulin is only necessary once, the rabies vaccine should be given on days 0, 3, 7, and 14 for optimal prevention of rabies.

A 32-year-old woman with a history of depression and obesity returns for a follow-up to discuss the management of her symptoms. She reports no improvement with aerobic exercises. She still has "pain all over" and is fatigued, although she has difficulty falling asleep most nights. Symptoms began insidiously at least 3 months ago. She has also begun seeing a counselor, as the constant pain and fatigue are triggering her depression. Vital signs are within normal limits. On exam, she is seated comfortably but is noted to be tender to palpation over her hips, back, neck, and lower extremities. A CBC, erythrocyte sedimentation rate, C-reactive protein, creatine kinase, and thyroid studies are all within normal limits. What is the most appropriate next step in the management of her condition? Amitriptyline Hydrocodone Naproxen Prednisone Sertraline

Amitriptyline Fibromyalgia is a complex illness of varying presentation. While thought of as a pain syndrome, patients may experience neuropsychological symptoms as well. They may report fatigue, minor memory problems, sleep disturbance, anxiety, and depression. Fibromyalgia is a diagnosis of exclusion. Laboratory studies and imaging should be undertaken on a case-by-case basis to rule out other conditions. Studies such as a CBC, erythrocyte sedimentation rate, and C-reactive protein are usually performed and should be normal. For patients with symptoms suggesting a thyroid disorder or inflammatory muscle condition, thyroid-stimulating hormone and creatine kinase are checked, respectively. Treatment is multifactorial and focused on treating the central nervous system dysfunction. Treating coexisting conditions that may be triggering fibromyalgia is also important. This patient has symptoms of depression as well, which should be treated with a combination of antidepressant medication and counseling. Patients should be counseled on the disease and treatment expectations, and the focus should be on regaining function and quality of life as opposed to solely treating pain. Improving sleep quality, weight reduction in patients who are overweight, and treatment of coexisting and exacerbating conditions should be discussed. Aerobic exercise is recommended for all patients as an initial treatment modality. Amitriptyline, a tricyclic medication, is a good choice in this patient because it is a sedating antidepressant. Taken at bedtime, this medication may help her to fall asleep faster while also treating her depression. When possible, tailoring medications to treat more than one symptom is preferred to avoid polypharmacy in addition to prescribing the lowest dose possible. Counseling should be suggested for patients with fibromyalgia with referrals given.

A 60-year-old man with a history of intravenous drug use presents to the emergency department with complaints of inability to open his mouth and difficulty swallowing. A review of his medical records shows that he is not up-to-date on his immunizations. On physical examination, stimulation of his posterior pharynx causes him to bite down instead of gag. You also note areas of inflammation, erythema and purulent discharge in bilateral antecubital fossae. Which of the following is the most appropriate management? Admission to the intensive care unit Consult with otolaryngology Tracheostomy Wound debridement

Admission to the intensive care unit Tetanus is a nervous system disorder that is now rarely seen in the developed world due to the increase in tetanus immunizations. The characteristic muscle spasms are caused by the toxin-producing anaerobe Clostridium tetani, which is found in soil. Risk factors for tetanus include individuals who are not immunized or insufficiently immunized, neonates, and individuals with penetrating injuries. Intravenous heroin use is also a risk factor due to agents used to dilute the drug supporting the growth of Clostridium tetani. Muscle rigidity starting in the jaw and facial muscles and descending to the limbs is the major clinical manifestation. The diagnosis of tetanus is a clinical one. Initial management is with admission to the intensive care unit. Treatment goals include halting toxin production through wound management, antimicrobial therapy, and neutralization of unbound toxin. Supportive therapy is also paramount. Patients are at risk for reflex spasms, so unnecessary procedures should be avoided and a dark, quiet environment should be maintained. Immunization with the full tetanus series is required for all patients who contract tetanus immediately upon diagnosis.

A 72-year-old man with a medical history of diabetes mellitus on insulin and hypertension on lisinopril and chlorthalidone presents to the emergency department with substernal aching chest pain that radiates to his right shoulder for the past 6 hours. The episode started when he was doing yard work. He reports having associated diaphoresis and nausea. The patient states that he has not previously had similar episodes of chest pain. His vital signs today include a heart rate of 95 bpm, blood pressure of 142/92 mm Hg, respiratory rate of 20 breaths per minute, pulse oxygenation of 99% on room air, and temperature of 98.6°F. Physical examination reveals a man in no apparent distress with a regular rate and rhythm and lungs that are clear to auscultation bilaterally. He has no lower extremity calf tenderness, edema, or erythema. The patient's initial ECG is shown above. The patient's initial troponin and 2-hour troponin are 0.1 ng per mL. The patient's chest radiograph shows no acute cardiopulmonary process. When you reassess the patient in 6 hours, his chest pain has resolved with medical management, his repeat ECG is unchanged, and his 6-hour troponin is normal. Which of the following is the most appropriate next step for this patient? Admit to hospital for cardiac catheterization Admit to observation for noninvasive cardiac testing to provoke ischemia CT angiogram chest to rule out pulmonary embolism and aortic pathology Discharge with aspirin, metoprolol, and atorvastatin without further testing Initiate thrombolytic therapy

Admit to observation for noninvasive cardiac testing to provoke ischemia The patient described in this vignette has chest pain that is concerning in quality for acute coronary syndrome, which includes unstable angina, non-ST elevation myocardial infarction, and ST elevation myocardial infarction. Unstable angina should be suspected in patients who have a new increase in frequency, intensity, and duration of angina-like chest pain without elevation in cardiac biomarkers. Chest pain due to ischemia (angina) is typically described as being aching or pressure-like in quality, substernal in location, and often radiating to the upper extremities. It is characteristically worse with exertion and improved with rest. Patients with angina often have associated nausea, vomiting, and diaphoresis. However, the chest pain associated with angina may be atypical, and patients can also present without chest pain with dyspnea, dizziness, weakness, epigastric pain, or fatigue. Patients with unstable angina may or may not have ECG changes associated with angina. The ECG changes, when present, are similar to the changes seen in patients who have non-ST elevation myocardial infarction and include new T wave inversion and ST segment depression. The patient in the vignette does not meet the criteria for ST elevation myocardial infarction or non-ST elevation myocardial infarction because he has two negative troponins. Patients with unstable angina are managed similarly to patients who have non-ST elevation myocardial infarction. Nitroglycerin is used for chest pain, and morphine can also be used for persistent pain. Patients with unstable angina should be initiated on intensive medical therapy, which includes a beta-blocker, statin, aspirin, an additional antiplatelet agent (clopidogrel, prasugrel, or ticagrelor), and anticoagulation (unfractionated heparin, enoxaparin, bivalirudin, or fondaparinux). Unstable angina is becoming an increasingly rare diagnosis due to the increased use of high-sensitivity troponins. The new high-sensitivity troponin cardiac biomarkers are able to detect troponin at much lower levels, and thus many of the patients who were previously diagnosed with unstable angina are now having elevated high-sensitivity troponin levels and are being diagnosed with non-ST elevation myocardial infarction. The patient in this vignette should be risk stratified according to the HEART (history, ECG, age, risk factors, and troponin) score. The HEART score produces a score between 1 and 10 based on the patient's age, cardiac risk factors, history, ECG changes, and cardiac biomarkers. Patients who have a score of ≤ 3 are considered to be low risk and appropriate for outpatient follow-up. Patients with a score of between 4 and 6 are considered to be intermediate risk, and patients with a score of 7-10 are considered to be high risk. Patients with intermediate or high risk for acute coronary syndrome without an elevation in cardiac biomarkers should undergo noninvasive testing to provoke ischemia. Patients are often admitted to an observation unit for this testing to occur, but it can be obtained outpatient in patients who have close follow-up and can undergo this testing within 72 hours. Noninvasive testing can include an exercise stress test, pharmacologic stress testing combined with imaging (myocardial perfusion imaging or echocardiography), or CT coronary angiography. Patients with abnormal noninvasive testing require hospital admission, cardiology consultation, and often further testing and treatment with coronary angiography and cardiac catheterization.

A 75-year-old man with a medical history of diabetes mellitus and coronary artery disease presents to the emergency department with a fever and cough for 2 days. His medication list includes metformin, aspirin, lisinopril, and metoprolol. He has not had any recent hospitalizations. Vital signs are notable for a heart rate of 108 bpm, blood pressure of 128/82 mm Hg, respiratory rate of 30 breaths per minute, pulse oxygenation of 94% on room air, and temperature of 101.6°F. Physical exam reveals a tachycardic rate with a normal rhythm, tachypnea, and crackles and decreased breath sounds in the right lower lung field. Laboratory findings include a white blood cell count of 18,000 cells per microliter, blood urea nitrogen of 15 mg/dL, and lactic acid of 1.8 mmol/L. The patient's chest X-ray is pictured above. Which of the following is the recommended treatment? Admit to the hospital and start intravenous ceftriaxone and azithromycin Admit to the hospital and start intravenous ciprofloxacin, piperacillin-tazobactam, and vancomycin Admit to the intensive care unit and start intravenous ciprofloxacin, piperacillin-tazobactam, and vancomycin Discharge with amoxicillin and doxycycline Discharge with azithromycin

Admit to the hospital and start intravenous ceftriaxone and azithromycin Pneumonia refers to an infection of the pulmonary parenchyma. Pneumonia is often classified as community-acquired or hospital-acquired. Hospital-acquired pneumonia is pneumonia that occurs 48 hours or more after admission and did not appear to be incubating at the time of admission. Community-acquired pneumonia can be caused by typical or atypical organisms. Streptococcus pneumoniae is the most common typical organism, and other typical organisms include Moraxella catarrhalis and Haemophilus influenzae. Atypical organisms include Chlamydia pneumoniae and Mycoplasma pneumoniae. The symptoms of bacterial pneumonia include cough and shortness of breath. Vital sign changes often include fever, tachycardia, and tachypnea. Physical exam findings may include decreased breath sounds or crackles on auscultation and dullness with percussion in the area of the consolidation. The diagnosis of pneumonia is confirmed when chest imaging (typically a chest radiograph) shows an infiltrate in a patient who clinically has findings consistent with pneumonia. The appropriate treatment setting for patients who have bacterial pneumonia varies based on the severity of symptoms and the patient's comorbidities. CURB-65 is a clinical prediction tool that can help predict adverse medical outcomes and guide clinicians in the appropriate treatment setting for patients who have community-acquired pneumonia. The variables included in the CURB-65 score are confusion (new disorientation to person, place, or time), urea (blood urea nitrogen > 20 mg/dL), respiratory rate ≥ 30 breaths per minute, blood pressure (systolic < 90 mm Hg or diastolic ≤ 60 mm Hg), and age ≥ 65 years. Each of these five criteria has a score of 1 point. Patients with a score of 0 can be treated outpatient. Patients who have a score of 1 or 2 should be hospitalized to the general medical ward unless the score is entirely based on an age ≥ 65 years. Patients who are ≥ 65 years who do not have comorbidities and are not toxic may be able to be treated outpatient. Patients who have a score of 3 or higher should be considered for intensive care unit admission. The CURB-65 clinical support tool should always be used in conjunction with patient-specific factors, such as ability to comply with oral antibiotics if discharged and outpatient follow-up. The patient in the vignette has a CURB-65 score of 2 because of his age and tachypnea. Patients who are admitted to the general medical ward of the hospital with community-acquired pneumonia and who do not have risk factors for Pseudomonas, methicillin-resistant Staphylococcus aureus, or other drug-resistant organisms are treated with ceftriaxone and azithromycin. Doxycycline is an appropriate alternative to azithromycin. Patients who have received intravenous antibiotics in the past 3 months or who have prior Pseudomonas aeruginosa or methicillin-resistant Staphylococcus aureus infections should have coverage added for these organisms.

An 80-year-old man with a history of hypertension, hyperlipidemia, benign prostatic hyperplasia, alcohol use, and tobacco use presents to the clinic. He reports fatigue and an unintended weight loss of 20 lbs over the last 2 months. His current medications include losartan, atorvastatin, and tamsulosin. Vital signs include a temperature of 37°C, blood pressure of 115/80 mm Hg, heart rate of 80 bpm, respiratory rate of 14/min, and SpO2 of 97% on room air. Physical exam reveals a man with obesity in no acute distress. Digital rectal exam reveals an enlarged, nodular prostate without induration. The remainder of his physical exam is unremarkable. Prostate-specific antigen is 15 ng/dL. A biopsy of the prostate reveals adenocarcinoma. Which of the following risk factors is most strongly related to the development of cancer in this patient? Advanced age Alcohol use Benign prostatic hyperplasia Cigarette smoking Obesity

Advanced age The patient presents with adenocarcinoma of the prostate, which is by far the most common form of prostate cancer. Prostate cancer is one of the most common cancers in men, affecting nearly one in seven men in the United States. There are many known risk factors for prostate cancer. The most significant risk factors are advanced age, African American ethnicity (due to many factors, including socioeconomic status, racial bias, and genetics), and a family history of prostate cancer. Diets high in animal fat and low in vegetables have also been found to play a role in the development of prostate cancer. Most patients are asymptomatic at the time of diagnosis due to high rates of screening and early detection. Uncommonly, patients can present with nonspecific urinary symptoms, hematuria, and hematospermia (blood in semen). If diagnosed in the metastatic stage, patients may present with bone pain (due to bone metastases), hematuria, incontinence, weight loss, and weakness or pain from spinal cord compression. Screening for prostate cancer should include prostate-specific antigen (PSA), which is often elevated. While not specific for malignancy, the likelihood of prostate cancer increases as the PSA becomes more elevated. If the PSA is elevated, further evaluation for prostate cancer is warranted. Patients should undergo a digital rectal exam (DRE), which may reveal nodules, induration, or asymmetry of the prostate. However, DRE is limited as the examiner can only palpate the posterior and lateral aspects of the prostate gland. If suspicion for prostate cancer is high despite benign DRE findings, a prostate biopsy should be obtained. This is most commonly performed with a transrectal ultrasound-guided approach, but an MRI-targeted biopsy can also be performed. Once a diagnosis of prostate cancer has been confirmed pathologically, the next step is risk stratification based on clinical and pathologic features, including the PSA level, anatomic extent of disease, and histologic grade. The treatment plan varies based on overall risk. For very low-risk and low-risk groups, active surveillance is often recommended. Patients with localized, intermediate-, and high-risk cancer are typically offered a combination of radical prostatectomy, radiation therapy (delivered by external beam source or brachytherapy), and androgen deprivation therapy. High-risk patients typically receive additional chemotherapy given the risk of metastatic disease development. Once treatment is complete, patients should undergo close follow-up with PSA level monitoring on a regular basis.

A 55-year-old man presents to the clinic for an annual follow-up of his well-controlled medical conditions. He is currently taking metformin, sitagliptin, lisinopril, aspirin, and atorvastatin. Vitals today include a HR of 87 bpm, RR of 17/min, BP of 139/81 mm Hg, T of 98.8°F, and SpO2 of 99% on room air. Which one of the following components of urine analysis is the best indicator of early-stage kidney disease in this patient? Albuminuria Creatinine Ketones Red blood cell concentration Waxy casts

Albuminuria Diabetic nephropathy is the most common cause of end-stage kidney disease in the United States. Patients with type 1 diabetes are much more likely to progress to end-stage kidney disease than patients with type 2 diabetes mellitus with risk factors, including male sex and those with a family history of the condition. This condition is also more prevalent in patients within the Black and American Indian populations. The prevalence in these populations is possibly related to access to health care rather than genetics, as there is a faster progression to end-stage kidney disease when a patient's diabetes and chronic kidney disease are not adequately managed. Diabetic nephropathy occurs about 10 years after a diabetes mellitus diagnosis, with initial indications including hyperfiltration with an increase in glomerular filtration rate (GFR) and progression to microalbuminuria (30-300 mg/day). As nephropathy worsens, albuminuria increases to > 300 mg/day and can be detected on urine dipstick as proteinuria. All patients with diabetes should receive yearly microalbuminuria testing to identify early diabetic nephropathy. Kidneys are often enlarged with chronic kidney disease progression, but biopsy is not often required. Patients with diabetes who are diagnosed with diabetic nephropathy should be treated with strict glycemic control and hypertension management. This involves a target BP of 140/90 mm Hg in patients with microalbuminuria and preserved GFR, while patients with overt proteinuria should be treated to a BP goal of 130/80 mm Hg. The rate of progression of end-stage kidney disease can be slowed with the use of angiotensin-converting enzyme inhibitors and angiotensin receptor blockers.

A 55-year-old female is awaiting adrenalectomy due to a pheochromocytoma. Prior to surgery, which of the following classes of medications is most likely to be used to control hypertension? Alpha-adrenergic blockers Angiotensin converting enzyme inhibitors Angiotensin receptor blockers Diuretics

Alpha-adrenergic blockers Alpha-adrenergic blockers are used prior to surgery in patients with a pheochromocytoma, to control hypertension from unopposed alpha stimulation when the tumor is manipulated. A pheochromocytoma will result in an increase in the production and release of catecholamines from the chromaffin cells of the adrenal medulla. This is a rare cause of secondary hypertension. Symptoms include episodes of hypertension, tachycardia, and diaphoresis with signs of sympathetic overstimulation. Other symptoms are tremor, dyspnea, palpitations, and general weakness. Laboratory testing reveals an increase in urinary and plasma metanephrines and catecholamines. A clonidine suppression test is positive. Location of the tumor is usually based by CT scan. Treatment involves using alpha blockers such as phenoxybenzamine. Beta blockers can be added following the administration of alpha blockers, but should not be used alone due to the possibility of worsening hypertension. Adrenalectomy is the definitive surgical treatment.

A 45-year-old man who is thin and has a medical history of hypertension, dyslipidemia, and a 25 pack-year smoking history presents to the clinic with reports of dyspnea. He states he becomes breathless after walking up a slight hill and reports an infrequent cough with clear sputum. Vital signs include HR of 85 bpm, RR of 18/minute, BP of 133/83 mm Hg, T of 98.3°F, and SpO2 of 96% on room air. Pulmonary function tests reveal FEV1/FVC of 0.53 and FEV1 of 2.60 L that increases to 2.65 L after inhaled albuterol administration. Chest radiograph findings are shown above. Which of the following is the predominant pathologic mechanism by which the suspected disease causes illness? Airway inflammation due to airway hyperresponsiveness Alveolar destruction due to inhibition of normal protease activity Bronchial constriction due to smooth muscle hypertrophy Microbial colonization of the bronchial tree due to poor mucociliary clearance Mucus hypersecretion due to gland hyperplasia

Alveolar destruction due to inhibition of normal protease activity Chronic obstructive pulmonary disease (COPD) is a chronic lung disease characterized by irreversible airway obstruction. The two subclassifications of this disease include chronic bronchitis and emphysema. Emphysema is characterized by abnormal permanent enlargement of the air spaces distal to the terminal bronchioles with destruction of the alveolar walls. This process occurs secondary to endogenous and exogenous oxidants that inhibit normal protease activity (**In the normal lungs, proteases maintain their homeostatic functions that regulate processes like its regeneration and repair**), which leads to loss of elastic tissue in alveolar walls and a decrease in the recoil tension necessary to support the structural integrity of the distal airways during the expiratory phase of respiration. This loss of distal airway support results in progressive hypoxia and dyspnea. Patients with emphysema may also exhibit a cough that produces scant, clear, mucoid sputum. The most important preventable cause of COPD is cigarette smoking, while other causes include environmental or occupational exposure to airborne toxins, dusts, and chemicals. Patients with emphysema are more commonly thin, with evident use of the muscles of respiration. In patients with advanced disease, chest auscultation reveals a quiet chest with decreased pulmonary and cardiac sounds, while breath and heart sounds are often normal in those with mild or moderate emphysema. COPD results in chronic hypercapnia due to the increased dead space in the lungs and the decreased ability of the patient to expel carbon dioxide. Patients with emphysema have flattened diaphragms with diminished vascular markings on chest radiograph. Patients with chronic bronchitis, on the other hand, are often overweight, with peripheral cyanosis and have normal hemidiaphragms and increased interstitial markings (dirty lungs on chest radiography). Pulmonary function test results in patients with COPD indicate obstructive patterns with reduced FEV1/FVC that is not reversible following inhaled beta-agonists (e.g., albuterol). The Global Initiative for Chronic Obstructive Lung Disease (GOLD) defines stages of the disorder according to the number of exacerbations and symptom severity, and treatment of COPD is guided by the stage of the condition. Smoking cessation is the most important intervention, which can be aided by behavioral counseling, nicotine replacement therapy, or pharmacologic regimens (bupropion, varenicline). First-line treatment for mild disease includes as-needed short-acting inhaled bronchodilators, such as ipratropium bromide and albuterol. Patients with moderate COPD should also be treated with a daily long-acting bronchodilator, such as salmeterol, formoterol, or tiotropium. If symptoms are not controlled well on monotherapy, patients in this category should be given an additional long-acting bronchodilator from the alternative class of agents (long-acting antimuscarinic agents or long-acting beta-agonists) rather than an inhaled glucocorticoid (fluticasone, mometasone). Severe disease should be treated with a long-acting antimuscarinic agent (tiotropium). An alternative treatment regimen for these patients includes combination of a long-acting beta-agonist with either an antimuscarinic agent or an inhaled glucocorticoid. Patients with the highest disease burden (category D or very severe disease) require regular treatment with a long-acting antimuscarinic agent and a long-acting beta-agonist. Patients with persistent symptoms can be prescribed triple therapy, with the addition of an inhaled glucocorticoid to the previous regimen. Theophylline has historically been used to control symptoms of COPD but is currently used only in patients who are refractory to other regimens. Pulmonary rehabilitation can improve quality of life, decrease dyspnea, and improve exercise capacity and is suggested for patients with category B, C, or D COPD. Lung transplantation is the definitive treatment for this condition with complications such as acute rejection, opportunistic infection, and obliterative bronchiolitis. Other surgical procedures include lung volume reduction surgery and bullectomy.

A 35-year-old woman is being managed for multiple episodes of chest pain that have been awakening her from sleep on several early mornings for the past 2 months. An electrocardiogram after one episode showed ST elevation. Coronary angiography did not reveal any stenotic lesions. She has no cardiac risk factors and is otherwise healthy. Which of the following daily medications is most likely to provide relief from her condition? Adenosine Amlodipine Aspirin Propranolol

Amlodipine Amlodipine is the most appropriate selection to use as daily prophylaxis against the anginal pain caused by Prinzmetal (variant) angina. Amlodipine, as well as other long-acting calcium channel blockers and nitrates, is effective at preventing the coronary vasospasm responsible for the chest pain seen in Prinzmetal's angina. This syndrome most commonly occurs in women under 50 years of age, with a classic presentation being angina in the early morning that awakens a patient from sleep. Of note, Prinzmetal's angina is not brought on by typical factors responsible for aggravating the angina of atherosclerosis, and patients may have no coronary risk factors. The work-up of any chest pain of suspected cardiac origin would include an electrocardiogram which, in this case, would show ST-segment elevation in the distribution of the vasoconstricted coronary artery. Coronary angiography will also be helpful to rule out a stenotic lesion in need of intervention. If Prinzmetal angina without a complicated stenotic lesion is diagnosed, patients should be counseled to avoid substances that contribute to vasoconstriction. Two important contributors are cigarette smoking and the use of cocaine. Treatment should consist of daily calcium channel blockers or nitrates to prevent vasoconstriction, and break-through nitrates as needed for acute episodes.

A 54-year-old woman presents to the office for her annual physical exam. Her only symptom is a mild headache for 4 or 5 days out of each week for the past 6 months. She reports a family history of high blood pressure and cerebrovascular accidents. She takes no medications and has no previous chronic diagnoses. Vital signs include blood pressure 190/100 mm Hg, pulse 82 beats per minute, temperature 98.6°F, and respirations 15 per minute. Her body mass index is 22 kg/m². Her complete metabolic panel and complete blood count are normal, as is her lipid panel and urinalysis. ECG and CXR are normal. Physical exam, including the fundoscopic exam, is within normal limits. Which of the following combination drug therapies would be the best choice for initial therapy for this patient? Amlodipine plus diltiazem Amlodipine plus lisinopril Captopril plus losartan Hydrochlorothiazide plus spironolactone Spironolactone plus captopril

Amlodipine plus lisinopril Hypertension is subdivided into three categories: elevated blood pressure, hypertension stage I, and hypertension stage II. Elevated blood pressure is a systolic pressure of 120-129 mm Hg, and stage I hypertension is a systolic pressure of 130-139 mm Hg or a diastolic pressure of 80-89 mm Hg. Stage II hypertension is a systolic pressure of 140 mm Hg or above or a diastolic pressure of 90 mm Hg or above. Severe hypertension is classified as a blood pressure over 180 mm Hg systolic or 110 mm Hg diastolic. Severe hypertension may be part of the clinical picture of a hypertensive crisis, where end-organ damage is acutely occurring, or may be asymptomatic. The patient in the above vignette has severe asymptomatic hypertension. She has no encephalopathy, proteinuria, creatinine changes, visual changes, cardiac changes, or neurologic changes. Diagnosis of hypertension in the office should be made after two or three individual office visits or after several weeks of home self-monitoring by the patient. However, in the case of severe hypertension, treatment can be initiated at the first visit. According to the International Society of Hypertension global hypertension guidelines, treatment of hypertension should begin with a calcium channel blocker, angiotensin-converting enzyme inhibitor, angiotensin II receptor blocker, or thiazide diuretics. Patients who present with a systolic blood pressure that is 20 mm Hg over goal or a diastolic pressure that is 10 mm Hg over goal may be given combination therapy as initial treatment. The combination therapy should include antihypertensives of two different classes, such as amlodipine (a calcium channel blocker) and lisinopril (an angiotensin-converting enzyme inhibitor). Combination regimens can lead to greater overall blood pressure reduction with fewer side effects than maximum doses of either medication alone.

A 12-year-old previously healthy girl presents to the clinic with acute onset of productive cough, fever to 102°F, and extreme fatigue over the past 3 days. She is awake and alert but appears tired. Her oxygen saturation is 96%, and respiratory rate is 15 breaths per minute. Lung auscultation reveals fine crackles in the left upper lobe. A chest radiograph reveals localized alveolar infiltrates with consolidation. Which of the following is the first-line treatment for this patient's most likely diagnosis based on her clinical presentation? Amoxicillin Azithromycin Cefotaxime Clindamycin

Amoxicillin This patient's constellation of symptoms are concerning for community-acquired pneumonia, and the onset of symptoms being acute and severe, the cough being productive, and the lung findings being focal are most suggestive of a typical bacterial etiology. Streptococcus pneumoniae is the most frequent cause of "typical" bacterial pneumonia in children of all ages. Amoxicillin is the recommended first-line treatment for outpatient pediatric pneumonia thought to be secondary to S. pneumoniae. Children with the more gradual onset of constitutional symptoms (e.g., fever, malaise, headache) accompanied by a nonproductive cough are more likely to have pneumonia secondary to atypical organisms, such as M. pneumoniae and C. pneumoniae. Patients with pneumonia secondary to atypical pathogens are also less likely to have focal ausculatory findings. A macrolide is the empirical treatment of choice for atypical bacterial pneumonia in otherwise healthy children > 5 years old being treated in outpatient setting. Macrolide antibiotics, such as azithromycin (B), are the treatment of choice for community-acquired pneumonia due to atypical pathogens. Among the macrolide antibiotics, clarithromycin and azithromycin have a more convenient dosing schedule and fewer side effects than erythromycin, but erythromycin is less expensive. Macrolide antibiotics may provide coverage for S. pneumoniae, which is the most frequent typical bacterial pathogen for all age groups. However, approximately 40 to 50 percent of S. pneumoniae isolates are resistant to macrolides. Failure to respond to macrolide therapy may indicate the development of a complication, a macrolide-resistant pathogen, or the need to alter therapy to provide better pneumococcal coverage. Cefotaxime (C) is an intravenous, third-generation cephalosporin used for inpatient treatment of community-acquired pneumonia for children with non-type-1 hypersensitivity reactions to penicillin. As the child is not hypoxemic, in respiratory distress, or toxic-appearing, she does not require inpatient treatment of her pneumonia at this time. A third-generation cephalosporin should also be used in the setting of confirmed beta-lactamase positive bacteria, such as Haemophilus influenzae or Moraxella catarrhalis or in the setting of complicated pneumonia without an identified pathogen. Clindamycin (D) may be used for treatment of typical bacterial pneumonias for children with type-1-hypersensitivity reactions to penicillin or for children with suspected aspiration pneumonia. Aspiration pneumonia typically involves the right middle or lower lobes and is more common in children with swallow dysfunction, such as those with static encephalopathy.

A 27-year-old G1P0 woman at 12 weeks gestation presents to the emergency department with the above physical exam finding after hiking through the woods in Wisconsin ("an erythematous blanching patch that may have central clearing and classically has a bull's-eye appearance"). What is the most appropriate therapy? Amoxicillin 500 milligrams orally three times daily Ceftriaxone 1 gram intravenously every 12 hours Doxycycline 100 milligrams orally two times daily Rifampin 600 milligrams orally once daily

Amoxicillin 500 milligrams orally three times daily This patient is exhibiting erythema migrans, a hallmark of Lyme disease. Lyme disease is the most common vector-borne disease in the United States. It is endemic to New England, the mid-Atlantic states, and the upper Midwest. It is caused by the spirochete Borrelia burgdorferi and transmitted by the Ixodes scapularis tick, more commonly known as the deer tick. The tick must be attached for more than 48 hours for transmission to occur. There are three stages of clinical Lyme disease. Early Lyme disease is characterized by erythema migrans, an erythematous blanching patch that may have central clearing and classically has a bull's-eye appearance. Hematogenous spread leads to diffuse erythema migrans, which spares the palms and soles. Acute disseminated Lyme disease occurs approximately 4 weeks after initial infection and can include meningoencephalitis, Bell palsy (which may be bilateral), or carditis, which often manifests with atrioventricular block. Late Lyme disease develops greater than 1 year after initial infection and includes chronic arthritis with or without chronic subtle encephalopathy. Only 50% of patients remember a tick bite; thus, diagnosis may be difficult. Erythema migrans is diagnostic, but not all patients present with this finding. Initial screening involves enzyme-linked immunosorbent assay (ELISA) testing with western blot and PCR to confirm the diagnosis. If the diagnosis is suspected, empiric treatment should be administered. Treatment for early Lyme disease and mild acute disseminated Lyme disease in pregnant patients is amoxicillin 500 mg PO three times daily.

A 40-year-old man with HIV presents with two weeks of progressive headache, malaise, and fever. On examination, he has mild nuchal rigidity, confusion, and a temperature of 38.2°C. Cerebrospinal fluid analysis shows a white blood cell count 360 cells/mL with a mononuclear predominance, glucose 28 mg/dL, and protein 220 mg/dL. What is the treatment of choice? Acyclovir Amphotericin B Ceftriaxone Vancomycin

Amphotericin B Cryptococcal meningoencephalitis is an opportunistic infection that occurs primarily in patients with advanced AIDS, although it can be seen in immunocompromised transplant patients as well. The majority of HIV-related cases occur when the CD4 count is < 100 cells/mm3. Patients present with progressive headache, nausea, malaise, and fever over the course of 1 - 2 weeks. Examination findings are typical for meningitis and include altered mental status, photophobia, and fever. Meningismus is less commonly seen. Presentations can be subtle and a high index of suspicion is needed for diagnosis. A CT scan of the brain is indicated if there are signs of increased intracranial pressure or focal neurologic deficits. Lumbar puncture should be performed with careful measurement of the opening pressure. Cerebrospinal fluid will show a mildly elevated white blood cell count with a mononuclear predominance, decreased glucose, and mildly elevated protein. However, in some cases the cerebrospinal fluid will only show minor, if any, abnormalities. Cryptococcal antigen testing of the CSF is nearly 100% sensitive and specific. India ink staining will show budding organisms. Treatment of choice is intravenous amphotericin B in addition to oral flucytosine for 14 days followed by an 8 week course of oral fluconazole.

An 80-year-old man presents to your clinic with complaints of malaise, myalgias, fever, cough, and abdominal pain for the past three weeks. One month ago, he vacationed in Ohio on a bird watching trip. He is febrile with mild tachycardia and pulse oximetry of 90%. A chest X-ray shows mediastinal lymphadenopathy and a focal infiltrate. Which of the following is the most appropriate therapy? Amphotericin B Azithromycin Fluconazole Zidovudine

Amphotericin B Histoplasmosis is an endemic mycosis that is generally asymptomatic, but may result in serious illness in patients with certain risk factors. In the United States, histoplasmosis is most commonly found in the midwestern states near the Ohio and Mississippi River valleys. Infected soil is found near areas where bats and birds live, such as chicken coops and caves. Birds cannot be infected with histoplasmosis and are not able to transmit the disease, but their excretions contaminate the soil, which provides the growth medium for the fungus. The majority of individuals with acute and subacute pulmonary histoplasmosis are asymptomatic. When symptoms occur, patients generally present with complaints of malaise, myalgias, fever, chills, cough and abdominal pain 3-14 days after exposure. Serious manifestations of histoplasmosis occur in immunocompromised individuals, such as those with human immunodeficiency virus (HIV) or the elderly. Treatment for more serious forms of the disease is with the antifungal medication, amphotericin B. Mild to moderate pulmonary disease can be treated with itraconazole.

Which of the following, based upon current literature, best defines a transient ischemic attack? A focal neurologic deficit lasting for less than 30 minutes, caused by reversible cerebral ischemia A focal neurologic sign or symptom lasting for at least 30 minutes, but less than 24 hours, caused by reversible central nervous system ischemia An episode of neurologic dysfunction caused by cerebral, spinal, or retinal ischemia, without acute tissue infarction An episode of neurologic dysfunction that resolves without residual functional deficit, but with evidence of acute tissue infarction on diffusion-weighted MRI studies

An episode of neurologic dysfunction caused by cerebral, spinal, or retinal ischemia, without acute tissue infarction Manifesting with a wide range of symptoms including speech, visual, sensory, motor, and vestibular deficits, recognition of the potential presenting symptoms of transient ischemic attacks and strokes is key to initiating and expediting workup and treatment of this spectrum of conditions. The importance of the temporal relationship between diagnosis, treatment, and outcomes is highlighted within the classic definitions of both ischemic strokes and transient ischemic attacks, with the World Health Organization formally characterizing an ischemic stroke as "a neurologic deficit of cerebrovascular cause that persists beyond 24 hours" over 40 years ago. This definition later evolved, with strokes characterized as "brain, spinal cord, or retinal cell death attributable to ischemia, based on neuropathological, neuroimaging, or clinical evidence of permanent injury." Dovetailing off of this, transient ischemic attacks had long been defined by the duration of their presenting neurologic symptoms, with resolution occurring before 24 hours. As the literature and research on the topic has grown, this definition no longer sufficiently characterizes transient ischemic attacks, as it has been shown that patients with less than one hour of neurologic deficits may have tissue infarction, and a large percentage of patients with classically diagnosed transient ischemic attacks based upon symptom duration is ultimately shown to have ischemic changes on diffusion-weighted imaging. Furthermore, reliance on duration of symptoms provides an impression that transient ischemic attacks are a benign entity without permanent injury or potential future sequelae when research has shown this is not the case, as even brief periods of ischemia have caused permanent tissue injury. With tissue infarction as the focal point of distinction between these conditions, the current definition of a transient ischemic attack is an episode of neurologic dysfunction caused by cerebral, spinal, or retinal ischemia, without acute tissue infarction.

What is anemia of chronic disease? What is the proposed pathophysiology?

Anemia of chronic disease refers to a low red blood cell, or RBC, count that may be associated with many chronic disease states like infections, malignancy, diabetes, or autoimmune disorders. The disease used to be called anemia of chronic inflammation because the underlying cause anemia is the continuous inflammation generated by chronic disease, which impairs iron metabolism and, in turn, RBC production. Patho: Inflammatory cytokines induce increased amounts of hepcidin by the liver. Hepcidin blocks ferroportin from releasing iron from the body stores. Inflammatory cytokines also decrease ferroportin expression and stops erythropoiesis by increasing bone marrow erythropoietin resistance. Apart from iron sequestration, white blood cells production is promoted by inflammatory cytokines. Bone marrow stem cells produce both red blood cells and white blood cells cells. Therefore, the upregulation of white blood cells causes fewer stem cells to differentiate into red blood cells. This may also have a role in inhibition of erythropoiesis, even when erythropoietin levels are normal, and aside from the effects of hepcidin.

A 66-year-old man with hypertension and type II diabetes mellitus presents to the clinic complaining of chest tightness, dyspnea, and dizziness while mowing his lawn earlier that day. He had similar symptoms two weeks ago. His symptoms resolved within a couple of minutes of sitting to rest. He is currently pain-free. Vital signs are pulse 88 beats per minute, blood pressure 134/88 mm Hg, temperature 98.8°F, oxygen saturation 97%, and 20 respirations per minute. Exam reveals an obese man in no acute distress and is otherwise unremarkable. Electrocardiogram is obtained and shown above ("J point or ST segment depression or normal"). Which of the following is the most likely diagnosis? Angina pectoris Pectoralis major strain Pulmonary embolism ST elevation myocardial infarction

Angina pectoris Angina pectoris is chest pain or discomfort attributed to myocardial ischemia, which occurs when myocardial oxygen demand exceeds oxygen supply. Oxygen demand is increased with physical exertion, mental stress, tachycardia, hypertension, and ventricular hypertrophy, all of which can lead to myocardial ischemia. Patients with angina classically present with chest pressure, tightness, or heaviness in the middle or left side of the chest that is preceded by exertion, lasts for minutes, and is relieved by rest. Some patients may also present with exertional shortness of breath, nausea, diaphoresis, dizziness, or fatigue. Patients usually have risk factors for coronary artery disease (family history, advancing age, hypertension, dyslipidemia, diabetes mellitus, chronic kidney disease, cigarette smoking). Electrocardiogram obtained when chest discomfort is present may reveal J point or ST segment depression indicative of subendocardial ischemia or the ECG may be normal. Further evaluation with stress testing and cardiac catheterization may be indicated to determine prognosis and to guide therapy.

A 17-year-old previously healthy boy presents with abdominal cramping, nausea, vomiting, and diarrhea 2 hours after eating at a picnic. Physical examination is unremarkable. What management is indicated? Anitiemetics and fluids Ciprofloxacin CT scan of the abdomen and pelvis Stool cultures

Anitiemetics and fluids This patient presents with a rapid onset gastroenteritis that is most consistent with Staphylococcal food poisoning resulting from ingestion of preformed enterotoxin. Staphylococcus-related food poisoning occurs when the bacteria multiplies in food prior to ingestion and creates enterotoxin. Protein rich foods (particularly ham, eggs, mayonnaise containing salads) support the bacteria's growth. Patients experience a sudden onset of nausea, vomiting, and diarrhea that begins 1 to 6 hours after ingestion of the contaminated food. The rapid onset is due to the fact that the toxin has already been formed prior to ingestion. Patients often recover quickly; usually between 6 and 8 hours of symptom onset and frequently do not present for medical treatment. Often, there will be outbreaks of Staphylococcal-related food poisoning from multiple people ingesting the same contaminated food. Treatment focuses on rehydration and treatment of nausea and vomiting if it is still present on presentation to the Emergency Department.

A 46-year-old woman presents with muscle weakness that has progressively worsened over the last several months. She has particularly noticed it when trying to stand up from a seated position, and she now has to use the armrests. Physical examination reveals weakness of the deltoids and hip flexors. Testing positive for which of the following autoantibodies is most consistent with the diagnosis? Anti-dsDNA Anti-Jo-1 Anti-SCL-70 Anti-SS-A

Anti-Jo-1 Anti-Jo-1 (and other autoantibodies recognizing the aminoacyl tRNA synthetases) are the most commonly positive myositis-specific autoantibodies. These occur in approximately 20 to 30 percent of patients with polymyositis. Individuals who test positive for these autoantibodies often have other manifestations in addition to the myopathy, including interstitial lung disease, nonerosive arthritis, fever, Raynaud phenomenon, and Gottron papules. In addition, antinuclear antibodies are found in more than half of patients with polymyositis. Polymyositis should be considered in patients who have progressive weakness of the proximal muscles over a period of months. Other lab abnormalities include elevated creatine kinase, aldolase, aspartate aminotransferase (AST), and alanine aminotransferase (ALT). Electromyography (EMG) will show an irritable myopathy, and muscle biopsy will reveal perivascular lymphocytic infiltrates.

What test is helpful in ruling out the diagnosis of systemic lupus erythematosus? Anti-Smith antibody Antinuclear antibody C-reactive protein Erythrocyte sedimentation rate

Antinuclear antibody The antinuclear antibodies (ANA) test is highly sensitive, as it is present in nearly all patients with systemic lupus erythematosus (SLE). SLE is a multisystem autoimmune disorder. The disease results in organ inflammation, dysfunction, and eventually destruction. Patients can present with nephritis, thrombosis, carditis, pneumonitis, pulmonary hypertension, cerebrovascular accident, or infection resulting from chronic immunomodulatory medications. The workup of patients with signs or symptoms concerning for SLE involves a number of serum tests, all of which are imperfect. However, nearly all patients with the disease will have antinuclear antibodies present on testing. The absence of this antibody virtually rules out the disease. Unfortunately, it is also present in up to 50% of patients without SLE, so a positive test does not clinch the diagnosis.

A 31-year-old woman is diagnosed with her sixth spontaneous abortion. Which of the following is the most likely underlying diagnosis? Antiphospholipid antibody syndrome Hemophilia A Thrombotic thrombocytopenic purpura Von Willebrand disease

Antiphospholipid antibody syndrome Antiphospholipid antibody syndrome is an autoimmune disorder that is a well-recognized cause of acquired hypercoagulability. Affected patients produce antibodies to a host of various proteins which predispose to thrombosis in a manner that is not well understood, but may involve interference with normal hemostasis pathways. The generally accepted "1 in 5 rule" states that 1 in 5 patients younger than age 45 with stroke, 1 in 5 patients with deep vein thromboses (DVT), and 1 in 5 patients with recurrent pregnancy loss will test positive for antiphospholipid antibodies. Antiphospholipid antibody syndrome should be suspected in patients with recurrent DVTs, recurrent spontaneous abortions, or recurrent cerebrovascular events, particularly in young people. Approximately 1% of patients with antiphospholipid antibody syndrome develop a rapidly progressive form known as catastrophic antiphospholipid antibody syndrome. This life-threatening condition involves widespread simultaneous small vessel occlusions in multiple organs. Despite treatment, the mortality of catastrophic antiphospholipid antibody syndrome is 50%.

A 9-year-old boy presents with complaints of low back pain for one week and dark urine. History reveals pharyngitis 2 weeks ago. The patient is afebrile with a BP of 124/88 mm Hg (elevated for his age) and a weight of 119 pounds. On exam, the location of the pain is at the right flank. Urinalysis is positive for leukocytes and moderate microscopic hematuria. Microscopic examination of urine reveals red blood cell casts. Which of the following laboratory tests should be serially measured to aid in confirmation of the diagnosis? Antistreptolysin O antibody Complete blood count Renal biopsy Urinalysis

Antistreptolysin O antibody Antistreptolysin O (ASO) antibody is evaluated in patients with suspected poststreptococcal glomerulonephritis. The initial test is ASO antibody titers, which can confirm recent invasive streptococcal infection and should be rechecked in 2 weeks for comparison. ASO antibody does not provide a definitive diagnosis, therefore, other tests including anti-hyaluronidase, anti-deoxyribonuclease B, and anti-streptokinase antibodies may be needed for confirmation. Poststreptococcal glomerulonephritis is an acute nephritic syndrome commonly seen in 2- to 14-year-olds, more often in males. Symptoms usually present 2 to 6 weeks post impetigo and 1 to 3 weeks after group A streptococcal pharyngitis. It is an immune-mediated disease involving putative streptococcal antigens, circulating immune complexes, and activation of complement in association with cell-mediated injury. Clinical findings include hematuria, pyuria, red blood cell casts, edema, hypertension, and oliguric symptoms. Symptoms may include a history of headache, malaise, anorexia, and flank pain in approximately 50% of patients due to enlargement of the renal capsule. Treatment is supportive, often with complete resolution in 3 to 6 weeks. Patients with edema and hypertension may benefit from furosemide and restriction of salt and water.

A 67-year-old man with a history of hypertension and remote abdominal aortic aneurysm repair presents to the emergency department after four episodes of gross hematemesis. He had an episode of emesis with some blood last week but dismissed it as a "stomach bug." He reports no history of heavy alcohol use or liver disease. His vital signs on arrival are T 38.5°C, HR 111 bpm, BP 100/80 mm Hg, RR 27/min. His stool is guaiac positive. Which of the following is the most likely cause this patient's symptoms? Aortoenteric fistula Esophageal varices Mallory-Weiss tear Peptic ulcer disease

Aortoenteric fistula An aortoenteric fistula is an abnormal communication between the aorta and the gastrointestinal tract. The most common risk factors for aortoenteric fistulas are abdominal aortic aneurysm and a history of aortic surgery. Gastrointestinal bleeding, including hematemesis, hematochezia, and melena, is often the presenting symptom. The classic triad of gastrointestinal bleeding, abdominal pain, and a palpable mass is rarely present and a history of aortic aneurysm is rarely known at the time of presentation. Fever and sepsis may be present due to seeding of the blood with gastrointestinal flora. Management of aortoenteric fistulas is emergent surgical repair.

A 62-year-old man with a history of ongoing tobacco abuse, hypertension, dyslipidemia and erectile dysfunction complains of progressive aching pain in his right buttock and hip. The pain is worse with walking and is relieved with rest. Physical exam of the lower extremities reveals slightly diminished femoral, popliteal, and dorsalis pedis pulses. Which of the following is the most likely site of this patient's peripheral arterial disease? Aortoiliac segment Common femoral artery Popliteal artery Superficial femoral artery

Aortoiliac segment This patient most likely has peripheral arterial disease in the right aortoiliac segment. Peripheral arterial disease is physiologically significant atherosclerosis of the aortic bifurcations or arteries of the lower limbs. It is strongly associated with smoking, diabetes mellitus, and aging and shares all the risk factors common to atherosclerosis. This patient presents with right hip and buttock claudication, diminished femoral pulses and erectile dysfunction. This presentation commonly represents atherosclerotic disease within the aortoiliac system and is sometimes referred to as Leriche syndrome. Classic claudication is characterized by leg pain that is consistently reproduced with exercise and relieved with rest. The degree of symptoms of claudication depends upon the severity of stenosis, the collateral circulation, and the vigor of exercise. Patients with claudication can present with buttock, hip, thigh, calf, or foot pain, alone or in combination. The usual relationships are between pain location and corresponding anatomic site of arterial occlusive disease.

A 25-year-old woman, whose last menses was one week ago, presents to the emergency department complaining of severe lower abdominal pain. She has a 4-hour history of right lower quadrant abdominal pain, anorexia, and nausea. She says the pain initially began around the bellybutton and then moved to the right lower quadrant. Her temperature is 38.3°C (101°F). Palpation of the left lower quadrant produces pain in the right lower quadrant. Pelvic examination shows right adnexal tenderness. Which of the following is the most likely diagnosis? Appendicitis Ectopic pregnancy Ovarian torsion Pelvic inflammatory disease

Appendicitis Right lower quadrant abdominal pain, anorexia, and nausea most likely suggests acute appendicitis. Appendicitis, an inflammation of the vestigial vermiform appendix, is one of the most common causes of the acute abdomen and one of the most frequent indications for an emergent abdominal surgical procedure worldwide. In women, right adnexal area tenderness may be present on pelvic examination. Rovsing's sign refers to pain in the right lower quadrant with palpation of the left lower quadrant, and is usually seen in patients with acute appendicitis.

A 45-year-old man with a history of paroxysmal atrial fibrillation presents to the ED with acute onset of severe pain and paresthesias in his right calf. On exam, you note lower extremity pallor and an absent dorsalis pedis pulse. Which of the following is the most likely diagnosis? Arterial atheroembolism Arterial thromboembolism Arterial thrombosis Arterial vasospasm

Arterial thromboembolism Arterial embolism can be divided into thromboembolic and atheroembolic causes. Most arterial thromboemboli originate in the left side of the heart and are frequently associated with a recent myocardial infarction, atrial fibrillation, or valvular abnormalities. Acute arterial thromboembolism results in the sudden loss of a previously present pulse. In general, patients with arterial thromboembolism have few physical findings suggestive of long-standing peripheral vascular disease and will have normal proximal and contralateral limb pulses. These patients typically do not have well-developed collateral circulation and are at high risk for limb ischemia.

A 45-year-old woman with a history of hypothyroidism on levothyroxine presents to the clinic for routine monitoring of her thyroid-stimulating hormone level. The patient reports she is feeling well and has been taking her levothyroxine as prescribed. Vital signs today include a heart rate of 105 bpm, blood pressure of 135/84 mm Hg, respiratory rate of 20/minute, oxygen saturation of 98% on room air, and temperature of 98.6°F. The patient's thyroid-stimulating hormone level is 0.03 mU/L. Which complication is this patient most at risk for on her current dose of levothyroxine? Acute kidney injury Atrial fibrillation Exophthalmos Goiter Hepatotoxicity

Atrial fibrillation Patients with hypothyroidism are treated with thyroid hormone replacement. Levothyroxine is the most frequently used preparation. Patients who intentionally or accidentally (with the prescribed dose being too high) take excessive amounts of thyroid hormone replacement are at risk for subclinical (low thyroid-stimulating hormone with normal free T4) or overt hyperthyroidism (low thyroid-stimulating hormone with elevated free T4). Two of the main risks of over-replacement hyperthyroidism are atrial fibrillation and decreased bone mineral density, resulting in osteoporosis and increased risk of fractures. The risk for atrial fibrillation or other cardiac dysrhythmias is particularly high in older adults with hyperthyroidism. Furthermore, the risk continues to increase the more the thyroid-stimulating hormone (TSH) concentration is suppressed. Patients with excessive amounts of thyroid hormone will have symptoms similar to other causes of hyperthyroidism, such as weight loss, heat intolerance, tremor, palpitations, anxiety, increased frequency of bowel movements, and shortness of breath. Patients with hypothyroidism due to under-replacement have similar symptoms to other causes of hypothyroidism, such as weight gain, cold intolerance, constipation, and menstrual irregularities. Exophthalmos and goiter do not occur due to excess thyroid hormone replacement in the absence of another underlying thyroid disease. The treatment of patients with hypothyroidism who have over-replacement of thyroid hormone is to reduce their dose. Patients who have overt symptoms of hyperthyroidism can be temporarily treated with a beta-blocker, such as metoprolol, to improve hyperadrenergic symptoms.

A 69-year-old man with atherosclerosis presents with calf pain during ambulation. He notices a blanching of the leg skin when the pain is at its worst. If he stops walking, the pain ceases within 5 minutes. He has a 120 pack-year history of tobacco use. Examination of the toes reveals prolonged capillary refill but normal sensation. A recent work-up revealed normal cardiac function. Which of the following abnormalities would most likely be found during examination of this patient's lower extremities? Atrophic, shiny skin Intrinsic foot muscle weakness Pitting edema Varicose veins

Atrophic, shiny skin Occlusive peripheral arterial disease results from atherosclerotic changes of vessels outside of the heart. It is heralded by intermittent claudication, a physical activity induced extremity pain that ceases with a few minutes of rest. Other classic symptoms include the 5 P's: pulselessness, pallor, pain, paresthesias and paralysis. Limb-threatening ischemia, suggested by paresthesias and paralysis, necessitate immediate evaluation and treatment. Prolonged capillary refill is also a common finding. Chronic poor blood supply causes the typical atrophic, shiny, dry skin changes seen with peripheral arterial disease. Evaluation includes pulse examination, abdominopelvic auscultation, Doppler assessment and ankle-brachial index testing. Diagnosis can be confirmed with Duplex ultrasonography, CT-angiography or MRI-angiography. Treatment begins with lifestyle modifications, namely tobacco cessation and regular exercise. Medications include lipid lowering statins, aspirin and other antiplatelet drugs like clopidogrel and cilostazol.

A 65-year-old woman presents to the emergency department with reports of severe pain and blurred vision occurring in the right eye that began 3 hours ago while watching a movie in the theater with her daughter. She also reports nausea and seeing halos around lights. Previous medical history is significant for COPD, well-controlled with tiotropium bromide daily, and seasonal allergies. Vital signs include a HR of 86 bpm, BP of 130/70 mm Hg, RR of 22/min, oxygen saturation of 98% on room air, and T of 98.6°F. Physical exam reveals a hazy cornea with a poorly reactive, mid-dilated pupil in the right eye. Which of the following medications should be avoided when treating this patient? Acetazolamide Atropine Latanoprost Pilocarpine Timolol

Atropine Acute angle-closure glaucoma is caused by narrowing of the anterior chamber angle in the eye resulting in a buildup of aqueous humor and in an increased intraocular pressure. Risk factors include female sex, age > 60 years, farsightedness, and positive family history of glaucoma. When the intraocular pressure builds up quickly, symptoms include decreased or blurry vision, headache, nausea, and severe eye pain. The blurred vision is often associated with halos around lights. On physical examination, a cloudy cornea with a red conjunctiva is often observed. The pupil of the affected eye will be moderately dilated and unresponsive to light. Intraocular pressure is usually > 30 mm Hg. Acute-angle closure glaucoma is a medical emergency and referral to an ophthalmologist should take place urgently. Medical treatment of acute angle-closure glaucoma includes agents that cause miosis, or pupillary constriction, thus allowing the blocked drainage angle to open and reducing the intraocular pressure. Topical eye drops such as timolol and apraclonidine that lower the intraocular pressure should be administered quickly after presentation. Systemic medications such as acetazolamide and mannitol can be given as part of a combination regimen with pressure lowering eye drops when a patient cannot be seen by an ophthalmologist within 1 hour. Peripheral iridotomy, either performed surgically or with a laser, is another option for treatment of acute-angle closure glaucoma. In this procedure, a small hole is created through the peripheral iris, which allows the aqueous humor to drain and thus lowers intraocular pressure. Atropine is an anticholinergic agent, which allows dilation of the pupil and would worsen acute angle-closure glaucoma.

A 5-year-old girl presents to the emergency department with a 2-day history of grossly bloody diarrhea, abdominal cramping, and fever. Other children from her daycare have similar symptoms. On physical exam, the child appears mildly dehydrated and has mild lower abdominal tenderness to palpation. A stool study reveals fecal leukocytes and stool culture grows nonmotile, facultatively anaerobic, gram-negative rods. After initial fluid resuscitation, which of the following is the most appropriate treatment? Azithromycin Ceftriaxone Ciprofloxacin Loperamide

Azithromycin Azithromycin is the most appropriate treatment for shigellosis in patients < 18 years of age. Shigellosis is a type of bacterial diarrhea. In the United States, most cases of shigellosis are caused by Shigella sonnei. Most cases in the United States are seen in daycare centers and in crowded living conditions. The majority of cases are transmitted through fecal-oral spread. Other modes of transmission include food-borne and sexual transmission. Shigella sp. have a relatively low infectious dose due to their ability to survive passage through the stomach's acidic environment. Upon reaching the colon, they invade the mucosal cells and induces an intense inflammatory response, which leads to death of epithelial and immune cells and creates colonic ulcerations and abscesses. The average incubation period is 3 days. Presenting symptoms may include abdominal pain, fever, mucoid or bloody diarrhea, and tenesmus. Stool frequency is usually 8-10 per day and significant fluid loss is not common. Diagnosis is based on small, volume bloody diarrhea, abdominal cramps, fever and with stool studies. Direct microscopy often reveals fecal leukocytes and red blood cells. Culture reveals non-motile, facultatively anaerobic, gram-negative rods that do not ferment lactose. Treatment should begin with oral or intravenous rehydration. For children with underlying immunodeficiency or who have toxemia or bacteremia, intravenous ceftriaxone is recommended. Azithromycin is the first-line oral treatment in children. Fluoroquinolone antibiotics are first-line treatment in adults. Complications of shigellosis include rectal prolapse, intestinal obstruction, colonic perforation, bacteremia, electrolyte imbalances, leukemoid reactions, and hemolytic-uremic syndrome.

A 25-year-old woman presents to your clinic with concerns about sexually transmitted infections. She states she has unprotected sex with multiple partners. She is asymptomatic, but her last partner told her that he recently tested positive for chlamydia. Her urine pregnancy test is positive. The most appropriate next step is administration of which of the following? Azithromycin Doxycycline Metronidazole Penicillin

Azithromycin Chlamydia trachomatis is the most common bacterial cause of sexually transmitted infections (STIs) in the United States. Expedited partner therapy was designed to increase the number of people treated for sexually transmitted infections and decrease the spread. All patients should be counseled about the risks of reinfection. Azithromycin may be given in a one-time oral dose of 1 gram in pregnant patients. Doxycycline is given to nonpregnant patients 100 mg twice per day for seven days and should be avoided in pregnant individuals. A single injection of 500 mg of ceftriaxone is also given for uncomplicated gonococcal infections if present. Co-infection with chlamydia and gonorrhea is common and empiric therapy for both should be given for symptomatic patients and those with recent known or uncertain sexual exposure to gonorrhea unless it has been definitively ruled out.

A 32-year old teacher is seen for a paroxysmal cough of 5 days' duration. He tells you that a student in his class was diagnosed with pertussis 3 weeks ago. Which one of the following is the best treatment? Amoxicillin Azithromycin Cephalexin Trimethoprim/sulfamethoxazole

Azithromycin Pertussis, also known as whooping cough, is an acute respiratory tract infection that has increased in incidence in recent years. The initial catarrhal stage presents with nonspecific symptoms of malaise, rhinorrhea, sneezing, lacrimation, and mild cough. During the paroxysmal stage, severe outbreaks of coughing often lead to the classic high-pitched whooping sound patients make when gasping for breath. The paroxysmal stage is followed by the convalescent stage and resolution of symptoms. Complications vary by age, with infants more likely to experience severe complications such as apnea, pneumonia, seizures, or death. In adolescents and adults, complications are the result of chronic cough. The diagnosis depends on clinical signs and laboratory testing. Both culture and polymerase chain reaction testing can be used to confirm the diagnosis. Although antibiotic treatment does not improve the clinical symptoms of pertussis, it can eradicate nasal bacterial carriage and may reduce transmission rates. Macrolide antibiotics such as azithromycin are first-line treatments to prevent transmission. Immunization against pertussis is essential for disease prevention. Current recommendations in the United States consist of administering five doses of the diphtheria and tetanus toxoids and acellular pertussis (DTaP) vaccine to children before 7 years of age, and administering a tetanus toxoid, reduced diphtheria toxoid, and acellular pertussis (Tdap) booster between 11 and 18 years of age.

A 32-year-old woman presents to family practice with weight loss and fatigue for the past few months. She has lost approximately 15 pounds in 3 months without trying. She also reports heart palpitations, excessive sweating, and oligomenorrhea. She reports no shortness of breath, illicit drug use, or night sweats. Her medical history includes anxiety, and she takes sertraline 50 mg daily. She is on no other medications. Her vitals include a BP of 128/82 mm Hg, HR of 101 bpm, RR of 12/min, T of 98.7°F, and SpO2 of 100% on room air. She has warm, moist skin, and examination of her thyroid reveals a palpable 1 cm nodule on the left lobe that is nontender. Lab values include a thyroid-stimulating hormone level of 0.2 mU/L, and radioactive iodine uptake scan shows a discrete area of increased uptake, corresponding to the left lobe nodule noted on exam. Which complication is most likely to develop if this patient is left untreated? Bone loss Coronary artery disease Loss of vision Megacolon Myxedema coma

Bone loss Hyperthyroidism occurs when thyroid hormone levels (T3 and T4) are elevated. Serum thyroid-stimulating hormone (TSH) levels are depressed in primary hyperthyroidism, and TSH receptor antibodies are elevated in certain situations, such as in patients with Graves disease. Fatigue, weight loss, menstrual changes, increased sweating, heat intolerance, and heart palpitations are all common symptoms of thyrotoxicosis. Physical examination findings in patients with hyperthyroidism include tachycardia, damp and warm skin, resting tremor, and hyperreflexia. Thyroid exam can vary, but an enlarged thyroid or presence of a thyroid nodule on an uptake scan can help differentiate the underlying cause and guide treatment. A discrete area of increased uptake indicates a hyperfunctioning thyroid nodule. Complications of hyperthyroidism include bone loss, hypercalcemia, cardiac dysrhythmias, and thyroid crisis. Thyroid hormone stimulates bone resorption and osteoclast activity. This results in a loss of cortical bone density and trabecular bone volume. The rate of bone turnover is increased, which can lead to elevated serum calcium and suppression of parathyroid hormone secretion. These changes lead to an increased risk of osteoporosis in these patients. Treatment for hyperthyroidism includes symptomatic control with beta-blockers, along with therapies to suppress thyroid hormone levels such as anti-thyroid medications, surgical intervention, and radioactive iodine to destroy thyroid tissue.

A 15-year-old boy is brought by his mother to the Emergency Department due to nausea, multiple episodes of vomiting previously ingested food, and three loose bowel movements. His mother notes that the boy reheated the leftover rice stored in the refrigerator the night before. Physical examination reveals a tired child who is afebrile with dry mucous membranes. He has pink palpebral conjunctivae, clear breath sounds, a soft abdomen with mild tenderness at the epigastric area, and full pulses. Which of the following is the most likely causative agent? Bacillus cereus Campylobacter jejuni Clostridium botulinum Vibrio cholerae

Bacillus cereus The boy presents with common manifestations of acute gastroenteritis, likely the result of a foodborne illness. Patients commonly present with diarrhea and vomiting but can have abdominal pain and fever. The classic setting of acute gastroenteritis caused by Bacillus cereus is intake of improperly refrigerated cooked or fried rice. Signs and symptoms appear within 24 hours of ingestion of the preformed enterotoxin. Laboratory tests are not routinely necessary, but if indicated, stool can be examined for culture and toxin identification. Management is supportive and includes oral rehydration salts, intravenous fluids, and antiemetics. Acute gastroenteritis caused by Campylobacter jejuni (B) occurs in the setting of ingestion of raw or undercooked poultry products, unpasteurized milk, or contaminated water. Signs and symptoms occur within 2 to 10 days from ingestion of contaminated food and include bloody diarrhea, abdominal cramps, fever, and vomiting. Management includes supportive treatment and antibiotics such as erythromycin and quinolones. Infection associated with Clostridium botulinum (C) occurs in the setting of ingestion of contaminated home-canned foods or improperly canned commercial foods. Signs and symptoms include vomiting, diarrhea, blurring of vision, dysphagia, and descending muscle weakness. Management includes supportive care with rehydration and administration of botulinum antitoxin early in the course of the illness. Acute gastroenteritis caused by Vibrio cholerae (D) occurs in the setting of ingestion of contaminated water, fish, and shellfish. Signs and symptoms include profuse watery diarrhea and vomiting which can lead to severe dehydration and death within hours. Management includes supportive care with aggressive oral and intravenous rehydration. Oral antibiotics are warranted in cases of confirmed cholera and include tetracycline or doxycycline for adults, and trimethoprim-sulfamethoxazole for children.

Which of the following is the most effective initial treatment for a benign esophageal stricture? Balloon dilation Balloon dilation and proton pump inhibitor Esophageal stent Nissen fundoplication

Balloon dilation and proton pump inhibitor Balloon dilation and proton pump inhibitor is the most effective initial treatment for a benign esophageal stricture. An esophageal stricture is the narrowing of the esophagus caused by the buildup of acid and is one of the complications of gastrointestinal reflux disease. Symptoms include heartburn, dysphagia, coughing, and shortness of breath. Treatment is aimed toward decreasing the symptoms of dysphagia and preventing the stricture from reoccurring. Therefore, the most effective initial treatment is to dilate the stricture using either a balloon dilator or a mechanical dilator followed by prescribing a proton pump inhibitor to prevent reoccurrence of acid buildup in the esophagus. Administering a proton pump inhibitor does not decrease the need to achieve adequate dilation with the balloon, but it has been proven to help reduce the need for subsequent dilations.

A 23-year-old woman comes to the emergency department complaining of a 3-week history of diarrhea, abdominal pain, fatigue, and weight loss. Her blood pressure is 100/65 mm Hg and her temperature is 37.1°C (98.7°F). She denies fever, chills, and night sweats. She has a 10-year history of type 1 diabetes mellitus that is well controlled with daily insulin. Physical examination of the abdomen reveals diffuse tenderness. Examination of the skin shows several areas of pruritic papules and vesicles occurring in groups on the elbows, forearms, and knees. Laboratory studies show anti-endomysial antibodies and anti-tTG antibodies. Which of the following is the most likely diagnosis? Celiac disease Crohn disease Diverticular disease Ulcerative colitis

Based on the constellation of findings, this patient most likely has Celiac disease (ie, gluten-sensitive enteropathy). Celiac disease is a disorder of the small bowel that is characterized by villous atrophy, symptoms of malabsorption (eg, steatorrhea, weight loss), and resolution of symptoms after withdrawal of gluten-containing foods from the diet. These patients typically present with diarrhea, weight loss, and have antibodies against gliadin. Additionally, anti-endomysial and anti-tTG antibodies are typically seen. Associated conditions with Celiac disease include dermatitis herpetiformis (multiple intensely pruritic papules and vesicles that occur in groups), Down syndrome, and other conditions which have autoimmune features (eg, type 1 diabetes mellitus). Treatment usually consists of eliminating gluten products from the diet. Crohn disease (A) is an inflammatory bowel disease (IBD) that causes inflammation of the lining of the GI tract. These patient's typically present with abdominal pain, bloody diarrhea, fever, and weight loss. Diverticular disease (C) (ie, diverticulosis) is the condition of having diverticula (out-pouchings of the colonic mucosa) in the colon. These patients typically present with painless rectal bleeding, and is rarely seen in patient's younger than 40 years of age. Ulcerative colitis (D) is another form of IBD and causes inflammation and ulcers in the colon. The main presenting symptom is constant diarrhea mixed with blood.

A previously healthy 20-year-old man presents to the emergency department assisted by his roommates. The patient is febrile, disoriented, has difficulty speaking, and is complaining of generalized abdominal pain. He has a rash that does not blanch on palpation. Laboratory findings include hematocrit 21%, platelets 10,000/mcL, MCV 90 fL, INR 1.6, and creatinine 4.75 mg/dL. Which of the following is the most appropriate next step in management? Begin emergent plasma exchange Hold treatment until labs for ADAMTS13 deficiency are completed Prescribe cortisone cream and have the patient follow up with their primary care provider Watchful waiting as the condition is self-limiting

Begin emergent plasma exchange A patient presenting to an emergency room with thrombotic thrombocytopenic purpura (TTP) should immediately begin emergent plasma exchange. This condition is almost always fatal if not treated immediately. This patient presented with the classic pentad of microangiopathic hemolytic anemia, thrombocytopenia, fever, acute renal failure, and severe neurologic findings. Microangiopathic hemolytic anemia and thrombocytopenia are the hallmarks of the condition and a presumptive diagnosis of TTP can be made without the entire pentad. Acquired TTP is characterized by severe ADAMTS13 deficiency and the presence of an inhibitor (autoantibody directed against ADAMTS13). ADAMTS13 deficiency with an ADAMTS13 inhibitor will confirm the diagnosis of TTP, but treatment is urgent and plasma exchange should not be postponed for testing. In addition to plasma exchange, glucocorticoids and rituximab should be administered. Platelet transfusion may also be necessary in the case of severe thrombocytopenia.

A 9-year-old boy has been diagnosed with stage III non-Hodgkin lymphoma. It is anticipated that he will develop nausea and vomiting during chemotherapy. What class of medication is recommended prior to starting treatment to decrease these side effects? Benzodiazepines Glucocorticoids Neurokinin-1 receptor antagonists Three 5-hydroxytryptamine receptor antagonists

Benzodiazepines Non-Hodgkin lymphoma is divided into four stages depending on location. Stage I is a single nodule or single tumor outside of the abdomen or mediastinum. Stage II is either a single tumor with extension to the regional lymph nodes or two single nodules on the same side of the diaphragm. Stage III is defined by nodules on both sides of the diaphragm, paraspinal or epidural tumors, primary intrathoracic disease, or extensive primary intra-abdominal disease. Stage IV would be any of the above with bone marrow or central nervous system involvement. Combination chemotherapy without radiation is the most common treatment in the pediatric patient. Side effects from combination therapy can include myelosuppression, neutropenia, and nausea and vomiting. In the pediatric population, pretreatment with benzodiazepines can help ameliorate the nausea and vomiting side effects. During the acute treatment, three 5-hydroxytryptamine receptor antagonists are recommended for the pediatric population, they are most effective in the first 24 hours following treatment.

What class of medication should be used for primary prophylaxis to prevent variceal hemorrhage? ACE inhibitor Antihistamine Beta blocker Proton pump inhibitor

Beta blocker Nonselective beta blockers are used for the primary prophylaxis of variceal bleeding and have demonstrated a decrease in the risk of variceal bleeding. All patients with large varices (diameter greater than 5 mm) should be considered for prophylactic therapy ("primary prophylaxis") to prevent variceal bleeding. The presence of additional endoscopic signs such as red wales does not influence the decision regarding prophylactic therapy. The mortality rate is reduced in patients taking a beta blocker. The number of patients needed to be treated to prevent one death is approximately 22. In patients who do not bleed during therapy and who do not experience side effects, treatment should be continued indefinitely because withdrawal of a beta blocker can result in an increased risk of bleeding. A long-acting preparation of propranolol or nadolol may be started. The usual starting dose of long-acting propranolol is 60 mg once daily and that of nadolol is 20 mg once daily. Because the risk of bleeding is greatest at night, the beta blocker should be administered in the evening.

A 45-year-old woman uses ibuprofen daily for the last 12 months to help with her chronic back pain. She comes to you with new onset epigastric pain. Endoscopic examination reveals a duodenal ulcer. Helicobacter pylori serologic testing is positive. She has no known drug allergies. In addition to stopping the ibuprofen, which of the following is the most appropriate therapy at this time? Bismuth plus metronidazole plus tetracycline plus a proton pump inhibitor Clarithromycin plus an H2-antagonist Metronidazole plus amoxicillin plus bismuth subsalicylate Metronidazole plus bismuth subsalicylate

Bismuth plus metronidazole plus tetracycline plus a proton pump inhibitor Peptic ulcer disease has a lifetime prevalence of 10%. It has many etiologies, including H.pylori infection, NSAIDs, aspirin, bisphosphonates, gastric hypersecretory states, gastric cancer, tobacco, psychological stress, chemo and radiotherapy, and infection with cytomegalovirus and herpes simplex virus. Half of the American population is colonized with H.pylori, but only 5-10% develop ulcers from it. Roughly 80% of duodenal ulcers and 60% of gastric ulcers are due to H.pylori. The most common presenting symptom is epigastric abdominal pain, however, many patients are asymptomatic. Asymptomatic patients may present with the main complications of peptic ulcer disease, such as upper GI bleeding from perforation or penetration. Diagnosis requires endoscopy. All gastric ulcers require biopsy to rule out malignancy and duodenal ulcers require biopsy when they have malignant features. Tests for H. pylori include serology, stool antigen, and rapid urease testing. Treatment includes H. pylori eradication if present, lifestyle changes, NSAID and aspirin cessation, and proton pump inhibitors (PPI). Surgery may be required if patients are refractory to medication management or perforation/penetration is present. Preferred treatment of H.pylori-positive peptic ulcer disease is the classic quadruple therapy of metronidazole + tetracycline + bismuth + PPI, for 10-14 days.

A 50-year-old man with a history of hypertension on amlodipine presents to the neurology clinic with gradually worsening bilateral headaches for the past 12 months. Vital signs include a heart rate of 92 bpm, blood pressure of 120/80 mm Hg, respiratory rate of 20/min, oxygen saturation of 98% on room air, and temperature of 98.6°F. The patient's MRI is shown above. What is the most common physical exam finding seen in a patient with this condition? Bitemporal hemianopsia Cerebrospinal fluid rhinorrhea Dysdiadochokinesia Hemiparesis Upward gaze palsy

Bitemporal hemianopsia Pituitary adenomas are benign tumors of the anterior pituitary gland. They may be functioning (hormone-secreting) or nonfunctioning. Pituitary adenomas can present with neurologic symptoms, symptoms due to abnormal hormone secretion, or as an incidental finding on a radiologic examination performed for another reason. The most common symptom in patients with a nonfunctioning adenoma is impaired vision, which is caused by suprasellar extension of the adenoma resulting in compression of the optic chiasm. This most often manifests as decreased vision in the bilateral temporal fields, called bitemporal hemianopsia. Therefore, patients with pituitary adenomas causing visual impairment will often have temporal visual field deficits on exam (hemianopsia), and one or both eyes may be affected. Visual acuity may decrease if there is severe compression of the optic chiasm. Although bitemporal hemianopsia is the most common manifestation, a pituitary adenoma can cause various patterns of visual impairment and thus should be considered with any unexplained pattern of vision loss. The visual impairment from pituitary adenomas typically has a slow progression, which can cause delays in diagnosis. The other neurologic symptoms that can be caused by pituitary adenomas include nonspecific headaches and diplopia (caused by compression of the oculomotor nerve). The most common hormonal abnormality from pituitary tumors is hypogonadism, which can cause amenorrhea in women and infertility and decreased libido in men. The best diagnostic imaging modality for identifying pituitary adenomas is an MRI of the brain with gadolinium contrast. The recommended treatment for pituitary adenomas causing neurologic symptoms is transsphenoidal surgery to remove the tumor.

A 35-year-old woman presents to her primary care clinician for palpitations for the past 2 months. She states she feels her heart racing, which is often accompanied by sweating, tremors, and anxiety. She also reports 10 pounds of unintentional weight loss over the past 2 months and menstrual irregularities. She has a history of celiac disease and takes no medications at this time. She reports no history of anxiety or depression, chest pain, or shortness of breath. Her vitals include BP of 142/85 mm Hg, HR of 145 bpm, RR of 14/min, T of 99.0°F, and SpO2 of 99% on room air. Physical exam reveals a diffusely enlarged, nontender thyroid, exophthalmos, and warm, moist skin. Her labs are significant for a thyroid-stimulating hormone of 0.2 mU/L, T3 of 400 ng/dL, and a free T4 of 3.5 ng/dL. ECG shows an irregularly irregular rhythm, and chest radiographs reveal no abnormalities. What is the best next step in the management of this patient's condition? Block sympathetic hyperactivity Electrical cardioversion Inhibit the release of stored thyroid hormone Inhibit thyroid hormone synthesis Radiofrequency ablation

Block sympathetic hyperactivity Graves disease is the most common cause of hyperthyroidism in the United States. In Graves disease, the thyroid-stimulating hormone (TSH) receptors are stimulated by antibodies, leading to hyperfunction of the thyroid gland. Risk factors include female sex, family history, and personal history of other autoimmune diseases (e.g., Addison or celiac disease). Patients often report heart palpitations, anxiety, heat intolerance, sweating, weight loss, menstrual changes, or tremors. Physical exam often reveals a diffusely enlarged thyroid, often with a bruit, sinus tachycardia, exophthalmos, sweaty skin, and tremor. Heart rhythm irregularities, such as atrial fibrillation, can develop if hyperthyroidism is left untreated. In Graves disease, TSH is suppressed while T3 and T4 hormone levels are elevated. TSH receptor antibodies and antithyroperoxidase are often elevated. Management of this condition starts with blocking sympathetic hyperactivity to help control symptoms and heart rate. Propranolol, a beta-blocker, is the initial treatment of choice for patients with hyperthyroidism. Beta-blockers can help control symptoms of hyperthyroidism by blocking beta-adrenergic receptors in the heart and should be started in patients with hyperthyroidism as soon as possible unless they have a contraindication. Atenolol is a beta-blocker that has the advantage of being a beta-1 selective beta-blocker and only requires once-daily dosing (25-50 mg/day). But any medication that blocks beta-adrenergic function will help control symptoms such as tachycardia, anxiety, tremors, and palpitations. Effective therapy also needs to be aimed at reducing thyroid hormone synthesis in patients with hyperthyroidism. Treatment options include anti-thyroid medications, radioiodine ablation, or surgery. Though all three of these options work well, they all have side effects and potential risks. The use of oral anti-thyroid medications, such as propylthiouracil, is a good option in pregnant patients since surgery and radiation therapy are not recommended.

A 28-year-old woman presents to the clinic with worsening symptoms related to her recently diagnosed ulcerative colitis. What would you expect her signs or symptoms to include? Bloody diarrhea Fistula formation Mouth sores Night sweats

Bloody diarrhea Bloody diarrhea is the classic finding in ulcerative colitis. This can occur in Crohn's disease as well, especially if the rectum is involved, but it is a more consistent finding with ulcerative colitis. Others symptoms include crampy abdominal pain, tenesmus, and loss of appetite. Ulcerative colitis is an inflammatory bowel disease that only affects the colon and spares the rest of the digestive system. Diagnosis is made by biopsy during colonoscopy. On biopsy, ulcerative colitis only affects the innermost lining (mucosal layer) of the colon. Crohn's disease, another inflammatory bowel disease, affects all layers of the gastrointestinal tract and can span the entire gastrointestinal tract from the mouth to the anus.

A 70-year-old man presents to the clinic reporting unsteady gait, frequent falls, left-sided arm and leg weakness, loss of balance, and difficulty walking. He also reports a headache that is severe and causes nausea. He states the headache began about 2 months ago and has progressively worsened. Valsalva maneuver makes the headache worse. The left-sided weakness started at about the same time as the headache and has gradually become more severe. His history is positive for hypertension, and he has a 15 pack-year smoking history. He takes lisinopril 10 mg daily. His vital signs are within normal limits. Physical exam reveals nystagmus, left-sided upper and lower extremity muscle weakness with hyperreflexia, and antalgic gait. Which of the following is the most likely diagnosis? Amyotrophic lateral sclerosis Brain tumor Hemorrhagic stroke Normal pressure hydrocephalus Parkinson disease

Brain tumor Brain tumors are an uncommon cause of headache but must be considered in a patient with a new, gradual-onset headache accompanied by neurological signs or seizures. The most common symptoms of brain tumors include a headache that may be worsened by Valsalva maneuver or straining and that may be more severe in the morning, focal neurological deficits, seizures, hemianopia, personality or behavioral changes, and disordered language. Older patients and children with new-onset headaches and neurologic deficits should raise clinical suspicion for a brain tumor. Brain tumors may be primary or metastatic. Advanced lung cancer commonly metastasizes to the brain, as does cancer of the breast, skin, colon, kidney, and thyroid. Risk factors for primary brain tumors include exposure to high-dose ionizing radiation and certain genetic syndromes such as Cowden disease, Gorlin syndrome, Li-Fraumeni syndrome, neurofibromatosis, and tuberous sclerosis. Brain tumors are diagnosed with gadolinium-enhanced magnetic resonance brain imaging or computed tomography if magnetic resonance imaging is not available. Treatment is tailored to the specific type of tumor.

A 12-year-old boy presented to your office one week ago with complaints of sore throat, bilateral knee pain, and chest pain. Treatment with penicillin was initiated and he has now returned for follow-up. Which of the following laboratory tests is most appropriate to monitor progress of his disease? Blood cultures C-reactive protein Complete blood count Throat culture

C-reactive protein Acute rheumatic fever (ARF) is an inflammatory, autoimmune response that develops after infection with Group A Streptococci (GAS). The incidence of ARF has significantly decreased in the past 50 years likely due to improved socioeconomic conditions and a decline in the prevalence of rheumatogenic strains of GAS. ARF has a number of signs and symptoms that may occur individually or in different combinations. Many patients initially have upper respiratory symptoms including pharyngitis. Some patients are asymptomatic, or if symptomatic do not seek medical treatment which leads to the sequelae of ARF. Other manifestations of ARF include polyarthritis, Sydenham's chorea, subcutaneous nodules, erythema marginatum, and carditis, which is the most serious manifestation. Diagnosis is based on clinical symptoms. Confirmation of GAS infection is helpful in making the diagnosis of ARF, but is not necessary. Treatment is with anti-inflammatory agents, antibiotics and therapy to manage the heart failure sequela. Recommended antibiotic treatment is with penicillin, which is used to eradicate the GAS infection. Measurement of C-reactive protein (CRP) or erythrocyte sedimentation rate (ESR) can be used to monitor the progression of the acute disease. Inflammatory markers are elevated with ARF. A normal result a few weeks after completion of treatment is indicative of resolution, whereas an increase suggests a rebound of inflammation and that the infection has not resolved.

Which of the following is an indication for prophylaxis against Pneumocystis jirovecii pneumonia in individuals infected with HIV? CD4 count of 125 CD4 count of 1500 Individuals taking anti-retroviral therapy Pregnant women with HIV regardless of CD4 count

CD4 count of 125 Pneumocystis jirovecii pneumonia (PCP), previously called Pneumocystis carinii pneumonia, is the most common cause of death in individuals with AIDS-related complications. Primary prophylaxis is indicated when CD4 counts fall below 200. Risk factors for the development of this opportunistic infection include a history of PCP, decreased CD4 count, as well as undiagnosed weight loss, oropharyngeal candidiasis, night sweats and fever in individuals with CD4 counts above 200 cells. Definitive diagnosis is by cytopathologic or histopathologic evidence of the organism in induced sputum samples, bronchoalveolar lavage fluid, or tissue. Trimethoprim-sulfamethoxazole is the recommended medication for prophylaxis and is also used for treatment of the disease.

A 47-year-old man presents with decreased libido and low energy levels. In addition to low testosterone levels, laboratory results show a prolactin level of 102 ng/mL. All other hormone levels tested are normal. The patient is sent for MRI of the brain, which reveals an 8 mm x 4 mm sellar mass. Which of the following is the most appropriate therapy? Cabergoline Quetiapine Radiation therapy Transsphenoidal surgery

Cabergoline Cabergoline is a dopamine agonist that is the preferred initial treatment of patients with a prolactin-secreting pituitary adenoma. Note that bromocriptine has historically been used, but cabergoline has been shown to be more effective and have less side effects. The most commonly occurring pituitary adenoma is a prolactinoma (prolactin-secreting). Indications for treatment of a prolactinoma are (a) mass size greater than 10 mm (termed a macroadenoma), (b) neurological symptoms due to the position of the mass, (c) hormonal symptoms, including hypogonadism or galactorrhea, and (d) women trying to conceive. Symptoms of a prolactinoma in a woman may include amenorrhea, oligomenorrhea, and galactorrhea. In men, symptoms may include decreased libido, fatigue, and eventually loss of muscle mass, gynecomastia and galactorrhea. Treatment with a dopamine agonist results in lowering of prolactin levels and shrinking of the tumor size in more than 90% of cases.

A patient complains of a change in bowel habits over the past 3 months. A fecal occult-blood test is positive. During a digital rectal examination, you palpate a solid 2cm by 2cm mass in the rectum. Which of the following serum tests would you order in the initial laboratory evaluation of a patient with suspected rectal neoplasia? Alpha fetoprotein (AFP) Cancer antigen 125 (CA-125) Cancer antigen 15-3 (CA-15-3) Carcinoembryonic antigen (CEA)

Carcinoembryonic antigen (CEA) Rectal cancer occurs most commonly as an adenocarcinoma. Other types are lymphoma, sarcoma and carcinoid. Symptoms include frank or occult bleeding, change in bowel habits, abdominopelvic pain, malaise, back pain and urinary symptoms. In addition to a thorough physical examination, evaluation usually requires a digital rectal examination and rigid proctoscopy. Lesions are biopsied for microscopic examination. If metastasis is suspected, further imaging is accomplished via ultrasound, CT and MRI modalities. Laboratory evaluation may include complete blood count, liver and kidney function tests, carcinoembryonic antigen (CEA) test and serum chemistries. Carcinoembryonic antigen is a glycoprotein involved in cell adhesion. It is normally produced in gastrointestinal tissue during fetal development, but the production of CEA stops before birth. Therefore, it is usually present only at very low levels in the blood of healthy adults. However, the serum levels are raised in some types of cancer, which means that it can be used as a tumor marker to monitor treatment response but not as a diagnostic modality.

A 47-year-old woman presents with several months of a persistent cough that recently resulted in hemoptysis. A review of system also confirms episodes of diarrhea, rash, and flushing. Physical exam is normal with the exception of focal wheezing over the right upper lobe. Which of the following is the most likely diagnosis?' Alpha-1-antitrypsin deficiency Carcinoid tumor Pulmonary hypertension Pulmonary tuberculosis

Carcinoid tumor Bronchial carcinoid tumors commonly cause persistent coughing with hemoptysis and focal wheezing. As these tumors begin to secrete hormonal mediators, they may cause carcinoid syndrome during which symptoms can include diarrhea, flushing, head and neck edema, bronchospasm, or hives. Specific manifestations depend on the type of hormone released, as this can vary. Carcinoid tumors should also be suspected in patients with recurrent pneumonia. Most diagnosed carcinoid tumors grow in the central bronchi. Peripherally-located bronchial carcinoid tumors are rare and usually do not produce symptoms. Suspicion of a bronchial carcinoid tumor should prompt work-up with imaging. A CT scan may help localize the lesion and assess growth over time. Octreotide scintigraphy can also aid in localization. Fiberoptic bronchoscopy usually allows for biopsy in centrally-located tumors. However, biopsy is often complicated by bleeding, as these tumors are generally very well vascularized. Though bronchial carcinoid tumors rarely metastasize, they commonly compromise respiratory function through bleeding and airway obstruction. These tumors are often resistant to radiation and chemotherapy. Fortunately, surgical excision generally gives a favorable prognosis.

Which of the following is associated with sudden onset hypotension, tachycardia, and impaired diastolic filling? Cardiac tamponade Constrictive pericarditis Pericardial effusion Pneumopericardium

Cardiac tamponade Cardiac tamponade results from acute compression of the myocardium by rapid fluid (or gas) accumulation in the pericardial sac. Tamponade develops when fluid filling the pericardial sac accumulates faster than the rate of stretch in the parietal pericardium. The resulting extrinsic pressure on the myocardium exceeds right atrial pressure leading to a reduction in right ventricular filling. With a continued rise in pericardial pressure, cardiac compliance decreases. Flow of blood into the right side of the heart ceases, leading to a precipitous decline in cardiac output. Key to remember: the rate of fluid accumulation, not the absolute volume, is the important factor in the development of tamponade.

A 39-year-old man presents to clinic with a complaint of penile purulent discharge and burning with urination that started after he had unprotected sexual intercourse with a stranger. Gram-negative diplococci are noted on gram stain of the urethral discharge. Which of the following antibiotics is used to target this organism? Benzathine penicillin G Ceftriaxone Doxycycline Levofloxacin

Ceftriaxone This patient presents with a gonococcal infection and the treatment of choice is ceftriaxone. Gonorrhea is an infection from Neisseria gonorrhoeae and affects both sexually active males and females. A gonococcal infection typically presents in males with symptoms of urethritis or epididymitis which includes discharge, dysuria, and testicular pain. Gram-negative diplococci on gram stain of urethral discharge is highly sensitive and specific for a gonococcal infection. Ceftriaxone, a cephalosporin, is used as treatment for gonococcal infections due to its low rate of resistance and therefore high efficacy. Because of the rising coinfection with Chlamydia trachomatis, it is recommended that azithromycin (in pregnancy) or doxycycline (in nonpregnant patients) be added to the treatment regimen in order to treat both organisms.

An 18-year-old sexually active man presents with testicular pain for 3 days. The pain is constant and is accompanied by fever. Examination reveals a mildly tender right testicle with a normal lie and no urethral discharge. Urinalysis shows 2+ leukocyte esterase, negative nitrites, and 10-25 WBC x 109/L. What treatment is indicated? Ceftriaxone 500 mg intramuscular Ceftriaxone 500 mg intramuscular once and doxycycline 100 mg twice a day for 7 days Ciprofloxacin 250 mg twice a day for 3 days Ciprofloxacin 500 mg twice a day for 14 days

Ceftriaxone 500 mg intramuscular once and doxycycline 100 mg twice a day for 7 days This patient has signs and symptoms consistent with epididymitis. Epididymitis results from infection of the vas deferens, causing edema from passive congestion and inflammation. It usually presents with a gradual onset of scrotal pain, unlike torsion, which presents with a more abrupt onset of symptoms. Approximately 10-30% of patients present with urinary symptoms, and 75% report fever. Early in the course, patients present with pain and tenderness localized to the epididymis, but symptoms quickly spread to the adjacent testicle. Classically, Prehn sign (decrease in pain with elevation of the scrotum) has been taught to be indicative of epididymitis, but this sign has a low sensitivity. Urinalysis typically demonstrates signs of infection. Pyuria is common (50-93%). If testicular torsion is suspected, scrotal ultrasound should be pursued. Antibiotic therapy should be tailored based on patient characteristics. In sexually active men, Chlamydia trachomatis and Neisseria gonorrhoeae infections are the most common cause of epididymitis, particularly in younger men (< 35 years). Therefore, treatment includes ceftriaxone and doxycycline. Older patients are more likely to have urinary tract abnormalities, predisposing them to epididymitis from typical urinary tract pathogens (e.g., E. coli). However, if they are sexually active as well, coverage of C. trachomatis and N. gonorrhoeae is also indicated. In addition to bacterial agents, amiodarone can cause epididymitis, as it concentrates in the testicle, resulting in lymphocytic infiltration. These patients usually do not have fever or pyuria.

A 58-year-old man presents to the emergency department complaining of chest pain. At presentation, the patient is alert, anxious, and able to verbalize a complaint of chest pain that started after he fell off his ladder while fixing the trim. There is some bruising on his chest wall. There are no other gross deformities on exam. Suddenly, the patient becomes unresponsive. Telemetry shows the rhythm above. No pulse can be palpated. In addition to securing the patient's airway, which of the following is the most appropriate next step in the management of this patient? Amiodarone drip Chest compressions Epinephrine Synchronized cardioversion

Chest compressions Pulseless Electrical Activity (PEA) is defined as an organized electrical rhythm without sufficient mechanical ventricular contraction to result in a palpable pulse or measurable blood pressure. PEA is thought to be the presenting rhythm in almost half of cardiac arrests. PEA is a non-perfusing rhythm and requires the immediate initiation of cardiopulmonary resuscitation with chest compressions. Cardiopulmonary resuscitation, vasopressor administration and recognition and reversal of the underlying cause are the cornerstones of the management of PEA. Causes of PEA can be summarized by The Hs and Ts, and include hypovolemia, hypoxia, hydrogen ion excess (acidosis), hyper- and hypokalemia, hypothermia, toxins, tamponade (cardiac), tension pneumothorax and thrombosis (coronary and pulmonary). Hypovolemia and hypoxia are the most common causes of noncardiac PEA.

A new thyroid nodule is found in an 82-year-old man. He questions the risk of this nodule being malignant. You perform a detailed history and discover that his maternal uncle worked as a nuclear energy technologist and had "some kind" of head and neck cancer. As a child, the patient received radiation treatments for severe acne. He has had 6 CT scans throughout his life. He is retired, but worked as a plumber in a hydroelectric plant for 35 years. Which of the following risk factors places him at highest risk for a malignant thyroid nodule? Childhood acne treatment Exposure to CT scans Family history Occupational exposure

Childhood acne treatment Thyroid cancers arise from the two distinct cell types found in the gland. The follicular cells can transform into papillary and follicular carcinomas, while the neuroendocrine calcitonin-secreting cells can transform into medullary carcinomas. Risk of malignant transformation correlates strongly with accidental radiation exposure, as in those exposed to atomic bombings (Japanese youth in Hiroshima and Nagasaki), those exposed to nuclear accidents (Chernobyl residents) and those treated with radiation therapy for the treatment of acne, ringworm or tonsillar hypertrophy (pre-1960s).

Which of the following infectious agents is associated with reactive arthritis? Adenovirus Chlamydia trachomatis Staphylococcus aureus Streptococcus pyogene

Chlamydia trachomatis The development of reactive arthritis is associated with a number of bacterial agents. Reactive arthritis is characterized by conjunctivitis or uveitis, urethritis, and oligoarthritis ("can't see, can't pee, can't climb a tree"). The syndrome occurs in susceptible hosts after infection with certain agents, including Chlamydia trachomatis. The arthritis typically develops 2-6 weeks after an episode of urethritis or dysentery. The most commonly involved joints are the ankles and knees, and the syndrome typically involves bilateral joints. Conjunctivitis may be present early and progress to uveitis later in the disease process. Management focuses on decreased inflammation with nonsteroidal anti-inflammatory medications. C. trachomatis-triggered reactive arthritis improves faster with antibiotics.

A 4-week-old boy presents with a 2-week history of increasing dyspnea, cough, and poor feeding. On examination you note conjunctivitis, tachypnea, and rales on lung auscultation. A chest X-ray shows hyperinflation and diffuse interstitial infiltrates. Which of the following is the most likely etiologic agent? Chlamydia trachomatis Parainfluenza virus Respiratory syncytial virus Staphylococcus species

Chlamydia trachomatis infections in newborns can manifest as pneumonia and conjunctivitis. Chlamydial pneumonia is usually seen in infants 3-19 weeks of age, and they frequently have been sick for several weeks. Fewer than 10% of neonates born to women with active chlamydia infection during labor develop pneumonia. The infant appears nontoxic and is afebrile, but is tachypneic with a prominent staccato cough. Physical examination reveals diffuse rales with few wheezes. The chest film shows hyperinflation and diffuse interstitial or patchy infiltrates and blood work frequently reveals eosinophilia. In addition to chest radiographs and blood work, specimens should be collected from the nasopharynx. Azithromycin, 10 mg/kg on day 1 and 5 mg/kg on days 2-5, is the treatment of choice. Conjunctivitis is present in many cases. Symptoms include conjunctival injection, various degrees of ocular discharge, and swollen eyelids. This infection is transmitted vaginally from an infected mother, and can present within the first 15 days of life. The diagnostic standard is to culture a conjunctival swab from an everted eyelid. Treatment for conjunctivitis is the same as for pneumonia. Prophylaxis with antibiotic ointments does not prevent vertical perinatal transmission of C. trachomatis, but it will prevent ocular gonococcal infection and should therefore be administered.

A 64-year-old man with chronic epigastric abdominal pain and steatorrhea presents to his primary care office to discuss the results of his recent abdominal CT scan and most recent lab results. His heart rate is 89 bpm, respiratory rate is 18/minute, oxygen saturation on room air is 99%, blood pressure is 125/86 mm Hg, and weight is 190 pounds, which is a 15-pound decrease from his visit 6 months ago. His pertinent labs are as follows: elevated bilirubin and alkaline phosphatase, normal white blood cell count, normal aspartate aminotransferase and alanine aminotransferase, normal amylase and lipase, and kidney function within normal limits. His CT scan is shows "calcified pancreas". Which of the following is most likely to be part of this patient's known medical history? Chronic alcohol consumption Family history of pancreatic cancer Known history of gallstones Personal history of cystic fibrosis Symptoms associated with dairy product consumption

Chronic alcohol consumption This patient has chronic pancreatitis. The condition results from chronic inflammation of the pancreas that causes parenchymal destruction, calcification, and fibrosis of the pancreas. The most common cause of chronic pancreatitis is chronic alcohol consumption. Patients present with chronic or intermittent epigastric or left upper quadrant abdominal pain, steatorrhea, weight loss, and abnormal pancreatic imaging. A CT or MRI can be used to diagnose chronic pancreatitis and would reveal a calcified pancreas. On laboratory tests, the amylase and lipase are usually normal, while serum alkaline phosphatase and bilirubin may be elevated due to the compression on the bile ducts. Treatment includes oral pancreatic enzyme replacement, low-fat diet, and pain medications as needed. In addition, patients should discontinue any and all alcohol consumption to decrease the number of flares.

A 60-year-old man with a past medical history of tobacco use presents to clinic with a chief complaint of cough. The cough has been present for five months, and he now has difficulty breathing when walking to his mailbox. He also had a "coughing fit" similar to this last year lasting three to four months. What is the most likely diagnosis? Acute bronchitis Asthma Chronic bronchitis Emphysema

Chronic bronchitis Chronic bronchitis is a form of chronic obstructive pulmonary disease that is defined as a chronic productive cough for at least three months in at least two successive years. In chronic bronchitis, the bronchi become irreversibly inflamed causing mucus hypersecretion and coughing. Patients are commonly overweight and cyanotic. Symptoms include chronic cough, purulent sputum, and dyspnea, particularly on exertion. Physical examination in a patient with chronic bronchitis reveals decreased breath sounds, increased resonance upon percussion of the lung fields, and use of accessory muscles to breathe. Treatment includes smoking cessation, lifestyle changes with pulmonary rehabilitation, inhaled bronchodilators, and inhaled steroids. Acute bronchitis (A) is an acute inflammation of the bronchi caused by a virus such as influenza, parainfluenza, or rhinovirus. Patients present with one to four weeks of a cough and sputum production. Worsening or chronic dyspnea is typically not associated with the disease, and treatment is aimed toward symptomatic comfort. Asthma (B) is the chronic course of airway obstruction, bronchial hyperresponsiveness, and inflammation. It is distinguished from chronic obstructive pulmonary disease in that asthma causes reversible airflow obstruction while chronic obstructive pulmonary disease is irreversible. Symptoms of asthma include coughing, wheezing, and shortness of breath that occur usually when exposed to an irritant. Emphysema (D) is another form of chronic obstructive pulmonary disease that is defined as the abnormal enlargement and destruction of the airspaces distal to the bronchioles. On physical examination, patients are thin, breathe with pursed lips, and have a barrel-chest. Patients present with dyspnea and irreversible airway obstruction but typically do not have a cough or sputum production.

A 53-year-old man wants to establish care at a new primary care clinic after moving. He reports a history of low back pain after a motor vehicle collision 5 years ago and takes ibuprofen 800 mg three times per day. He reports no fever, blood in urine, or abdominal pain. He reports no tobacco, alcohol, or drug use but says he consumes five to six cups of coffee per day. Today, his vitals are a T of 99.1°F, BP of 144/84 mm Hg, RR of 13/min, HR of 88 bpm, and oxygen saturation of 99% on room air. On physical examination, he has normal bowel sounds in all four quadrants and is not tender to palpation over the entire abdomen. Lungs are clear to auscultation, and auscultation of his chest reveals a regular rate and rhythm without any murmurs. Laboratory results reveal a creatinine of 2 mg/dL, and urinalysis reveals mild proteinuria and hematuria. CT scan of the abdomen without contrast reveals bilaterally reduced kidney size with irregular renal contours and papillary calcifications. Which of the following is the most likely diagnosis? Chronic interstitial nephritis Focal segmental glomerulosclerosis Ischemic tubular necrosis Polycystic kidney disease Renal cell carcinoma

Chronic interstitial nephritis Analgesic nephropathy is kidney damage due to analgesic medications and is characterized by chronic interstitial nephritis and papillary necrosis. Common medications that can cause analgesic nephropathy include NSAIDs, aspirin, and acetaminophen alone or in combination with aspirin, codeine, or caffeine. Patients who ingest caffeine in addition to these medications are at a higher risk. Clinicians need to be aware of patients with a history consistent with analgesic nephropathy since most patients are asymptomatic. Physical exam is often benign in these patients, with hypertension being the most common physical exam finding. Laboratory abnormalities in patients with analgesic nephropathy include an elevated creatinine and blood urea nitrogen, proteinuria, pyuria, and hematuria. CT abdomen without contrast findings include bilateral small kidney size, papillary calcifications, and kidneys with irregular contours. Treatment involves removing the causal agent and providing supportive measures.

A 70-year-old man presents for his annual exam. The patient says he feels well and has had no recent illness. He states, "Maybe a bit of slowing down, but that's to be expected." Vital signs are BP of 135/60 mm Hg, HR of 85 bpm, RR of 15/min, oxygen saturation of 97% on room air, and T of 98.3°F. The patient's BMI is 18 kg/m2. Physical exam reveals palpable cervical and supraclavicular lymph nodes. A thorough palpation of the abdomen reveals an enlarged spleen with a sharp edge. Findings on exam are otherwise normal. You order a CBC, CMP, and blood smear. Complete blood count reveals a hemoglobin of 14 g/dL, hematocrit of 38 L/L, platelets of 130 K/μL, and WBC of 8,000/μL with 5,000 lymphocytes and 2,700 neutrophils. Blood smear results show isolated lymphocytosis and fragile lymphocytes. Which of the following is the most likely concern? Chronic lymphocytic leukemia Chronic myeloid leukemia Hodgkin lymphoma Multiple myeloma Sarcoidosis

Chronic lymphocytic leukemia This patient presents signs of chronic lymphocytic leukemia (CLL), the most common kind of leukemia in older adults. The most frequent presenting symptoms are lymphadenopathy, splenomegaly, and lymphocytosis. Many patients with CLL are asymptomatic, discovered incidentally from their CBC. Only a small percentage present with classic constitutional symptoms of fevers, fatigue, night sweats, and weight loss. CLL is caused by a mutation in the B lymphocytes, whose monoclonal population then gradually expands. This patient's lymphocyte count is almost twice that of his neutrophils, the inverse of the usual proportions. On the peripheral blood smear, fragile lymphocytes result in smudge cells with no discernible membrane or nucleus. CLL can develop slowly over time, with gradual increases in nonfunctioning lymphocytes accompanied by gradual declines in functional blood cells. For asymptomatic patients in the early stage, observation is the best course. Therapy is reserved for patients with advanced stage or active disease, showing anemia, thrombocytopenia, or severe B symptoms. This patient's bone marrow function is reasonably well preserved but needs to be monitored.

A 23-year-old woman presents to the clinic stating she has begun a new relationship and her new male partner is seronegative for herpes simplex virus. She is seropositive for the virus and has three to five symptomatic episodes per year. She wishes to protect her partner from infection. What is the most effective way to decrease the risk of herpes simplex virus transmission to her partner? Chronic oral acyclovir Chronic topical acyclovir Circumcision of the partner Condom use only during outbreaks Episodic oral acyclovir during outbreaks

Chronic oral acyclovir Genital herpes is caused by the herpes simplex virus type 1 and type 2, with type 2 being most prevalent. The mode of transmission is through skin-to-skin contact with infected individuals. The lesions of active genital herpes are single or grouped vesicles that become painful, shallow ulcers before resolving. Herpes simplex virus lies dormant in the nerve root and reactivates in times of systemic illness, stress, or idiopathically. Viral shedding occurs even in the absence of active lesions, and patients with genital herpes are estimated to shed the virus 10-20% of all days and more frequently during the first year of infection. Treatment for genital herpes is aimed at symptom resolution and diminished transmission. For patients with partners who are seronegative for herpes simplex virus, chronic oral acyclovir taken on a daily basis is recommended to suppress outbreaks and decrease the possibility of transmission. Acyclovir 400 mg taken twice daily, valacyclovir 1 g taken once daily, valacyclovir 500 mg taken once daily, or famciclovir 250 mg taken twice daily are all acceptable options for suppressive treatment. A vaccine for herpes simplex virus does not currently exist.

A 71-year-old man is discovered to have gross hematuria during a routine physical exam. This is a new finding, and is confirmed by a repeat, clean-catch urinalysis. He denies any urinary urgency, frequency, or dysuria. No flank masses are palpated. Which of the following risk factors would have contributed to his most likely disease? Cigarette smoking Cryptorchism Heavy alcohol use HPV infection

Cigarette smoking Cigarette smoking is strongly related to the development of bladder carcinoma, of which this patient is showing symptoms. Bladder carcinoma typically presents in the 8th decade of life, most often in men. In addition to cigarettes, other risk factors include exposure to industrial dyes and solvents. The most common presentation is as painless gross or microscopic hematuria, as seen on a urinalysis. Rarely, urinary urgency, frequency, or dysuria may develop. After a urinalysis is performed, the next best step in diagnosing bladder cancer is a cystoscopy with biopsy. Abdominal ultrasounds, CT, or MRI may later be necessary for staging. Treatment depends on staging. More bladder cancers are superficial at diagnosis and can be managed with transurethral resection and intravesical chemotherapy. More invasive bladder carcinomas generally require radical cystectomy, irradiation, and combination chemotherapy regimens. Prognosis is usually good, with survival rates for superficial bladder cancers near 80%.

Thyroid storm is a severe variant of thyrotoxicosis. Which of the following is the most appropriate way to diagnose it? Antithyroid antibodies Clinical findings Levels of circulating T4 Levels of TSH

Clinical findings The diagnosis of thyroid storm is based on clinical findings. Thyroid storm is often precipitated by infection, which can cause symptoms that mask a thyrotoxic state. Clinical findings in thyroid storm include hyperpyrexia (> 38.8°C), tachycardia out of proportion to temperature, gastrointestinal dysfunction (nausea, vomiting, diarrhea, jaundice), and CNS dysfunction (marked hyperirritability, anxiety, confusion, apathy, coma). There is usually pronounced decompensation of one or more organ systems. Any patient presenting with goiter, fever, and marked tachycardia should be considered to be in thyroid storm and treated accordingly. Admission to the intensive care unit and consultation with an endocrinologist is appropriate. Treatment of thyroid storm includes beta-blockers, antithyroid drugs (propylthiouracil), glucocorticoids, iodide, antipyretics, aggressive fluid replacement, and identification and treatment of any precipitating process. Lugol solution or potassium iodide inhibits release of T4 into the peripheral circulation. If either is used, it should be given only after loading doses of antithyroid drugs to block iodine-induced synthesis of T4. Lithium also has an antithyroid effect and can be used in severe cases of thyroid storm. Severe thyrotoxic symptoms unresponsive to all these regimens may respond to sodium ipodate at 500 mg/day.

A 28-year-old man presents to the emergency department with blurry vision, arm and trunk weakness, and dyspnea. Physical exam reveals bilateral fixed pupillary dilation, symmetrical upper extremity weakness, and palsies of cranial nerves III, IV, and VI. The exam is also notable for five puncture wounds on the left forearm. During his stay, the patient develops respiratory failure and requires intubation. Which of the following is the most likely etiology of this patient's condition? Autoimmune demyelination of peripheral nerves Cervical disk herniation causing spinal cord compression Clostridium botulinum infection Viral destruction of anterior horn cells in the spinal cord

Clostridium botulinum infection This man most likely has wound botulism caused by Clostridium botulinum infection. Botulism is an uncommon but potentially lethal neuroparalysis condition. Botulism is caused by a neurotoxin secreted by C. botulinum, a gram-positive, rod-shaped, obligate anaerobic bacteria that secretes spores. There are five main modes of botulism acquisition: foodborne, infant, wound, inhalation, and iatrogenic. In the United States, the majority of cases are infant botulism, followed by foodborne botulism and wound botulism. Infant botulism is most commonly associated with raw honey ingestion. Foodborne botulism is most commonly associated with home-canned food. Wound botulism is most commonly associated with puncture wounds and black tar heroin injection. Classic clinical manifestations of botulism include the acute onset of bilateral cranial neuropathies, symmetric descending weakness, absence of fever, normal or slow heart rate, normal blood pressure, and no sensory deficits other than blurred vision. Cranial nerve involvement is common, which may manifest as diplopia, blurred vision, nystagmus, ptosis, and dysphagia. Other symptoms may include urinary retention, constipation, and respiratory distress. Infants often present with hypotonia, drooling, and weak cry. Foodborne botulism is often accompanied by nausea, vomiting, abdominal pain, and diarrhea. Laboratory studies should include stool analysis for spores, electromyography, and serum analysis of neurotoxin. The Center for Disease Control should be contacted as soon as the diagnosis of botulism is suspected so that equine serum antitoxin can be obtained. Additional management consists of close respiratory status, antibiotics, and nutritional support.

A 4-year-old boy presents to the emergency department with right knee pain and swelling after he tripped and fell at home. His parent states that he was walking before he tripped over a toy and fell on his right side, and his knee started swelling immediately. His temperature is 98.9°F, blood pressure is 115/78 mm Hg, heart rate is 105 bpm, oxygen saturation is 99% on room air, and respiratory rate is 24/min. A physical exam shows moderate edema over the right knee that is mildly tender to palpation with decreased range of motion. Laboratory findings reveal a hemoglobin of 12.5 g/dL, a platelet count of 250 × 109/L, and a WBC count of 6,000/µL. A knee X-ray shows a large effusion but no evidence of fracture. What is the best next step in the management of this patient's condition? Aspiration of fluid in the knee Coagulation studies Complete skeletal survey Computed tomography of the right knee Reassurance and observation

Coagulation studies Hemophilia is a congenital coagulation disorder that can be divided into two types: hemophilia A and hemophilia B. Hemophilia A is characterized by a congenital deficiency of coagulation factor VIII, whereas hemophilia B is associated with a deficiency of coagulation factor IX. Hemophilia A is more common than hemophilia B. Inheritance of this disorder is X-linked recessive, affecting primarily male patients and some female carriers. Clinicians should suspect hemophilia in any male patient who has symptoms of hemorrhage with a positive family history. Symptoms are identical across the two types and typically present in infant males or in early childhood with varying degrees of bleeding, depending on the level of factor VIII or IX activity. Patients with severe disease will present with spontaneous bleeding into the joints (the knee is the most common site) and soft tissues, whereas those with moderate disease will have bleeding with surgery or trauma, as seen in this patient. Intracranial bleeding is the most concerning symptom since it is a common cause of death and requires immediate evaluation and intervention. If caught in early childhood, patients can receive long-term prophylaxis with factor concentrate to prevent hemophilic arthropathy. Recurrent hemarthrosis can cause progressive joint disease. Hemophilia A and B can be diagnosed with coagulation studies, particularly the measurements of factor VIII and factor IX activity levels, which will be low. An activated partial thromboplastin clotting time (aPTT) will be markedly prolonged in severe hemophilia but may or may not be in milder cases. The aPTT will normalize if mixed with plasma from a patient who does not have hemophilia. Hemoglobin will be within normal limits unless there is recurrent hemorrhage that is untreated. Prothrombin time (PT), plasma von Willebrand factor, and platelet count will also be normal. Treatment with plasma-derived or recombinant factor VIII or IX products is standard. Primary prophylaxis in children with severe hemophilia is also a mainstay of treatment to prevent recurrent hemarthrosis and future joint destruction. In milder cases, treatment with factor products can be given before surgical procedures, before high-risk activities (such as sports), or on an as-needed basis for bleeding episodes. Patients with mild hemophilia A may respond to intravenous or intranasal treatment with desmopressin. Desmopressin is not used in the treatment of hemophilia B. Antifibrinolytic agents (e.g., aminocaproic acid) can be used for mucosal bleeding or following dental procedures. Acute hemarthrosis can be treated with analgesics (avoiding aspirin and NSAIDs), joint immobilization, and ice packs.

A 55-year-old man with a history of peptic ulcer disease presents to your office with a complaint of right knee pain that started last night. On physical exam his knee is erythematous, warm and exquisitely tender to palpation. Synovial fluid analysis reveals the presence of positively birefringent calcium pyrophosphate dihydrate crystals. Which of the following is the most appropriate therapy? Allopurinol Colchicine Indomethacin Vancomycin

Colchicine Calcium pyrophosphate crystal deposition disease, also referred to as pseudogout, is a common crystal-induced arthropathy that generally affects the large joints. Pseudogout has a similar clinical presentation to gout, but the etiology is different. Pseudogout may be idiopathic, especially in the elderly. It may also be caused by trauma, hyperparathyroidism, hemochromatosis, and medications that cause hypomagnesemia such as loop diuretics or proton pump inhibitors used in peptic ulcer disease. Patients present with acute onset of severe pain, inflammation and edema in the knees, ankles, elbows or wrists. Pseudogout is generally monoarticular, but may present in multiple joints as well. Diagnosis is by synovial fluid analysis, so arthrocentesis is necessary for patients with monoarticular arthritis. Initial treatment for pseudogout when two or less joints are inflammed is aspiration and intraarticular glucorticoid injection. When injection is not feasible, treatment is with nonsteroidal anti-inflammatory drugs (NSAIDs) or colchicine. Workup of the underlying metabolic problem causing the pseudogout attack will help to prevent future flare-ups.

A 56-year-old man underwent a prolonged pelvic surgery. Two weeks later, he reports frequent tripping over his right foot. Strength examination of the right foot reveals: 5/5 plantar flexion, 5/5 ankle inversion, 3/5 ankle eversion, and 2/5 dorsiflexion. Light touch sensation is diminished over the lateral but not medial shin. Ankle and knee reflexes are intact. Injury to which one of the following nerves is most likely causative? Common peroneal Femoral Sciatic Tibial

Common peroneal The common peroneal nerve is a division of the sciatic nerve. As it descends, it wraps around the fibular head and then divides into its two distal branches, the superficial and deep peroneal nerves. Compression injury frequently occurs at the fibular head, mainly due to prolonged external pressure, as in prolonged bed rest, high casts, or lengthy surgeries. Common peroneal neuropathy typically presents as a foot drop, in which the patient complains of a limp, tripping over the foot as they ambulate. Patients display a steppage gait to compensate, flexing the hip and knee more to help "clear" the dropped foot. Ankle weakness, namely dorsiflexion, is the culprit, as the main dorsiflexor (tibialis anterior) is a deep peroneal innervated muscle. Ankle eversion, powered by peroneus longus and brevis (two superficial peroneal innervated muscles), may also be weak. However, plantar flexion and ankle inversion have normal strength, as these muscles are innervated by the posterior tibial nerve. Paresthesias and numbness often accompany the dorsiflexion weakness, appearing primarily in the distribution of the superficial peroneal nerve (lateral lower leg and dorsum of the foot).

A previously healthy 30-year-old woman presents to your office with complaints of abdominal pain and diarrhea for the past year. Her abdominal pain is located in the lower abdomen with relief after defecation. Her diarrhea symptoms include a small volume of loose stool, typically after meals. She has experienced some improvement in symptoms with dietary modifications, but overall is still having regular loose stools. Recently she has been feeling more fatigue than normal. Which of the following is the most appropriate initial laboratory test? Complete blood count Giardia antigen Stool culture Thyroid stimulating hormone

Complete blood count Irritable bowel syndrome (IBS) is an idiopathic chronic disorder of the gastrointestinal system characterized by altered bowel habits and abdominal pain. Patients typically present with complaints of abdominal pain, bloating, and constipation or diarrhea, or both. Initial workup is with a complete history and physical exam. Laboratory testing and diagnostic imaging are generally not recommended in patients younger than aged 50 years. Patients with alarm features including iron deficiency anemia, weight loss, and family history of inflammatory bowel disease should have further evaluation. When laboratory testing is indicated, initial tests include a complete blood count to screen for anemia, infection, and inflammation. History-specific diagnostic testing can include gallbladder ultrasound, abdominal computed tomography, and colonoscopy.

A 33-year-old woman is being evaluated in clinic for pain in her left lower leg. She underwent an arthroscopic meniscus repair of the left knee three months ago. She denies any recent trauma. Patient reports sudden onset of pain and burning sensation in her leg. It does not worsen with physical activity or elevation. She denies fever or chills. On physical exam her left leg is erythematous, warm, swollen and tender to the lightest touch. Lower extremity pulses are 2+ bilaterally. Initial lab work reveals a normal complete blood count, erythrocyte sedimentation rate, and C-reactive protein. What is the most likely diagnosis? Cellulitis Complex regional pain syndrome Erysipelas Peripheral vascular disease

Complex regional pain syndrome Complex regional pain syndrome is a form of chronic pain in a body region, but it most commonly affects the extremities. It is characterized by pain, swelling, skin changes, vasomotor instability, limited range of motion, and patchy bone demineralization. It frequently occurs after a soft tissue injury, surgery, or vascular event such as a myocardial infarction or stroke. The pathophysiology of complex regional pain syndrome is incompletely understood and most of the time there is no definable nerve lesion. Symptoms of complex regional pain syndrome include pain that is described as burning, tingling, throbbing or aching. The pain can be quite severe and out of proportion to physical exam. Many patients experience allodynia, which is pain from a stimulus that does not usually provoke pain. Other symptoms include sensitivity to cold or touch, swelling, skin changes such as thickened, rough skin, or muscle atrophy. A diagnosis of complex regional pain syndrome requires the presence of regional pain and sensory changes following a noxious event. It is usually made clinically as there is no laboratory or radiologic testing that alone is diagnostic. Both cellulitis (A) and erysipelas (C) present with erythema, warmth, edema and pain. However, these are caused by bacterial infections, most commonly S. aureus and beta-hemolytic streptococci. Cellulitis involves the deeper dermis while erysipelas affects more superficial layers. Erysipelas is characterized by erythema that is raised above the level of surrounding skin with sharply demarcated borders. Cellulitis is more diffuse, without clear borders and is not raised. Both of these conditions are associated with a systemic response such as fevers and chills, usually with a white count and elevated inflammatory markers. Erythrocyte sedimentation rate, C-reactive protein and the peripheral blood white blood cell count are unremarkable in complex regional pain syndrome. Peripheral vascular disease (D) is the result of compromised blood supply to the extremities caused by atherosclerosis in the peripheral arteries. This results in cold, pale, extremities and claudication. Claudication is pain or cramping that is worse with activity such as walking.

Which of the following findings is most likely to occur with acute diverticulitis? Constipation Gross lower gastrointestinal bleeding Right lower quadrant tenderness Testicular pain

Constipation The clinical signs and symptoms of acute diverticulitis vary with the severity of the disease. Steady pain is the most common symptom in acute diverticulitis, not colicky pain. Most cases in the United States start with vague and generalized pain that localizes to the left lower quadrant. Diffuse tenderness is associated with perforation or abscess formation. Bowel habits are commonly altered with diverticulitis, with constipation being more common than diarrhea, although an alternation between the two is also common. Urinary symptoms can occur due to intestinal inflammation near the ureter or bladder. Although stool samples may be guaiac positive for occult blood, gross bleeding is unusual.

Which of the following is most characteristic of a complex febrile seizure? Convulsions are focal in nature Convulsions lasting 12 minutes Second seizure occurs with second febrile illness Single seizure that occurs at a temperature of 40°C

Convulsions are focal in nature Complex febrile seizures are diagnosed when multiple seizures occur during the same febrile illness, seizures are prolonged (> 15 minutes), or the seizures have a focal component. Not all criteria are needed to diagnose a complex febrile seizure (i.e., a focal seizure would be considered complex even if it only lasted 5 minutes). Simple febrile seizures are associated with generalized convulsions.

A 41-year-old previously healthy woman presents to the ED with gingival bleeding and epistaxis over the past four days. She takes no medications and has no recent travel. You note scattered petechiae, but otherwise physical exam is normal. Her hemoglobin is 12.5 g/dL, international normalized ratio 1.1, PTT 25 seconds, and platelets 14,000/µL. A peripheral blood smear shows few large, well-granulated platelets. Which of the following is the most appropriate initial treatment? Corticosteroids and immunoglobulin Observation Platelet transfusion Splenectomy

Corticosteroids and immunoglobulin Immune thrombocytopenia (ITP) is an acquired autoimmune disease that results in rapid destruction of platelets. It is characterized by thrombocytopenia, the presence of purpura or petechiae, normal bone marrow, and no other identifiable cause for the thrombocytopenia. It is more common in children than in adults. The physical exam may reveal petechiae, epistaxis, gingival bleeding, and menorrhagia. The presence of lymphadenopathy, hepatosplenomegaly, pallor, or hyperbilirubinemia should suggest an alternative diagnosis such as leukemia, lymphoma, lupus, mono, or hemolytic anemia. Management is predicated primarily on the severity of thrombocytopenia and bleeding. Drugs that interfere with platelet function are discontinued. In general, asymptomatic patients with platelet counts > 20,000/µL require no treatment. When the platelet count is < 20,000/µL or the patient is symptomatic, corticosteroid treatment is indicated. Intravenous (IV) immunoglobulin may be coadministered in more severe cases.

A six-year-old boy is brought to clinic because of a sore throat. For the past two days, he has had low-grade fever, decreased activity, and sore throat. Review of his medical records show that his parents declined routine vaccinations. On physical examination, his vital signs are normal. Findings include enlarged cervical lymph nodes and gray and white exudates on the pharynx. Upon scraping of the exudate, bleeding is noted. Complete blood count is normal. Throat culture is pending. Which of the following is the most likely etiologic agent? Candida albicans Corynebacterium diphtheria Epstein-Barr virus Haemophilus influenzae type b

Corynebacterium diphtheria The boy is manifesting signs and symptoms suspicious for diphtheria. Diphtheria is an infectious disease caused by the Gram-positive bacillus Corynebacterium diphtheriae. Infection may lead to respiratory disease, cutaneous disease, or an asymptomatic carrier state. The onset of symptoms is typically gradual; the most common presenting symptoms are sore throat, malaise, cervical lymphadenopathy, and low-grade fever. The earliest pharyngeal finding is mild erythema, which can progress to isolated spots of gray and white exudate. In at least one-third of cases, local elaboration of toxin induces the formation of a coalescing pseudomembrane (composed of necrotic fibrin, leukocytes, erythrocytes, epithelial cells, and organisms). This membrane adheres tightly to the underlying tissue and bleeds with scraping. Definitive diagnosis of diphtheria requires culture of C. diphtheriae from respiratory tract secretions or cutaneous lesions and a positive toxin assay. Routine laboratory results are usually nonspecific. The treatment of respiratory diphtheria consists of antibiotic therapy and diphtheria antitoxin for severe cases. Careful airway management is important due to risk of airway obstruction.

A 38-year-old man presents with the worst headache of his life. A magnetic resonance angiogram is obtained which demonstrates a large unruptured cerebral aneurysm of the posterior communicating artery. Which of the following are you most likely to find on physical examination? Cranial nerve III palsy Cranial nerve V palsy Cranial nerve VI palsy Cranial nerve VII palsy

Cranial nerve III palsy Cranial nerve III palsy is a likely physical examination finding in a patient presenting with a subarachnoid hemorrhage due to a cerebral aneurysm. A subarachnoid hemorrhage is bleeding within the subarachnoid space of the brain and is most commonly caused by a cerebral aneurysm or an arteriovenous malformation. A cerebral aneurysm is a ballooning of the blood vessels of the brain that when under continued pressure can rupture. This is a medical emergency which requires immediate intervention. An unruptured cerebral aneurysm is often incidentally found on brain imaging or when screening for a cerebral aneurysm if a patient has a strong family history. Patients are typically asymptomatic with an unruptured aneurysm but may become symptomatic if increased pressure on the aneurysm continues. Characteristic findings of both unruptured and ruptured cerebral aneurysms include headache, often the worst headache of life, cranial nerve III deficit, and visual loss. Cranial nerve III is the oculomotor nerve which can become compressed when a cerebral aneurysm is present at the junction of the internal carotid artery and posterior communicating artery, causing a unilateral strict downward gaze, ptosis, anisocoria, and blurred vision. When a cerebral aneurysm causes a cranial nerve III deficit with no other neurological symptoms, the aneurysm is likely to be unruptured, but is large enough to compress the nerve.

Over the last 7 months, a 13-year-old girl has had intermittent abdominal pain, which has made her quite irritable. The abdominal pain is associated with arthralgias and general malaise. Review of systems reveals that she has lost 5 kg (11 lb) and has painful bowel movements. Which one of the following is the most likely cause of these symptoms? Celiac disease (gluten enteropathy) Crohn's disease Irritable bowel syndrome Ulcerative colitis

Crohn's disease Crohn's disease is the most common chronic inflammatory bowel disease which occurs during adolescence and young adulthood, with a second peak at 50-80 years of age. The manifestations of Crohn's disease are dependent on the site of involvement, but systemic signs and symptoms are more common than with ulcerative colitis. Crohn's disease presents with chronic diarrhea, crampy abdominal pain, fever, weight loss, and fatigue. Perianal disease (anal fissures, perirectal abscesses, and anorectal fistulas) is also common in Crohn's disease. Crohn's disease may affect any part of the GI tract, from the mouth to the anus, but typically affects the small bowel and colon. Irritable colon and celiac disease may mimic symptoms of Crohn's disease, but objective findings of weight loss and anal lesions are extremely uncommon.

A patient with AIDS presents with fever, severe headache, and positive Kernig sign. Cerebrospinal fluid findings reveal lymphocytosis and increased protein levels. India ink staining reveals encapsulated yeast forms. Which of the following is the most likely pathogen? Borrelia burgdorferi Candida albicans Cryptococcus neoformans Neisseria meningitidis

Cryptococcus neoformans Although all of the above pathogens can cause meningitis in a patient with AIDS, Cryptococcus neoformans has the following spinal fluid findings: increased opening pressure, variable pleocytosis, increased protein, and decreased glucose. India ink stain of the cerebrospinal fluid (CSF) usually reveals budding, encapsulated fungal cells. Prior to waiting on stain results, the CSF can be sent for detection of the cryptococcal antigen (highest sensitivity in AIDS patients). Detecting a serum cryptococcal antigen also aids in the diagnosis. Treatment of cryptococcal meningitis involves antifungal therapy such as amphotericin B and flucytosine, followed by fluconazole suppression until adequate immune reconstitution or, if this does not occur, indefinitely.

A 55-year-old woman with a history of hypertension and diabetes mellitus presents to the emergency department after becoming incoherent during a work meeting 45 minutes prior to arrival. Examination reveals a right facial droop and right upper extremity weakness. It is decided to proceed with thrombolysis with recombinant tissue-type plasminogen activator. Which of the following is an exclusion criterion for using reperfusion therapy? Current warfarin use with INR > 1.7 Lack of intracranial hemorrhage on computed tomographic imaging Myocardial infarction 6 months prior Significant head trauma one year prior

Current warfarin use with INR > 1.7 Current warfarin use with INR > 1.7 is an exclusion criterion for using reperfusion therapy. Inclusion criteria for the treatment of acute ischemic stroke with recombinant tissue plasminogen activator include age ≥ 18 years old, a clinical diagnosis of ischemic stroke causing measurable neurologic deficit, and onset of symptoms < 4.5 hours before beginning treatment. If the exact time of stroke onset is not known, it is defined as the last time the patient was known to be normal. There is a long list of exclusion criteria and relative exclusion criteria including, but not limited to, significant stroke or head trauma in the previous three months, previous intracranial hemorrhage, intracranial neoplasm, arteriovenous malformation or aneurysm, recent intracranial or intraspinal surgery, arterial puncture at a noncompressible site in the previous seven days, symptoms suggestive of subarachnoid hemorrhage, persistent blood pressure elevation (systolic ≥ 185 mmHg or diastolic ≥ 110 mmHg), serum glucose < 50 mg/dL, active internal bleeding, acute bleeding diathesis, platelet count < 100,000/mm3, current anticoagulant use with an INR > 1.7 or PT > 15 seconds, heparin use within 48 hours and an abnormally elevated aPTT, current use of a direct thrombin inhibitor or direct factor Xa inhibitor with evidence of anticoagulant effect by laboratory tests, evidence of hemorrhage or extensive regions of obvious hypodensity consistent with irreversible injury on head CT scan.

A 23-year-old childcare worker presents to the hospital in labor. She has had no prenatal health care because she has felt well, except for one mild cold. She does not know her due date. The 6.0 lb, 18 cm neonate has Apgar scores of 5 and 7 at 1 and 5 minutes, respectively. Physical exam shows microcephaly and splenomegaly. Skin is slightly jaundiced but with no vesicles or erosions. A seizure prompts a CT of the head, which shows a solid ring of solid calcification encircling the two lateral ventricles. What is the most likely causative agent for these newborn findings? Cytomegalovirus Herpes simplex virus Listeria monocytogenes Parvovirus B19 Toxoplasma gondii

Cytomegalovirus This child shows signs of congenital cytomegalovirus (CMV), the most common of the TORCH congenital disorders (which include toxoplasmosis, other, rubella, CMV, and herpes simplex virus). CMV is best known for causing progressive hearing loss in the infant and is a member of the Herpesviridae family that is ubiquitous in the human population, with some regions of the world having 100% positive serology. Most immunocompetent carriers are asymptomatic, and most exposed neonates remain asymptomatic. Mothers who have their primary infection during pregnancy are many times more likely to have symptomatic fetuses and neonates. This mother's job as a childcare worker predisposes her to such infection through contact with saliva and other body fluids of infants. This infant's microcephaly, seizure, and periventricular calcifications shown on CT are among the disease's most common neurological manifestations. The periventricular calcifications can be also seen on prenatal ultrasound. Nonspecific evidence includes splenomegaly, hepatomegaly, and jaundice. Neonates with congenital CMV may pass the universal hearing screen at birth, only to have sensorineural hearing loss months later when hearing becomes more critical to the child's development. Their eyes may also be affected with chorioretinitis. They will be born small for gestational age due to intrauterine growth restriction. Some cases will exhibit purpuric papules and macules known as blueberry muffin rash. CMV can be diagnosed with real-time PCR of a saliva swab, and this may become a universal screening test at birth. Treatment is with intravenous ganciclovir or oral valganciclovir, the prodrug. Although treatment carries risks, they are not as great as the risks of nontreatment.

A 76-year-old man presents complaining of generalized weakness for the past two weeks. His wife states that he has almost no appetite and he spends all day lying in bed. Physical exam reveals a mildly emaciated male who responds slowly to questions. He has hyperpigmented palmar creases. Laboratory workup reveals a serum sodium level of 130 mmol/L and a serum potassium level of 5.8 mmol/L. Which of the following laboratory values is consistent with this patient's presentation? Decreased antidiuretic hormone level Decreased cortisol level Decreased thyroid-stimulating hormone level Decreased thyroxine level

Decreased cortisol level Primary adrenal insufficiency, also known as Addison disease, results from the inability of the adrenal cortex to produce sufficient glucocorticoids and mineralocorticoids. Most common signs and symptoms of Addison disease include anorexia, generalized weakness, skin hyperpigmentation, and hypotension. Hyponatremia and hyperkalemia result from a decrease in aldosterone production, which decreases the number of sodium-potassium antiporters present in the renal tubules. This leads to increased excretion of sodium and retention of potassium. Further workup would reveal a decreased cortisol level. Treatment consists of glucocorticoid and mineralocorticoid replacement.

A 68-year-old woman is diagnosed with hyperparathyroidism. Both her serum calcium level and parathyroid hormone level are elevated. Which of the following findings would be most consistent with the diagnosis? Chvostek sign Decreased serum magnesium level Decreased serum phosphate level Hypotension

Decreased serum phosphate level Calcium and phosphate have an inverse relationship with one another. Typically, as serum calcium levels increase, parathyroid hormone (PTH) levels will decrease due to feedback inhibitor; however, in hyperparathyroidism, PTH release will remain elevated despite having elevated calcium. This increase in parathyroid hormone (PTH) levels will promote the renal excretion of phosphate, causing serum levels of phosphate to decrease. Hyperparathyroidism is the most common cause of hypercalcemia and can be classified as primary, secondary, or tertiary. Primary hyperparathyroidism is typically due to a defect in the parathyroid glands that cause PTH release in the setting of elevated calcium levels. Causes include parathyroid cancer, adenoma, or dysfunctional calcium sensing receptors on the surface of chief cells. Other lab abnormalities include serum hypophosphatemia and an increased urine calcium level. Surgical intervention with a parathyroidectomy is the mainstay of therapy.

A 57-year-old woman with a long history of poorly-controlled diabetes mellitus presents with worsening edema in her legs and an occasional "wet-sounding" cough. Physical exam shows 2+ pitting edema extending to mid-calf bilaterally and trace periorbital edema. Blood tests show hypoalbuminemia and hyperlipidemia and urinary protein excretion is 3.5 grams/24 hours. Which of these complications may occur as a direct result of her new condition? Deep vein thrombosis Gouty arthritis Peripheral neuropathy Struvite calculi

Deep vein thrombosis Deep vein thrombi secondary to a hypercoagulable state may occur in patients with nephrotic syndrome due to excessive urinary protein losses. As serum albumin levels drop below 2 grams/dL, patients typically become deficient in antithrombin, protein C, and protein S, which causes an increased propensity to clotting. Patients with nephrotic syndrome who develops a thrombosis in any location should be anticoagulated for 3-6 months; recurrent thrombi may warrant anticoagulation indefinitely. Nephrotic syndrome is characterized by the triad of hypoalbuminemia, hyperlipidemia, and proteinuria greater than 3 grams/24 hours. Symptoms include dependent edema that can become generalized, ascites, and pulmonary edema. One third of adults with nephrotic syndrome have an underlying systemic disease such as diabetes mellitus, systemic lupus erythematous, or amyloidosis, as opposed to one of many primary renal diseases. Nephrotic syndrome is diagnosed based on the above blood and urinalysis findings. Renal biopsy is typically needed to determine the etiology unless there is a known likely secondary cause. Treatment requires increasing the patient's daily protein intake and using an ACE-inhibitor or angiotensin receptor-blocker to reduce protein loss. Salt restriction and thiazide or loop diuretics aid in edema management. Statins and diet changes may help manage the hyperlipidemia. Gouty arthritis (B) causes acute joint point and is related to overproduction of uric acid or underexcretion of uric acid by the kidney. Neither etiology is directly related to nephrotic syndrome. Peripheral neuropathy (C) may occur in this patient secondary to her poorly controlled diabetes but it is not caused by nephrotic syndrome. Struvite calculi (D) are referred to as "infection stones" as they typically occur in patients with recurrent urinary tract infections with urease-splitting organisms. These stones generally do not occur as a result of nephrotic syndrome.

A 72-year-old man presents to the emergency department with chest pain. During triage, he collapses, and the nursing staff cannot feel his pulse. The patient is taken to a treatment room where cardiopulmonary resuscitation is initiated, and the above cardiac rhythm is noted ("Vfib"). What is the best next step? Administration of amiodarone Cardioversion Defibrillation Two minutes of chest compressions

Defibrillation Sudden cardiac death is a leading cause of mortality. It occurs in a bimodal age distribution: infancy and individuals > 45 years of age. There is a slight predominance in men. In a sudden episode of cardiac death, the initial rhythm is most likely ventricular fibrillation, or ventricular tachycardia that degenerates into ventricular fibrillation. In a witnessed cardiac arrest, if the initial rhythm is amenable to defibrillation, a shock should be administered as soon as possible. Defibrillation is the best chance of recovering a perfusing rhythm from a nonperfusing rhythm. The longer a heart is in a nonperfusing rhythm, the less likely it is to respond to defibrillation and cardiac life support.

A 64-year-old man presents to the clinic for management of chronic symptomatic hypercalcemia of malignancy. Despite the continued use of bisphosphonates, his total calcium level remains elevated. Which of the following is the most appropriate medication this patient should receive next? Calcitonin Cinacalcet Denosumab Zoledronic acid

Denosumab Hypercalcemia occurs when the entry of calcium into circulation exceeds the excretion of calcium into urine or the deposition of calcium into bone. Two of the most common causes of hypercalcemia are primary hyperparathyroidism (commonly caused by a parathyroid adenoma) and malignancy. Clinical manifestations vary. For patients with mild hypercalcemia (serum total calcium level of less than < 12 mg/dL), symptoms are often vague and can include constipation, fatigue, and depression. Moderate hypercalcemia, defined as total serum calcium levels of 12 to 14 mg/dL, may cause additional symptoms such as polyuria (most common renal manifestation), polydipsia, dehydration, muscle weakness, and altered sensorium. Severe hypercalcemia is diagnosed when serum total calcium levels rise above 14 mg/dL and can be associated with lethargy, stupor, and even coma. Treatment of mild hypercalcemia and asymptomatic moderate hypercalcemia focuses on avoidance of aggravating factors, such as thiazide diuretics and volume depletion. Treatment for acute hypercalcemia is reserved for patients who present with either altered sensorium or severe hypercalcemia and includes volume expansion with isotonic saline, administration of calcitonin, and concurrent administration of zoledronic acid or pamidronate (bisphosphonates). Ultimately, the underlying disease causing the hypercalcemia should be treated. In patients with chronic hypercalcemia due to malignancy, it is recommended to use intravenous zoledronic acid or pamidronate to prevent long term skeletal complications. In the case of a patient with chronic symptomatic hypercalcemia of malignancy that is refractory to bisphosphonate therapy, denosumab, a monoclonal antibody which inhibits bone resorption via inhibition of receptor activator of nuclear factor kappa-B ligand (RANKL) can be used. In addition, denosumab is an option for patients in whom bisphosphonates are contraindicated due to severe renal impairment, as it is not renally excreted.

A patient with irritable bowel syndrome complains mainly of lower abdominal pain. She denies diarrhea or constipation, and rarely has problematic flatulence. For this patient's abdominal pain, which of the following medications do you recommend? Desipramine Loperamide Lubiprostone Rifaximin

Desipramine Irritable bowel syndrome is considered an intestinal motility disorder in which chronic diarrhea is the main symptom. Associated symptoms include abdominal pain, bloating, flatulence and constipation. Some dysmotility causes include improper secretion due to luminal or environmental stimuli, decreased pain thresholds (hence, increased pain sensitivity) and dysregulation of the brain-intestinal axis. Rome IV diagnostic criteria are based on symptom presence on at least one symptom, on average, one day per week over the past 3 months plus 2 or more of the following: defecation decreases symptoms, a change in the frequency of stooling or a change in the shape of stool. Treatment regimens are geared toward which of the symptoms a patient with irritable bowel syndrome suffers from most: pain, diarrhea, bloating or constipation. The pain associated with irritable bowel syndrome can be treated with selective-serotonin reuptake inhibitors like citalopram, tricyclic antidepressants like desipramine, and antispasmodics such as atropine, hyoscyamine, dicyclomine or scopolamine.

A patient is diagnosed with central diabetes insipidus. Which of the following medications should you prescribe? Carvedilol Demeclocycline Desmopressin Metformin

Desmopressin Diabetes insipidus is a disorder of antidiuretic hormone (ADH) function that leads to excessive urination and thirst. Treatment is geared at monitoring, replacing lost fluid and ADH analogues. Most patients are able to take in enough oral fluids to replace that lost in the urine. If dehydration is prominent, intravenous fluids may be necessary, and are usually given at 500-750 ml/h. If diabetes insipidus is found to be of the central type, the drug of choice is desmopressin (DDAVP), an ADH analogue. It can be given orally, intranasally, intravenously or subcutaneously. Electrolyte and fluid status monitoring is still important in those taking desmopressin. Alternatives to desmopressin include chlorpropamide, the thiazides and carbamazepine. A low protein, low sodium diet may also help in decreasing urinary output.

A patient presents with excessive urination and thirst. A comprehensive metabolic panel is normal. Urinalysis is significant for a low specific gravity. Which of the following is the most appropriate next test to order for the evaluation of these symptoms? Cosyntropin stimulation test Desmopressin stimulation test Dexamethasone suppression test Octreotide suppression test

Desmopressin stimulation test Diabetes insipidus is a disorder of antidiuretic hormone (ADH) function, either production and release (central) or action (nephrogenic). Excessive urination (polyuria) and extreme thirst (polydipsia) are the main symptoms. These symptoms also occur in diabetes mellitus (DM). However, in DM, the urine typically contains high levels of glucose or protein, whereas in DI there is no component of glucose dysregulation and the urine is free of glucose and protein. DI commonly presents with the signs of dehydration and hypernatremia. If DI is suspected in a patient who presents with polydipsia and polyuria, the initial work-up is used to distinguish DI from other causes. At the minimum, this includes a serum glucose, bicarbonate and calcium level, which typically can be covered with ordering a comprehensive metabolic panel. In DI, urinalysis will show low specific gravity, low osmolarity and dilute color. Once other causes, namely diabetes mellitus, are ruled-out, a fluid deprivation test or desmopressin stimulation test is used to distinguish between the different types of diabetes insipidus.

Type 1 diabetes mellitus is the result of which of the following? Destruction of alpha cells in the islets of Langerhans Destruction of beta cells in the islets of Langerhans Destruction of delta cells in the islets of Langerhans The presence of Leydig cells

Destruction of beta cells in the islets of Langerhans The islets of Langerhans in the pancreas contain beta cells that are responsible for producing insulin. Type 1 diabetes mellitus occurs when these cells are destroyed, either through an autoimmune process (type 1A) or a non-autoimmune process (type 1B). Individuals are genetically susceptible to this disorder which is thought to be initiated by environmental triggers. The genetic markers for diabetes mellitus type 1 are present from birth. Type 1 diabetes mellitus is different from type 2 diabetes mellitus. In type 2, decreased insulin release and insulin resistance are responsible for the disorder. Treatment for type 1 diabetes mellitus involves following a proper diet and exercise plan coordinated with insulin replacement therapy.

A 2-year-old boy presents to the Emergency Department with his grandmother due to concern for dehydration. He has had two wet diapers and eight watery, malodorous, green stools in the past 24 hours. He has been extremely thirsty and drinking plenty of water and juice. Serum chemistries are significant for a sodium 151 mEq/L, chloride 117 mEq/L, potassium of 3.0 mEq/L, and bicarbonate of 19 mEq/L. Urinalysis is significant for 1+ ketones but is otherwise normal. What is the most likely cause of this child's hypernatremia? Diabetes insipidus Diarrhea Salt poisoning Water deprivation

Diarrhea The child presents with dehydration and labs significant for hypernatremia, hyperchloremia, hypokalemia, and an anion gap metabolic acidosis. Hypernatremia may result from one of two broad categories: excessive water loss or excessive salt intake. This child's history is significant for poor urine output in conjunction with diarrhea. Diarrhea results in excessive free water loss if it is hypotonic. The hypernatremia may be augmented if diarrheal losses are primarily replaced with hypotonic solutions such as water, soda, or juice. Diarrhea also results in potassium losses, as well as loss of bicarbonate. Of note, the metabolic acidosis of diarrhea alone typically does not result in an elevated anion gap. However, dehydration may cause lactic acidosis, which causes an anion gap metabolic acidosis. The child's hyperchloremia is likely a result of compensation for his bicarbonate losses.

A 23-year-old man presents to the emergency department via ambulance from the scene of a motor vehicle collision in which he was an unrestrained passenger. The patient was intubated at the scene due to a Glasgow Coma Scale score of 4. On arrival to the emergency department, the patient has an oxygen saturation of 92% intubated, HR of 90 bpm, T of 98°F, and RR of 13/min with the ventilator. The patient is assessed for other injuries via the Acute Trauma Life Support protocol with no acute findings. He is in a cervical collar. Noncontrast head CT is obtained in the emergency department and shows no skull fracture but does show multiple cerebral contusions. After several days in the ICU, the patient is not improving. He remains intubated and unresponsive, except to painful stimuli. What is the suspected diagnosis based on the patient's symptoms? Diffuse axonal injury Epidural hematoma Intraventricular hemorrhage Subarachnoid hemorrhage Subdural hematoma

Diffuse axonal injury Traumatic brain injury is defined as an alteration in brain function due to an external force. The force could be from the patient's head being struck by an object, the head striking an object, penetration of the brain, acceleration/deceleration injury, or other forces. Risk factors include having a lower socioeconomic status, alcohol and drug use, and underlying psychiatric or cognitive disorders. Younger age and increased age are also risk factors. A traumatic brain injury can be categorized in many different ways, most commonly by using the Glasgow Coma Scale (GCS). The GCS categorizes injury based on eye opening, motor, and verbal response. A score of 13-15 is mild injury, 9-12 is moderate injury, and 8 or less is severe injury. Diagnosis of a mild traumatic brain injury can be made clinically after a patient has head trauma, and the patient may have memory impairment, confusion, nausea, and dizziness but no neurological deficits. Imaging is not usually needed. Treatment includes observation and supportive care with close follow-up. Moderate and severe traumatic brain injury are diagnosed from physical exam, GCS score, and findings on imaging with CT head. Treatment includes possible intubation based on chest rise, oxygen saturation, and GCS score. Management of blood pressure and oxygen saturation are important, as well as involvement of the neurosurgical team. Diffuse axonal injury, considered a moderate to severe traumatic brain injury, occurs from shearing forces along white matter tracts causing focal cerebral contusions and hematomas.MRI is the most diagnostic, but CT can also aid in the diagnosis. As noted above, multiple cerebral contusions will be present. It can also be characterized by coma without focal lesion with axonal damage present in multiple regions of the brain parenchyma. Treatment is focused on managing vital signs, glucose, and intracranial pressure. The damage to the brain with diffuse axonal injury is often irreparable, and the prognosis is usually poor.

You are examining an afebrile 78-year-old woman in the emergency department. During cardiac examination, you auscultate a low-intensity, low-pitch extra heart sound, which occurs in early diastole. You do not hear any murmurs. Her ECG appears normal. Which of the following is the most likely diagnosis? Bacterial endocarditis Dilated cardiomyopathy Right bundle branch block Tricuspid stenosis

Dilated cardiomyopathy The third heart sound (S3) is a low-frequency, brief vibration occurring in early diastole at the end of the rapid diastolic filling period of the right or left ventricle. It is best appreciated at the apex in the left lateral position. It may be a normal variant in patients younger than age 40. After age 40, the presence of an S3 is usually abnormal and correlates well with ventricular dysfunction, namely volume overload. However, there are many causes of ventricular dysfunction: dilated or ischemic cardiomyopathy, conduction abnormalities, left-to-right intracardiac shunts, ischemic heart disease, myocarditis, or valvular regurgitation. High output states, such as anemia, thyrotoxicosis, or pregnancy, also are causative

A 7-year-old boy presents to the emergency department with cough and chest pain. The cough has been present for several days, but the chest pain has been present for only one day. His vital signs are notable for a temperature of 38.8°C, heart rate of 150, blood pressure of 78/49, respiratory rate of 50, and oxygen saturation of 94%. He is ill appearing, has coarse breath sounds on lung auscultation, a systolic murmur and a gallop on his cardiac exam, and his liver is palpable 4 centimeters below the right costal margin. His pulses are palpable and capillary refill is 5 seconds. Which of the following is the most likely complication? Aortic valve insufficiency Constrictive pericarditis Dilated cardiomyopathy Left ventricular outflow tract obstruction

Dilated cardiomyopathy This patient has acute systolic dysfunction in the setting of an upper respiratory infection, which is most likely secondary to myocarditis. Myocarditis can be caused by infections, connective tissues disease, genetic diseases, toxins, and granulomatous diseases. The most common viral etiologies are enterovirus, adenovirus, parvovirus, EBV, and CMV. Myocardial damage may occur from inflammation and direct damage to the myocardium. This results in ventricular enlargement, poor systolic function, and dilated cardiomyopathy. The systolic dysfunction may result in congestive heart failure, arrhythmias, and even death. Patients typically present with chest pain, fever, and fatigue, and in fulminant disease they may have respiratory distress, tachycardia, hypotension, a gallop rhythm, and a murmur. Rash and end-organ involvement (i.e. hepatitis) may be seen. Dilated cardiomyopathy is the most common type of cardiomyopathy, and is characterized by left ventricular systolic dysfunction and left ventricular dilation. In pediatrics, it is usually caused by genetic mutations, myocarditis, or Kawasaki disease.

A 17-year-old girl with a history of well-managed cystic fibrosis is being evaluated for a steadily worsening chronic cough with shortness of breath and wheezing. She is producing copious purulent malodorous sputum and occasional hemoptysis. Crackles are heard at her bilateral lung bases. Which of the following findings would be most expected on this patient's chest radiograph? Dilated, thickened bronchi with "tram-track" marks Low lung volumes and ground glass opacities Lung hyperinflation with flattening of the diaphragm Normal chest radiograph

Dilated, thickened bronchi with "tram-track" marks Dilated, thickened bronchi are classic radiograph findings in patients with bronchiectasis, the most likely disease in this patient. The bronchi are often described as having "tram-track" or ring-like markings. Scattered, irregular opacities, atelectasis, or focal consolidations may also be noted. A suspicion of bronchiectasis based on radiographic findings and patient symptoms warrants a high-resolution CT scan which is a more precise diagnostic study. Bronchiectasis is suspected in patients who have a chronic cough, dyspnea, wheezing, and heavy production of purulent, foul-smelling sputum. Pleuritic chest pain, weight loss, and anemia are commonly associated. An exam usually reveals crackles at the lung bases and nail clubbing in severe disease. In the United States, the greatest percentage of patients with bronchiectasis develop it secondary to cystic fibrosis. Lung infections, tumor presence, and immunodeficiency states are other less common causes. Antibiotics are usually needed with the choice being guided by sputum cultures. Haemophilus influenza, Streptococcus pneumonia, and Staphylococcus aureus are often isolated. In addition to proper antibiotic coverage, bronchiectasis should be treated with daily chest physiotherapy and inhaled bronchodilators.

A 42-year-old man with no significant medical history and no known allergies presents to the emergency department with concerns of a tick bite that occurred earlier this morning, about 8 hours ago. The patient reports that he was on a hike with his family in upstate New York when he noticed the tick on his left calf. His vital signs are a HR of 89 bpm, RR of 19/min, oxygen saturation on room air of 99%, BP of 132/88 mm Hg, T of 97.8°F, and BMI of 29.6 kg/m2. On physical exam, there is an attached tick to his left calf that is then removed. There is no surrounding erythema or warmth to the affected area on exam. Which of the following is the most appropriate management for this patient's condition? Discharge with a 10-day course of doxycycline Discharge with a single dose of ciprofloxacin Discharge with a single dose of doxycycline Discharge with a single dose of doxycycline and obtain Lyme disease serology Discharge without prophylaxis

Discharge without prophylaxis The patient in the above vignette had a recent tick bite from a high-risk Lyme disease area. Lyme disease is an infection caused by Borrelia burgdorferi, which is a gram-negative spirochete that is spread by Ixodes (deer) ticks. Most cases in the United States occur from the mid-Atlantic, northeastern, and north central regions of the country. There are three stages of Lyme disease infection based on early or late manifestations of the disease. Stage 1, early localized infection, presents most commonly with erythema migrans, which is an erythematous rash with central clearing, commonly referred to as a bull's eye rash. This can also be accompanied with viral-like symptoms such as body aches, fatigue, headache, and fevers. Stage 2, early disseminated infection, presents with either neurologic or cardiac manifestations. Patients may present with aseptic meningitis with a headache and neck stiffness or they may present with facial palsy due to cranial nerve VII neuropathy. Cardiac presentations consist of atrioventricular (AV) block. Lastly, stage 3, late persistent infection, occurs months or even years after the initial infection. At this stage, the patient would have ongoing musculoskeletal, neurological, and skin conditions. Prophylactic treatment for patients with a known tick bite should be given if all of the following criteria are met: a deer tick has been attached for at least 36 hours, prophylaxis treatment would be started within 72 hours from the removal of the tick, more than 20% of the ticks from that area are known to be infected with B. burgdorferi, and there are no contraindications for using doxycycline, which may include pregnancy, children under 8 years old, and known allergy. These cases need to be based on shared decision making. Regarding the patient in the above vignette, the tick was removed within 8 hours of the bite, and thus he does not need prophylactic treatment. Therefore, the patient should be discharged from the ED without prophylactic treatment. If the patient had met all the above criteria for prophylactic treatment, then a single dose of doxycycline would be indicated. Lastly, if the patient presented in stage 1 of a Lyme disease infection, then the appropriate treatment would be doxycycline 100 mg twice a day for 10-14 days.

A 67-year-old man with diabetes mellitus, chronic lower back pain and previously stable systolic heart failure now has increasing orthopnea and shortness of breath. Physical exam reveals pulmonary crackles, jugular venous distention and lower extremity edema. He is on carvedilol, lisinopril, furosemide, insulin and ibuprofen. Which of the following is appropriate for management of this patient? Add a calcium channel blocker Decrease the dosage of furosemide Discontinue the carvedilol Discontinue the ibuprofen

Discontinue the ibuprofen Non-steroidal anti-inflammatory drugs (e.g. ibuprofen) can worsen symptoms of heart failure. In fact, a number of medications that are in common clinical use are relatively or absolutely contraindicated in patients with heart failure. This is either because they can cause exacerbations of heart failure or because there is a higher risk of adverse events in such patients. Mechanisms by which some of these drugs can exacerbate heart failure relate to sodium retention, negative inotropic effects or direct cardiotoxicity. Non-steroidal anti-inflammatory drugs are associated with an increased risk of heart failure exacerbation, increased renal dysfunction and impairment of the response of angiotensin converting enzyme inhibitors and diuretics. Observational data has reported an association with non-steroidal anti-inflammatory drugs and increased mortality in patients with heart failure.

A 72-year-old man is being observed by your clinic for benign prostatic hyperplasia. His symptoms of a weak stream, straining, and post-void dribbling are worsening and he would like to begin medical treatment. You initiate tamsulosin and advise him of which of the following most likely side effect? Decreased libido Dizziness Erectile dysfunction Hypertension

Dizziness Dizziness is commonly caused by tamsulosin, an alpha-blocker that offers relatively quick improvement of symptoms in benign prostatic hyperplasia (BPH). BPH affects 50% of men by age 60 and is a common cause of obstructive voiding symptoms. These can include hesitancy and straining to urinate, decreased force and caliber of the urine stream, and the need to urinate twice within 2 hours due to incomplete emptying. Post void dribbling can occur. Irritative bladder symptoms, including urgency, frequency, and nocturia, may also be present. A distended bladder may be palpated, and a rectal exam will usually show some degree of smooth, firm prostate enlargement. The American Urological Association symptoms index should be used to gauge severity of BPH and guide treatment. Recurrent urinary tract infections, gross hematuria, bladder stones, or kidney disease merit surgical resection and require referral to urology. Mild symptoms can be observed. Moderate to severe symptoms may merit medical therapy, for which several options exist. Alpha-blockers, like prazosin and tamsulosin, will improve urine flow by improving urine flow at the bladder neck. However, side effects include hypotension, dizziness, headache, and tiredness. Additionally, finasteride, a 5-alpha reductase inhibitor, blocks the conversion of testosterone to active dihydrotestosterone. Because of this, it can effectively decrease the size of the prostate and improve symptoms within 6 months. Due to this mechanism of action, finasteride is only helpful for reducing symptoms in patients with an enlarged prostate. Side effects may include decreased libido and erectile dysfunction. Phosphodiesterase-5 inhibitors, like tadalafil, offer improvement in urinary function as well as some improvement of erectile dysfunction. Combination therapy is usually beneficial and tolerated, and it has been proven to slow progression of the disease significantly more than either drug alone. Decreased libido (A) and erectile dysfunction (C) are more likely to be due to treatment with 5-alpha-reductase inhibitors, like finasteride. These side effects are due to the impaired conversion of testosterone to dihydrotestosterone, which does not occur with the use of an alpha-blocker. Hypertension (D) is not a likely side effect of alpha-blockers, as these medications typically cause a decrease in blood pressure, as opposed to an increase.

A 23-year-old man presents with 2 days of rice water diarrhea after a recent trip to India. What is the appropriate pharmacologic treatment for his condition? Amoxicillin Cephalexin Doxycycline Piperacillin

Doxycycline Cholera is an acute form of diarrhea caused by toxin-forming strains of the gram-negative bacterium Vibrio cholerae, which is transmitted via the fecal-oral route. It is endemic in certain regions, such as Asia and Africa, and cases in the U.S. are most often imported via travelers such as in this patients case. Isolation of V. cholerae in a stool sample establishes a definitive diagnosis, but providers may make a presumptive diagnosis of cholera based on its clinical presentation. The diarrhea produced by cholera is frequently described as having the consistency of rice water, and may be profuse, leading to dehydration. Administration of appropriate rehydration, therefore, is the cornerstone of treatment. However, antibiotic treatment has been shown to reduce the duration and quantity of diarrhea, as well as decrease the shedding of V. cholerae. Appropriate antibiotic selection for cholera may include doxycycline, with ciprofloxacin, azithromycin, or tetracycline as reasonable alternatives depending on local patterns of susceptibility and resistance.

A 6-year-old boy presents to your office with a 5-day history of fever and headache. He denies any changes in vision or nuchal rigiditiy. He denies any international travel but he does mention that he went hiking in Tennessee one week ago. Physical exam reveals a nonpruritic, small, maculopapular rash noted on the arms and wrist that began yesterday. Laboratory values are significant for thrombocytopenia and hyponatremia. Which of the following is the first-line treatment? Amoxicillin Chloramphenicol Corticosteroids Doxycycline

Doxycycline Rocky Mountain spotted fever (RMSF) is a tick-borne disease caused by the organism Rickettsia rickettsii. The classic triad of symptoms associated with the disease includes fever, headache, and rash in a person with a history of tick bite or exposure to endemic areas in the Southeast United States. The rash appears two to five days after the onset of fever in the majority of cases. The rash begins as small, blanching, nonpruritic macules that develop into a maculopapular rash and progress after several days to petechial lesions. It begins around the wrists and ankles and spreads to the palms and soles, eventually covering the extremities and trunk. A rash on the palms and soles is a distinct distribution that occurs in approximately 40% of patients with Rocky Mountain spotted fever. In patients with severe disease in whom treatment is delayed, complications include cardiac and renal failure, meningitis, and pneumonitis. Common laboratory manifestations include thrombocytopenia and hyponatremia. Doxycycline therapy also treats Lyme disease, ehrlichiosis, and relapsing fever; differential diagnoses often clinically confused with Rocky Mountain spotted fever. Doxycycline became the preferred drug choice for treating children of any age because of the potential for severe or fatal cases. Short courses of doxycycline to treat Rocky Mountain spotted fever do not cause significant dental staining.

A 15-year-old girl presents to clinic with vaginal discharge. She recently became sexually active and states that her partner was recently diagnosed with gonorrhea. Speculum examination reveals white discharge and an erythematous cervix. Bimanual examination is unremarkable. A urine pregnancy test is negative. Which of the following is the most preferred treatment option? Azithromycin Azithromycin and metronidazole Ceftriaxone, amoxicillin, and metronidazole Doxycycline and ceftriaxone

Doxycycline and ceftriaxone The most common manifestation of symptomatic chlamydial infection in females is cervicitis and in males, urethritis. Infection may be asymptomatic or may result in vaginal or urethral discharge. The treatment of choice for C. trachomatis cervicitis in non-pregnant women is seven days of doxycycline. Alternatively, a single dose of one gram of azithromycin can be used, and this regimen is preferred in pregnant patients. Sexual partners should be treated, and each partner should abstain from sexual activity for one week. Follow-up testing is not recommended unless non-compliance is suspected, symptoms persist or return, or the patient is pregnant. Patients diagnosed with cervicitis or urethritis should also be treated for assumed N. gonorrhoeae co-infection with 500 mg ceftriaxone IM unless definitively ruled out.

A 49-year-old man presents to his gastroenterologist's office for a follow-up on his abdominal pain and recent diagnosis of a gastrointestinal bleed. For the past few weeks, the patient has had intermittent epigastric pains, which he has correlated to occur a few hours after eating, and at times, he has even been woken up at night. The pain improves after he eats a snack. His vitals today are heart rate of 90 bpm, respiratory rate of 17/minute, oxygen saturation on room air of 99%, and blood pressure of 124/82 mm Hg. The patient had a recent positive fecal occult blood test and was scheduled for an endoscopy and colonoscopy. Both procedures were completed 2 days ago. There were no acute findings on the colonoscopy, but on the endoscopy, the presence and location of ulcers were confirmed. The patient was told the ulcers were unlikely to be malignant, given their location. Where is the most likely location of this patient's ulcer? Duodenum Esophagus Rectum Sigmoid colon Stomach

Duodenum This patient has peptic ulcer disease, most likely located in the duodenum. Peptic ulcer disease is caused by a change or imbalance of the mucosal lining by either an increase in aggressive luminal factors, such as acid or pepsin, or by a decrease in mucosal protective factors. The most common causes of peptic ulcers are Helicobacter pylori infections or chronic or heavy use of NSAIDs. There are two forms of peptic ulcer disease, which are characterized by their location. The first form results from stomach or gastric ulcers, which commonly occur between the ages of 55 and 70 years old. The second form results from duodenal ulcers, which is what the patient in the above vignette has. These ulcers most commonly present between the ages of 30 and 55 years old. Gastric ulcers have a 4% chance of malignancy, whereas duodenal ulcers are almost always benign. The most common presenting symptom for peptic ulcer disease is intermittent epigastric abdominal pain, commonly described as dull, aching, or hunger-like. The pain for gastric ulcers is commonly noted to occur immediately after eating and is worse with food. However, duodenal ulcers lead to pain about 1-2 hours after eating and the pain is relieved with food. The patient's physical exam can be normal, or they might present with mild localized epigastric tenderness. It is common for patients to have a positive fecal occult blood test. The gold standard for diagnosis is an upper endoscopy. Treatment depends on the cause of the ulcers. If the patient has a known diagnosis of H. pylori, treatment entails a triple therapy with clarithromycin, amoxicillin, and a proton pump inhibitor. If no H. pylori is detected, a proton pump inhibitor or H2 receptor blocker is appropriate.

A 38-year-old woman presents to the emergency department with right upper extremity swelling and erythema along her inner arm, which she noticed a few hours ago. She reports that she has just finished treatment for colon cancer and had her peripherally inserted central catheter line removed from her right upper arm 3 days ago. On her physical exam, she has a palpable cord-like inflammation of her inner right upper extremity with tenderness and erythema above the peripherally inserted central catheter line site. No purulent discharge is noted. Her right upper arm is visually larger than her left arm. Her vital signs show regular heart rate and temperature. Which of the following is considered the most important next step in this patient's care? Duplex ultrasound Intravenous antibiotics MRI venography Right upper extremity X-rays Warm compresses and NSAIDs

Duplex ultrasound Venous thrombosis is the formation of a blood clot in a vein. If the blood clot or thrombus is in a deep vein, there is a risk that it will travel to the lungs causing a pulmonary embolism which can be fatal. It is important to accurately and quickly identify a deep vein thrombosis (DVT) in order for treatment to be initiated. The major risk factors include history of previous blood clots, venous stasis, immobility, pregnancy, malignancy, and recent trauma or surgery. Patients with these risk factors and symptoms of extremity swelling, pain, warmth, and erythema will need a duplex ultrasound to diagnose. Identification and treatment of a DVT is vital to prevent a resulting pulmonary embolism or possible post-thrombotic syndrome of the involved extremity. The next step in this patient's evaluation and treatment is duplex ultrasound due to a high suspicion of upper extremity DVT. This patient is considered high risk for DVT due to her hypercoagulable state caused by malignancy and recent peripherally inserted central catheter (PICC) line. PICC lines can cause upper extremity DVT, as the catheter can damage the vein wall, causing inflammation and trauma, which can cause thrombus formation in the vein. Treatment is with anticoagulation such as direct oral anticoagulants or warfarin depending on patient's past medical history, kidney function, and compliance.

A 65-year-old smoker with no documented cardiac disease presents with several months of worsening cough and exertional dyspnea. He denies exposure to inhalational toxins or dusts. Examination reveals bibasilar inspiratory crackles and fingernail clubbing. Spirometry reveals a restrictive lung pattern. A chest computed tomography scan is read as "honeycombing in the periphery and bases." A bronchoalveolar lavage report is only significant for lymphocytosis and a histological classification of usual interstitial pneumonia. Which of the following is the most likely diagnosis? Chronic obstructive pulmonary disease Community-acquired pneumonia Idiopathic pulmonary fibrosis Pneumoconiosis

Idiopathic pulmonary fibrosis Nomenclature of restrictive pulmonary disease can be confusing. It can be simplified if one first dissects the lung into airspace (mostly obstructive diseases), parenchyma (the lung "meat" which doesn't contain air, aka interstitium) and chest wall (the mechanism that runs the airspaces and parenchyma). Restrictive disease encompasses abnormalities of the parenchyma and chest wall. Idiopathic pulmonary fibrosis (IPF) represents a main subclassification of the more than 200 interstitial lung diseases. IPF is a chronic, progressive, and often fatal, interstitial lung disease which affects adults over 50 years of age (men > women and smokers > non-smokers). Although linked to tobacco, occupational toxin exposure, gastroesophageal reflux, and genetic predisposition, the exact etiology remains unknown, hence the term idiopathic. It is felt the pathology is based on abnormal wound healing, oxidant-antioxidant imbalance, and excessive deposition of collagen. Patients experience a progressive dry cough, exertional dyspnea, inspiratory "velcro-like" crackles, and fingernail clubbing. Radiography is nonspecific, but high resolution computed tomography reveals the diagnostic findings of fibrotic changes referred to as "honeycombing" (small cystic spaces enveloped by thick, fibrotic, well-defined walls, mainly occurring in the periphery and bases). Bronchoalveolar lavage (BAL) is an important diagnostic test, however, it is mainly used to rule-out other pulmonary disease such as malignancy and infections. In IPF, the tissue obtained from lung washings will show a histological appearance of usual interstitial pneumonia, the classic pathological description of IPF.

A 25-year-old woman presents to the emergency room with a fever of 104°F complaining of chills, right back and side pain, as well as frequency and pain on urination. She reports being sexually active for the first time the previous week, without the use of a condom. History is otherwise unremarkable. Physical examination is that of a well-developed and well-nourished young female in mild to moderate distress. Vital signs are pulse is 94, respiration is 24, blood pressure is 100/60, and temperature is 103.5°F. There is significant right CVA tenderness and tenderness over her bladder. Gynecologic exam demonstrates erythema of the vaginal and urethral mucous membranes and scant purulent urethral discharge. There is no abdominal, uterine, or fallopian pain nor pain on lateral motion of the cervix. What is the most likely diagnosis? Candida vaginitis E. coli urinary tract infection N. gonorrheae cervicitis Pelvic inflammatory disease

E. coli urinary tract infection This young woman has an E. coli urinary tract infection (UTI). E. coli is the etiology of the majority of UTI's, reflecting the colonization of the lower gastrointestinal tract and the ascending nature of many UTI's. It is not unusual for the first UTI in a female to occur at the time that she is sexually active for the first time giving rise to the name "honeymoon cystitis." In this case, there is evidence that both the upper and lower urinary tracts are involved because of the high-grade fever, chills, and CVA tenderness as well as the elevated pulse and respiratory rates and the slightly low blood pressure.

A 72-year-old man with a history of chronic tobacco use presents to clinic with a complaint of increasing dyspnea while walking to his mailbox. Physical examination reveals decreased breath sounds, hyperresonance to percussion, and a barrel chest. A chest X-ray is obtained that demonstrates hyperinflation with flattened diaphragms. What is the most likely diagnosis? Acute bronchitis Asthma Chronic bronchitis Emphysema

Emphysema Emphysema is a subset of chronic obstructive pulmonary disease and is characterized by severe dyspnea with or without exertion, decreased breath sounds on auscultation, and increased resonance to percussion due to hyperinflation. It occurs when there is a permanent abnormal enlargement and destruction of the airspaces distal to the terminal bronchioles. Airflow obstruction is irreversible. Patients are typically older and thin and have a past medical history of chronic tobacco use. As the disease progresses, they begin to breathe with pursed lips and develop a barrel chest. Pulmonary function tests with spirometry are used to determine the severity of airflow limitation and to monitor progression of the disease.

Which of the following findings is most suggestive of Pneumocystis jirovecii pneumonia in a patient with suspected HIV infection and shortness of breath? Bradycardia despite relative volume depletion Elevated serum lactate dehydrogenase Hyponatremia Unilateral lobar consolidation on chest radiograph

Elevated serum lactate dehydrogenase Pneumocystis pneumonia is one of the most common opportunistic infections in patients with HIV. Patients typically present with an insidious nonproductive cough, dyspnea, unexplained fever for longer than two weeks, chest pain, and fatigue. The greater the elevation of lactate dehydrogenase (LDH), the worse the prognosis.

Which of the following tests is diagnostic for primary hypothyroidism? Elevated thyroid-stimulating hormone, high free thyroxine, high free triiodothyronine Elevated thyroid-stimulating hormone, low free thyroxine, low free triiodothyronine Low thyroid-stimulating hormone, normal free thyroxine, high free triiodothyronine Low thyroxine-binding globulin

Elevated thyroid-stimulating hormone, low free thyroxine, low free triiodothyronine An elevated TSH level with low free T3 and T4 is diagnostic for primary hypothyroidism. Hypothyroidism is a condition of decreased production or secretion of thyroid hormone from the thyroid gland. It can result from decreased stimulation of the thyroid gland (central or secondary hypothyroidism) or gland dysfunction (primary hypothyroidism). It is a common endocrine disorder and is 4-5 times more common in women than in men. Although there are numerous causes of the disease and varied presentations, an elevated TSH is the most sensitive and best screening test in the diagnosis of primary hypothyroidism. In primary hypothyroidism, lack of thyroid hormone production leads to decreased negative feedback from the thyroid to the pituitary gland. The pituitary gland, in turn, increases production of TSH in an attempt to increase the thyroid gland's hormone production. A high TSH with a low T4 level is indicative of primary hypothyroidism, while the presence of a low TSH and low T4 level is indicative of central hypothyroidism.

A 64-year-old man with a history of diabetes mellitus on metformin and a previous non-ST elevation myocardial infarction presents to the clinic to follow up on his diabetes management. Vital signs today include a heart rate of 80 bpm, blood pressure of 120/80 mm Hg, respiratory rate of 20/minute, oxygen saturation of 98% on room air, and a temperature of 98.6°F. Physical examination reveals a regular cardiac rate and rhythm and lungs that are clear to auscultation. The patient's laboratory findings include a creatinine of 1.2 mg/dL and a hemoglobin A1C of 7.7%. Which of the following medications would decrease cardiovascular morbidity and mortality in this patient? Empagliflozin Glimepiride Glipizide Linagliptin Repaglinide

Empagliflozin Sodium-glucose cotransporter 2 (SGLT2) inhibitors, such as empagliflozin and canagliflozin, are a class of medication used to lower blood glucose in patients with diabetes mellitus. This medication class works by blocking the reabsorption of glucose in the proximal tubules of the kidneys, which leads to increased renal excretion of glucose. Studies have shown that in patients who have both diabetes mellitus and cardiovascular disease (coronary artery disease, cerebrovascular disease, or peripheral vascular disease), empagliflozin and canagliflozin reduce cardiovascular morbidity and mortality. Empagliflozin is often preferred to canagliflozin because lower limb amputations and fractures occurred more often in the canagliflozin branch of the clinical trial. SGLT2 inhibitors are one of the pharmacologic classes available for use when a patient's blood glucose is not adequately controlled with lifestyle modifications and metformin. Possible adverse effects of SGLT2 inhibitors include an increased risk of yeast infections, increased risk of urinary tract infections, and hypotension. They should not be used in patients with an estimated glomerular filtration rate < 45 mL per minute. SGLT2 inhibitors do not cause hypoglycemia unless used with another agent that causes hypoglycemia.

A 45-year-old woman presents to her primary care provider because of recurrent epigastric pain that usually occurs after heavy meals and with sitting in a reclined position. She reports transient relief when she sits upright or drinks milk. She denies weight loss, early satiety, dysphagia, and melena. Physical examination is notable for obesity, but is otherwise unremarkable. Which of the following is most appropriate next step? Ambulatory 24-hour pH monitoring Empiric trial of omeprazole Fundoplication Upper endoscopy

Empiric trial of omeprazole An empiric trial of omeprazole or other proton pump inhibitor (PPI) is the most appropriate next step in the management of uncomplicated gastroesophageal reflux disease (GERD). Gastroesophageal reflux disease is defined as symptomatic reflux of gastric juice into the esophagus without associated esophageal mucosal injury. GERD is an extremely common disorder in the United States, affecting 10-20% of certain populations. In the majority of patients with GERD, most reflux episodes are the result of transient relaxations of the lower esophageal sphincter (LES). Transient relaxation of the LES can be caused by foods (e.g. coffee, alcohol, chocolate, fatty foods), medications (e.g. beta-agonists, calcium channel blockers, nitrates), hormones (e.g. progesterone), and nicotine. Other risk factors include obesity, gastroparesis, and hiatal hernia. Heartburn and regurgitation are the most common symptoms of GERD. Heartburn associated with GERD is typically described as retrosternal burning that occurs in the postprandial period. Heartburn is typically exacerbated by reclining and relieved with antacids. Less common symptoms or atypical symptoms of GERD are wheezing, chronic cough, or chest pain. Initial diagnostic studies are unwarranted in patients who present with typical symptoms of GERD and who do not have any alarm features such as weight loss, dysphagia, odynophagia, anemia, or recurrent vomiting. Upper endoscopy should be performed in patients with alarm or atypical symptom or in patients who do not respond to adequate medical treatment. Lifestyle and dietary modification are recommended in all patients with GERD. Weight loss is beneficial in overweight patients. Medical treatment typically involves antacids, histamine-2 receptor antagonists, and PPIs. PPIs are recommended in patients with GERD symptoms 2 or more times per week or in patients with severe symptoms. Mild and intermittent GERD symptoms are typically managed with a histamine-2 receptor antagonist. Surgical treatment is reserved for patients who fail medical therapy, patients with Barrett esophagus, or in patients with extraesophageal symptoms. Complications of GERD include Barrett esophagus, peptic strictures, and aspiration.

A patient with excessive alcohol use and cirrhosis presents with acute upper gastrointestinal bleeding, hypotension, and tachycardia. His past medical history also includes portal hypertension. In addition to cardiopulmonary stabilization and blood transfusion, which of the following is the most appropriate initial emergent interventional treatment? Distal splenorenal shunt Endoscopic band ligation Large volume paracentesis Transjugular intrahepatic portosystemic shunt

Endoscopic band ligation Esophageal varices are dilated submucosal veins which commonly occur in the lower third of the esophagus. They are mainly due to portal hypertension secondary to cirrhosis. They typically present with massive upper gastrointestinal bleeding. Emergency care is based on hemorrhage control, maintaining cardiopulmonary function, and correcting concurrent cirrhosis-induced coagulopathies. Blood transfusions are often necessary. Therapeutic endoscopy with variceal ligation, banding, or sclerotherapy, is considered by many to be the definitive emergent treatment. Octreotide, a somatostatin analogue, can also be used in the emergent care of bleeding esophageal varices. Refractory cases may require balloon tamponade.

A 75-year-old woman with a history of hypertension presents to the ED with right upper quadrant pain that has been gradually worsening over the past day. Her vital signs are T 103°F (39.4°C), BP 100/60 mm Hg, HR 100 bpm, RR 22/min, and SaO2 97% on room air. Physical exam reveals scleral icterus and right upper quadrant tenderness without rebound or guarding. A bedside right upper quadrant ultrasound demonstrates a common bile duct measuring 1 cm. Laboratory results are pending. What is the definitive management of this condition? Broad spectrum antibiotics Cholecystectomy Endoscopic retrograde cholangiopancreatography Percutaneous transhepatic cholangiography

Endoscopic retrograde cholangiopancreatography The patient presents with severe (suppurative) ascending cholangitis and requires biliary decompression with endoscopic retrograde cholangiopancreatography (ERCP). Ascending (or acute) cholangitis is a bacterial infection of the biliary system and is most frequently associated with common bile duct stones and obstruction. The disease classically presents with the Charcot triad (fever, jaundice, and right upper quadrant pain). However, only 50-75% of patients present classically, so clinicians must maintain a high index of suspicion for this life-threatening condition. While administration of broad-spectrum antibiotics is appropriate, those with signs of severe disease, including persistent abdominal pain, hypotension despite adequate fluid resuscitation, fever greater than 102°F (38.9°C), and confusion, warrant urgent ERCP.

A 31-year-old man presents with 10/10 flank pain that radiates to his testicle. He has no significant past medical history. Temperature and vital signs are normal. Examination reveals no edema or skin lesions. Urinalysis is positive for a microhematuria. Further evaluation would most likely reveal which of the following abnormalities? Calcium deposition in the renal parenchyma Enlarged renal pelvis and proximal ureter Glomerular capillary angiopathy Parenchymal hemorrhage, neutrophil casts and suppurative necrosis

Enlarged renal pelvis and proximal ureter Hydronephrosis is the distension of the renal calyces and pelvis due to obstruction of distal urinary flow which commonly accompanies hydroureter. It can occur in both acute and chronic forms, and can affect one or both kidneys. Major childhood causes include congenital ureteropelvic junction abnormalities, urethral valves and urethral stricture. Major adult causes include urolithiasis, benign prostatic hyperplasia and other bladder outlet obstruction, prostate carcinoma, bladder prolapse and retroperitoneal or pelvic neoplasms. Acute hydronephrosis leads to changes in renal function and acute renal failure, while chronic forms cause infection and sepsis, renal scarring, permanent nephron loss and calculous formation. Acute cases typically present with pain (renal or ureteral colic), which alludes to the level of obstruction. Renal pelvic or superior ureteral obstruction causes flank pain, while inferior ureteral obstruction refers pain to the ipsilateral testicle or labia. Severe pain usually is the result of acute complete or partial bilateral stone obstruction, or acute fluid overload in previously asymptomatic and undiagnosed ureteropelvic junction abnormality. Leukocytosis and pyuria suggests an infectious etiology. Hematuria suggests urolithiasis as the cause. Renal ultrasonography is the initial test of choice to diagnose the level of urinary tract obstruction. If unremarkable, intravenous pyelography or CT scanning may be necessary. Treatment is geared at treating the underlying etiology. Nephrocalcinosis (A), most common in premature infants, is a disorder of serum or urine hypercalcemia (hyperparathyroidism) that leads to diffuse, fine calcium deposition in the renal parenchyma. It is related to, but different than, nephrolithiasis. Nephrotic syndrome is the result of some type of glomerular abnormality. In adults, the most common primary cause is focal segmental glomerulosclerosis (glomerular scarring), while one of the most common secondary causes is diabetic nephropathy (glomerular inflammation, angiopathy (C) and scarring). Nephrotic syndrome is characterized by proteinuria, hypoalbuminemia, hyperlipidemia and (typically facial) edema. Pyelonephritis is inflammation of the renal parenchyma, calyces and pelvis commonly due to urinary tract or serum bacterial infection. Patients present with fever, tachycardia, dysuria, nausea and costovertebral angle tenderness. Histopathological samples typically consist of hemorrhage, neutrophil casts and suppurative necrosis (D).

A 58-year-old man presents with bilateral buttock and shoulder pain. Other than hypertension, he has no significant medical history. He states that he has felt "stiff" and "tired" for the past few weeks. He denies any specific injury, but does report being more active in the garage working on his classic automobile. He also denies back pain. Examination reveals no skin or digit abnormalities. Ophthalmologic examination is within normal limits. The gluteal and periscapular muscles are tender to palpation, but there is no discernable trigger point, warmth or induration. Which of the following laboratory abnormalities is most likely present in this patient? Antinuclear antibody positivity Erythrocyte sedimentation rate > 50 mm/h HLA-B27 positivity Sideroblastic microcytic anemia

Erythrocyte sedimentation rate > 50 mm/h Polymyalgia rheumatica (PMR) is an acute vasculitic condition marked by acute proximal muscle pain and stiffness. Age of onset is > 50 years. Areas commonly affected are the muscles of the shoulder and pelvic girdle. Distal extremity muscles are usually spared. Common associated findings are malaise, weight loss, night sweats, morning stiffness and muscle tenderness. There may also be some peripheral edema, low-grade fever and variable areas of proximal bursitis. In other words, signs of the systemic inflammatory response are present. Treatment is corticosteroids, usually prednisone 10-20 mg per day.

A 34-year-old woman with a history of seizures presents to the office reporting symptoms of progressive fatigue for the past 6 months. She is taking phenytoin and has not had any seizures in over a year. Her temperature is 98.7°F, blood pressure is 118/82 mm Hg, heart rate is 92 bpm, oxygen saturation is 98% on room air, and respiratory rate is 20/min. She has some skin pallor and mild gingival hyperplasia on physical exam. Laboratory studies reveal a hemoglobin of 10.0 g/dL and a mean corpuscular volume of 118 fL. Supplementation with which of the following could have prevented this condition? Cobalamin Folic acid Pyridoxine Riboflavin Thiamine

Folic acid Folate deficiency anemia is a type of macrocytic anemia caused by an inadequate level of folic acid within the RBCs and serum. Folic acid is found in most fruits and vegetables, with a daily dietary requirement of 50-100 mcg. It is absorbed throughout the entire gastrointestinal tract, but its stores in the body are limited, and thus inadequate intake over a 3-month period can lead to deficiency. The most common cause of deficiency is inadequate dietary intake, often seen in patients with alcohol use disorder or anorexia. Other causes include decreased absorption, which can be caused by certain medications that interfere with its absorption within the gut, such as phenytoin, methotrexate, trimethoprim-sulfamethoxazole, and sulfasalazine. Folic acid is also poorly absorbed in patients with concomitant vitamin B12 (cobalamin) deficiency. Patients who are pregnant, have hemolytic anemia, or have certain skin diseases require increased folic acid in their diets. They will present with signs and symptoms of anemia, such as fatigue, dyspnea, headache, palpitations, and pallor. The clinical features will be similar to those of B12 deficiency, except that isolated folate deficiency does not result in neurological symptoms. Patients will have a decreased hemoglobin and an elevated mean corpuscular volume (> 100 fL). An RBC folic acid level < 150 ng/mL or serum folic acid level < 2.5 ng/mL confirms the diagnosis. A peripheral smear will reveal megaloblastic anemia with macro-ovalocytes and hypersegmented neutrophils. A serum vitamin B12 level should always be measured since both conditions cause identical megaloblastic anemia. Some cases of folic acid deficiency result from gastrointestinal mucosal atrophy due to vitamin B12 deficiency. This condition is treated with daily oral folic acid supplementation (1 mg). Rapid improvement of symptoms occurs within a few days, with full correction within 2 months of treatment.

A 43-year-old man comes to the clinic complaining of a 3-week history of a "burning" sensation in his chest and nocturnal cough. He says he wakes up several times at night coughing and wheezing. He weighs 350 lbs and has a BMI of 48 kg/m2. If this patient's condition is left untreated, which of the following complications can most likely develop? Esophageal adenocarcinoma Hepatocellular carcinoma Small cell carcinoma of the lung Squamous cell carcinoma of the esophagus

Esophageal adenocarcinoma This patient most likely has gastroesophageal reflux disease (GERD). GERD typically presents with heartburn, regurgitation, noncardiogenic chest pain, and nocturnal cough. Complications from longstanding GERD can result in reflux esophagitis, esophageal strictures, Barrett esophagus (intestinal metaplasia), and esophageal adenocarcinoma. The vast majority of esophageal adenocarcinomas arise from a region of Barrett metaplasia, which is due to GERD. Additionally, in patients who eventually develop Barrett esophagus, their risk of developing esophageal adenocarcinoma increases to at least 30-fold above the general population.

A 52-year-old man comes to the clinic complaining of difficulty swallowing, upper abdominal pain, and heartburn. He says he has a difficult time swallowing solid foods and the food frequently gets "stuck" in his throat. He has a history of food allergies, asthma, and atopic dermatitis. Laboratory studies show markedly elevated serum IgE levels. Upper endoscopy shows stacked circular rings. Which of the following is the most likely diagnosis? Achalasia Esophageal stricture Esophagitis Gastroesophageal reflux disease (GERD)

Esophagitis Dysphagia to solids, upper abdominal pain, GERD-like symptoms (eg, heartburn), food impaction, and a history of allergies suggests eosinophilic esophagitis (EE). EE is an allergic inflammatory condition of the esophagus that primarily involves eosinophils. EE is strongly associated with allergic diseases (eg, food allergies, asthma, atopic dermatitis). Laboratory studies typically show elevated levels of serum IgE. Endoscopic findings typically show stacked circular rings that can be transient or fixed along with white nodules with granularity. Treatment usually consists of dietary modification (avoidance of food allergens), medical therapy (treatment with corticosteroids and other anti-inflammatory agents), and mechanical dilation of the esophagus.

A 68-year-old man presents with complaints of vague abdominal pain, unintentional weight loss, and early satiety for the past two months. Which of the following diagnostic studies is likely to provide a definitive diagnosis? Contrast-enhanced computed tomography scan of the abdomen and pelvis Double-contrast radiography Endoscopic sonography Esophagogastroduodenoscopy

Esophagogastroduodenoscopy Esophagogastroduodenoscopy (EGD) will allow biopsy of suspicious lesions and lends itself to diagnosing gastric cancer. Gastric cancer is most commonly adenocarcinoma, which is a malignant neoplasm of the stomach. The median age at diagnosis is 70 years of age. Stomach cancer is more prevalent in men and more prevalent in those of Japanese or Chinese descent. Risk factors include exogenous sources of nitrates, including foods that are dried, smoked, and salted. The bacteria Helicobacter pylori may contribute to chronic gastritis and loss of acidity which is a risk factor in developing gastric cancer. It is also more prevalent in individuals with a history of tobacco abuse. Gastric cancer can metastasize by local extension through the gastric wall into the omenta, pancreas, colon, or liver. Clinical features include upper abdominal pain that can be vague or severe, postprandial fullness, early satiety, nausea, weight loss, or recurrent vomiting. Physical exam findings can include a palpable, enlarged stomach, or an enlarged liver. Management is dependent on the extent of disease but includes chemotherapy, surgical resection, or radiation.

In which of the following ways does essential tremor differ from the tremor of Parkinson disease? Essential tremor can be treated with dopamine agonists Essential tremor is a pill-rolling tremor Essential tremor is exacerbated by action Essential tremor is unilateral

Essential tremor is exacerbated by action Essential tremor is usually symmetric and exacerbated by action, whereas the tremor of Parkinson disease is usually asymmetric and occurs at rest. Essential tremor affects up to 5% of the general population after the age of 60. It is often inherited in an autosomal dominant fashion. The age of onset may be as early as the first or second decade of life, but senile tremor is frequently delayed until the mid-60s. Patients first become aware of mild postural and action tremor in the hands, which is indistinguishable from an enhanced physiologic tremor, and may result in minimal functional impairment for many years until it gradually progresses. Treatment of essential tremor does not influence the course of the illness and thus is justified only when the tremor interferes with function. At least 50% of patients note improvement or complete amelioration of tremor following the ingestion of a small amount of ethanol. First-line drug treatment includes trials of a non-selective beta-adrenergic blocker (e.g., propranolol) or primidone.

Which of the following medications is considered first line therapy for absence seizure? Carbamazepine Diazepam Ethosuximide Phenytoin

Ethosuximide Absence seizures are sometimes referred to as petit mal seizures. The hallmark of absence seizures is an abrupt and sudden-onset impairment of consciousness, interruption of ongoing activities, a blank stare, possibly a brief upward rotation of the eyes. If the patient is speaking, speech is slowed or interrupted; if walking, he or she stands transfixed; if eating, the food will stop on its way to the mouth. Usually, the patient will be unresponsive when addressed. In some cases, attacks are aborted when the patient is called. The attack lasts from a few seconds to half a minute, and evaporates as rapidly as it commenced. Absence seizures generally are not followed by a post-ictal state. This is in contrast to the majority of seizure disorders. Electroencephalography (EEG) shows generalized spike-and-slow wave discharges. The traditional treatment of absence seizures is with ethosuximide. An alternative agent is valproic acid.

A 30-year-old man presents to clinic with pain in the penis and rectum for two days. He had both oral and anal receptive sex with a new partner one week ago without the use of a condom. He denies penile discharge and dysuria, but did have a low-grade fever for one day prior to the onset of pain. Physical examination shows shallow, painful ulcers on the shaft of the penis and a clustered patch of clear vesicles on the right side of the buttocks adjacent to the anus. No inguinal lymphadenopathy is present. Rapid HIV antibody test is negative. Cultures are obtained from both sites. Which of the following is the most likely diagnosis? Chancroid Genital herpes Lymphogranuloma venereum Primary syphilis

Genital herpes Herpes simplex virus (HSV) is a lifelong viral infection affecting over 15% of individuals in United States. Two types of HSV cause genital herpes: HSV-1 and HSV-2. Most cases of genital herpes are caused by HSV-2, however, there is an increased prevalence of HSV-1 in young women and men who have sex with men.Transmission occurs via contact with lesions, mucosal surfaces, genital secretions, and oral secretions. In addition, herpes simplex virus can be passed from mother to child during pregnancy, childbirth, or in the newborn period, causing complications such as neonatal herpes. The first episode of infection may have more severe signs and symptoms such as fever, generalized myalgias, mucopurulent vaginal discharge, and dysuria. The lesions may be clustered, clear vesicles or shallow, coalescent painful ulcers with an erythematous border depending on the phase of the infection. Recurrent episodes are usually less severe. Individuals may experience mild paresthesias and burning two to five days prior to vesicular eruption. Diagnosis for active ulcers is made via viral culture or herpes simplex virus DNA polymerase chain reaction. Tzanck preparations have a low sensitivity and specificity and are not recommended. Serologic tests are available to detect antibodies to HSV; however, false positives may occur for early, new infections so confirmatory testing is recommended. There are multiple treatments allowing for symptomatic relief and viral suppression. First-line treatment for an initial outbreak is acyclovir 400 mg three times daily for 7-10 days, valacyclovir 1 gram orally twice daily for 7-10 days or famciclovir 250 mg orally three times daily for 7-10 days.

A 63-year-old man with a medical history significant for type 2 diabetes mellitus, hypertension, and chronic kidney disease presents with fatigue and dyspnea on exertion for the past month. A physical examination is unremarkable except for conjunctival pallor. His vital signs are within normal limits except for a HR of 106 bpm. A fecal occult blood test is negative. Laboratory tests are ordered and the results show a creatinine of 2.1 mg/dL, a potassium of 3.8 mmol/L, a hemoglobin of 9.8 gm/dL, and a mean corpuscular volume of 82 fL. Which of the following is the best next step in management of this patient's condition? Begin erythropoietin therapy Evaluate iron stores Order a hemoglobin electrophoresis Refer for emergent dialysis Supplement folate and vitamin B12

Evaluate iron stores In patients with chronic kidney disease that does not require dialysis, anemia is common. The severity of the anemia typically correlates with the severity of the kidney disease. The anemia is primarily secondary to low levels of erythropoietin produced by the kidneys. The red cells are morphologically normal and normochromic although their life span is decreased. Reticulocyte counts will be decreased. Iron stores should be assessed initially to determine if the patient has iron deficiency. Patients with iron deficiency and nondialysis chronic kidney disease need to be treated with iron before the administration of erythropoietin. All patients with chronic kidney disease who have a transferrin saturation level at or below 20% and a serum ferritin concentration at or below 100 ng/mL should be treated with iron. It is important to correct the iron deficiency before administering erythropoietin so that there are adequate stores of iron to create new red blood cells after erythropoietin administration. Patients with anemia in chronic kidney disease should be monitored every 3-6 months to ensure that blood counts remain stable

An 8-year-old girl with a history of sickle cell anemia presents with diffuse pain consistent with an acute sickle cell pain crisis. While in the emergency department, she develops acute onset headache, right-sided facial droop, and right arm weakness. A CT scan confirms the diagnosis. Which of the following is the next best step in management? Alteplase Exchange transfusion MRI brain Dranexamic acid

Exchange transfusion Cerebrovascular events are a potential complication of sickle cell disease. This patient developed symptoms concerning for acute ischemic stroke. For pediatric patients with acute ischemic stroke in the setting of sickle cell disease, exchange transfusion is the treatment of choice. Transfusion goals include decreasing hemoglobin S levels to less than 30% and obtaining a total hemoglobin level of 10 g/dL. For adults with acute ischemic stroke in the setting of sickle cell disease, use of tissue plasminogen activator (tPA) is an area of controversy. However, tPA is not indicated or approved for use in pediatric patients.

A 20-year-old man presents to the clinic with fatigue for the past 6 weeks. He describes constantly feeling tired and being unable to focus on work. His weight at his last visit 6 months ago was 180 pounds, and his weight today is 150 pounds. Vital signs include a HR of 98 bpm, BP of 120/80 mm Hg, RR of 20/min, oxygen saturation of 98% on room air, and T of 100.6°F. Physical examination is significant for nontender anterior cervical lymphadenopathy and excoriations on his trunk. Which of the following is the most appropriate test to confirm the suspected diagnosis? Antinuclear antibodies assay Bone marrow biopsy Excisional lymph node biopsy Fine-needle aspiration Heterophile antibody testing

Excisional lymph node biopsy Hodgkin lymphoma is a B cell lymphoma that accounts for 10% of lymphomas diagnosed in resource-limited countries. The majority of cases of Hodgkin lymphoma are classic Hodgkin lymphoma with a minority being nodular lymphocyte-predominant lymphoma. Hodgkin lymphoma has a bimodal age distribution occurring most often around 20 years of age or 65 years of age. The risk factors for Hodgkin lymphoma include prior infectious mononucleosis infection, immunocompromised state, history of human immunodeficiency virus infection, or a family history of Hodgkin lymphoma. The most common presentation of Hodgkin lymphoma is nontender lymphadenopathy. In many cases, the lymphadenopathy is chronic and asymptomatic, and the involved lymph node is usually above the diaphragm. Patients may also report pruritus or nonspecific constitutional (B) symptoms, such as fever, night sweats, and weight loss. Physical exam findings include palpable lymphadenopathy (often cervical, axillary, or supraclavicular), hepatomegaly, and splenomegaly. The diagnosis may also be initially suspected when a mediastinal mass is identified on chest X-ray. The diagnosis of Hodgkin lymphoma is confirmed with a tissue biopsy. Excisional biopsy of a peripheral lymph node is the preferred method, but multiple core-needle biopsies may be adequate. The classic finding that is seen on histologic examination of the biopsy is Reed-Sternberg cells. These cells have a bilobed nucleus and prominent nucleoli, which is often described as an owl eyes appearance. The Reed-Sternberg cells are mixed with pleomorphic inflammatory cell infiltrate and express CD30 but do not express CD45 or CD3. Staging is guided by the Ann Arbor staging system with Cotswolds modifications. Stage I Hodgkin lymphoma involves a single lymph node, such as cervical, axillary, inguinal, or mediastinal. Stage II indicates involvement of two or more lymph node regions on the same side of the diaphragm. Stage III involves lymph node regions on both sides of the diaphragm, and stage IV involves one or more extranodal organs. Patients typically require intravenous contrast-enhanced CT of the neck, chest, abdomen, and pelvis for staging. Hodgkin lymphoma generally has a high cure rate. The management varies depending on the stage of disease, with the goal being to maximize the cure rate while minimizing side effects of treatment. Patients with early-stage disease (stages I or II) are typically treated with a combination of chemotherapy and radiation. Combination chemotherapy alone is typically the treatment for stage III or IV disease. Complications that occur after treatment include additional malignancies (breast, lung, and skin cancers) and cardiovascular disease.

A 70-year-old woman with a medical history of hypertension presents to clinic for her annual visit. She is currently taking lisinopril for her hypertension. During her visit, she reports several months of fatigue and dyspnea on exertion. She denies shortness of breath at rest, orthopnea, and swelling of her lower extremities. On physical exam, the patient is afebrile, with a pulse of 92 bpm, a blood pressure of 125/82 mm Hg, and SpO2 of 97% on room air. She has a regular heart rate and rhythm with normal S1 and S2 heart sounds. The patient has normal breath effort, and lungs are clear to auscultation bilaterally. There is no lower extremity edema. Labs reveal a normal CBC, ferritin, TSH, and BNP. An echocardiogram demonstrates left atrial dilation and a nondilated left ventricle with concentric remodeling and an ejection fraction of 57%. Which of the following is the most effective management? Exercise training Furosemide Metoprolol No additional interventions Sodium restriction

Exercise training This patient presents with heart failure with preserved ejection fraction (HFpEF), or diastolic heart failure. HFpEF is characterized by signs and symptoms of heart failure along with a left ventricular ejection fraction (LVEF) ≥ 50%. Clinical manifestations of HFpEF are similar to those of heart failure with reduced ejection fraction (HFrEF) and include fatigue, weakness, dyspnea on exertion, orthopnea, and paroxysmal nocturnal dyspnea. Patients may present with elevated jugular venous pressure, pulmonary rales, lower extremity edema, and an S4 heart sound (or S4 gallop). Risk factors for HFpEF include advanced age, female sex, obesity, tobacco use, hypertension, diabetes, coronary artery disease, valvular heart disease, and atrial fibrillation. Echocardiogram is the most useful test to diagnose HFpEF and will reveal a normal or slightly increased LVEF and diastolic dysfunction. Concentric remodeling or left ventricular hypertrophy may also be present. Brain natriuretic peptide (BNP) levels may aid in the diagnosis and clinical decision-making of heart failure, but given its limited sensitivity and specificity, normal levels should not be used to exclude the disease. The treatment of HFpEF is largely centered around exercise training to improve performance status, diastolic function, and blood pressure control for patients with associated hypertension. Diuretics should only be used to relieve symptoms of volume overload. Furosemide (B) is incorrect. Furosemide and other diuretics are only indicated for treatment of HFpEF in patients with symptomatic pulmonary or lower extremity edema. The patient in the vignette above is not experiencing symptoms related to volume overload and thus should not be treated with furosemide. Metoprolol (C) and other beta-blockers are not indicated in the treatment of HFpEF. However, the use of beta-blockers is recommended in the treatment of HFrEF, as it has proven to reduce mortality related to the disease. No additional interventions (D) and sodium restriction (E) are both incorrect. These are not the most effective management of HFpEF.

A 15-year-old girl presents to urgent care with a nonproductive cough and low-grade fever that developed insidiously over the previous week. Physical exam reveals diffuse rales on pulmonary auscultation, and chest X-ray shows diffuse non-focal infiltrates. Which of the following is the most accurate statement regarding this patient's condition? Extrapulmonary manifestations may include hemolysis and CNS involvement Mycoplasma pneumoniae is the most common etiology in all age groups Penicillin is the most appropriate treatment The formation of cold agglutinins is a positive confirmatory test

Extrapulmonary manifestations may include hemolysis and CNS involvement This patient's clinical presentation is characteristic of community-acquired pneumonia. There are numerous pathogens that cause community-acquired pneumonia in children with a predilection for causing disease in particular age groups. Mycoplasma pneumonia, Chlamydophila pneumonia, and S. pneumoniae are most common in school age through the teen years. S. pneumoniae is classically associated with "typical" pneumonia, which is marked by rapid onset of high fever accompanied by chills, a high white count (>15,000), and findings of lobar consolidation on chest X-ray. M. pneumoniae and C. pneumoniae are classically associated with "atypical" pneumonia and present with a more gradual onset of symptoms and a pattern of interstitial infiltrates on chest radiography. Atypical bacterial infections are also more likely to present with extrapulmonary manifestations such as skin rashes, hemolysis (secondary to the formation of cold agglutinin antibodies), CNS involvement (aseptic meningitis, cerebellar ataxia, cranial nerve palsies), and gastrointestinal symptoms. However, these typical and atypical features frequently overlap and cannot be used consistently to distinguish between different etiologic agents.

You receive the laboratory report of a pericardial effusion sample sent yesterday from an inpatient with metastatic lung cancer. Which of the following results would you most expect? Exudate with Gram-positive bacilli Exudate with low-protein and high glucose levels Exudate with positive cytology Transudate with elevated carcinoembryonic antigen levels

Exudate with positive cytology There are several different reasons why the pericardial sac fills with fluid. Pericardial effusion often poses several diagnostic questions. In order to work through a differential, a sample of pericardial fluid can be sent to the lab for analysis testing. As is common with other fluid analyses, the first step in evaluating pericardial fluid is to differentiate transudate from exudate. Transudate represents an imbalance between vessel hydrostatic and oncotic pressure. Transudates are usually associated with some cardiac disease, such as congestive heart failure, or hepatic disease, such as cirrhosis. Other causes of transudative effusion include nephrotic syndrome, hypothyroidism and amyloidosis. On the other hand, exudates herald the presence of some traumatic injury or inflammation. Exudate can be infectious in nature, as in viral, bacterial or fungal pericarditis, or even myocarditis and endocarditis. Exudates are also commonly associated with autoimmune rheumatic conditions, such as rheumatoid arthritis or systemic lupus erythematosus. Cancer, either primary or metastatic, can also produce a pericardial exudate. In addition, exudates can be bloody, as in bleeding disorders or direct trauma. Once the pericardial effusion is deemed exudative, other tests are employed. Total cell counts, WBC differentials, fluid glucose, total protein and lactate dehydrogenase levels, microscopic examination, Gram stain, culture and susceptibility testing, AFB smear and culture, cytology and parasitic testing round out the typical battery of tests used to determine the source of a pericardial exudative effusion. Light's criteria are used to help differentiate transudative from exudative effusions - most commonly in pleural effusions but the table below can also be applied to pericardial effusions.

A 45-year-old woman comes to the urgent care clinic with a 5-hour history of right upper quadrant pain, fever, nausea, vomiting, and anorexia. She says the pain radiates to her shoulder and back. She says she began experiencing these symptoms shortly after eating a hot dog and chili cheese fries. She smokes 2 packs of cigarettes per day, drinks 2-3 alcoholic beverages every night, and occasionally smokes marijuana. The patient states that her two sisters both have had cholecystectomies. Her temperature is 38.8°C (101.8°F). Palpation of the abdomen shows voluntary guarding. An abdominal ultrasound shows a gallbladder wall of 6 mm. Which of the following contributed the most to the development of this patient's condition? Alcohol consumption Cigarette smoking Family history Marijuana smoking

Family history Right upper quadrant pain (radiating to the shoulder or back), fever, nausea, vomiting, and anorexia most likely suggests acute cholecystitis. Acute cholecystitis refers to a syndrome of right upper quadrant pain, fever, and leukocytosis (with a left shift) associated with gallbladder inflammation that is usually related to gallstone disease. Onset of symptoms generally occurs following ingestion of a fatty meal (as in this case). Several risk factors have been identified with the development of gallstones. They include female sex, advancing age (> 40 years), previous pregnancies, family history of gallstone disease, oral contraceptive pills (and estrogen replacement therapy), sedentary lifestyle (e.g., obesity), and type 2 diabetes. In this clinical scenario, the patient has a family history of gallbladder disease.

Which of the following is a minor rather than a major Jones criterion in the diagnosis for acute rheumatic fever? Fever New murmur Polyarthritis Subcutaneous nodules

Fever Rheumatic fever is a known delayed sequela of infection with group A streptococcal infection. Fever is a minor criterion for acute rheumatic heart disease. The Jones criteria, created in 1944 and updated in 1992, are used to diagnose acute rheumatic fever (ARF). Rheumatic heart disease is an important yet uncommon sequela of rheumatic fever. Acute rheumatic fever incidence is declining in the US and other industrialized countries and is reported to be fewer than 20 cases per 100,000 individuals in school-aged children. Diagnosis of ARF is possible using one major plus two minor Jones criteria or two major Jones criteria. Major criteria include carditis (new murmur), polyarthritis, chorea, erythema marginatum (nonpruritic, erythematous rings, found on trunk and inner limbs), and subcutaneous nodules (painless collagen collections on back of wrists, elbows, and knees). Minor criteria include arthralgias, fever, elevated acute phase reactants (erythrocyte sedimentation rate and C-reactive protein).

Which of the following best describes the most common symptoms of acute human immunodeficiency virus infection? Diarrhea and abdominal pain Fever, malaise, myalgias, sore throat, and rash Fever, productive cough, and shortness of breath Most individuals are asymptomatic

Fever, malaise, myalgias, sore throat, and rash In the United States, an estimated 1.2 million individuals are HIV positive, with approximately 50,000 people newly diagnosed each year. The clinical presentation of acute HIV infection, also known as acute retroviral syndrome, is variable both in type and severity of symptoms. The classic presentation is abrupt onset of a viral-like illness, which develops 10 to 14 days following exposure to the virus. Symptoms may include fever, malaise, sore throat, headache, arthralgias, anorexia, nausea, vomiting, and rash. Physical examination findings vary and may include generalized lymphadenopathy, nonexudative pharyngitis, mild hepatosplenomegaly, mucocutaneous ulcers, and oral thrush. A maculopapular rash on the thorax, face, and limbs may also be present. Due to the nonspecific and self-limited nature of symptoms, many affected individuals do not seek care and thus the true incidence of acute HIV is unknown. However, experts estimate that 60 to 90 percent of individuals who contract HIV develop an acute illness. Identification of acute HIV has tremendous public health implications, as acute infection represents the peak infectivity of the disease, characterized by high levels of viral shedding. Routine HIV tests used in most emergency departments measure anti-HIV antibodies, which are often falsely negative during acute infection. Therefore, when acute HIV is suspected, nucleic acid amplification tests, which directly measure viral RNA in the patient's blood, should be used.

Which of the following meets criteria for a diagnosis of systemic inflammatory response syndrome (SIRS)? Heart rate > 80 and respiratory rate > 16 Heart rate > 90 Heart rate > 90 and temperature > 100.4°F Temperature > 100.4°F and WBC > 10,000

Heart rate > 90 and temperature > 100.4°F The SIRS criteria is part of the sepsis syndrome spectrum of diseases. SIRS often represents the body's host response to an infection. Although research has found that SIRS criteria alone does not predict an increased mortality, it should prompt continued investigation for an underlying pathology. The presence of organ dysfunction and shock, however are significant predictors of adverse outcomes and should be fully addressed. Sepsis is the tenth most common cause of death in the US.

A 26-year-old woman is found to have a blood pressure of 160/90 mm Hg. Similar values are obtained on two subsequent visits. She denies episodic headaches, palpitations, and diaphoresis. She is not obese. On abdominal exam, she is found to have a renal bruit. No abdominal masses are palpated. Her serum creatinine is 1.5 mg/dL. What is the most likely diagnosis? Autosomal dominant polycystic kidney disease Fibromuscular dysplasia Minimal change disease Pheochromocytoma

Fibromuscular dysplasia Fibromuscular dysplasia should be suspected in women younger than 35 years with unexplained hypertension. Fibromuscular dysplasia (FMD) is most commonly seen in women. The mean age of diagnosis is 52 years. The renal arteries and extracranial cerebrovascular arteries are most commonly affected and most patients have involvement of multiple arteries. FMD is a noninflammatory, nonatherosclerotic disease that results in arterial occlusion, stenosis, and dissection. The etiology of FMD is unknown, however, there are reported autosomal inheritance in some families. Hormones have also been implicated given the predominance of middle-aged women affected. Hypertension is the most common finding in renal FMD. Flank pain or abdominal pain can result from renal or mesenteric artery ischemia, aneurysm, or dissection. Typical findings of extracranial cerebrovascular FMD include headache, pulsatile tinnitus, and neck pain. These symptoms are indicative of stenosis, dissection, aneurysmal rupture, or thromboembolism. FMD should be suspected in women with hypertension before 35 years of age, women with severe or resistant hypertension, women with an epigastric bruit, in any woman less than 60 years of age with a transient ischemic attack or stroke. Non-invasive diagnostic imaging, like computed tomography angiography (CTA), magnetic resonance angiography (MRA), or duplex ultrasonography, are usually sufficient to diagnose FMD. Treatment of FMD depends on severity of disease and the arteries involved. Patients presenting with acute stroke symptoms should be managed as an ischemic stroke regardless of FMD. Angioplasty with or without stenting for renal FMD has a good prognosis. Medical management involves blood pressure management and secondary prevention of strokes with antiplatelet agents.

A 72-year-old man has been more fatigued lately. As a result, his primary practitioner orders an electrocardiogram. A P wave is present for every QRS complex and the PR interval is 225 milliseconds in length. Which of the following is the diagnosis? First degree atrioventricular block High-grade atrioventricular block Mobitz type I atrioventricular block Third degree atrioventricular block

First degree atrioventricular block A first degree atrioventricular (AV) block is characterized by a long PR interval (>200 milliseconds). The normal PR interval is 120 to 200 milliseconds in length. The PR interval represents the time between atrial depolarization (P wave) and the ventricular depolarization (QRS complex). A prolonged PR interval is a sign of a delayed conduction at the AV node before ventricular depolarization. Causes of first degree AV block include a benign increase in vagal tone, drugs that slow AV conduction (e.g., beta-blockers, digoxin), previous myocardial infarction, dilated cardiomyopathy, and some types of muscular dystrophy. Typically, patients exhibit no symptoms and there are no signs on physical exam.

A 24-year-old man with HIV presents with severe dysphagia and odynophagia for one week. Physical examination reveals the above image. What management is indicated? Acyclovir Clotrimazole troches CT scan of the neck Fluconazole

Fluconazole This patient presents with infectious esophagitis secondary to a candidal infection and can be treated with fluconazole. Esophagitis is defined as inflammation of the esophagus and can be caused by a number of mechanisms including infection, allergy related (eosinophillic), or pills. Infectious esophagitis typically occurs in patients that are immunocompromised (diabetes, HIV/AIDS, chronic steroids etc.). Esophageal candidiasis is common in the HIV/AIDS population and is considered an AIDS-defining illness. In addition to candida, esophagitis can be caused by HSV1 and CMV. Patients with infectious esophagitis present with severe odynophagia that may be to solids, liquids, or both. They may also have chest pain, nausea, and dyspepsia. Dehydration is common secondary to pain. Diagnosis can be made based on clinical presentation but an endoscopy can be performed to confirm the diagnosis. In immunocompromised patients with candidal esophagitis, treatment should be with fluconazole for 14 to 21 days. If they are unable to tolerate oral medications, intravenous fluconazole can be given.

An 18-year-old woman presents to the clinic reporting several episodes that she describes as blackouts. She reports feeling a fluttering in her stomach and a sense of anxiety and approaching doom, then does not remember anything for a few seconds to a few minutes. Her mother states when her daughter has these episodes, she stares into space, tilts her head to the left, and smacks her lips repetitively, although the patient has no recollection of this. The patient reports having no prior illness, trauma, or surgery and does not take any medications or illicit drugs. Her vital signs are within normal limits. Her physical exam, including the neurologic exam, is normal. CBC and CMP results are normal, and her urine drug screen is negative. Which of the following is the most likely diagnosis? Acute dystonia Focal onset seizure Generalized tonic-clonic seizure Psychogenic nonepileptic seizure Tourette syndrome

Focal onset seizure The patient in the vignette has classic signs and symptoms of focal onset seizures. Epileptic seizures have three components: abnormal synchronous electrical brain activity, clinical signs, and a distinct onset and offset. Focal onset seizures take place when abnormal synchronous electrical activity affects one specific area of the brain, causing focal signs. Common signs and symptoms of focal onset seizures are an aura, automatisms, and dystonic posturing, while uncommon signs and symptoms are affective disordered behavior, vocalizations, or hypermotor behavior. Examples of auras encountered in focal onset seizures include a feeling of impending doom, anxiety, fear, nausea, or deja vu. Examples of automatisms include lip-smacking, chewing, fumbling, picking, or fidgeting. Dystonic posturing usually manifests as head turning. Patients may retain consciousness throughout the episode (focal onset seizure with retained consciousness) or may lose awareness (focal onset seizure with impaired awareness). Those patients with impaired awareness have an arrest in their normal behavior and stare off into space for 30 to 120 seconds. Patients may experience postictal confusion, which is usually brief. Definitive diagnosis is with electroencephalography, although the results may be normal if this exam is performed between seizure episodes. Magnetic resonance imaging of the brain should be performed to rule out mass lesions or other causes of focal seizure activity (stroke, vascular malformation, trauma, or degenerative disorder). Treatment is with broad-spectrum antiepileptic drugs, such as levetiracetam or divalproex sodium, or with more narrow-spectrum agents, such as gabapentin.

A 65-year-old man presents with fatigue that has been worsening over the last several months. A complete blood count demonstrates megaloblastic anemia. Serum vitamin B12 levels are within normal limits. Which of the following is the most likely diagnosis? Autoimmune hemolytic anemia Folate deficiency Hypothyroidism Iron deficiency anemia

Folate deficiency Folic acid deficiency can cause a megaloblastic anemia. Typically the cause of folic acid deficiency is reduced intake from a person who is anorexic, alcoholic, or malnourished. Symptoms of folic acid deficiency include fatigue and glossitis. The neurologic disturbances that often occur with vitamin B12 deficiency are absent in folic acid deficiency. Treatment is with supplementation of folic acid daily. The response is typically rapid with a complete correction of the anemia usually occurring within two months.

A 58-year-old man ran out of his congestive heart failure medications. He presents with significant dyspnea and altered mental status. Examination reveals bibasilar crackles and jugular venous distension. An electrocardiogram shows sinus rhythm with low voltage complexes. Which of the following is most appropriate at this time? Amiodarone Disopyramide Furosemide Ventricular assist device

Furosemide Acute or decompensated congestive heart failure may be the result of myocardial infarction, dysrhythmias, dietary noncompliance, medical noncompliance, kidney failure, pulmonary embolism, anemia or toxins. Acute failure typically requires hospitalization and critical care. In-house care is comprised of oxygen, nitrates, and furosemide. Furosemide is a loop diuretic that inhibits the transporter at the loop of Henle in the kidneys leading to free water clearance. Inotropic medications, such as dobutamine and dopamine, may also be necessary if there is evidence of impaired perfusion. Severe cases may require an intraaortic balloon pump or a ventricular assist device.

A 24-year-old athlete undergoes anterior cruciate ligament repair surgery. Two weeks later, he presents with excruciating pain distal to the knee. Fracture, soft tissue injury and intraarticular infection are ruled-out. Examination reveals a swollen, warm, red foot and ankle that is painfully sensitive to light touch. The other leg appears normal. The patient guards this area and active range-of-motion is restricted. Complete pinprick sensory testing is deferred as initial testing is too painful to continue. Vibration testing results in continued pain even after removal of the tuning fork. Which of the following medications will you most likely prescribe? Gabapentin Intranasal desmopressin Intravenous immunoglobulin Pyridostigmine

Gabapentin Complex regional pain syndrome (CRPS) is not commonly encountered in primary care. However, CRPS presents with profound signs and symptoms. CRPS-1 occurs after a noxious neurological event, such as soft tissue crush injury, immobilization, orthopedic surgery and podiatric surgery. Any insult to the integrity of peripheral nerves is a possible etiology. This condition is felt to be due to activation of peripheral nociceptors, causing an increase in neuroexcitatory agents in the spinal cord, leading to upregulation and sensitization of peripheral and central pain pathways. Lowered neuronal thresholds result in over activity and dysregulation of the sympathetic nervous system. The most common presenting symptoms include: neuropathic pain (spontaneous, burning, dysesthetic, diffuse), allodynia (pain felt from a nonpainful stimulus, such as clothes or bed sheets on the skin), hyperesthesia (exaggerated pain response to a painful stimulus), hypoesthesia (decreased sensation/numbness in a painful area), hyperpathia (continued sensation after a stimulus is removed, such as continuing to feel vibration after a tuning fork is removed), decreased range-of-motion and joint guarding, skin changes (edema, erythema, temperature alterations, changes in hair growth and nail composition) and motor impairment. A classic presentation is post-surgical patients with days-to-weeks onset of distal extremity diffuse neuropathic pain, edema and erythema, as in the above patient. Most patients are treated with a mix of medication options: corticosteroid burst, intranasal calcitonin, NSAIDs, short course of opioids, gabapentin, tricyclic antidepressants and transdermal clonidine or lidocaine. Intranasal desmopressin (B) is an antidiuretic hormone analogue used in treating central diabetes insipidus, not CRPS. Intranasal calcitonin has been shown to be beneficial in treating CRPS. IVIG (C) is used in treating immune deficiencies, autoimmune disease and some infections. It has no role in the treatment of CRPS. Pyridostigmine (D) is an anticholinesterase agent used in treating the motor disorder myasthenia gravis. It plays no role in treating neuropathic pain such as CRPS.

A morbidly obese 32-year-old woman has a 4-month history of hypertension that has been nonresponsive to multiple antihypertensive medications. Blood pressure readings over the past month have an average diastolic pressure of 115 mm Hg. Her serum creatinine is 1.2 mg/dL. Which of the following diagnostic tests is most appropriate at ruling-in the most common cause of this refractory hypertension? Captopril renography Duplex Doppler ultrasonography Gadolinium-enhanced magnetic resonance angiography KUB radiograph

Gadolinium-enhanced magnetic resonance angiography Systemic hypertension is a common disease in America. Most cases are essential, however, secondary hypertension exists and is commonly caused by renal vascular hypertension, with renal artery stenosis (RAS) being a common pathology. Atherosclerosis and fibromuscular dysplasia are the most common underlying sources of renal artery stenosis. RAS accounts for 5% of all systemic hypertension. RAS is especially prevalent in cases of refractory, malignant or accelerated hypertension, as well as in significant diastolic hypertension in those less than 35-years-old. Clinical manifestations include abdominal or renal bruit, deteriorating renal function and other signs of generalized atherosclerosis. Magnetic resonance angiography (MRA) has the highest sensitivity for detecting renal artery stenosis. Due to the risk of gadolinium-induced nephrogenic systemic fibrosis, MRA should be avoided in patients with a creatinine clearance less than 30 mlL/min. Diagnosis can also be made with computed tomography angiography which has a comparable sensitivity for detecting renal artery stenosis. Limitations include use of contrast (contraindicated in patients with limited renal function) and radiation exposure.

You examine a two-day-old boy in the neonatal intensive care unit. He was born to a 26-year-old G2, P2 mother via normal spontaneous vaginal delivery. The mother had limited prenatal care. The nurse notes that the infant has a poor suck. On examination, you note that the neonate is small for gestational age, with microcephaly, enlarged liver, and petechiae. On complete blood count, you note thrombocytopenia. You suspect congenital cytomegalovirus infection. Which of the following is the most appropriate therapy? Acyclovir Cidofovir Foscarnet Ganciclovir

Ganciclovir The boy has findings that are suspicious for congenital cytomegalovirus (CMV) infection. Congenital CMV is common worldwide. It is the leading cause of nonhereditary sensorineural hearing loss (SNHL) and can cause other long-term neurodevelopmental disabilities, including cerebral palsy, intellectual disability, vision impairment, and seizures. At birth, most infants with congenital CMV are asymptomatic, but approximately 10 percent have symptoms. Clinical findings in the symptomatic neonate include petechiae, jaundice at birth, hepatosplenomegaly, small size for gestational age, microcephaly, sensorineural hearing loss, lethargy or hypotonia, poor suck, chorioretinitis, seizures, hemolytic anemia, and pneumonia. The diagnosis of congenital CMV infection can be established within the first three weeks of life by detection of CMV in the urine or saliva. Testing should be carried out as soon as the diagnosis is suspected so that appropriate antiviral therapy can be started promptly. Intravenous (IV) ganciclovir and its orally available prodrug, valganciclovir, are the first-line antiviral agents of choice for treatment of congenital CMV disease. These medications have been evaluated in multicenter, randomized clinical trials in newborns with symptomatic congenital CMV infection and have demonstrated a benefit when treatment is initiated within the first month of life.

A woman presents with significant family and work related stress. She has been drinking alcohol more frequently than usual. For the past two weeks, she has noticed upper central abdominal pain, nausea, and loss of appetite. Initial laboratory testing reveals a mild anemia. You decide to order an esophagogastroduodenoscopy, which only reveals gastric epithelial inflammation. Which of the following is the most likely diagnosis? Esophagitis Gastritis Pancreatitis Peptic ulcer disease

Gastritis Gastritis refers to inflammation of the stomach epithelium secondary to an inflammatory response. The most common cause of acute gastritis is due to H. pylori infection. There is poor correlation between symptoms (pain and dyspepsia) and endoscopic findings. However, acute H. pylori gastritis can cause sudden onset of epigastric pain, nausea, and vomiting, while gastropathy caused by NSAID use, alcohol, or portal hypertension can present with GI bleeding. Exam is usually unremarkable. Diagnosis is mainly clinical but can include noninvasive H. pylori testing, evaluation for suspected autoimmune gastritis via antiparietal cell and intrinsic factor antibodies, and analysis of the serum pepsinogen I-to-II ratio. Endoscopy with biopsy can provide a pathologic diagnosis. Treatment generally consists of avoidance of causative factors (e.g. NSAIDs, alcohol) along with prophylaxis with proton-pump inhibitors.

A 64-year-old man presents to the clinic with generalized pain in his neck, shoulders, and hips. He states the pain is worse in the mornings and is sometimes associated with one-sided headaches. His erythrocyte sedimentation rate and C-reactive protein are both elevated. You diagnose him with polymyalgia rheumatica. What other condition would you suspect in this patient? Fibromyalgia Giant cell arteritis Systemic lupus erythematosus Trigeminal neuralgia

Giant cell arteritis Polymyalgia rheumatica is a systemic, inflammatory rheumatic arthritis that is characterized by morning stiffness and muscle aches in the shoulders, neck, and hips. Patients may present with functional limitations, including difficulty putting on a coat or shirt, getting out of a chair, combing hair, donning socks and shoes. Systemic manifestations of malaise, fatigue, depression, anorexia, weight-loss, and low-grade fever may be present as well. The incidence increases with age, and it is most commonly found in elderly patients > 50 years of age. Women and individuals of Scandinavian and northern European descent are most commonly affected. Polymyalgia rheumatica is a clinical diagnosis. Elevated erythrocyte sedimentation rate (ESR) and C-reactive protein (CRP) aid in diagnosing. Polymyalgia rheumatica is associated with an increased incidence of giant cell arteritis. New-onset headache, monocular visual impairment, or jaw pain with mastication in patients with a diagnosis of polymyalgia rheumatica should be concerning for giant cell arteritis. Diagnosis of giant cell arteritis is confirmed by temporal artery biopsy. Treatment for both conditions is glucocorticoid therapy.

A 26-year-old woman presents complaining of yellow vaginal discharge after starting a relationship with a new sexual partner. Physical exam reveals purulent discharge from the external os of the cervix. Microscopy reveals gram-negative diplococci. What is the most likely diagnosis? Bacterial vaginosis Chlamydial cervicitis Gonococcal cervicitis Trichomonas vaginitis

Gonococcal cervicitis Gonorrhea is an infection of Neisseria gonorrhoeae, one of the most common sexually transmitted infections in the United States. Co-infection of C. trachomatis occurs in up to 30% of persons infected with gonorrhea. Symptoms are often similar between men and women and consist of dysuria and purulent discharge. There are a number of patients that remain asymptomatic, and this is often a factor that leads to further transmission of infections. Though it is more common in the genitourinary tract, gonorrhea may also infect the anal and oropharyngeal tract, or disseminate to joints where it results in arthritis. The preferred treatment for gonococcal cervicitis (or urethritis) is ceftriaxone 500 mg intramuscularly plus coverage for chlamydia (doxycycline 100 mg BID for 7 days in nonpregnant patients). Diagnosis is made with gram stain and culture, nucleic acid hybridization tests, or nucleic acid amplification. Microscopic evaluation reveals a gram-negative, kidney bean-shaped diplococci.

A 48-year-old man with a history of HIV presents to your office with complaints of fever, cough and shortness of breath. Laboratory testing reveals a CD4 count of 130 cells/microL. Which of the following findings is most likely to be seen on chest X-ray? Ground glass opacification Hampton's hump Honeycombing Kerley B lines

Ground glass opacification Pneumocystis jirovecii pneumonia (PCP), previously called Pneumocystis carinii pneumonia, is an opportunistic infection found in individuals infected with HIV who have CD4 counts of less than 200 cells/microL. Symptoms of PCP develop gradually, with fever, cough and shortness of breath being the most common complaints. Patients may also present with fatigue, weight loss, chest pain and chills. Chest X-ray will reveal bilateral, fine, diffuse opacification, often with a ground glass appearance. Patients with HIV whose CD4 counts drop below 200 cells/microL should be given prophylaxis with trimethoprim-sulfamethoxazole and treatment of the infection is with the same agent.

An 81-year-old man is admitted to the hospital for agitation and combative behavior. He has a history of Alzheimer disease and hyperlipidemia, for which he takes donepezil and simvastatin. His family, who cares for him, reports that this combative behavior is atypical. The agitation started suddenly this afternoon and worsened over a period of 5 hours. There has been no recent change to his medications, and his family reports no recent falls or known head injury. He is afebrile. His blood pressure is 108/74 mm Hg, heart rate is 102 bpm, and the rest of his vital signs are within normal range. He appears anxious and is trying to pull out his intravenous lines. There is no localization of neurologic findings on examination. Baseline labs are drawn and a urine sample is obtained, which is positive for nitrites. The patient is started on empiric antibiotics for a presumptive urinary tract infection. What would be the most appropriate initial management for the patient's agitation at this time? Administer intramuscular olanzapine Apply soft restraints Have a sitter or family member at the bedside Increase dose of donepezil Start patient on a low-dose benzodiazepine

Have a sitter or family member at the bedside Delirium is an acute, reversible confusional state. It is distinguished by an altered level of consciousness and disturbance of attention. It is common in hospitalized patients, particularly those in intensive care units. Risk factors for delirium include underlying infection, recent surgical procedure, underlying neurological disorder (e.g., stroke, dementia), dehydration, and polypharmacy. Unlike dementia, symptoms can develop over a period of hours to days. One of the first signs of delirium is alteration of mental status. Varying degrees of cognitive impairment can also be observed, such as difficulty with language and memory. Perceptual disturbances are common in delirium, with patients experiencing delusions and hallucinations. Delirium can be classified as hypoactive or hyperactive. Hypoactive symptoms can be easily missed, as the patient may appear quiet, somnolent, or inattentive. Hyperactive symptoms include agitation, anxiety, wandering, and emotional lability. Sleep-wake cycle disturbances are common, and patients may be more sensitive to light and sound. Bedside screening tools can be used to assess for delirium. These include the Confusion Assessment Method (CAM). A thorough review of the patient's history is important to identify potential causes of delirium, such as medications, substance use, or symptoms of infection. The physical examination may also provide clues into underlying etiology, such as signs and symptoms of liver failure, diabetic ketoacidosis, trauma, localizing neurological findings, and substance withdrawal or overdose. In general, complete blood count, complete metabolic panel, urinalysis, and urine culture should be ordered. Chest radiograph and electrocardiogram are frequently performed. Diagnostic testing varies by patient. Toxicology, arterial blood gas, thyroid function testing, serum drug levels, neuroimaging, lumbar puncture, and electroencephalogram are additional tests that may be indicated depending on patient presentation. Identifying causes, such as infection, and underlying medical conditions should be appropriately addressed and managed. Nonpharmacologic interventions are preferred for initial management of delirium and agitation. These interventions include reorienting the patient to their environment, having regular visitors at the bedside who are familiar to the patient, implementing a sitter at the bedside, maintaining a day-night schedule, encouraging use of visual aids and hearing aids if applicable to the patient, treating pain, and discharging intravenous lines, drains, or catheters once they are no longer needed. Physical restraints are considered a last resort. Antipsychotic medications may be considered in the setting of severe agitation, such as when symptoms are threatening self-harm or not responding to initial intervention. Antipsychotics should be used at the lowest dose necessary and for the shortest duration required, and patients should be closely monitored due to the risk of QT prolongation. Benzodiazepines should be avoided unless pharmacologic intervention is required and there is a contraindication to antipsychotic drug use. Benzodiazepines can cause paradoxical confusion.

A new nursing home employee is being screened for tuberculosis and develops a 6 mm induration on his right volar forearm after injection of a purified protein derivative (PPD). This reading indicates prior tuberculosis exposure if he has had which of the following life circumstances? He is a former guard at a correctional facility He is a HIV-negative injection drug user He is a recent immigrant from Latin America He is an organ recipient on daily immunosuppressives

He is an organ recipient on daily immunosuppressives An organ recipient on daily immunosuppression therapy is among the group of people who would be considered to have a positive purified protein derivative tuberculin skin test, or PPD, at a reading of 5 mm or greater of induration. Other patients considered to have a positive PPD at 5 mm would include those with recent contact to people with active tuberculosis, those with chest radiographs suggestive of infection, and any HIV-positive patients. Patients who recently emigrated from countries with a high tuberculosis prevalence, are injection drug users, or are residents or employees of high-congregate areas are considered to have a positive PPD reading at 10 mm. Many immunocompromising conditions also allow for a positive reading at 10 mm. Otherwise healthy adults without significant risk of prior exposure would have a positive PPD read at 15 mm of induration. The tuberculin skin test, in which purified protein derivative is placed under the skin of the volar forearm, is the most common method used in screening at-risk people for tuberculosis exposure. This test has high sensitivity and specificity. However, patients who were previously vaccinated with bacillus Calmette-Géurin may falsely test positive. In these patients, an interferon gamma release assay should replace the PPD for tuberculosis screening. All suspected cases of tuberculosis should be reported to government health authorities. A sputum culture should be ordered to determine drug sensitivities of the isolate. Beyond routine screenings, clinicians should be alert for signs and symptoms of pulmonary tuberculosis in patients. These include persistent cough, anorexia, weight loss, fever, night sweats, and blood-streaked sputum production. If tuberculosis is diagnosed, patients must complete treatment with a 6-9 month course of an appropriate anti-tuberculosis drug regimen to prevent the spread of illness.

A previously healthy 42-year-old man presents to your office with questions about screening for colon cancer. He has no family history of colon cancer, but heard that he should start getting a colonoscopy every 5 years starting at age 40. Which of the following recommendations do you provide? He does not need screening because he has no family history of colon cancer He should be scheduled for a colonoscopy as soon as possible He should start screening at age 45 and continue until age 75 He should start screening at age 50 and continue indefinitely

He should start screening at age 45 and continue until age 75 Colorectal cancer is the third most common type of cancer for both men and women in the United States. The United States Preventative Services Task Force recommends colon cancer screening for average-risk individuals starting at age 45 years through age 75 years. Screening may occur using one of three modalities: high-sensitivity fecal occult blood testing annually, sigmoidoscopy every 5 years with fecal occult blood testing every 3 years, or colonoscopy every 10 years. Patients with a first-degree relative (parent, sibling or child) who has a history of colorectal cancer or adenomas should be screened earlier than age 45 years. Screening for these patients is determined on an individualized basis and generally starts at age 40 years or 10 years younger than the age the affected relative was diagnosed.

A 49-year-old woman presents with headaches that occur nearly every afternoon since she switched jobs 6 months ago. She describes squeezing pain at her forehead, accompanied by fatigue and poor concentration. Her exam is normal except for posterior neck tenderness, which she attributes to stress at work. Which of the following interventions would be most helpful to this patient? Headache abortive therapy with sumatriptan Headache prevention with amitriptyline Headache prevention with daily verapamil High dose prednisone taper

Headache prevention with amitriptyline Amitriptyline would be most helpful for headache prevention in this patient who is describing tension-type headaches. Tension headaches are very common and tend to present as a bilateral headache in a band-like distribution with pain in the frontal and occipital regions. It is the most common type of primary headache. A physical exam may reveal focal areas of tenderness at the posterior neck, where pain and stiffness radiate upward causing pain at the back of the head and a vice-like discomfort around the skull. Neurologic exams should be normal. Tension headaches frequently occur during times of stress, and the clinician should be alert to underlying, untreated depression or anxiety. Tricyclic antidepressants such as amitriptyline have been shown to be effective for depressive symptoms as well as headaches and other chronic pain symptoms. Beyond this, avoidance of triggers, such as stress, poor sleep, loud noises, and light glare, can help. Patients should also be counseled to try daily stretches, biofeedback exercises, hot compresses on the posterior neck, and massage as ways of inducing relaxation of the neck and body in general. If these interventions are not helpful or new symptoms such as nausea and vomiting develop, clinicians should reconsider whether a diagnostic overlap with migraines may be possible as tension headaches and migraine headaches commonly occur together.

A 12-year-old girl presents for her annual well child check. She had menarche eight months prior, and has had menses every month. Each cycle lasts two weeks. The patient uses at least six pads on most days of the cycle. On exam, she is pale but otherwise well appearing. Her heart rate is 80 beats per minute, blood pressure 110/65, and respiratory rate 14. The remainder of her exam is within normal limits. Which of the following are the most likely laboratory findings? Hemoglobin 12 g/dL, mean corpuscular volume 80, RDW 10 Hemoglobin 6 g/dL, mean corpuscular volume 60, RDW 12 Hemoglobin 6 g/dL, mean corpuscular volume 60, RDW 18 Hemoglobin 8 g/dL, mean corpuscular volume 100, RDW 10

Hemoglobin 6 g/dL, mean corpuscular volume 60, RDW 18 This patient has iron deficiency anemia secondary to dysfunctional uterine bleeding. Iron deficiency anemia is characterized by a microcytic anemia (low hemoglobin and low MCV). Approximately two percent of adolescent females experience iron deficiency anemia due to menstrual blood loss and the adolescent growth spurt. Most children with iron deficiency anemia are asymptomatic. The most common sign is pallor, which usually does not occur until the hemoglobin level is < 8 g/dL. Iron studies in this patient show reduced iron stores (low ferritin), increased iron-binding capacity (high transferrin and elevated TIBC), and decreased transferrin saturation. The mean corpuscular volume (MCV) describes red blood cells as microcytic, normocytic, or macrocytic. A normal MCV in an adolescent female is approximately 80. The red cell distribution width (RDW) is a measure of the variability in size of red blood cells and is normally 11.5 - 14.5. Iron deficiency anemia is characterized by an elevated RDW.

A 16-year-old boy presents with a marked hemarthrosis of his knee after tripping and striking it against the edge of a step. This has happened in the past. His activated partial thromboplastin time is prolonged and his prothrombin time and thrombin time are normal. What is the most likely diagnosis? Factor II deficiency Hemophilia B Vitamin K deficiency Von Willebrand disease

Hemophilia B Hemophilia B (Christmas disease) is an X-linked inherited bleeding disorder caused by a deficiency in factor IX. Patients with hemophilia have spontaneous bleeding episodes involving skin, mucous membranes, and joints. Minor trauma can cause large amounts of bleeding or hemarthrosis. Complications include compartment syndrome from intramuscular hematoma, joint destruction from recurrent hemarthroses, or the development of antibodies to exogenous factor IX. Exogenous factor IX concentrate is the mainstay of therapy. Diagnosis is made by examining factor IX activity. Other tests include an activated partial thromboplastin time (aPTT) which will be prolonged. The prothrombin time (PT) and thrombin time will be normal.

Which of the following best describes the etiology of the jaundice seen in patients with thyroid storm? Direct constricting effects of thyroid hormone on the biliary duct Hepatic tissue hypoxia due to increased peripheral consumption of oxygen Hypotension leading to decreased gut motility Impaired reabsorption of thyroid hormone in the enterohepatic circulation

Hepatic tissue hypoxia due to increased peripheral consumption of oxygen Thyroid storm refers to thyrotoxicosis in its extreme form. Patients with Grave's disease or toxic multinodular goiter can become acutely decompensated through an excess of circulating thyroid hormone and subsequent hypermetabolism. Signs and symptoms of thyroid storm include hyperpyrexia, nausea, vomiting, diarrhea, mental status changes, jaundice, high-output congestive heart failure, cardiac tachyarrhythmias, hypertension, and diaphoresis. The exact mechanism whereby thyrotoxicosis evolves into thyroid storm in certain patients is unknown, although stress, infection, trauma to the thyroid gland, diabetic ketoacidosis, surgery, pregnancy, and radioactive iodine therapy are all known precipitating factors. Certain drugs, such as anesthesia agents, anticholinergics, adrenergics, nonsteroidal anti-inflammatory drugs, salicylates, and chemotherapeutic agents, can also trigger thyroid storm in a patient with underlying thyrotoxicosis. Heart failure in thyroid storm is the result of increased oxygen demand by the tissues and the heart's inability to keep up with such a high demand (high-output heart failure). Likewise, jaundice is the result of hepatic tissue hypoxia due to increased peripheral consumption of oxygen and unmet oxygen needs of the hepatocytes. Jaundice can also result from hepatic congestion secondary to heart failure. While less common than other signs and symptoms of thyroid storm, the presence of jaundice portends a poor prognosis. Treatment of thyroid storm typically begins with administration of propranolol to slow the heart rate, lower the body's metabolic demands, and block peripheral conversion of T4 to T3. Next, propylthiouracil 600 mg is administered, followed by iodine administration. Correction of volume and electrolyte imbalance as well and cooling of body temperature is essential. Corticosteroids are given to decrease the rate of peripheral conversion of T4 to T3.

A 31-year-old nurse presents to the emergency department after being stuck with a blood-filled hollow bore needle during a resuscitation. She discovers the patient has hepatitis B. That same day, she has her blood drawn. Which of the following serologic markers indicates prior hepatitis B immunization? Hepatitis B core antibody (anti-HBc) Hepatitis B envelope antigen (HBeAg) Hepatitis B surface antibody (anti-HBs) Hepatitis B surface antigen (HBsAg)

Hepatitis B surface antibody (anti-HBs) Presence of the hepatitis B surface antibody (anti-HBs) immediately postexposure indicates prior immunization. Although anti-HBs may also be seen with acute infection, its appearance is associated with a postexposure lag time of roughly two months. A person negative for HBsAg but positive for anti-HBs either has cleared an infection or has been vaccinated previously. The vaccination for hepatitis B contains only the surface antigen, thus an immunized patient will only have surface IgG present if they have been vaccinated.

A 60-year-old woman presents with two days of right upper quadrant abdominal pain that is constant in nature and associated with subjective fever, nausea, and vomiting. Vital signs are significant for a temperature of 38.1°C, heart rate 87 bpm, blood pressure 140/80 mm Hg, respiratory rate 14 breaths/min, and oxygen saturation of 99% on room air. On physical examination, her abdomen is soft with right upper quadrant tenderness and a positive Murphy sign. Which of the following tests is most sensitive and specific in diagnosing this patient's most likely condition? Computed tomography scan with intravenous contrast Hepatobiliary iminodiacetic acid (HIDA) scan MRI with gadolinium Ultrasound

Hepatobiliary iminodiacetic acid (HIDA) scan This patient has suspected acute cholecystitis and requires an imaging study for confirmation. Although an ultrasound of the gallbladder is the usual initial diagnostic imaging study for clinically suspected acute cholecystitis, hepatobiliary iminodiacetic acid (HIDA) scanning is considered the most sensitive and specific test for diagnosing acute cholecystitis. IDA is administered intravenously, taken up by hepatocytes, and excreted into the bile canaliculi. Failure to obtain an outline of the gallbladder within one hour proves cystic duct obstruction and, in the appropriate clinical setting, confirms the diagnosis of acute cholecystitis. Visualization of the gallbladder and common duct within one hour has a high negative predictive value. A HIDA scan is usually obtained when the ultrasound study is equivocal.

A 12-year-old boy with a medical history significant for eczema presents with a painful rash and general malaise. The rash has been worsening over the last week. Vital signs are HR of 98 bpm, RR of 18/min, SpO2 of 97% on room air, BP of 114/66 mm Hg, and a T of 100.9°F. A physical examination reveals a rash that includes well-circumscribed erosions overlying skin that appears eczematous in the periorbital area, the bilateral antecubital fossa, and the bilateral popliteal fossa. There are several crops of vesicles in normal-appearing skin near the eczematous patches. Which of the following is the most likely pathogen? Herpes simplex virus Measles virus Staphylococcus aureus Streptococcus pyogenes Trichophyton rubrum

Herpes simplex virus Primary eczema herpeticum is a herpes simplex virus infection in a patient with concurrent atopic dermatitis. The infection typically affects eczematous skin and then spreads to adjacent normal-appearing skin. The lesions initially appear as vesicles and then eventually appear as punched-out erosions that result in denuded areas of skin. This often is painful and can be associated with fever, general malaise, and irritability. Successive new lesions, spreading out from the eczematous skin, may continue to appear over several weeks. Diagnosis can be made clinically, but antigen detection tests or viral cultures will confirm the diagnosis. The treatment for eczema herpeticum consists of beginning oral antiviral therapy promptly to help avoid hospitalization and complications from infection. Immunocompromised patients are at greater risk of developing a severe disseminated infection. As with other herpes infections, recurrent subsequent infections will be milder in presentation.

A 28-year-old woman presents to the clinic with gradual weight gain for the past 6 months. She also reports that her menstrual cycles have been infrequent and heavy. Vital signs today include a heart rate of 80 bpm, blood pressure of 155/95 mm Hg, respiratory rate of 20/minute, oxygen saturation of 98% on room air, and temperature of 98.6°F. Physical examination reveals a regular heart rate and rhythm and lungs that are clear to auscultation. You also notice excess dark hair around the patient's mouth and chin and that she has increased abdominal and facial fat with proximal muscle wasting. Which of the following laboratory findings is most likely to be seen? High 24-hour urinary free cortisol level, high adrenocorticotropic hormone, and high dehydroepiandrosterone sulfate High 24-hour urinary free cortisol level, low adrenocorticotropic hormone, and high dehydroepiandrosterone sulfate High 24-hour urinary free cortisol level, low adrenocorticotropic hormone, and low dehydroepiandrosterone sulfate Low 24-hour urinary free cortisol level, high adrenocorticotropic hormone, and low dehydroepiandrosterone sulfate Low 24-hour urinary free cortisol level, low adrenocorticotropic hormone, and low dehydroepiandrosterone sulfate

High 24-hour urinary free cortisol level, high adrenocorticotropic hormone, and high dehydroepiandrosterone sulfate This patient has symptoms consistent with Cushing syndrome, which refers to a collection of symptoms and signs caused by excess levels of glucocorticoids. This may be due to exogenous etiologies (which are caused by administration of glucocorticoids for another condition) or endogenous etiologies. The endogenous causes are further classified as adrenocorticotropic hormone (ACTH) dependent or independent. The most common endogenous cause of Cushing syndrome is Cushing disease, which is caused by an ACTH-producing pituitary tumor and accounts for 65-70% of Cushing syndrome cases. Other endogenous causes include adrenal tumors, ectopic ACTH secretion by nonpituitary tumors such as small cell lung cancer, and ectopic corticotropin-releasing hormone secretion by nonhypothalamic tumors. The symptoms and signs seen in Cushing syndrome include obesity, hypertension, menstrual irregularities, glucose intolerance, and hirsutism. Patients also often have an atypical pattern of fat distribution, which includes increased abdominal and facial fat but proximal muscle wasting and weakness. Cushing syndrome should be considered in patients with the characteristic symptoms (facial plethora, proximal muscle weakness, striae > 1 cm, and easy bruising), unusual findings for their age (osteoporosis or hypertension in young adults), and adrenal adenomas identified on imaging. The diagnosis requires biochemical testing with initial screening for Cushing syndrome with late-night salivary cortisol, 24-hour urinary free cortisol, and overnight dexamethasone suppression test. An elevated 24-hour urinary free cortisol level is suggestive of Cushing syndrome. The next step in evaluation is to determine the cause. It is important to rule out an exogenous cause of Cushing syndrome by ascertaining whether the patient is taking glucocorticoids, such as prednisone. Measuring the ACTH level distinguishes whether the syndrome is ACTH independent or dependent. Patients with ACTH-independent causes, such as adrenal tumors, have a low ACTH level, while patients with ACTH-dependent causes, including pituitary adenoma and nonpituitary ACTH-secreting tumor, have an elevated level of ACTH. It is also helpful to measure the dehydroepiandrosterone sulfate (DHEAS) level since it is typically decreased in adrenal causes of Cushing syndrome but is normal or increased in ACTH-dependent causes, including Cushing disease. Because Cushing disease is the most likely endogenous cause of Cushing syndrome, the expected laboratory findings for this patient would be a high 24-hour urinary free cortisol level, high ACTH level, and high DHEAS level. The next step in confirming the cause is imaging to localize the tumor. Patients with a suspected pituitary adenoma should have a brain MRI, while those with a suspected adrenal tumor should have a CT scan of the abdomen and pelvis with contrast. The treatment of endogenous Cushing syndrome consists of surgical removal of the causative tumor. Treating exogenous causes requires tapering of steroids.

A 28-year-old woman with no past medical history presents to the emergency department with acute dyspnea. Physical exam reveals tachycardia, warm extremities, wide-pulse pressure, bounding pulses, a systolic flow murmur, exophthalmos and a neck mass. Which of the following is the most likely diagnosis? Aortic regurgitation High output heart failure Low output heart failure Methamphetamine intoxication

High output heart failure This patient most likely has high-output heart failure secondary to thyrotoxicosis. High output heart failure occurs when cardiac output is elevated in patients with reduced systemic vascular resistance. Examples include thyrotoxicosis, anemia, pregnancy, beriberi and Paget's disease. Patients with high output heart failure usually have normal pump function, but it is not adequate to meet the high metabolic demands. In high output heart failure the heart rate is typically elevated, the pulse is usually bounding and the pulse pressure wide. Pistol-shot sounds may be auscultated over the femoral arteries, which is referred to as Traube's sign. Subungual capillary pulsations, often referred to as Quincke's pulse, may also be present. Although these findings may be seen in other cardiac conditions, such as aortic regurgitation or patent ductus arteriosus, in the absence of those conditions, these signs are highly suggestive of elevated left ventricular stroke volume due to a hyperdynamic state. Patients with chronic high output also may develop signs and symptoms classically associated with the more common low-output heart failure; specifically, they may develop pulmonary or systemic venous congestion or both, while maintaining the above normal cardiac output.

Tonya is a 35-year-old nurse who presented to the emergency department last week for a neurologic event. Today she presents with fatigue, generalized dull aching, decreased color vision in her left eye, and right arm and leg weakness that has lasted for 48 hours. A magnetic resonance imaging study of the brain showed no evidence of a stroke but revealed scattered T2 lesions. What is the treatment for this acute episode? Glatiramer High-dose corticosteroids Interferon beta Low-dose corticosteroids

High-dose corticosteroids This woman presents with the classic signs and symptoms of multiple sclerosis. In most cases, the disease follows a relapsing-remitting pattern, with short-term episodes of neurologic deficits that resolve completely or almost completely. A minority of patients experience steadily progressive neurologic deterioration. Typical symptoms include sensory disturbances, motor weakness, optic neuritis, impaired coordination, and fatigue. Multiple sclerosis is a clinical diagnosis. Two neurologic deficits lasting for at least 24 hours, separated in space and time are considered diagnostic. Magnetic resonance imaging is highly sensitive for revealing sharply delineated regions of demyelination throughout the central nervous system white matter, especially in periventricular areas, and can be used to diagnose multiple sclerosis in cases not meeting the threshold for clinical diagnosis. Cerebrospinal fluid is also obtained for diagnostic purposes. The analysis typically demonstrates oligoclonal bands and an increased immunoglobulin G concentration. Exacerbations affect 85% of patients with multiple sclerosis; infections and stress may play a role. For those with significant, acute symptoms, corticosteroids are the treatment of choice. The initial drug therapy for acute episodes, whether CIS or a subsequent acute episode, is intravenous corticosteroids. Corticosteroids decrease T-cell response to antigens, production of cytokines, secretion of immunoglobulins, and levels of myelin basic protein in cerebrospinal fluid. Treatment with high-dose intravenous methylprednisolone (more than 500 mg/day for at least 3 days) is a typical recommended treatment. Disease-modifying agents should be initiated early in the treatment of multiple sclerosis to forestall disease and preserve function. Symptom management constitutes a large part of care; neurogenic bladder and bowel, sexual dysfunction, pain, spasticity, and fatigue are best treated with a multidisciplinary approach to improve quality of life.

An 18-year-old man presents to the ED near where he resides in southern Missouri, complaining of fever and shortness of breath for two weeks. His temperature is 37.5°C, heart rate 107 bpm, blood pressure 115/76 mm Hg, respiratory rate 22 breaths per minute, and his oxygen saturation on room air is 94%. He is a construction worker whose hobbies include hiking, caving, and hunting. A chest radiograph shows multiple granulomas and hilar adenopathy. Which of the following organisms is most likely responsible for his presentation? Blastomyces dermatitidis Candida glabrata Coccidioides immitis Histoplasma capsulatum Taenia saginata

Histoplasma capsulatum This man has multiple factors in his history that are consistent with a fungal pulmonary infection caused by Histoplasma. He lives in southern Missouri, the correct region. He also works in construction and has a hobby of caving. Histoplasmosis is typically linked to contact with bird or bat droppings or exposure to construction sites. It is found almost exclusively in the Mississippi and Ohio River valleys. The disseminated form of this illness can be asymptomatic or can cause a pneumonia-like illness. Findings on chest radiograph can be variable but often include hilar adenopathy.

A 34-year old resident physician from Iowa presents for a health examination prior to hospital employment. His examination is unremarkable, but a chest radiograph shows bilateral lung fields with BB-sized calcifications and hilar adenopathy. A PPD skin test is negative. The findings in this patient are most likely a result of which of the following? Coccidioidomycosis Cryptococcosis Histoplasmosis Tuberculosis

Histoplasmosis The majority of people with normal immunity who develop histoplasmosis manifest an asymptomatic or clinically insignificant infection. The most common abnormality on chest radiograph is a solitary pulmonary calcification. Cavitation is rare, but hilar and mediastinal adenopathy is seen frequently. Nodules, if present, are multiple and have a "BB-sized" appearance. It is highly prevalent in the Midwestern United States and exposure to bird or bat excrement is a common cause. The spectrum of this illness ranges from asymptomatic infection to severe disseminated disease. Culture remains the gold standard for diagnosis but requires a lengthy incubation period. Fungal staining produces quicker results than culture but is less sensitive. Therapy is indicated in chronic or disseminated disease and severe, acute infection. Amphotericin B is the agent of choice in severe cases and itraconazole is effective in moderate disease

The patient shown above presents to the clinic reporting new-onset right shoulder pain. He has a medical history significant for hypertension, atherosclerosis, and peripheral venous insufficiency. He is currently taking lisinopril, atorvastatin, propranolol, and aspirin. Physical examination reveals no obvious deformity of the shoulder with normal, pain-free active and passive range of motion of the neck, shoulder, and elbow. Circulation, motor innervation, and sensory innervation are intact and normal compared bilaterally. Chest auscultation reveals a normal rate and rhythm with regular respiratory rate and breath sounds bilaterally. An ECG demonstrates no acute pathology with normal sinus rhythm. Which of the following is most likely also reported by the patient? Catching and locking with shoulder motion History of tobacco use History of varicella-zoster Recent trauma Uncontrolled diabetes mellitus

History of tobacco use Superior pulmonary sulcus tumors, or Pancoast tumors, are lung neoplasms located at the apical pleuropulmonary groove, adjacent to the subclavian vessels. Like other pulmonary neoplasms, the primary cause of a pancoast tumor is tobacco use, with other risk factors including occupational and environmental carcinogens, such as asbestos, radon, and diesel exhausts. Shoulder pain in the C8, T1, and T2 dermatomes is the most common presenting symptom related to Pancoast tumors and results from invasion of the brachial plexus or extension of the tumor into the parietal pleura, endothoracic fascia, first and second ribs, or vertebral bodies. Involvement of the paravertebral sympathetic chain and inferior cervical ganglion results in ipsilateral ptosis with narrowing of the palpebral fissure, miosis, enophthalmos, and anhidrosis. This constellation of symptoms is commonly referred to as Horner syndrome. As these lesions are located in the lung periphery, pulmonary symptoms are not common initially but rather are seen with disease progression. Diagnosis is confirmed using core-needle biopsy to collect tissue for histologic evaluation. Most superior sulcus tumors are non-small cell lung cancers, predominantly squamous cell carcinoma or adenocarcinoma. Initial radiographic findings include a unilateral apical cap of more than 5 mm, asymmetry of bilateral apical caps of more than 5 mm, an apical mass, and local bone destruction. Computed tomography of the chest and upper abdomen should be implemented to evaluate for metastatic spread of the disease. Magnetic resonance imaging is useful in evaluating involvement of soft tissue structures, such as the brachial plexus and subclavian vessels, in addition to the vertebral bodies and spinal canal. Staging is determined using the tumor, node, metastasis (TNM) system with most Pancoast tumors being T3 or T4 lesions. Treatment depends on staging and the presence of metastatic disease, with concurrent chemoradiotherapy being the initial management for patients without metastatic spread. Surgical resection is the definitive treatment for patients without distant metastases or local progression. Patients should be followed with a history, physical examination, and computed tomography of the chest every 3-6 months for the first 3 years, then every 6 months for 2 years, and annually thereafter.

A 20-year-old woman presents to the emergency department with a complaint of acute onset generalized severe pain after playing a soccer match. A complete blood count demonstrates a hematocrit of 20 percent with reticulocytosis. Which of the following are you likely to find on peripheral blood smear? Basophilic stippling Burr cells Heinz bodies Howell-Jolly bodies

Howell-Jolly bodies Howell-Jolly bodies are nuclear remnants of red blood cells that are normally removed by the spleen but become present on peripheral blood smear when there is a malfunction of the spleen as seen in sickle cell anemia. Sickle cell anemia is a blood disorder characterized by defective hemoglobin that inhibits the ability to carry oxygen through blood vessels. These sickled cells tend to cluster together in vessels, eventually causing blockages leading to a vaso-occlusive crisis known as sickle cell crisis. This is characterized by acute excruciating pain and can be caused by dehydration, infection, stress, or even weather conditions. Laboratory findings demonstrate a low hematocrit with reticulocytosis, and peripheral blood smear demonstrates sickled red blood cells and Howell-Jolly bodies, which represents hypofunction of the spleen due to repeated splenic infarcts.

A 16-year old boy presents with a superficial bite to his right arm. He states he was camping in the woods and a bat flew into his tent. Which of the following treatments should this patient receive? Human rabies immunoglobulin and 3 doses of inactivated rabies vaccine over 7 days Human rabies immunoglobulin and 4 doses of inactivated rabies vaccine over 14 days Human rabies immunoglobulin only Inactivated rabies vaccine only

Human rabies immunoglobulin and 4 doses of inactivated rabies vaccine over 14 days The rabies virus enters the CNS and causes a wide range of symptoms from headaches, anorexia, hallucinations, agitation to seizures, and hydrophobia. Bats are the most common animals to infect humans. Raccoons are the most common animal infected with rabies. The patient's history and presentation suggest an infection with the rabies virus and post-exposure prophylaxis with the inactivated rabies vaccine and the human rabies immunoglobulin should be administered. The human rabies immunoglobulin is a one time dose where as the inactivated rabies vaccine is given 4 times over 14 days. In addition the bite wound should be examined and cleaned and antibiotics may be of benefit.

A 4-month-old baby presents to your office with symptoms of worsening constipation, poor feeding, listlessness, and generalized weakness for two weeks. Physical exam findings include temperature of 98.6°F, ptosis, poor head control and poor ability to suck. Which of the following is the most appropriate initial therapy? Bisacodyl Human-derived botulism immune globulin Magnesium citrate Senna

Human-derived botulism immune globulin Infant botulism, while rare, is the most common form of botulism seen in the United States. Raw honey is a frequently mentioned etiology of infant botulism. Other causes include corn syrup and soil and vacuum dust. The initial clinical manifestation is constipation, followed by motor function symptoms such as ptosis, facial and generalized weakness. Clinical presentation and electromyography findings consistent with infant botulism allow for a presumptive diagnosis while confirmatory stool studies are pending. Treatment in infants under age 1 year is with human-derived botulism immune globulin (BabyBIG) and should be administered as early as possible in the course of the infection. Equine-derived heptavalent botulinum antitoxin is used in non-infant cases of botulism.

You suspect hypoparathyroidism in a patient who recently underwent a course of neck radiotherapy. Which of the following laboratory results would you expect to find if this patient began showing signs of carpopedal spasm and lip paresthesias? Hypercalcemia and hyperphosphatemia Hypercalcemia and hypophosphatemia Hypocalcemia and hyperphosphatemia Hypocalcemia and hypophosphatemia

Hypocalcemia and hyperphosphatemia Calcium and phosphate homeostasis is maintained by an interplay between the parathyroid glands, intestines, kidneys and bones. The normal action of parathyroid hormone (PTH) is to increase serum calcium levels and decrease serum phosphate levels. PTH affects the kidneys by increasing calcium reabsorption, decreasing renal phosphate reabsorption, and increasing active-vitamin D formation, and affects the bones by increasing osteoclast activity. Hypoparathyroidism can have primary and secondary causes. In primary hypoparathyroidism, the causes are congenital, acquired (such as thyroidectomy, parathyroidectomy, neck radiation), autoimmune or associated with magnesium deficiency or heavy metal diseases. Patients may present with symptoms of hypocalcemia: paresthesias (especially of the lips, mouth and fingertips), cramping, tetany, carpopedal spasm, altered mental status, irritability, mood swings, anxiety, stridor and hoarseness, wheezing and bronchospasm, diaphoresis and seizures. Laboratory testing which suggests primary hypoparathyroidism includes decreased PTH, hypocalcemia and hyperphosphatemia.

A 40-year-old man presents to the office with his wife, who states she has noticed some personality changes and aggressive behavior that began a few months ago. He is now having brief, irregular involuntary movement of his arms, is unable to focus, and is having trouble problem-solving. His wife also describes excessive blinking and head thrusts. Which of the following conditions are these findings suspicious for? Alzheimer disease Huntington disease Parkinson disease Wilson disease

Huntington disease Huntington disease is a fully penetrant autosomal dominant neurodegenerative disorder caused by an expanded trinucleotide (CAG) repeat in the gene for the protein huntingtin. The prevalence is approximately 10 per 100,000. Symptoms include a combination of a movement disorder, psychiatric disturbances, and cognitive dysfunction. Early on, the movement disorder is predominantly chorea, but parkinsonism and dystonia develop later. Some patients, especially those with juvenile onset, have a more rapidly progressive akinetic-rigid and dystonic form (the Westphal variant). Psychiatric manifestations, which are universal but widely variable, include personality changes, impulsiveness, aggressive behavior, depression, and paranoid psychosis. These psychiatric symptoms may precede the motor manifestations, and psychotropic drug therapy may be incorrectly blamed for the subsequent development of the movement disorder. Cognitive changes result in progressive subcortical dementia with disturbed attention, concentration, judgment, and problem-solving that differs from the typical cortical dementia of Alzheimer disease. Oculomotor dysfunction most often manifests in difficulties with refixating the gaze, and a resulting tendency to use blinks and head thrusts is common. The diagnosis is confirmed by genetic testing.

An 8-month-old boy has a painless swelling on the right side of his scrotum since birth that gets worse with crying. Transillumination of this side of the scrotum reveals increased fluid and the child does not cry on palpation of the testicle which lies in its normal position. Based on these findings, which of the following is the most likely diagnosis? Epididymitis Hydrocele Testicular torsion Varicocele

Hydrocele A hydrocele is a collection of fluid that accumulates in the tunica vaginalis. Communicating hydroceles result when the upper processus vaginalis fails to be obliterated, leaving an open tract between the peritoneum and the scrotum. The tract is closed in non-communicating hydroceles. Most hydroceles are right sided. Hydroceles may be present at birth, but they usually are painless and worsen with crying or exertion. Hydroceles often resolve spontaneously by the age of 18 months. Examination reveals transillumination and enlargement of the scrotum. Patients with a hydrocele that has persisted for more than 1 year or who are older than 18 months should undergo ultrasonography to ensure that the hydrocele is not a reactive hydrocele caused by testicular tumor or inflammatory process.

An 18-year-old African-American man presents with questions about his diagnosis of sickle cell anemia. He has been managing his condition since childhood by treating his symptoms and having occasional blood transfusions. Now he is experiencing more frequent painful episodes and wants to know if there are other treatment options. Which of the following is the most appropriate pharmacologic treatment? Ferrous sulfate Hydroxyurea Oxycodone Prednisone

Hydroxyurea Sickle cell anemia is an autosomal recessive genetic disorder that causes a mutated form of hemoglobin S. This mutation leads to chronic vaso-occlusive crises in affected individuals and causes a number of other health problems. It is most commonly found in persons of African ancestry. Individuals may be carriers of the sickle cell trait, meaning that they carry one mutated gene for hemoglobin S and one normal gene. Carriers generally do not experience the painful or vaso-occlusive crises that those with the disease experience and have some resistance to malaria. Generally diagnosed in early childhood, patients with sickle cell disease experience clinical manifestations that include acute and chronic pain, anemia, splenic sequestration, infection and involvement of multiple organ systems. Aspects of treatment include preventive of infection through immunizations, prevention of vaso-occlusive crises with hydroxyurea and management of painful episodes with rest, analgesia and hydration. Hydroxyurea increases fetal hemoglobin (HbF) production and slightly raises the total hemoglobin concentration in the body. Fetal hemoglobin reduces the chance that red blood cells will sickle in a person who has sickle cell disease. So increased production of HbF can reduce the occurrence of sickling-related complications. Treatment and management of the disease is best provided by a sickle cell disease specialist.

A 68-year-old woman with no significant medical history presents to the clinic with her daughter for cognitive changes. The daughter says that her mother gets "lost" in conversations, and that she would "sleep all day" if permitted. Further questioning reveals a recent history of constipation, as well as passing two kidney stones within the past 12 months. Which of the following electrolyte imbalances is most consistent with these symptoms? Hypercalcemia Hyperkalemia Hypocalcemia Hypokalemia

Hypercalcemia Hypercalcemia is associated with cognitive changes, malaise and fatigue, constipation, and renal calculi. Other symptoms may include polyuria, polydipsia, and bone pain. This can be remembered by the phrase "stones, bones, abdominal groans, psychic moans, and fatigued overtones." Hypercalcemia has many possible etiologies, though primary hyperparathyroidism remains the most common. Others include hypercalcemia of malignancy, multiple myeloma, sarcoidosis, prolonged immobilization, or excessive intake of calcium/vitamin D supplements. The most accurate measurement of serum calcium is the ionized calcium concentration and should be ordered whenever the total serum calcium abnormal. Until the primary cause is identified, hypercalcemia is initially managed with fluids and forced calciuresis. Intravenous saline is needed in severe cases. Intravenous bisphosphonates may be used in hypercalcemia that is due to hyperparathyroidism or a malignancy. Calcitonin can be given to manage hypercalcemia while the bisphosphonates reach therapeutic levels. Unmanaged hypercalcemia may lead to serious complications including neuromuscular manifestations such as weakness or paresthesia, or cardiac arrhythmias secondary to QT interval shortening or heart block.

Which of the following is the first-line treatment for acute pericarditis? Citalopram Ibuprofen Oxycodone Prednisone

Ibuprofen If there is a specific etiology of the pericarditis, then therapy should be directed at that etiology. Otherwise, nonsteroidal anti-inflammatory drugs (NSAIDs) and colchicine are the mainstays of therapy for acute pericarditis. Ibuprofen has the best side effect profile, but other NSAIDs should be equally effective. While many patients will have resolution of symptoms with NSAIDs alone, when used as an adjunct to NSAID therapy, colchicine reduces symptoms, decreases the rate of recurrent pericarditis, and is generally well-tolerated. Colchicine has not proven to be effective in bacterial or malignancy-related pericarditis.

A 70-year-old woman with multiple myeloma presents with confusion, lethargy, abdominal cramping, nausea, and generalized weakness. Her vital signs are normal. What electrolyte abnormality is likely responsible for this presentation? Hypercalcemia Hyperkalemia Hypocalcemia Hypokalemia

Hypercalcemia Patients with multiple myeloma are at an increased risk for hypercalcemia secondary to increased breakdown of bone. In a patient with multiple nonspecific symptoms, electrolyte disturbances should be suspected. Hypercalcemia can present with a number of symptoms, including altered mental status, abdominal pain, nausea, vomiting, weakness, lethargy, muscle aches, and depression. The most common cause of hypercalcemia is primary hyperparathyroidism. Malignancy is also a common cause. About 30% of patients with multiple myeloma will experience hypercalcemia at some point in their clinical course. Severe hypercalcemia (> 14 mg/dL) can lead to both high-degree atrioventricular blocks and ventricular dysrhythmias. Treatment of hypercalcemia focuses first on restoring intravascular volume with intravenous fluids. Patients at risk for hypercalcemia are often volume depleted secondary to chronic illness and decreased oral intake. Restoring circulating volume allows for increased perfusion to the kidneys and elimination of calcium through urine. Once intravascular volume has been repleted, the clinician can attempt to enhance renal elimination (forced diuresis) by administering a loop diuretic (e.g., furosemide). Although not typically started in the ED, bisphosphonates can be given to inhibit osteoclast activity, thus, reducing the amount of calcium entering the bloodstream.

A 72-year-old woman with hypertension, hyperlipidemia, and diabetes mellitus presents to the emergency department with left-sided numbness and weakness for the past 1 hour. The patient reports taking amlodipine, atorvastatin, and metformin daily. Vital signs include a HR of 103 bpm, BP of 142/95 mm Hg, RR of 20/min, SpO2 of 98% on room air, and T of 98.6°F. Physical examination reveals intact facial expression and 2/5 strength in the left upper and lower extremities. Sensation and motor function are intact on the right side. Head CT reveals no hemorrhage. Which of the following risk factors is most strongly associated with the patient's condition? Cigarette smoking Diabetes mellitus Family history of ischemic stroke Hyperlipidemia Hypertension

Hypertension he major modifiable risk factors associated with ischemic stroke include hypertension, dyslipidemia, diabetes mellitus, smoking, and physical inactivity. Of these risk factors, hypertension is the most significant and is present in about 80% of individuals with ischemic stroke. Other modifiable risk factors include cardiac disease (coronary artery disease, atrial fibrillation, and congestive heart failure), obstructive sleep apnea, migraine with aura, excessive alcohol consumption, illicit drug use, diet, and physical inactivity. The nonmodifiable risk factors include age, male sex, and family history of stroke. Acute ischemic strokes present with an acute onset of focal neurologic deficits. Common symptoms include focal numbness, focal weakness, visual field defects, aphasia, dysarthria, and coordination deficits. Headache is atypical for ischemic stroke and occurs more often in hemorrhagic stroke. Alterations in level of consciousness are uncommon with infarcts in one hemisphere but may occur in bihemispheric infarcts or brainstem strokes. It is important to determine the time of symptom onset in patients with acute ischemic stroke since the treatment options are time sensitive. It is also key to check a serum glucose level because hypoglycemia can mimic stroke symptoms. CT scan of the head without contrast is the initial imaging study obtained in patients with acute ischemic stroke to rule out intracerebral hemorrhage as the cause of the patient's symptoms. Evidence of ischemia may not be seen on CT for several hours. CT angiogram of the head and neck should also be obtained in patients with a suspected acute ischemic stroke to help identify large-vessel occlusion that may be amenable to endovascular thrombectomy. The treatments that may be used in acute ischemic stroke include tissue plasminogen activator (alteplase), endovascular thrombectomy, and risk factor modification for secondary prevention. Tissue plasminogen activator must be administered within 4.5 hours of symptom onset (last known normal time), and the patient must not have any contraindications.

A 74-year-old man presents to the ED with chest pain radiating to the jaw and dyspnea. His past medical history is significant for hypercholesterolemia, hypertension and diabetes. He denies illicit drug use. His blood pressure is 210/122 mm Hg. Physical exam and chest X-ray are normal. His ECG is consistent with left ventricular hypertrophy. Which of the following is the most likely diagnosis? Autonomic dysreflexia Hypertensive emergency Hypertensive urgency Sympathetic crisis

Hypertensive emergency A hypertensive emergency is a severe elevation in blood pressure with evidence of end-organ damage. This requires immediate lowering of blood pressure. There is no specific blood pressure at which hypertensive emergency occurs, however, end-organ damage is less likely if the diastolic BP is < 130 mm Hg. With that being said, the well-accepted criteria for hypertensive crisis are systolic pressure ≥180 mm Hg or diastolic pressure ≥ 110 mm Hg. One must further consider the patient's baseline blood pressure, as a patient with chronic hypertension may not have end-organ damage with pressures around 200/150 mm Hg. Precipitants of hypertensive emergencies include progression of essential hypertension (especially if there is medical noncompliance), progression of renovascular disease, acute cardiac or cerebral ischemic injury and undiagnosed or progressive endocrinopathies. Symptoms of hypertensive emergency include chest pain, dyspnea and neurologic deficits. Associated clinical scenarios include encephalopathy, hemorrhagic or ischemic stroke, aortic dissection, acute myocardial infarction, acute coronary syndrome, acute renal failure, pulmonary edema with respiratory failure, microangiopathic hemolytic anemia and pre-eclampsia/eclampsia/HELLP syndrome.

A patient is found to be hyponatremic. Laboratory evaluation reveals low serum osmolality, urine sodium concentration > 20 mmol/L, and a fractional excretion of sodium (FENa) > 1%. He appears to be "fluid overloaded." Which of the following is the most likely cause of this hyponatremia? Cirrhosis Heart failure Hypertensive nephropathy Syndrome of inappropriate ADH release (SIADH)

Hypertensive nephropathy Hyponatremia is defined as sodium less than 135 mEq/L. Hyponatremia can occur in a hypovolemic, euvolemic, or hypervolemic state. Hypervolemic hypo-osmolar hyponatremia is associated with fluid overload. The etiology is usually from a perceived low intravascular volume by the kidneys and active water reabsorption in excess to sodium retention. If urine sodium is low (<20) causes include liver failure, cirrhosis, hepatorenal syndrome, nephrotic syndrome, and heart failure. If urine sodium is high (>20) causes include acute or chronic renal failure, such as that caused by hypertensive nephropathy. Treatment of hypervolemic hypo-osmolar hyponatremia is dialysis. Cirrhosis (A) and heart failure (B) is often the cause of hypervolemic hypo-osmolar hyponatremia when the urine sodium is low (<20). SIADH (D) results in euvolemic hyponatremia with urine osmolality greater than serum osmolality. The excess ADH causes total body water to increase thereby diluting total body sodium. Despite the increased total body water, these patients typically do not show evidence of edema or heart failure as the increased water is intracellular not intravascular

A man with dyspnea-on-exertion presents for cardiac evaluation. Physical exam is significant for a left sternal border systolic murmur which is louder during a Valsalva maneuver. An S4 is also appreciated. The ECG shows large QRS complexes. An echocardiogram reveals a decrease in left ventricular chamber volume and normal left atrial measurements. Which of the following is the most likely diagnosis? Aortic stenosis Dilated cardiomyopathy Hypertrophic cardiomyopathy Mitral stenosis

Hypertrophic cardiomyopathy Hypertrophic cardiomyopathy (HCM) results from left or right ventricular hypertrophy or both. This condition can be primarily caused by autosomal dominant genetic mutations of the cardiac sarcomere genes and myocardial fiber hypertrophy. Secondary causes include aortic stenosis, mitral valve abnormalities, coronary heart disease and chronic systemic hypertension. Most patients are asymptomatic upon presentation, however, the common clinical manifestations are dyspnea, angina and dysrhythmia. Decreased chamber volume and increased ventricular wall thickness are the key echocardiographic findings in HCM. Furthermore, the ECG typically shows left ventricular hypertrophy, T-wave inversion and large QRS complexes. A harsh, left sternal border, systolic, crescendo-decrescendo murmur which is worse with Valsalva maneuver is quite typical of HCM.

A young adult with a family history of type 1 diabetes mellitus presents to the emergency room severely weak with altered mental status, nausea, and vomiting. On exam, she is hypotensive with increased pigmentation at her mucus membranes and palmar creases. Which of the following laboratory findings would you expect in this patient? Hyperglycemia Hypocalcemia Hypokalemia Hyponatremia

Hyponatremia Hyponatremia is an expected laboratory finding in patients with acute adrenal insufficiency, of which this patient is showing signs and symptoms. Acute adrenal insufficiency is a medical emergency caused by insufficient plasma cortisol. It may occur for many reason, such as pituitary hypofunction, rapid withdrawal of steroid medications, or following extreme physiologic stress. However, this patient is most likely suffering from primary adrenal insufficiency, or Addison disease, as evidenced by the hyperpigmentation at this patient's skin and mucus membranes. Her family history of type 1 diabetes mellitus, an autoimmune condition, also increases her risk of Addison's disease. Other signs and symptoms of primary adrenal insufficiency include hypotension, abdominal pain, nausea, vomiting, diarrhea, fever, and confusion. Coma may result if untreated. Lab findings can include persistent hypoglycemia, eosinophilia, and hypercalcemia. Hyponatremia and hyperkalemia will result if there is a chronic mineralocorticoid deficiency. Differentiation of primary adrenal insufficiency from secondary adrenal insufficiency is accomplished with a cosyntropin, or ACTH, stimulation test. Though both types will present with depressed plasma cortisol, administration of synthetic ACTH will prompt a cortisol increase in patients with secondary adrenal insufficiency as their adrenal cortex is intact. However, this test will not result in a cortisol increase in primary adrenal insufficiency due to the lack of a functional adrenal cortex. Treatment of secondary adrenal insufficiency is dependent on the cause. Treatment of primary adrenal insufficiency generally requires daily hormone replacement with oral hydrocortisone, dexamethasone, or prednisone to prevent future adrenal crises.

A 40-year-old woman presents with weakness, fatigue, nausea, and diarrhea. Physical exam reveals orthostatic hypotension and axillary fold hyperpigmentation. Which of the following laboratory abnormalities would you expect to find in this patient? Hypercalcemia Hypermagnesemia Hypokalemia Hyponatremia

Hyponatremia Primary adrenal insufficiency (Addison disease), or hypocortisolism, is most commonly caused by autoimmune destruction of the adrenal cortex. Nonspecific symptoms of hypocortisolism include weakness, fatigue, weight loss, anorexia, orthostasis, and listlessness. The most specific sign of primary adrenal insufficiency is hyperpigmentation, typically of the mucous membranes, axillary folds, and nipples. Another specific symptom of primary adrenal insufficiency is salt craving. Because the adrenal cortex is damaged, aldosterone levels are also affected. A decrease in aldosterone will lead to less renal sodium reabsorption and less renal potassium excretion, with subsequent hyponatremia and hyperkalemia. A net loss of sodium to the urine may lead to polyuria and hypovolemia. Hyponatremia associated with adrenal insufficiency may cause seizures, delirium, coma, or death.

A 60-year-old man is admitted to the general medicine floor. He presented to the emergency department 1 week ago with bilateral leg weakness that had worsened to include bilateral arm and hand weakness after recovering from a recent gastrointestinal infection. You are performing your daily rounds. The patient reports feeling well this morning. On exam, he has a T of 97°F, RR of 18/min, oxygen saturation of 99% on room air, HR of 85 bpm, and BP of 117/77 mm Hg. The patient has not had trouble breathing independently. Based on the suspected diagnosis, what is the most likely exam finding? Bilateral papilledema Cogwheel rigidity Hyporeflexia Paresthesias Urinary retention

Hyporeflexia Guillain-Barré syndrome is an immune-mediated polyneuropathy that is triggered from an event such as recent illness, commonly gastrointestinal illness or influenza. It can also be triggered by vaccinations, although the risk is very low. Gastrointestinal illness is the most common preceding event. It will present with symmetric, ascending, progressive muscle weakness, and up to 90% of patients will have diminished or absent deep tendon reflexes on initial exam and hyporeflexia will be present at the nadir of the disease. Patients may develop complete paralysis. Dysautonomia or autonomic dysfunction can also be a physical exam finding and may present as hypertension, ileus, fever, tachycardia, or urinary retention. Many patients will have to be intubated due to the ascending muscle weakness affecting their respiratory system. Diagnosis is clinical and must include progressive, ascending weakness with hyporeflexia. Diagnosis is confirmed via lumbar puncture which reveals elevated protein but normal white blood cell counts. If diagnosed within 4 weeks of symptom onset, intravenous immune globulin (IVIG) therapy is recommended. Otherwise, care is supportive and focused on rehabilitation. About 80% of patients will recover fully within 1 year.

A 63-year-old man with prostate cancer and bony metastasis presents with nausea, decreased oral intake, constipation, generalized fatigue, and mild confusion. His vitals are T 37.1°C, HR 102 bpm, BP 95/57 mm Hg, RR 20/min, oxygen saturation 96%, and finger stick blood glucose 102 mg/dL. On examination, he has slow mentation, normal pupil size, dry mucous membranes, and decreased bowel sounds. A chemistry panel shows the following: Sodium: 133 mEq/L Potassium: 3.3 mEq/L Chloride: 97 mEq/L Bicarbonate: 23 mEq/L Blood urea nitrogen: 27 mg/dL Creatinine: 2.2 mg/dL Calcium: 13.1 mg/dL Magnesium: 2.1 mg/dL Which of the following treatments should be initiated immediately? Bisphosphonates Calcitonin Intravenous furosemide IV normal saline

IV normal saline This patient's multiple symptoms are caused by hypercalcemia secondary to osteolytic bone metastasis from his prostate cancer. Hypercalcemia often presents with nonspecific symptoms, including fatigue, weakness, confusion, hypertension, bradycardia, polyuria, polydipsia, dehydration, nausea, vomiting, constipation, ataxia, and coma. Any symptomatic hypercalcemia or asymptomatic hypercalcemia with a serum level > 14 mg/dL should be aggressively treated. Treatment should start with repletion of intravascular volume with isotonic saline. Restoring glomerular filtration rate (GFR) will lead to renal calcium clearance.

A one-year-old boy of Mediterranean descent is found to have mild anemia on routine blood work. Further workup confirms the presence of beta-thalassemia trait. Which of the following is true regarding beta-thalassemia trait? If both parents have thalassemia trait with genotype β/β0, the risk of having a child with beta-thalassemia major is 25 percent It is inherited as an autosomal-dominant trait Management is with chronic blood transfusions and iron chelation therapy Mean cell volume (MCV) is usually above 80 fL/cell

If both parents have thalassemia trait with genotype β/β0, the risk of having a child with beta-thalassemia major is 25 percent A patient with anemia who is from the Mediterranean region and has a family history of anemia should raise the possibility of beta-thalassemia trait. Beta-thalassemia trait is caused by a defect in the single beta-globin. It is inherited as an autosomal recessive trait and if both parents are affected with genotype β/β0, the risk of having a child with beta thalassemia major is 25 percent. There are few findings on physical exam except for mild pallor. Beta-thalassemia trait is characterized as a mild hypochromic microcytic anemia. Older children may even have hemoglobin levels very close to normal. The peripheral blood smear often shows a variation in the size and shape of the red blood cells, hypochromia and basophilic stippling. A quantitative hemoglobin electrophoresis is usually done for confirmation. There is no specific treatment as beta-thalassemia trait is a benign condition. Establishing the diagnosis is important to prevent unnecessary treatment for iron deficiency.

A 32-year-old woman presents with abdominal pain, nausea, vomiting, and change in skin color for 6 days. She states that she had unprotected intercourse 4 weeks ago. Which one of the following tests indicates acute infection with hepatitis B as the cause of the patient's symptoms? Antibody to hepatitis B e antigen (Anti-HBe) Antibody to hepatitis B surface antigen (Anti-HBs) IgG antibody to B core antigen (Anti-HBc-IgG) IgM antibody to B core antigen (Anti-HBc-IgM)

IgM antibody to B core antigen (Anti-HBc-IgM) This patient presents with symptoms consistent with acute Hepatitis B virus (HBV) infection. HBV is primarily transmitted through parenteral exposure (needle stick, intravenous drug use) or through unprotected intercourse. Transmission through blood transfusion is rare due to advances in screening techniques. Acute viral hepatitis presents with malaise, fever, anorexia, nausea, vomiting, abdominal discomfort, and diarrhea. Often, jaundice leads patients to consult a physician. Fulminant hepatitis is characterized by the acute onset of hepatic failure and encephalopathy over a short period of time (usually days). Measurement of hepatic enzymes can demonstrate 10- to 100-fold elevations of aspartate aminotransferase (AST) and alanine aminotransferase (ALT) levels. ALT is usually elevated to a greater degree than AST in viral hepatitis (the reverse is usually true in alcoholic hepatitis). Hyperbilirubinemia can be moderate (5-10 mg/dL) or severe (15-25 mg/dL) and usually presents days to weeks after the onset of symptoms. Both direct and indirect bilirubin will be elevated in viral hepatitis. Serum testing can also be used to diagnose the type of viral hepatitis. Acute hepatitis B is characterized by the presence of IgM antibody to B core antigen (Anti-HBc-IgM).

A 35-year-old woman with obesity presents to the emergency department with epigastric pain and vomiting for the past 12 hours. Her vital signs today include a heart rate of 85 bpm, blood pressure of 125/85 mm Hg, respiratory rate of 20/minute, oxygen saturation of 97% on room air, and temperature of 98.6°F. Physical examination reveals epigastric tenderness to palpation without guarding or rebound tenderness. The patient halts inspiration as you palpate the right upper quadrant after asking her to take a deep breath. Laboratory studies include a white blood cell count of 7,000/μL, lipase of 65 U/L, aspartate aminotransferase of 450 U/L, alanine aminotransferase of 480 U/L, alkaline phosphatase of 600 U/L, and total bilirubin of 2 mg/dL. Gallbladder ultrasound reveals gallstones in the gallbladder and a proximal common bile duct diameter of 8 mm. There is no cholecystic fluid or gallbladder wall thickening. Which of the following explains the etiology of the most likely condition? Impacted stone in the common bile duct Impacted stone in the cystic duct Infection of the biliary tree secondary to obstruction Inflammation of the pancreas due to a stone in the main pancreatic duct Viral hepatitis

Impacted stone in the common bile duct Choledocholithiasis refers to the presence of gallstones impacted in the common bile duct. In most cases, choledocholithiasis occurs because a stone has passed from the gallbladder into the common bile duct. The clinical manifestations include pain consistent with biliary colic, such as epigastric or right upper quadrant pain associated with nausea and vomiting. Patients with uncomplicated choledocholithiasis are afebrile and without leukocytosis or elevated pancreatic enzyme (lipase) levels. Physical exam findings include epigastric or right upper quadrant tenderness. The patient may have scleral icterus or appear jaundiced, depending on the degree of bilirubin elevation. Laboratory findings include elevation in aspartate aminotransferase (AST), alanine aminotransferase (ALT), alkaline phosphatase, and bilirubin. The liver enzymes are usually elevated in a cholestatic pattern (alkaline phosphatase elevated more than AST and ALT) in later presentations. Abdominal ultrasound is the initial recommended imaging modality. Abdominal ultrasound may show stones in the gallbladder and dilation of the common bile duct. However, it is not a sensitive study for visualizing stones in the common bile duct. After laboratory studies and abdominal ultrasound, patients are typically stratified as being low, intermediate, or high risk of choledocholithiasis. Patients who have a common bile duct stone identified on abdominal ultrasound are at high risk of choledocholithiasis and are treated with cholecystectomy and endoscopic retrograde cholangiopancreatography (ERCP), an endoscopic procedure to remove stones in the common bile duct. Patients who present with symptoms of biliary colic and have either elevated liver biochemical tests or dilation of the common bile duct on ultrasound should undergo either a cholecystectomy with intraoperative cholangiogram or a magnetic resonance cholangiopancreatography (MRCP). MRCP is a more sensitive diagnostic modality to image the biliary tract and identify stones. Patients who have a stone on MRCP require ERCP to remove it. Patients who have biliary pain with gallstones but normal liver biochemical tests and normal common bile duct diameter are at low risk for choledocholithiasis and should be treated with cholecystectomy.

A 35-year-old man presents to the primary care clinic with heartburn after eating. Pain occurs mostly in the evening, approximately 30 minutes after consuming his evening meal. He also reports mild regurgitation and belching at night. He reports no dysphagia, bloody stools, or weight loss. For the past 2 months, he has taken omeprazole 20 mg every morning but is on no other medication, and he has no other significant medical history. He does not smoke. His vitals are BP 122/78 mm Hg, pulse 78 bpm, RR 15 breaths/minute, temperature 98.6°F, and O2 saturation 99% on room air. He is nontender on abdominal physical exam with no guarding or rigidity. What is the best next step in the treatment of this patient? Change omeprazole dose from morning to evening Increase omeprazole to twice a day Perform Helicobacter pylori breath test Recommend esophageal pH and impedance testing Schedule an upper gastrointestinal endoscopy

Increase omeprazole to twice a day Gastroesophageal reflux disease (GERD) is a common condition where reflux of stomach contents produces symptoms of heartburn and dyspepsia. Other symptoms include regurgitation, burning chest pain, cough, or burping. Relaxation of the lower esophageal sphincter allows reflux into the esophagus, and GERD symptoms are usually worse after meals or when reclining or laying down. Uncomplicated GERD is a clinical diagnosis and usually does not require any further diagnostics or laboratory tests. The goal of treatment is to prevent complications and provide patients with relief from their symptoms. Initially, medication therapy is a once-daily proton pump inhibitor (PPI) given 30 minutes before the morning meal. H2 receptor antagonists are sometimes used but are not as effective as PPIs. Lifestyle modifications, such as eating smaller meals and not reclining after eating, should also be discussed with patients diagnosed with GERD. If symptoms do not resolve, increasing the PPI (e.g., omeprazole) to twice daily is the next step in treatment. The second dose should be taken 30 minutes before the evening meal, and patients should avoid laying down for 3 hours.

A 23-year-old woman presents to the emergency department with sudden onset of fever, chills, cough, sore throat, and headache that began yesterday. Physical exam reveals temperature of 101.2F, clear rhinorrhea, and an injected posterior oropharynx. Lungs are clear to auscultation. She has no past medical history. Which of the following is the most likely diagnosis? Bronchitis Influenza Pneumonia Streptococcal pharyngitis

Influenza Influenza is a virus that causes an acute respiratory illness that classically presents with sudden onset fever, cough, sore throat, rhinorrhea, myalgias and headache. Early in the illness, fever and frontal headache are common. As these symptoms subside, the respiratory symptoms become more apparent. Lung auscultation will not reveal any adventitious sounds unless the infection becomes more complicated or severe. Symptoms usually last between two and five days, although they may persist for more than one week. Influenza A and B are the major causes of illness in humans, and outbreaks of the virus occur in sporadic events. Infection is initially caused by the contact with respiratory secretions from infected individuals. Complications are more common in children < 5 years of age (greatest risk < 2 years of age), adults > 65 years of age, pregnant women, individuals with medical co-morbidities (e.g. COPD, asthma, diabetes), or immunocompromise. Pneumonia is the most common complication. During the acute illness, influenza can be isolated from nasopharyngeal swabs and used in rapid influenza diagnostic tests (RIDTs). If necessary (high risk of complication), antiviral therapy can be initiated within the first 48 hours of symptoms to decrease the length of the illness. Typically, treatment includes oseltamivir or zanamivir.

14-year-old boy is brought by his mother to your clinic due to orange colored tears. Further questioning reveals that the boy was diagnosed with latent tuberculosis and was prescribed with an unrecalled antimycobacterial agent three weeks ago. Two days ago, the boy noted orange colored urine, tears and saliva. You suspect that this is an adverse effect of the drug previously prescribed. What is the mechanism of action of the drug? Binds irreversibly to the 30s subunit of ribosomes inhibiting protein synthesis Competitive antagonist of para-aminobenzoic acid which is needed for the bacterial synthesis of folic acid Inhibition of DNA-dependent RNA polymerase resulting in decreased RNA synthesis Involvement of the isoniazid gene that encodes the enoyl-acyl carrier protein reductase needed for the last step of the mycolic acid biosynthesis pathway of cell wall production

Inhibition of DNA-dependent RNA polymerase resulting in decreased RNA synthesis The boy is suffering from tuberculosis. There are five closely related mycobacteria in the Mycobacterium tuberculosis complex: M. tuberculosis, M. bovis, M. africanum, M. microti, and M. canetti. M. tuberculosis is the most important cause of tuberculosis disease in humans. The tubercle bacilli are non-spore-forming, nonmotile, pleomorphic, weakly Gram-positive curved rods 1-5 μm long, typically slender and slightly bent. Rifampin is active against M. tuberculosis, Mycobacterium leprae, M. kansasii, and Mycobacterium avium complex. Rifampin is an integral drug in standard combination treatment of active M. tuberculosis disease and can be used as an alternative to isoniazid (INH) in the treatment of latent tuberculosis infection in children who cannot tolerate INH. All rifamycins including rifampin can turn urine and other secretions (e.g., tears, saliva, stool, sputum) orange. The rifamycins inhibit the DNA-dependent RNA polymerase of mycobacteria, resulting in decreased RNA synthesis.

A 53-year-old man with a history of obesity, hypertension, and diabetes mellitus is being treated with allopurinol for recurrent gout flares. Which of the following mechanisms describes the action of allopurinol? Converts uric acid to a more soluble purine degradation product, allantoin Inhibits microtubule polymerization, preventing activation, degranulation, and migration of neutrophils Inhibits xanthine oxidase, the enzyme responsible for the conversion of hypoxanthine to xanthine to uric acid Prevents the reabsorption of uric acid at the proximal convoluted tubule, thereby promoting excretion of uric acid

Inhibits xanthine oxidase, the enzyme responsible for the conversion of hypoxanthine to xanthine to uric acid Allopurinol and its active metabolite, oxypurinol, inhibit xanthine oxidase, the enzyme responsible for the conversion of hypoxanthine to xanthine to uric acid, resulting in diminished urate production and decreased serum urate concentration. Hyperuricemia is necessary but not sufficient for development of gout, a progressive monosodium urate crystal deposition disease, and epidemiological data suggests that asymptomatic hyperuricemia may proceed onset of gout symptoms by at least 10 years. The three clinical stages in the natural history of gout are the gout flare, intercritical or interval gout, and chronic arthropathy and tophaceous gout. Acute onset monoarticular redness, warmth, swelling, and pain, often involving the knee or first metatarsal phalangeal joint, characterizes an initial gout flare. Involvement of other joints, such as a wrist, finger, elbow, shoulder, or hip, or involvement of multiple joints is more common in a recurrent gout flare. Between gout flares, patients are usually asymptomatic. Tophi, collections of solid urate, specifically distinguishes tophaceous gout. Prevention of recurrent gout flares and disease progression is achieved and maintained by reducing serum urate levels using lifestyle modifications and pharmacotherapy. For overweight or obese patients with established gout, weight loss towards ideal body weight may result in fewer recurrent gout flares. Avoidance of potential triggers for gout flares, such as alcohol binges, high purine/protein and fatty meals, and variable medication adherence, may also decrease the likelihood of recurrence. Pharmacologic urate-lowering therapy is indicated for patients with multiple recurrent gout flares annually or with clinical or imaging findings indicative of arthropathy or the development of tophi. Allopurinol is appropriate initial medical therapy for most patients. Recommended initial dosing of allopurinol is 100 mg orally daily, with dose titration every two to four weeks to reach and maintain the urate-lowering goal. The starting dose of allopurinol as well as the rate of dose adjustment should be modified for patients with chronic kidney disease. Patients should receive prophylaxis with colchicine or nonsteroidal anti-inflammatory drugs for flares when initiating urate-lowering medication due to increased likelihood of gout recurrence early in treatment.

A 52-year-old man with a long history of alcohol use disorder presents with epigastric pain and vomiting. You diagnose him with pancreatitis. Which of the following laboratory values is associated with an increased risk of mortality on admission? ALT 350 mmol/L LDH 400 IU/L Lipase 14,000 U/L WBC 15,000 cells/mm3

LDH 400 IU/L The Ranson criteria include a number of parameters which may be used in order to calculate a score predictive of mortality from pancreatitis. The criteria actually has two parts, the first score calculated at the time of admission and the second within 48 hours of admission. With a rising score, the risk of mortality increases and many sources advise consideration of admission to the ICU for a score of 3 or more. An LDH value of >350 IU/L on admission is part of the criteria.

A previously healthy 30-year-old man presents to your clinic with concerns about screening for cholesterol. He has never smoked, is physically active, and is in the normal range for body mass index and blood pressure. He says that his father started taking medication for hyperlipidemia at age 66 and he wants to know when he should start being screened. Which of the following is the most appropriate next step in management according to the United States Preventative Services Task Force? Initiate lipid screening at age 35 Initiate lipid screening at age 45 Initiate preventative therapy with aspirin Initiate preventative therapy with lovastatin

Initiate lipid screening at age 35 Screening for lipid abnormalities is a part of determining overall cardiovascular risk. Lipid-lowering therapy is one way to improve cardiovascular outcomes and patients at risk should be screened to evaluate the need for treatment with aspirin or statins. The need for and timing of screening is determined by assessing patient risk factors including age, sex, and other risk factors for cardiovascular disease such as smoking, hypertension and family history of premature coronary heart disease. Patients are at higher risk if they have more than one risk factor or a single risk factor that is severe. Examples of severe single risk factors include a heavy smoking history or multiple first-degree family members with coronary heart disease at a young age. Men determined to be at low risk should start screening for lipid abnormalities at age 35.

Which of the following history and physical examination findings are most suggestive of epididymitis? Acute, unilateral scrotal pain with negative cremasteric reflex Firm, enlarged unilateral testis that does not transilluminate Indurated, enlarged, tender testis accompanied by high fever, nausea, and vomiting Insidious scrotal pain with posterolateral testicular tenderness and positive Prehn sign

Insidious scrotal pain with posterolateral testicular tenderness and positive Prehn sign Acute epididymitis is a clinical syndrome that consists of gradual pain, swelling, and inflammation of the epididymis lasting less than six weeks. It is the most common cause of scrotal pain in postpubertal males. The most common causes of acute epididymitis in sexually active men less than 35 years are Neisseria gonorrhoeae and Chlamydia trachomatis. For individuals less than 14 years and greater than 35 years of age, epididymitis is thought to be caused by the common urinary tract pathogen Escherichia coli. Risk factors for epididymitis include sexual activity, anatomic abnormalities, strenuous physical activity, bicycle or motorcycle riding, and prolonged periods of sitting. Clinical manifestations include gradual onset of pain that may radiate to the lower abdomen. Other symptoms include dysuria, frequency, urgency, hematuria, discharge, and fever. Physical examination shows posterolateral swelling and induration of the epididymis with tenderness on palpation and a normal cremasteric reflex. Prehn sign, pain relief with testicular elevation, is often positive. Color Doppler ultrasonography is necessary to rule out testicular torsion. Additional diagnostic testing includes Gram stain, urethral culture, and urinalysis. Empiric treatment should be initiated to prevent complications. Ceftriaxone 500 mg intramuscular in a single dose plus doxycycline 100 mg orally twice a day for 10 days is recommended for treatment of epididymitis in men less than 35 years of age. In men greater than 35 years of age, levofloxacin 500 mg orally once daily for 10 days is recommended for coverage of enteric organisms unless there is a high suspicion for sexually transmitted pathogens. Additional treatment includes bed rest, scrotal elevation, and nonsteroidal anti-inflammatory drugs. All partners within sixty days of preceding symptoms should be referred, tested, and presumptively treated. Complete resolution of symptoms may take up to two weeks.

A newly diagnosed type 1 diabetic complains of severe dyspnea. The patient is diagnosed with acute bacterial pneumonia, admitted to the hospital, and started on antibiotic therapy. During the second day of admission, he develops mental status changes, increased urinary output, vomiting and abdominal pain. You notice deep and labored breathing on examination. Laboratory evaluation shows declining renal function and elevated levels of β-OH-butyrate. Arterial blood gas evaluation reveals an elevated anion-gap, pH of 7.18 and bicarbonate of 20 mmol/L. Which of the following is the most appropriate management at this time? Insulin and intravenous fluids Nephrology consultation Pulmonology and infectious Disease consultation Sublingual glucose and intravenous glucagon

Insulin and intravenous fluids The patient is suffering from diabetic ketoacidosis (DKA), which is most commonly seen as a complication of type 1 diabetes, but also can occur in ketosis-prone type 2 diabetes. Precipitants include infection, exogenous steroids, failure to take prescribed insulin, inflammation, ischemia, intoxication and infarction. In the setting of insulin deficiency and depleted hepatic glycogen stores, fatty acids become the energy metabolism substrate of choice. This metabolism shift in the liver causes more ketone production, and as a result, increased serum and urine ketosis, namely elevated levels of acetoacetate and β-OH-butyrate. Symptoms include polyuria, polydipsia, dehydration, nausea and vomiting, Kussmaul respirations (deep, labored breathing) and mental status changes. Laboratory changes include hyperglycemia, early increased anion-gap metabolic acidosis, late nonanion gap metabolic acidosis, elevated urine and serum ketones, elevated BUN and Cr, sodium and potassium depletion, leukocytosis and elevated amylase. Treatment includes evaluating for precipitants, aggressive hydration, intravenous insulin, laboratory monitoring and electrolyte replacement.

A 64-year-old man presents to his primary care clinician for his annual visit. In reviewing concerns, he reports sexual dysfunction over the past 6 months. He reports difficulty obtaining and maintaining an erection during intercourse with his spouse of 35 years. He reports this first occurred while they were on vacation, and he attributed it to an overindulgence of alcohol. However, his symptoms continued even once arriving home and settling back into his normal routine. His medical history includes diabetes mellitus, benign prostatic hyperplasia, and hyperlipidemia. His medications include metformin, doxazosin, and atorvastatin. His only surgical history is bilateral knee replacements 2 years ago. He does not smoke and only drinks alcohol when on vacation. He obtained his annual blood work, which showed an A1C of 6.5% and LDL of 115 mg/dL. His vital signs are within normal limits, including a BP of 116/74 mm Hg. He mentions that a friend of his with similar symptoms has used sildenafil with success, and he would like to try this medication as well. Which of the following is the most important concern in prescribing this medication? Chronic kidney disease Diabetic retinopathy Interaction with doxazosin Peripheral neuropathy Peripheral vascular disease

Interaction with doxazosin Erectile dysfunction (ED) is the inability to consistently generate or maintain an erection for the duration of sexual intercourse. The most common cause of ED is progressive vascular disease, which causes a reduction in arterial blood flow. ED affects more than one-half of men aged 40-70, and the incidence further increases with age. Doxazosin is an alpha-blocker that treats benign prostatic hyperplasia by relaxing the muscles of the bladder and prostate but also works as an antihypertensive by relaxing the blood vessels, which allows blood to flow more easily through the body. Sildenafil is a phosphodiesterase type 5 (PDE-5) inhibitor, which works by increasing blood flow to the penis via a vasodilatory effect. Based on their individual mechanisms of action, the combination of doxazosin and sildenafil can lead to increased hypotension and orthostasis. This can cause the patient to experience dizziness, lightheadedness, and syncopal episodes. Chronic kidney disease (A) can manifest as end-organ damage from long-term uncontrolled diabetes but would not be a concern for prescribing sildenafil as both doxazosin and sildenafil are primarily metabolized in the liver. Diabetic retinopathy (B) is more likely to occur secondary to poor chronic diabetes management and is unlikely to be a result of initiating sildenafil. Peripheral neuropathy (D) is more likely to occur secondary to long-term uncontrolled diabetes as compared to the addition of a sildenafil prescription for this patient. It is important to recognize that peripheral vascular disease (E) would be a concern in this patient based on his history of diabetes mellitus and the possibility that this presentation of ED may precede or be an early presentation of more advanced atherosclerotic disease. However, this concern is based on an understanding of disease pathology and progression. It does not answer the question regarding concerns of prescribing sildenafil for this patient. Administration of PDE-5 inhibitors has shown the potential to possibly improve peripheral neuropathy and peripheral vascular disease by improving blood flow.

A 35-year-old man presents with intermittent rectal bleeding and mucoid discharge for two weeks. His diet is low in fiber. He reports some bright red bloody streaks on the toilet paper after each bowel movement. Rectal pain is absent. On physical exam, no nodules are initially seen. With Valsalva, a purple nodule covered by mucosa is seen protruding through the anus. Which of the following is the most likely diagnosis? Anal abscess Anal fissure External hemorrhoid Internal hemorrhoid Supralevator abscess

Internal hemorrhoid Internal hemorrhoids are subepithelial vascular cushions that contribute to normal anal pressures and ensure proper closure of the anal canal. They consist of connective tissues, smooth muscle fibers, and arteriovenous communication between the superior rectal artery and rectal vein. Internal hemorrhoids typically occur proximal to the dentate line. Hemorrhoids become symptomatic with activities that increase venous pressure (such as straining, constipation, prolonged sitting, pregnancy, low fiber diet, and obesity). As the disease progresses, bleeding and protrusion may occur. Patients with internal hemorrhoids typically present with bleeding, hemorrhoid prolapse, and mucoid discharge. Bleeding is typically bright red with the amount ranging from streaks on the toilet paper to severe bleeding causing anemia (rare finding). Stages of internal hemorrhoids include stage I (hemorrhoid is confined to the anal canal), stage II (hemorrhoid protrudes through the anal canal but reduces spontaneously), stage III (hemorrhoid requires manual reduction after bowel movements), and stage IV (hemorrhoid chronically protrudes and is at risk of strangulation). Pain with internal hemorrhoids is an unusual finding. Stage IV internal hemorrhoids may result in a sense of fullness, discomfort, and mucoid discharge resulting in irritation of the skin and soiling of underclothes. Nonprotruding internal hemorrhoids are not visible on physical exam but may protrude through the anus with gentle straining while the clinician spreads the buttocks. Prolapsed internal hemorrhoids are readily visualized as purple nodules covered by rectal mucosa. Anoscopy provides optimal visualization of internal hemorrhoids. Stages I and II internal hemorrhoids can be managed by conservative measures, including high fiber diets, increased hydration, and bulk-forming laxatives. The mucoid discharge may be managed by the local application of a cotton ball near the anus. Injection sclerotherapy, rubber band ligation, or application of electrocoagulation may be used to treat stages I and II internal hemorrhoids refractory to conservative measures and stage III internal hemorrhoids. For acute treatment of all stages of internal hemorrhoids, topical creams, foams, or suppositories containing topical anesthetics, vasoconstrictors, and corticosteroids are effective. Surgical resection is reserved for patients with recurrent severe bleeding due to stages III and IV.

A 55-year-old woman with a history of hypertension and diabetes mellitus presents to the emergency department after becoming incoherent during a work meeting 45 minutes prior to arrival. On examination, a right facial droop and right upper extremity weakness is appreciated. Vital signs are within normal limits. Her serum blood glucose is within normal limits. A non-contrast head computed tomography scan is normal. Electrocardiogram reveals normal sinus rhythm. Which of the following is the next appropriate step in management? Intravenous heparin bolus and infusion Intravenous recombinant tissue-type plasminogen activator bolus and infusion Magnetic resonance imaging of the brain Watchful waiting

Intravenous recombinant tissue-type plasminogen activator bolus and infusion Intravenous recombinant tissue-type plasminogen activator bolus and infusion is the next appropriate step in management. Recombinant tissue-type plasminogen activator bolus and infusion, or rtPA, is a fibrinolytic agent that converts plasminogen to plasmin in the presence of fibrin to initiate fibrinolysis at the site of thrombus formation, which improves blood flow to areas of the brain that are ischemic, but not yet infarcted, called the ischemic penumbra. A small number of patients with acute ischemic stroke will be eligible for reperfusion therapy, and an even smaller number will actually receive it. Recombinant tissue-type plasminogen activator has been shown to improve functional outcomes from ischemic stroke and that the benefits outweigh the risks for patients who receive treatment within 4.5 hours of symptom onset. The benefit of intravenous thrombolysis decreases continuously over time from symptom onset. Therefore, treatment must be given as soon as possible, rather than near the end of the time window. The most important factor in successful thrombolytic treatment of acute ischemic stroke is early treatment. Nonetheless, selection of appropriate candidates for thrombolysis demands a neurologic evaluation, a neuroimaging study, and determination of eligibility based upon inclusion and exclusion criteria.

A man presents to the emergency department with a blood pressure of 200/136 mm Hg and laboratory evidence of acute renal failure. Which of the following medications and initial blood pressure response rates are the most appropriate at this time? Intravenous labetalol, to goal within 1 to 2 hours Intravenous nicardipine, to goal within 6 to 12 hours Oral clonidine, to a goal within 3 to 6 days Oral magnesium sulfate, to a goal within 1 to 2 days

Intravenous labetalol, to goal within 1 to 2 hours A hypertensive emergency is a severe elevation in blood pressure with evidence of end-organ damage. This requires immediate lowering of blood pressure. In hypertensive emergencies, treatment begins in the emergency department and continues in the intensive care unit. It begins with obtaining intravenous access for medication delivery, obtaining intra-arterial access for monitoring and serial measurements of urinary output, blood pressures, mental status, and renal indices. Medication options include labetalol, esmolol, nicardipine, hydralazine, fenoldopam, nitroglycerin, nitroprusside, and phentolamine. IV labetalol (a beta-adrenergic and alpha-adrenergic blocker) has a rapid onset of action (within minutes), making it very useful for the treatment of hypertensive emergencies. In most hypertensive emergencies, the mean arterial pressure (MAP) should be reduced by about 10-20% in the first hour, followed by a gradual reduction during the next 23 hours. The treatment goal is to achieve a final pressure reduction by 25% compared with baseline.

A 58-year-old man presents with 3 months of headache and transient diplopia. He also reports that chewing tough foods has been progressively painful. Laboratory testing reveals a normocytic anemia and ESR of 88 mm/hour. A brain CT is normal. Biopsy reveals fragmented elastic lamina, CD4+ lymphocytes and macrophages. Which of the following is the best next step to manage this patient's condition? Intravenous methylprednisolone Oral methotrexate Oral prednisone Temporal artery resection

Intravenous methylprednisolone Giant cell arteritis, as known as temporal arteritis, is the most common primary vasculitis of the elderly, and predominantly affects those aged over 50 years. It is a syndrome of systemic inflammation which mainly affects the branches of the internal and external carotid arteries, but it can affect any vessel in the body. The most common symptoms are a temporal or occipital headache, jaw claudication (mandibular, tooth and tongue pain with chewing tough foods), visual changes (partial or complete blindness, diplopia, visual field cuts or amaurosis fugax) and arthralgias. It is associated with constitutional symptoms, anemia, an enlarged tender nodular erythematous temporal artery, ESR > 50 mm/hour and polymyalgia rheumatica. Temporal artery biopsy confirms the diagnosis and may show fragmented internal elastic lamina and panarteritis with CD4 lymphocytes and macrophages. Since unilateral partial or complete blindness occurs in up to 20% of patients with giant cell arteritis, immediate treatment is necessary once the diagnosis is confirmed or if there is a high suspicion for the diagnosis of giant cell arteritis. High-dose corticosteroids are the standard treatment, and typical options include intravenous methylprednisolone for patients with threatened or established visual loss at presentation. Intravenous methylprednisolone is administered daily for three days, followed by oral prednisone for 2 to 4 weeks. After symptoms lessen, steroids are slowly tapered over 6 months, with complete tapering off in 2 to 3 years. As such, the clinician must monitor for complications of long-term corticosteroid therapy. These include hypertension, diabetes, osteoporosis, steroid myopathy (proximal weakness), fluid retention, bruising, insomnia, restlessness, hypomania and hypercholesterolemia.

A 28-year-old man presents with scrotal swelling and pain. Examination reveals a left, nontender, swollen spermatic cord that feels like a "bag of worms." The swelling reduces when the patient moves to a supine position. There are no skin lesions, hernia or testicular abnormalities. In addition to scrotal ultrasonography, you may consider ordering further imaging with attention to which of the following organs? Ascending colon Descending colon Kidney Urinary bladde

Kidney Scrotal mass and pain are common complaints in the primary care office. Etiologies range from incidental benign conditions to serious cancers to surgical emergencies. Any patient reporting scrotal swelling needs to be evaluated immediately. A careful history and physical examination, including transillumination, is necessary. The evaluation begins with determining which structure is swollen: skin, epididymis, testicle or spermatic cord. Dilation of the pampiniform venous plexus along the spermatic cord is referred to as varicocele. This condition is suspected in any swelling of the spermatic cord, especially when palpation reveals a nontender "bag of worms" consistency. There are primary and secondary types of varicocele. Primary varicocele is usually idiopathic in nature. Secondary varicocele can occur due to abdominal mass compression of the renal veins (more common with right sided varicocele), or superior mesenteric artery compression of the left renal vein (aka "Nutcracker Syndrome"; most commonly due to renal cell carcinoma and retroperitoneal fibrosis/adhesions). Patients with varicocele usually complain of scrotal heaviness and a "dragging" aching pain. It mostly occurs in tall, thin men between the ages of 15 and 30 years, and affects the left side more than the right. The "wormy" mass or swelling typically reduces when the patient is supine. Varicoceles, although not life threatening may cause decreased testicular artery flow and infertility, and therefore should be evaluated further with ultrasonography and possibly abdominal imaging.

A 43-year-old woman presents with a several year history of irregular bowel habits characterized by alternating diarrhea and constipation and associated with intermittent, occasional crampy abdominal pain and bloating. She denies loss of appetite, weight loss or vomiting, and she has never seen blood in her stools. The pain does not awaken her at night, and she has been unable to identify any particular dietary triggers. Her body mass index (BMI) is 24, she is afebrile, and her abdominal exam is unremarkable. A series of three fecal occult blood tests are all negative. What is the most likely diagnosis? Abdominal migraine Acute diverticulitis Crohn's disease Irritable bowel syndrome

Irritable bowel syndrome Irritable bowel syndrome (IBS) is the most likely diagnosis based on this patient's chronic history of the above described symptoms, as well as her normal physical examination. IBS is the most commonly diagnosed GI condition, is more common in women, and presents most often between the ages of 30 and 50. Diagnosis of IBS is based on clinical features, though patients with features such as rectal bleeding, weight loss, iron deficiency anemia, nocturnal symptoms, or those with a family history of colorectal cancer, inflammatory bowel disease or celiac disease should undergo additional testing such as endoscopy and laboratory assessment to rule out more serious pathology. Differentials to consider include inflammatory bowel disease, colon cancer, hyper- or hypothyroidism, chronic diarrhea, chronic constipation, or celiac sprue. Studies suggest avoiding foods that trigger symptoms, as well as regular exercise may help reduce symptoms of IBS. Antispasmodics and antidepressants may be helpful in reducing symptoms for some patients, and loperamide hydrochloride may be used to reduce episodes of diarrhea specifically. Lubiprostone is indicated for constipation-predominant IBS. 5-hydroxytryptamine 4 agonists and 3-hydroxytryptamine 3 antagonists are reserved for severe forms of constipation-predominant and diarrhea-predominant forms of IBS, respectively, due to the severe side effects associated with their use.

Which of the following is the most common cause of sudden cardiac arrest and sudden cardiac death? Anomalous coronary artery Cardiomyopathy Ischemic heart disease Left ventricular hypertrophy

Ischemic heart disease Sudden cardiac arrest and sudden cardiac death usually occurs in people with some form of underlying structural heart disease, most notably ischemic heart disease. Sudden cardiac arrest and sudden cardiac death refer to the sudden cessation of cardiac activity with hemodynamic collapse. Events that are successfully treated leading to patient survival are referred to as sudden cardiac arrest, while those that lead to death are referred to as sudden cardiac death. As much as 70% of sudden cardiac arrest have been attributed to coronary or ischemic heart disease. Among patients with coronary heart disease, sudden cardiac arrest or death can occur both during an acute coronary syndrome and in the setting of chronic, otherwise stable coronary heart disease. Often such patients have had prior myocardial damage and scar that serves as a substrate for sudden cardiac arrest.

Which of the following is the first-line preferred treatment for a latent tuberculosis infection? Doxycycline Isoniazid plus rifampin Lamivudine plus zidovudine Penicillin

Isoniazid plus rifampin First-line preferred therapy for latent tuberculosis infection (LTBI) is isoniazid (INH) plus rifampin, rifampin alone, or INH plus rifapentine according to the 2020 updated guidelines on treatment of LTBI set forth by the Centers for Disease Control and Prevention and the National Tuberculosis Controllers Association. Alternative therapy is INH alone for either 6 months or 9 months. In the United States, LTBI is the most prevalent form of tuberculosis (TB). LTBI is the term given to patients with a positive purified protein derivative (PPD) skin test without evidence of active TB. PPD testing has been used for more than 100 years and relies on delayed-type hypersensitivity to M. tuberculosis cellular proteins. Because PPD relies on a hypersensitivity reaction, any factor that reduces that reaction affects the host response to PPD. The most common clinical example is the use of corticosteroids, which blunt the delayed hypersensitivity response and can complicate PPD interpretation. Therefore, PPD testing should not be performed while a patient is taking corticosteroids. Also, TB testing should be targeted to those with a higher risk of infection and should not routinely be done in low-risk patients. Patients at increased risk for progression to active TB include those who have been recently infected (recent PPD converters); patients who are HIV seropositive; patients who have silicosis, diabetes, or chronic renal failure (including those receiving hemodialysis); solid-organ transplant recipients; patients with gastrectomy or jejunoileal bypass or head and neck cancer; injection drug users; patients with chest radiograph evidence of prior TB; and patients who weigh at least 5% less than ideal body weight. Patients taking chronic corticosteroid therapy and those who are to receive tumor necrosis factor alpha (TNF-alpha) blockers (e.g., infliximab) are also at risk.

A man presents to the clinic at his correctional facility with reports of night sweats, unintended weight loss, and a cough for the past 3 weeks. A review of his medical records reveals negative HIV immunoassay results and medical history of hypertension, for which he takes lisinopril. Physical examination reveals an ill-appearing man in no acute distress, with cervical lymphadenopathy, dullness to chest percussion, crackles with inspiration, and vitals of HR 95 bpm, RR 18 breaths per minute, BP 144/94 mm Hg, T 102.1°F, and SpO2 97% on room air. Chest radiography reveals cavitary infiltrates in the upper lobes bilaterally. Nucleic acid amplification testing confirms the suspected diagnosis. Which of the following is the preferred regimen for treating this condition? Isoniazid, rifampin, pyrazinamide, and ethambutol for 2 months, then administer isoniazid and rifampin for 4 months Isoniazid, rifampin, pyrazinamide, and ethambutol for 2 months, then administer levofloxacin and rifampin for 2 months Isoniazid, rifampin, pyrazinamide, and ethambutol for 4 months, then administer ciprofloxacin and moxifloxacin for 4 months Isoniazid, rifampin, pyrazinamide, and ethambutol for 4 months, then administer isoniazid and pyrazinamide for 2 months Isoniazid, rifampin, pyrazinamide, and ethambutol for 4 months, then repeat diagnostic testing

Isoniazid, rifampin, pyrazinamide, and ethambutol for 2 months, then administer isoniazid and rifampin for 4 months Tuberculosis is a respiratory disease caused by Mycobacterium tuberculosis that clinically presents as either primary disease, latent infection, or reactivation disease. The pathogen is transmitted by aerosolized droplets that are small enough to reach the alveolar space within the lungs, wherein the tubercle bacilli proliferate and spread to local lymph nodes. Primary disease classically presents with cough, weight loss, fatigue, lymphadenopathy, fever, night sweats, chest pain, dyspnea, and hemoptysis. Physical exam may reveal dullness to percussion, decreased tactile fremitus, and crackles with inspiration or after a cough. An absence of abnormal physical exam findings does not exclude the condition. Risk factors include children < 4 years old, residents and employees in high-risk settings (e.g., correctional facilities, health care facilities, homeless shelters), those with chronic diseases that increase the risk of tuberculosis reactivation (e.g., silicosis, end-stage renal disease, diabetes mellitus), and immunosuppressed patients, among others. Active tuberculosis disease should be further examined using chest radiography, which most commonly demonstrates hilar adenopathy and infiltrates with or without cavitation in the upper lobes or the superior aspect of the lower lobes. Computed tomography is useful in further evaluating the severity of infection and diagnosing complications of pulmonary tuberculosis, such as pneumothorax, bronchiectasis, extensive pulmonary destruction, malignancy, venous thromboembolism, and chronic pulmonary aspergillosis. If active disease is suspected, three sputum specimens should be submitted for acid-fast bacillus smear, mycobacterial culture, and nucleic acid amplification testing. First-line treatment for tuberculosis includes the concomitant use of rifampin, isoniazid, pyrazinamide, and ethambutol (RIPE therapy) for 2 months, followed by continuation of isoniazid and rifampin for 4 months. To ensure these medications are taken as directed, this drug regimen requires directly observed therapy, that is, the administration of doses with observation by a health care clinician. Additionally, patients with suspected infection should be isolated, and airborne precautions should be implemented. Confirmed tuberculosis diagnoses are required to be reported to public health agencies.

A 19-year-old woman at 34 weeks gestation presents with a 1-day history of progressive left-sided facial weakness. She notes a recent history of an upper respiratory infection. On exam, she is unable to raise her left eyebrow, close her left eye, or smile on her left side. She also reports decreased taste sensation. Her neurologic exam is otherwise unremarkable. Which of the following complications is most commonly associated with the patient's diagnosis? Cavernous sinus thrombosis Descending paralysis Hearing loss Keratitis

Keratitis The patient has unilateral facial nerve palsy with no other focal neurologic or systemic findings. Although the differential diagnosis for facial nerve paralysis is extensive and includes infectious, traumatic, and neoplastic etiologies, the most common diagnosis is idiopathic Bell palsy. Bell palsy is characterized by the sudden onset of facial nerve paralysis with symptoms usually peaking by 48 hours. Some 60% of patients have a viral prodrome. In making the diagnosis, careful consideration must be given to the motor function of the forehead; sparing of this region would be characteristic of a central, and not peripheral, etiology. Because the orbicularis oculi muscles are involved, this results in incomplete closure of the eyelids on the affected side and leads to corneal exposure keratitis. It is important to prescribe lubricating eye drops, to keep the corneal epithelium from breaking down, and to patch the affected eye at nighttime.

A 21-year-old man presents to the emergency department with headache, fever, and vomiting. He has been febrile and acting lethargic for the past three days. On exam, a petechial rash is noted on his lower extremities and sacral area. Which of the following signs is most likely to be positive in this patient? Kehr sign Kernig sign Murphy sign Obturator sign

Kernig sign The Kernig sign is most likely to be positive in patients with meningitis. Meningitis is an inflammatory disorder of the meninges. Meningitis can be caused by bacteria, viruses, parasites, or drugs. The most common causes of community acquired bacterial meningitis are Streptococcus pneumoniae, Neisseria meningitidis, and Listeria monocytogenes. Healthcare-associated bacterial meningitis is usually occurs after neurosurgery and is usually caused by Staphylococci and gram-negative bacilli. Some of the most common risk factors for meningitis include age extremes (< 5 years or > 60 years), diabetes mellitus, renal insufficiency, immunosuppression, human immunodeficiency virus (HIV) infection, crowded housing (e.g. military recruits, college dorms), intravenous drug use, and alcoholism. Patients with bacterial meningitis typically present with fever, headache, and altered mental status. Nuchal rigidity is present in the majority of patients. Nuchal rigidity can be assessed by actively or passively flexing the neck, or with Brudzinski sign or Kernig sign. A positive Brudzinski sign occurs when the patient flexes his or her hips and knees in response to neck flexion. Kernig sign is positive when the knee cannot be fully extended when the patient is lying supine with the hip flexed. Many patients with meningitis caused by N. meningitidis will have characteristic skin manifestations like petechiae and purpura. Laboratory studies should include complete blood count, blood cultures, and lumbar puncture. A computed tomography (CT) scan may be required prior to lumbar puncture to assess for increased intracranial pressure. The typical cerebrospinal fluid findings, in patients with bacterial meningitis, are increased white blood cell count and decreased glucose. Antibiotics should be initiated as quickly as possible after the lumbar puncture is performed however antibiotics should be initiated empirically if the CT or LP is delayed. Intravenous antibiotics should be directed against the most likely bacteria based on the patient's age, underlying risk factors, and comorbid diseases. Dexamethasone is a common adjunctive therapy used to diminish rates of permanent neurologic sequelae such as hearing loss, intellectual impairment, and focal deficits.

A 62-year-old man with a history of hypertension that is controlled well with lisinopril, hyperlipidemia that is managed with simvastatin, and cocaine use presents to the emergency department via ambulance with chest pain that started approximately 20 minutes ago. He describes the chest pain as an intense pressure in his chest that radiates to his left arm. His vital signs are heart rate of 145 bpm, blood pressure of 150/95 mm Hg, respirations of 38/minute, SpO2 of 92%, and temperature of 98.9°F. Upon physical exam, the patient is noted to be diaphoretic. Cardiac exam is notable for tachycardia with a regular rhythm and no murmurs. A troponin level is drawn and is 1.0 ng/mL. An electrocardiogram is performed and is shown above. Which of the following medications should be avoided in the treatment of this patient? Aspirin Diazepam Diltiazem Labetalol Nitroglycerin

Labetalol The most common cardiac condition associated with cocaine use is acute coronary syndrome (ACS). Approximately 6% of patients who use cocaine and have chest pain will experience myocardial infarction (MI). This occurs regardless of the route of cocaine ingestion. Cocaine use causes an increase in myocardial oxygen demand via inhibition of norepinephrine uptake. Cocaine also stimulates alpha-adrenergic receptors, which can constrict coronary vessels. Additionally, cocaine use is thought to promote thrombus formation in the coronary arteries. The treatment of cocaine-associated myocardial infarction is similar to the treatment of myocardial infarction from other causes, with early reperfusion an important part of treatment. In the patient above the ECG shows ST depression in the inferolateral leads with an elevated troponin. Coronary angiography and primary percutaneous coronary intervention play a key role in early reperfusion and should be initiated quickly after the initial assessment of the patient. In patients with suspected cocaine use, there is one primary difference in medication management. Beta-blockers, such as labetalol, should not be administered to patients with suspected cocaine ingestion due to unopposed alpha-adrenergic stimulation, which could result in coronary artery vasoconstriction and systemic hypertension.

A 60-year-old woman presents to the clinic for her annual exam. She reports she feels well. Her current medications include amlodipine 10 mg daily and hydrochlorothiazide 12.5 mg daily. Her vital signs are within normal limits except a body mass index of 28 kg/m2. As part of her yearly exam, she receives a visual acuity test that shows acuity of 20/20 in the right eye, 20/30 in the left eye, and 20/25 with both eyes. Her intraocular pressure is 18 mm Hg on the right and 24 mm Hg on the left. Which of the following clinical interventions is recommended at this time? Acetazolamide 250 mg orally twice daily Latanoprost ophthalmic solution 1 drop left eye at bedtime Pilocarpine ophthalmic solution 1 drop three times daily left eye Propranolol 20 mg orally once daily Timolol 0.5% ophthalmic solution 1 drop left eye at bedtime

Latanoprost ophthalmic solution 1 drop left eye at bedtime Open-angle glaucoma is defined as atrophy of the optic nerve head and is usually accompanied by elevated intraocular pressure, leading to defects in the visual field with eventual progression to blindness if left untreated. Normal intraocular pressure ranges from 12-22 mm Hg. The etiology of open-angle glaucoma is unknown, although it is more prevalent in Black and Hispanic populations, in those with a family history of open-angle glaucoma, and in those with advanced age, hypertension, and diabetes mellitus. Patients with open-angle glaucoma are often asymptomatic. Peripheral visual field loss progresses slowly and often goes unnoticed. Diagnosis of open-angle glaucoma is made when characteristic nerve damage is noted on funduscopic exam, along with visual field deficits. Tonometry is used as a screening tool in the primary care setting and also as a tool to assess the effectiveness of pressure-lowering treatments. First-line therapy for glaucoma is with ophthalmic preparations of prostaglandin analogs, such as latanoprost. These medications work by increasing the outflow of aqueous humor through the uveoscleral pathway, thus decreasing intraocular pressure. Common side effects of prostaglandin analogs include lengthening of eyelashes, change of iris color, and keratitis. Prevention of blindness is the goal of treatment, but even with treatment, one in seven patients with glaucoma will experience blindness in at least one eye within 20 years of diagnosis. Advanced disease progression at diagnosis and poor adherence to treatment protocols are indicators of a poorer prognosis. Patients who present with severe visual field loss may be referred for laser therapy as a first-line measure. Acetazolamide 250 mg twice daily (A) is a carbonic anhydrase inhibitor taken systemically that can be used to lower intraocular pressure but is not recommended as first-line therapy due to systemic side effects such as aplastic anemia, metabolic acidosis, flushing, and paresthesias. Pilocarpine ophthalmic solution 1 drop three times daily left eye (C) is a topical cholinergic that increases outflow of aqueous humor through the trabecular meshwork. Pilocarpine can cause blurred vision, eye pain, headache, and poor night vision. For this reason, it is not first-line therapy in the treatment of glaucoma. Propranolol 20 mg orally once daily (D) is a systemic beta-blocking agent with minimal effects on intraocular pressure. Direct application of beta-blocking agents to the eye is more effective. Timolol 0.5% ophthalmic solution 1 drop left eye at bedtime (E) is a beta-blocking topical agent that decreases aqueous humor production. Side effects of timolol include bradycardia, fatigue, bronchospasm, and ocular dryness. Timolol drops, unlike latanoprost, are not recommended for nighttime administration. They may be used if a patient is unable to afford a topical prostaglandin. Topical prostaglandins are still recommended first.

Which of the following findings seen on rectal examination is most consistent with a concomitant systemic process? Anal fissure with bleeding Anal fissure with deep ulcer Anterior midline anal fissure Lateral anal fissure

Lateral anal fissure A lateral anal fissure is concerning for an underlying systemic illness in the patient. Most anal fissures occur in the midline. Fissures that occur in other locations should raise concern for diseases like Crohn's, HIV, leukemia, tuberculosis, or syphilis. Most commonly, fissures develop from a superficial tear in the anoderm when hard stool is passed often associated with constipation. They occur in areas of weakness of the muscle fibers most commonly in the posterior midline but also in the anterior midline. Patients complain of minor rectal bleeding and significant pain with defecation.

A 40-year-old man presents with bloody stools for six months. He also reports a severe, tearing pain during defecation followed by throbbing discomfort. Medical history is significant for constipation. He has undergone multiple conservative treatments without improvement of his symptoms. He continues to have painful defecations. A fibrotic skin tag at the outermost edge of the anoderm is noted on physical exam. Which of the following is the most appropriate next step in management? Diltiazem 2% ointment Lateral internal sphincterotomy Silver nitrate 2% Sitz baths Topical hydrocortisone

Lateral internal sphincterotomy The patient presents with a chronic anal fissure that is refractory to conservative treatment. Surgical correction is indicated. Anal fissures are benign lesions that are rocket-shaped or linear and measure less than 5 mm. They are most commonly found in the posterior midline but can also occur anteriorly. Infants and middle-aged individuals are often affected. Anal fissures that do not occur in the midline should raise concerns for Crohn disease, human immunodeficiency virus, tuberculosis, syphilis, or carcinoma. Ischemia and increased anal pressure are the main pathogenetic mechanisms of anal fissures. Patients typically present with a severe, tearing pain during defecation followed by throbbing discomfort. Constipation may accompany painful defecation. The patient may also report mild hematochezia with blood on the toilet paper. Diagnosis is by visual inspection of the anal verge. Acute anal fissures may resemble cracks in the anal epithelium, while chronic fissures usually have a fibrotic skin tag and hypertrophied anal papilla at the outermost edge. A physical exam may show an acute or chronic longitudinal tear in the anoderm that usually extends no further than the dentate line. Conservative treatments, such as increasing dietary fiber, sitz baths, and topical anesthetics, should be prescribed initially. Topical anesthetics, such as 5% lidocaine or 2.5% lidocaine plus 2.5% prilocaine, may relieve symptoms temporarily. Following conservative treatment, healing of anal fissures usually occurs in two months. Anal fissures that persist beyond two to three months are considered chronic. Chronic fissures may be treated with 0.2 to 0.4% nitroglycerine or diltiazem 2% ointment. Chronic fissures refractory to medical therapy may benefit from lateral internal sphincterotomy. The purpose of the surgery is to excise the hypertrophied internal sphincter. There are two approaches for lateral internal sphincterotomy: open and closed. Complications of lateral internal sphincterotomy include infection, bleeding, fistula development, and incontinence.

A 62-year-old man with a medical history of hypertension and hyperlipidemia presents to the emergency department by ambulance with sudden-onset neurological deficit. His current medications include losartan, amlodipine, hydrochlorothiazide, and atorvastatin. A stat MRI reveals acute right lacunar infarct. Which of the following is the most likely physical exam finding? Expressive aphasia Left hemineglect Left hemiparesis Right gaze deviation Right hemiparesis

Left hemiparesis The patient in the vignette with a right lacunar stroke is most likely to present with left hemiparesis. While there are many types of lacunar syndromes based on location of the infarct, the most common is contralateral motor hemiparesis. Lacunar strokes are small, ischemic infarcts located in deep, subcortical areas such as the thalamus, basal ganglia, or pons. They are caused by occlusion of a single penetrating branch of large cerebral arteries from the circle of Willis, middle cerebral artery, or basilar artery. Risk factors for lacunar strokes include hypertension, diabetes mellitus, hyperlipidemia, carotid artery atherosclerosis, peripheral artery disease, tobacco use, and history of previous transient ischemic attack. There are also certain genetic factors that can predispose patients to an increased risk of lacunar infarcts. While many lacunar strokes are asymptomatic, the most common presentation is pure motor hemiparesis with contralateral hemiparesis of the face, arms, or legs. Due to the noncortical location of the infarcts, lacunar strokes never present with cortical symptoms such as aphasia, hemianopia (loss of half the visual field), agnosia (loss of one or more senses), or neglect. While hemiparesis is the most common lacunar syndrome, other classic lacunar syndromes include pure sensory loss, ataxic hemiparesis, sensorimotor loss, and dysarthria-clumsy hand syndrome. Emergent imaging is important in the evaluation of sudden-onset neurological deficit with suspected stroke. While a noncontrast CT of the head is most appropriate for rapid evaluation, MRI is more sensitive in the detection of lacunar infarcts. Patients with suspected stroke should also be screened for eligibility for thrombolytic therapy. Treatment includes intravenous thrombolytic therapy with tissue plasminogen activator. Given the small size of lacunar infarcts, mechanical thrombectomy is never performed. In addition to acute treatment of the stroke, patients should be treated with antiplatelet therapy for secondary stroke prevention, be treated for comorbidities, and make lifestyle modifications.

A patient presents with nausea, vomiting, right-sided hemiplegia, and non-occipital headache. His gaze is deviated to the left. Thirty minutes later, he becomes stuporous and progresses into coma. The pupils are now fixed and dilated. Abnormal posturing is absent. A brain CT scan is ordered. In which of the following sites would you expect to find intracerebral hemorrhage? Left pons Left putamen Right cerebellum Right thalamus

Left putamen A large putamen hemorrhage results in a rapid progression of hemiplegia, nausea, vomiting and headache over 30 minutes, which is quickly followed by ipsilateral deviation of the eyes, stupor, coma and mydriatic pupils ("blown-pupil", associated with brainstem compression and oculomotor nerve palsy). Acute management includes controlling intracranial pressure and hypertension. Surgical removal of clots is seldom successful. However, some neurologic function may be salvaged in those without coma or those with lobar clots.

A 28-year-old man presents to his primary care clinician with concerns for low libido and erectile dysfunction. His medical history includes insulin resistance, hypertension, and carpal tunnel syndrome. He takes metformin 500 mg once daily and lisinopril 10 mg once daily. Vitals include BP of 130/78 mm Hg, HR of 90 bpm, RR of 13/min, T of 98.7°F, SpO2 of 99% on room air, height of 6 feet 6 inches, and he weighs 250 pounds. On physical exam, he has large hands with wide fingers and a moist, doughy handshake. His mandible is prominent with coarse facial features. What additional finding is most likely to be associated with this patient's condition? Aortic root dilation Aortic stenosis Enlarged atrial septum Left ventricular hypertrophy Mitral valve prolapse

Left ventricular hypertrophy Acromegaly is almost always caused by a pituitary adenoma, which causes an excess of growth hormone. If diagnosed before the closure of the growth plates, it is referred to as gigantism. Elevated growth hormone levels cause tall stature and increased growth in specific areas of the body. The hands, feet, cranium, and mandible are often enlarged, and coarse facial features are common. A soft, doughy, moist handshake is a common way to describe the soft tissue overgrowth seen in these patients. Patients with acromegaly often experience cardiac complications such as hypertension, cardiomyopathy, left ventricular hypertrophy, insulin resistance, weight gain, carpal tunnel, and erectile dysfunction or amenorrhea. Around 10% of patients with acromegaly will already have signs of left ventricular hypertrophy by the time a diagnosis is made. Elevated growth hormone levels and insulin-like growth factor 1 (IGF-1) help make the diagnosis, and MRI is the test of choice for visualizing the pituitary tumor.

A 63-year-old man presents to your office accompanied by his wife. She reports a cognitive decline over the last several months. Physical exam shows a shuffling gait and a reduced arm swing. He has no delusions, but is reacting to visual hallucinations. His affect is flat. He has deficits in short-term and long-term memory and confabulates when answering questions. A previous medical provider gave the patient an antipsychotic medication which caused catatonia. Which of the following is the most likely diagnosis? Frontotemporal dementia Lewy body dementia Normal pressure hydrocephalus Parkinson's disease

Lewy body dementia Lewy body dementia (LBD) is a dementia similar to Alzheimer's and is characterized by hallucinations, parkinsonian features, and extrapyramidal signs, as well as the cognitive dysfunction common in all dementias. Patients with LBD lose cholinergic neurons which cause the loss in cognitive functioning and a loss in dopaminergic neurons causes the loss in motor control. Lewy bodies occur in the brainstem, midbrain, olfactory bulb, and neocortex. Patients with LBD are very sensitive to neuroleptic and antiemetic medications that affect dopaminergic and cholinergic systems. They respond with catatonia, loss of cognitive function, or develop life-threatening muscle rigidity. Frontotemporal dementia (A) is an umbrella term for a diverse group of uncommon disorders that primarily affect the frontal and temporal lobes of the brain — the areas generally associated with personality, behavior and language. Parkinsonism, and motor neuron disease. Normal pressure hydrocephalus (C) presents with abnormal gait, dementia, and urinary incontinence. Parkinson's disease (D) presents with bradykinesia, resting tremor, postural instability, autonomic dysfunction, and behavioral changes.

A 66-year-old woman has been on estrogen-replacement therapy for nine years. Her social history is significant for 20 years of heavy alcohol use and 30 years of tobacco use. Her family history is positive for familial adenomatous polyposis. Which of the following organs is most likely to undergo malignant transformation due to this patient's alcohol use? Breast Colorectal Liver Lung

Liver Hepatocellular carcinoma, also known as malignant hepatoma, is a primary cancer of the liver. It is rare in the United States, where most of liver cancer is secondary from a metastatic site. However, in countries where hepatitis is endemic, such as China, this cancer represents the most common of all liver cancers. The most common causes are viral hepatitis (hepatitis B and C) and alcoholic cirrhosis. Other risk factors include aflatoxin exposure, hemochromatosis, Wilson disease, and type 2 diabetes mellitus. 20-25% of cases are due to hepatitis B or C infection, while 80% of cases are due to alcoholic cirrhosis. Alpha-fetoprotein is elevated in the majority of cases but it is nonspecific. Triple or quad-phase liver CT scan aides in the diagnosis. Biopsy provides a definitive diagnosis. Ultrasound plays an important role in the surveillance of high-risk patients, however, contrast enhanced abdominal CT with triple phase scanning remains the primary diagnostic tool. Treatment options include surgical resection, transplantation, chemotherapy, and radiotherapy.

A 41-year-old woman presents to the emergency department with an 8-hour history of left-sided epigastric pain that radiates to her back. She vomited four times overnight after going out to eat. The patient's vitals are BP of 105/70 mm Hg, HR of 112 bpm, T of 99.4°F, RR of 20/min, and oxygen saturation of 98% on room air. Urine is negative for pregnancy but positive for glucose. She has epigastric tenderness but not rigidity and cannot lie down due to pain. You order a 2 L bolus of saline with an opioid for pain. Laboratory results show a serum glucose of 280 mg/dL, serum amylase of 240 U/L, and serum lipase of 311 U/L. Liver enzymes are within normal limits. Bedside ultrasound of the gallbladder is negative for cholelithiasis. Patient is admitted to the surgery floor, and pain has improved significantly after 2 days. Additional blood work shows a triglyceride level of 1,090 mg/dL. What key management should be discussed with her besides diabetic medication and dietary changes? Elective cholecystectomy Extended-release niacin therapy Long-term fenofibrate Long-term rosuvastatin Trial of disulfiram

Long-term fenofibrate The patient shows signs and symptoms of acute pancreatitis due to hypertriglyceridemia. Central or left-central chest pain that radiates to the back is a classic sign of pancreatitis. Patients with pancreatitis also prefer to sit up or lean forward. Nausea and vomiting are common in pancreatitis from all etiologies. Fever, tachycardia, and hypotension are common in more severe pancreatitis. Such signs may add up to systemic inflammatory response syndrome and would require hospitalization. Jaundice is sometimes present. Serum amylase (reference range: 50-140 U/L) and lipase (0-160 U/L) are the two key labs that show elevation in pancreatitis. Treatment of acute pancreatitis involves aggressive fluid replacement, pain relief with an intravenous opioid, and NPO status until stabilization. Repeat complete metabolic panels are necessary with special attention to glucose, electrolytes, and calcium levels, as some patients are hypocalcemic. The main etiologies of acute pancreatitis include gallstones that cause obstruction of the ductal system (including the ampulla) and reflux of bile and pancreatic enzymes or edema of these structures from the passing of stones, also causing obstruction, stasis, and reflux; chronic alcohol use; and hypertriglyceridemia, possibly causing the capillaries of the pancreas to become blocked. Gallstones and alcohol use are less likely in this case, given the normal liver enzymes and ultrasound findings. By contrast, elevated levels of serum triglyceride is a direct clue to her hypertriglyceridemia. Triglyceride levels > 500 mg/dL pose a risk for acute pancreatitis, with the risk increasing for those with levels > 1,000 mg/dL or prior episodes. The most effective agents for treatment of hypertriglyceridemia are fibrates, such as fenofibrate or gemfibrozil (the former is preferred). Acute pancreatitis caused by hypertriglyceridemia is most often associated with uncontrolled diabetes. Controlling diabetes, providing dietary counseling, and investigating potential drug-related causes (e.g., estrogen) are also important. More severe pancreatitis cases have signs of infection (fever, elevated white blood cell count) and pancreatic necrosis.

You have made a new diagnosis of polycystic kidney disease in one of your primary care patients. Proper maintenance of normal blood pressure should be obtained with which of the following medications? Furosemide Losartan Metoprolol Verapamil

Losartan In a patient with polycystic kidney disease, the development of hypertension signifies disease progression. Therefore, it is clinically necessary to maintain proper blood pressure in these patients. Target levels for adults are 140/90 mmHg or below the seventy-fifth percentile for children. Angiotensin-converting-enzyme inhibitors (ACEi) or angiotensin II receptor blockers (ARBs) are the preferred therapeutics. Studies have shown that treatment with these drugs, as compared to other antihypertensives, is associated with preservation of normal renal function in those with PKD. Common ARBs are medications ending in -sartan, such as losartan, irbesartan and valsartan.

Which of the following would you most expect to find in a patient who presents to the emergency department in cardiogenic shock? Dehydration High pulse pressure Low cardiac index Warm, hyperemic extremities

Low cardiac index Cardiogenic shock is the leading cause of death in acute myocardial infarction. It is characterized by decreased cardiac output and tissue hypoxia in the presence of sufficient intravascular volume. Patients present with hypotension, tachycardia, altered mentation, cool cyanotic extremities, faint peripheral pulses, and oliguria. A low pulse pressure is also typically encountered. Coronary angiography is indicated if myocardial ischemia or infarct is present. Invasive hemodynamic monitoring may be used, the results of which show a high pulmonary capillary wedge pressure (> 15 mm Hg) and a low cardiac index (stroke volume x heart rate / body surface area; < 2.2 L/min/m2). The cardiac index is a hemodynamic parameter that relates the cardiac output from the left ventricle in one minute to body surface area. Thus, relating cardiac performance to the size of the individual. Treatment mainstay includes prompt inotropic medications such as dopamine, dobutamine and phosphodiesterase inhibitors, and norepinephrine. Other measures include the placement of central and peripheral arterial lines, possible fluid resuscitation, ICU care, electrolyte and acid-base correction, intra-aortic balloon pump, percutaneous coronary intervention or coronary artery bypass grafting.

A 23-year-old man presents to the emergency room with intermittent severe left-sided flank pain that radiates to his groin. He reports being healthy otherwise and is training for a marathon. He also consumes very little water on a daily basis. Which of the following is the preferred imaging modality to diagnose the suspected condition? Low-dose computed tomography of the abdomen and pelvis without contrast Magnetic resonance of the abdomen and pelvis Ultrasound of the abdomen and kidneys X-ray of kidneys, ureters and bladder

Low-dose computed tomography of the abdomen and pelvis without contrast A CT of the abdomen and pelvis without contrast is the preferred method of imaging for adults when nephrolithiasis is suspected because it has the highest diagnostic accuracy for nephrolithiasis and can also identify hydronephrosis. CT imaging can also accurately measure stone size as well as identify the location of the stone, which allows for improved treatment planning. Most stones 5 mm in diameter can usually pass on their own. Stones 10 mm in diameter, as well as proximal ureteral stones, are unlikely to pass spontaneously and may require intervention beyond pharmacotherapy. Low radiation-dose CT scanning protocols are used in these instances to assure the minimal amount of radiation for the patient. Women who could be pregnant should undergo a pregnancy test before a CT is performed. If pregnancy is confirmed, then ultrasound is the preferred imaging choice. Risk factors for developing nephrolithiasis include a history of prior nephrolithiasis, family history of nephrolithiasis, low fluid intake, excessive physical exercise, persistently acidic urine, diabetes, obesity, and hypertension.

A 42-year-old woman presents to a free clinic with reports of malaise and a persistent headache that has lasted 2 weeks. For the past few months, she has been living out of her camper with her dog in various parts of the Midwest. She has a history of HIV but discontinued her antiretroviral therapy a couple of years ago because her viral load had been undetectable. She reports no current drug use or sexual activity. Vitals are BP of 120/95 mm Hg, HR of 75 bpm, RR of 22/min, oxygen saturation of 98% on room air, and T of 100.4°F. Exam reveals marked cervical lymphadenopathy, and she mentions her neck has been stiff. Her head CT scan shows no masses, allowing a lumbar puncture. India ink staining of CSF demonstrates round encapsulated yeast organisms. What is the most likely primary site of her infection? Eyes Intestinal tract Lungs Nasopharynx Skin

Lungs The scenario presents a case of Cryptococcus meningitis in a patient with HIV who is not taking her antiretroviral therapy (ART). Exposure to the fungal organism Cryptococcus neoformans occurs through inhalation of aerosolized dried animal droppings, especially those of pigeons, and is common. However, those with healthy immune systems are not affected. Lung lesions and dissemination to the CNS are typically limited to highly immunocompromised patients with CD4 count < 50/μL, more commonly in global regions with limited access to ART. Pulmonary symptoms include a cough with scant sputum and pleuritic pain. Cryptococcosis is the most common fungal cause of meningitis. Apart from fever, malaise, and headache, common symptoms include photophobia, nausea, vomiting, and confusion. Dimpled skin lesions, such as those of molluscum contagiosum, may be present, along with tachypnea and high diastolic blood pressure. Tapped CSF will reveal round encapsulated yeast organisms upon India ink staining. Lumbar puncture may need to be repeated throughout the early treatment phase to relieve intracranial pressure. Effective antifungal treatment, regardless of immunocompetence, has three stages: induction, consolidation, and maintenance. The current recommendation for induction therapy, lasting at least 2 weeks, is a combination of intravenous amphotericin B and oral flucytosine. Consolidation therapy comprises high-dose fluconazole, followed by lower doses of fluconazole for maintenance therapy, which should last 1 year. If left untreated, this condition is fatal. This patient would also need to be restarted on ART.

A 35-year-old man presents to the emergency department with slurred speech and right facial drooping. He had a similar episode a few months ago and did not experience any residual effects. He also has a history of deep vein thrombosis in the left leg but has no history of surgeries, hypertension, or diabetes. The patient is a White man. Which of the following should be included in the evaluation to determine the cause of this condition? Bone marrow biopsy D-dimer Hemoglobin electrophoresis Lupus anticoagulant

Lupus anticoagulant Antiphospholipid syndrome is an acquired autoimmune disorder that causes a hypercoagulable state. Individuals with this disorder are known to have recurrent venous or arterial thrombosis at an early age, although not all patients are symptomatic. Any patients with the following presentations should be tested for antiphospholipid syndrome: history of thrombosis in the absence of known risk factors, stillbirth, recurrent pregnancy loss, history of heart murmurs or cardiac valvular vegetations, history of hematologic abnormalities, or pulmonary hypertension. The cause of this disorder is unknown, but it is associated with a variety of autoimmune and rheumatic diseases, certain infections (e.g., syphilis, hepatitis C, malaria), and drugs (e.g., procainamide, phenytoin, amoxicillin). Genetic disposition and familial association have also been linked. Important lab markers include the presence of lupus anticoagulant (associated with the strongest risk for thrombosis), anticardiolipin, and anti-beta 2-glycoprotein I antibodies. Treatment is individualized based on the patient's current clinical status and history of thrombotic events. Known risk factors for hypercoagulability must be addressed, including discontinuing oral contraceptives, tobacco cessation, and treating hypertension and hyperlipidemia, if present. Any associated autoimmune diseases must be managed as well. Low-dose aspirin is a common therapy for primary prevention of thrombotic events. Thrombotic events are initially treated with intravenous heparin or subcutaneous enoxaparin. Life-long treatment with warfarin is standard for patients who experience recurrent thrombotic events.

An 80-year-old man who does not have housing is found in his riverside tent with confusion and fever in July. On physical examination, the patient is disoriented with T 38.2°C, HR 122 bpm, BP 106/84 mm Hg, and oxygen saturation 99% on room air. You note splenomegaly, flaccid paralysis, and multiple mosquito bites. Which lab abnormality would lead you to suspect West Nile encephalitis in this patient? Decreased cerebrospinal fluid glucose level Leukocytosis Lymphopenia Markedly increased serum ALT and AST

Lymphopenia Patients with West Nile encephalitis will have leukopenia with a pronounced and prolonged lymphopenia, which can aid in distinguishing it from other causes of encephalitis. West Nile encephalitis is an arthropod-borne virus (arbovirus) endemic to the Middle East and now found throughout the United States. It is transmitted by the Culex mosquito, with birds serving as the intermediate host. Most people with West Nile encephalitis will remain asymptomatic or have a mild viral syndrome. Patients with symptomatic encephalitis will present with new psychiatric symptoms, cognitive deficits, seizures, flaccid paralysis, and tremors. Most will have a headache and low-grade fever. In patients with meningoencephalitis, a lumbar puncture will show a pleocytosis with mostly lymphocytes, normal to elevated glucose, and increased protein.

An 82-year-old man presents with slurred speech and unilateral arm weakness that have resolved. His non-contrast head CT is negative and he is given aspirin. Which of the following is the most appropriate management plan? Discharge home with PCP follow-up Low molecular weight heparin MRI and MRA of the head and neck Transcranial dopplers

MRI and MRA of the head and neck This patients symptoms are concerning for a transient ischemic attack (TIA), neurologic deficits that occur and then resolve within 24 hours. Most patients with a TIA will resolve within one hour (the previous definition). Patients who have a TIA are at high risk for stroke in the period immediately following a TIA and therefore further testing is warranted in a timely fashion. In most cases, patients should be admitted to an observation unit or hospital in order to rule out any reversible causes of the TIA. Testing includes vascular imaging to identify any critical stenosis. The MRI will also identify the subset of patients with TIA symptoms who ultimately have an infarct on MRI. If MRA is not available, carotid dopplers are an acceptable alternative. Once the vasculature has been evaluated, patients may be discharged with outpatient follow-up. Routine testing that often occurs during the observation period includes cardiac telemetry for dysrhythmia (particularly atrial fibrillation) and echocardiography to evaluate for valvular lesions or intracardiac thrombus.

A 27-year-old woman presents with a complaint of transient vision loss in her right eye. She states that she has had multiple similar episodes in the past six months. She also complains of incomplete bladder emptying, intermittent tremors, and intermittent weakness in her left arm. Which of the following is the best diagnostic test for the suspected diagnosis? CSF testing for myelin basic protein CT scan of the brain and spine MRI of the brain and spine Serum oligoclonal bands

MRI of the brain and spine MRI of the spine is the diagnostic modality of choice for multiple sclerosis (MS). Multiple sclerosis is characterized by demyelination of axons within the central nervous system. Patients present with symptoms that are "scattered in time and space" meaning that they have intermittent symptoms in anatomically disparate locations. Patients may have patchy motor and sensory deficits and bladder dysfunction. Spinal cord lesions typically involve the posterior columns, lateral spinothalamic tracts, and the corticospinal tract. Brain and spinal MRI is the diagnostic modality of choice because it can be used to rule out causes of cord compression while also showing lesions diagnostic for MS

A 40-year-old man presents to the emergency department after an episode of blood-streaked vomiting. This happened following a night of excessive drinking and a few episodes of forceful vomiting. He presents with no other symptoms and is in otherwise good health. Which of the following is the most likely diagnosis? Bleeding esophageal varices Boerhaave syndrome Mallory-Weiss syndrome Ruptured peptic ulcer

Mallory-Weiss syndrome Mallory Weiss syndrome is a common cause of upper GI bleeding. It is often associated with alcoholism and eating disorders that are associated with forceful and recurrent vomiting or retching. Mallory Weiss syndrome is characterized by a focal, longitudinal tear that occurs at the gastroesophageal junction at the distal esophagus or proximal stomach. Tears are usually superficial, only involving the mucosal layer. Most heal spontaneously in 24-48 hrs. Endoscopy is used to visualize the tear and identify the source of bleeding. This procedure can be both diagnostic and therapeutic.

A 21-year-old previously healthy man presents to clinic after several nights of partying. He complains of headache, nausea, and emesis that now contains blood. He also endorses a constant burning sensation in the left upper quadrant of his abdomen that started after the retching began. What is the most likely diagnosis? Esophageal varices Esophagitis Mallory-Weiss tear Peptic ulcer disease

Mallory-Weiss tear Mallory-Weiss tear is caused by a longitudinal mucosal tear of the distal esophagus and proximal stomach. Patients typically present with a history of repeated forceful retching, and tears occur because of a sudden increase in intra-abdominal pressure. Patients with a history of alcoholism and hiatal hernia are at increased risk of a Mallory-Weiss tear. Signs and symptoms include hematemesis and epigastric or back pain. Endoscopy is required to make the diagnosis and can also be used as a therapeutic modality to control the actively bleeding laceration. Patients with a Mallory-Weiss tear may require a blood transfusion. Some tears heal spontaneously on their own without further intervention.

An 18-year-old college student with a history of HIV (CD4+ 250/µL) presents to the ED with a headache, fever, and stiff neck for 2 days. He thought he had a cold and has been taking acetaminophen without relief of his headache. Vital signs are T 102.38°F (39.1°C), BP 100/50 mm Hg, HR 140 bpm, and RR 30/min. He is sleepy but arousable. On exam, you place the patient's right hip and knee into a flexed position and then proceed to extend the knee. The patient winces when the knee is just beyond 90 degrees of flexion. You also note petechiae on his trunk and extremities, with one small area on his right forearm that looks like a purple patch with a gray necrotic center. Which of the following is the most likely diagnosis? Cryptococcal meningitis Herpes encephalitis Meningococcemia Pneumococcal meningitis Toxoplasmosis

Meningococcemia This patient has meningococcemia, a disease caused by Neisseria meningitidis. The clinical presentation ranges from a mild febrile illness to fulminant disease progressing to death within hours. Patients with meningococcal meningitis may present similarly to patients with meningitis of other origins with headache, photophobia, vomiting, fever, and signs of meningeal inflammation. Petechiae generally appear on the extremities and may progress to involve almost any body surface. Macular lesions may progress to purpura and ecchymoses in fulminant meningococcemia (purpura fulminans). The patient in this scenario exhibits a positive Kernig sign, representing meningeal irritation, and has a purpuric lesion on his right forearm characterized by a gray necrotic center surrounded by a purple ring. Morbidity and mortality are high in meningococcemia but reduced with prompt recognition and immediate initiation of antibiotic therapy. Ceftriaxone and vancomycin are acceptable first-line agents.

A 50-year-old man with a history of hyperlipidemia and a recent diagnosis of colon cancer presents to the office. He is scheduled for surgical resection in 6 weeks. He is in no apparent distress today in the office. He currently takes atorvastatin 10 mg daily. Vital signs show a heart rate of 80 bpm, blood pressure 127/75 mm Hg, respiratory rate 18/minute, and oxygen saturation of 98% on room air. Physical exam reveals a normal cardiac exam with audible S1 and S2 without murmur. Lungs are clear to auscultation in all lung fields, and his abdomen is nondistended and soft, with no rigidity or guarding on palpation. Various labs are ordered as part of his presurgical workup, including a carcinoembryonic antigen. In which aspect of the patient's care would the results of this test be most helpful? Determining the type of chemotherapy Determining the type of surgery Diagnosis of concurrent non-oncologic disease Monitoring for residual disease Staging of the tumor

Monitoring for residual disease Colorectal cancers account for about 10% of cancer-related deaths in the United States per year. They usually arise from mutations in adenomatous polyps. Risk factors can be genetic or environmental, with genetic susceptibilities carrying the most increased risk. Genetic risk factors include first-degree relatives with colon cancer, a history of breast or gynecologic cancer, and familial polyp syndromes. Environmental risk factors include hypercholesterolemia, coronary artery disease, low-fiber and high-animal-fat diets, obesity, tobacco or alcohol use, long-term red meat ingestion, and low socioeconomic status. Patients may be asymptomatic and diagnosed via screening or present with intestinal obstruction, perforation, or acute bleed. There may also be a change in bowel habits, hematochezia, abdominal pain, or blood per rectum. Diagnosis is made via biopsy through colonoscopy or surgery. While routine laboratory tests do not aid in the diagnosis or prognosis, the tumor marker carcinoembryonic antigen (CEA) is helpful. CEA is recommended to be drawn after diagnosis to aid in monitoring for residual disease. With a level > 5 ng/mL in newly diagnosed colon colorectal cancers, there is a poorer prognosis. If this level normalizes or lowers after surgical resection, the prognosis is improved. However, if levels remain elevated, the patient's prognosis remains very poor or may indicate residual or persistent disease.

An 18-year-old woman, who was just fired from her job, reports ingesting 50 tablets of 325 mg aspirin three hours prior to arrival in the ED. She has dyspnea and tinnitus but does not have any nausea and vomiting. On physical examination, you find a moderately distressed diaphoretic woman with tachycardia and tachypnea but clear lungs bilaterally and an oxygen saturation of 98%. You ask the nurse to initiate intravenous access, draw labs, and start continuous cardiac monitoring. The arterial blood gas results are called back to you by the lab. Which of the following is the most likely acid-base disturbance in this patient? Metabolic acidosis Metabolic acidosis and respiratory alkalosis Metabolic and respiratory acidosis Triple acid/base disorder (respiratory alkalosis, high anion gap metabolic acidosis, and metabolic alkalosis)

Metabolic acidosis and respiratory alkalosis Salicylates (aspirin) directly stimulate the respiratory center of the brain to cause hyperventilation and a subsequent primary respiratory alkalosis. A concurrent metabolic acidosis also develops, primarily from the uncoupling of oxidative phosphorylation, which leads to anaerobic metabolism, lactate production (with development of an anion gap), and hyperthermia. There is increased fatty acid metabolism as well, which results in excess ketone production and superimposed ketoacidosis. The respiratory alkalosis is not compensation for the metabolic acidosis.

You are evaluating a 73-year-old woman with a history of weakness, fatigue, and difficulty walking. She lives alone and unassisted. She has no past medical history and takes no medications. Physical examination shows an enlarged, smooth, and tender tongue. Pallor of the conjunctivae is also seen. Which of the following laboratory studies would be most likely to confirm the diagnosis? Iron panel Mean corpuscular volume Methylmalonic acid and homocysteine levels Peripheral blood smear

Methylmalonic acid and homocysteine levels This patient most likely has vitamin B12 (cobalamin) deficiency. Vitamin B12 deficiency typically presents with paresthesias (eg, numbness and tingling), ataxia, glossitis, and megaloblastic anemia (eg, weakness, fatigue, easy bruising). Laboratory studies will show increased serum (and urine) concentrations of both methylmalonic acid (MMA) and homocysteine (HC). These increased levels occur as a result of their decreased rate of metabolism. In contrast, isolated levels of HC is only seen in folate deficiency. Thus, this test is critical in differentiating the specific vitamin deficiency as the etiology of the anemia.

A 60-year-old man with insulin-dependent diabetes mellitus, gastroparesis, hypertension, and congestive heart failure has just received a new diagnosis of pheochromocytoma. Which of the following medications should be discontinued? Furosemide Insulin lispro Lisinopril Metoclopramide

Metoclopramide Pheochromocytoma is a rare neuroendocrine neoplasm which secretes catecholamines and can cause hypertension, headache, sweating, tachycardia and can lead to hypertensive crises. Most cases of pheochromocytoma are discovered incidentally on abdominal CT scan or MRI. The discovery of an adrenal mass should lead to further testing, including plasma and urinary measurements of catecholamines and fractionated metanephrines. Once pheochromocytoma is diagnosed, surgical excision is the next step in treatment. To prepare for surgical excision, medications which cause stimulation of pheochromocytoma activity must be discontinued. These medications include beta-blockers (in the absence of alpha-blocking agents), glucagon, metoclopramide, and histamine. Metoclopramide blocks the inhibitory effects of dopamine receptor activation on sympathetic nerves, which can potentially lead to catecholamine release in the setting of a pheochromocytoma. Beta-blockers are routinely administered to patients with pheochromocytoma, but only after they have been established on treatment with alpha-adrenergic blocking agents such as phenoxybenzamine. If beta-blockers are administered prior to alpha-adrenergic blockade, the result can be further elevation in blood pressure due to unopposed alpha-adrenergic action in the peripheral blood vessels. Surgical excision of a pheochromocytoma is a high-risk procedure and must be performed by a skilled surgical team. Proper preoperative medical preparation to ensure volume expansion and blood pressure control is essential.

A patient presents to the emergency department with a complaint of recurrent symptoms of vomiting, sensitivity to loud noises, and a severe pulsating headache over the right temporal region that has lasted over the past ten hours. What is the most likely diagnosis? Cluster headache Migraine headache with aura Migraine headache without aura Tension-type headache

Migraine headache without aura Migraine headache without aura is defined as a unilateral, pulsatile severe headache that lasts between 4-72 hours and is accompanied by nausea, vomiting, photophobia, or phonophobia. The onset of symptoms is typically triggered by emotional stress, menstruation, malnutrition, or sleep deprivation. Migraine headaches is a genetic disorder and is more common in women than men. It can be divided into multiple subtypes of migraine headaches including migraine headaches without aura, migraine headaches with aura, menstrual migraines, and vestibular migraines. An aura is a focal neurological symptom that occurs before the onset of headache. This phase includes the gradual development of positive or negative symptoms such as bright visual lights, tinnitus, rhythmic movements, visual loss, loss of hearing, or the inability to move a body part. This stage typically lasts no longer than one hour before the onset of a headache. If a patient does not experience symptoms of an aura, but does exhibit signs and symptoms of a migraine headache, a diagnosis of migraine headache without aura can be made. Treatment includes administration of non-steroidal anti-inflammatory medications for mild symptoms and oral triptans plus antiemetic medication for moderate to severe attacks. Cluster headache (A) is a unilateral severe orbital, supraorbital, or temporal pain accompanied by forehead sweating, conjunctival injection, lacrimation, nasal rhinorrhea, or miosis on the same side as the pain. This head pain is typically not accompanied by nausea, vomiting, phonophobia, or photophobia. Treatment includes administration of oxygen via a high flow nonrebreathing facial mask and triptans, such as sumatriptan. Migraine headache with aura (B) includes symptoms of an aura with visual, sensory, or motor changes that gradually spread over five minutes and last less than one hour. Typically these aura symptoms are unilateral and are followed by the onset of a unilateral severe headache. Treatment is similar to migraine headaches without aura which includes triptans and antiemetic medication. Tension-type headache (D) is the most common type of headache and is characterized by a mild to moderately severe, bilateral, non-throbbing headache. These headaches are not associated with nausea, vomiting, photophobia, or phonophobia and treatment includes over the counter analgesics including nonsteroidal anti-inflammatory medications, acetaminophen, and aspirin.

A 9-year-old girl is in the clinic for a follow-up. She was diagnosed with asthma a few days ago after an ED visit. She has had night awakenings about twice a week. She has been using her inhaled short-acting beta agonist almost every day for relief of shortness of breath. On physical examination, you note occasional wheezing on both lung fields. Which of the following is the correct classification of this patient's asthma severity? Intermittent Mild persistent Moderate persistent Severe

Moderate persistent The patient has symptoms consistent with moderate persistent asthma. The Expert Panel Report 3 of the National Asthma Education and Prevention Program (NAEPP) describes moderate persistent asthma as being characterized by any of the following: daily symptoms of asthma, nocturnal awakenings more than once per week (although not every night), daily need for inhaled short-acting beta agonists (SABAs) for symptom relief, some limitation in normal activity, forced expiratory flow in 1 second (FEV1) between 60 percent and 80 percent of predicted, ratio of FEV1 to forced vital capacity (FEV1) between 1 to 5 percent below normal range for age, and two or more exacerbations requiring oral glucocorticoids per year. NAEPP definitions of severity apply best to patients in the untreated state. For patients already receiving therapy, the focus should shift to assessing control and adjusting medications accordingly.

A 70-year-old man with a history of hypertension presents with complaints of fatigue and worsening lower back and pelvic pain. He denies history of trauma or fever. On examination, he is pale with diffuse tenderness to palpation of his lumbar spine and pelvis. Neurological examination is nonfocal. Laboratory studies show a normocytic anemia and a creatinine of 1.9 mg/dL. His X-ray is shown above. What is the most likely diagnosis? Cauda equina syndrome Lymphoma Metastatic prostate cancer Multiple myeloma

Multiple myeloma Multiple myeloma is a neoplastic proliferation of plasma cells in the bone marrow resulting in a monoclonal immunoglobulin. Occurring primarily in older adults, patients present with a range of symptoms best remembered by the mnemonic CRAB (elevated Calcium, Renal insufficiency, Anemia and Bone lesions). Calcium levels are elevated which can result in symptoms of renal colic, constipation, and confusion. Renal insufficiency is often the result of deposition of monoclonal light chains (Bence Jones proteins) in the renal tubules (also known as myeloma kidney). Hypercalcemia can also contribute to the renal insufficiency as well. Patients often have a normocytic, normochromic anemia with associated symptoms of fatigue, pallor, and generalized weakness. Lastly, osteopenia and extensive osteolytic lesions are commonly seen and may result in pathologic fractures. Patients experience bone pain primarily in the spine and trunk and less so in the extremities. Radiographs will show characteristic punched-out lytic lesions. Diagnosis is confirmed by finding > 10% clonal bone marrow plasma cells in addition to evidence of tissue or organ impairment as noted above.

A 4-year-old girl is brought to the ED by her parents due to lethargy. A week prior, the girl had a cough and viral symptoms. Later, she developed fever and malaise. She has been less active with decreased appetite. A few hours prior to arrival in the ED, she was having difficulty breathing. On exam, the T is 38.3°C, RR is 35/min, HR is 126 bpm, and BP is 90/60 mm Hg. She has clear breath sounds, hepatomegaly, and decreased pulses. Which of the following is the most likely diagnosis? Bronchiolitis Dysrhythmia Myocarditis Pneumonia

Myocarditis This girl demonstrates signs and symptoms that are suspicious for myocarditis, which is a condition that results from inflammation of the heart muscle. The majority of children present with acute or fulminant disease. Myocarditis can be caused by infectious, toxic, or autoimmune conditions. Common causes of viral myocarditis include enterovirus (coxsackie group B), adenovirus, parvovirus B19, Epstein-Barr virus, cytomegalovirus, and human herpes 6 (HHV-6). The presentation of the disease is variable, and patients can present with symptoms that range from subclinical disease to cardiogenic shock, dysrhythmias, and sudden death. There is usually a history of a recent respiratory or gastrointestinal illness within the previous weeks. There is a prodrome of fever, myalgia, and malaise several days prior to the onset of symptoms of heart dysfunction. Later, patients present with heart failure symptoms, including dyspnea at rest, exercise intolerance, syncope, tachypnea, tachycardia, and hepatomegaly. Testing is focused on determining the severity of cardiac dysfunction and includes ECG, cardiac biomarkers, chest radiography, and echocardiography. Confirmation of myocarditis is generally made by cardiac magnetic resonance imaging or endomyocardial biopsy.

An 18-year-old agricultural laborer presents with an intensely sore throat and fever. He reports he has never been sick before. The patient's vaccination history is not available. Vitals are BP of 110/65 mm Hg, HR of 70 bpm, RR of 12 bpm, T of 100.8°F, and BMI of 20 kg/m2. The submandibular area shows significant edema, and laryngoscopic exam reveals a dull-colored, leathery plaque adhering to the right tonsil, which bleeds upon scraping. What is one of the potential complications of this patient's disease? Esophagitis Hemolytic uremic syndrome Myocarditis Reactive arthritis Splenic rupture

Myocarditis This patient has the typical presentation of diphtheria, caused by gram-positive Corynebacterium diphtheriae, which can produce a toxin that destroys tissue. Diphtheria, once common in the U.S. and elsewhere, has been fairly well eliminated by the development of the diphtheria toxoid in the 1920s and its routine inclusion in the Tdap vaccine and booster by the mid-20th century. Diphtheria should be considered in patients with concerning signs and symptoms and an unknown or incomplete vaccination history. The bacterium enters the nasopharynx and, if it is a toxin-producing strain, develops a lesion called a pseudomembrane on the nasopharynx or brachial tree. The word diphtheria comes from the Greek word for leather, descriptive of its thickness. Diphtheria lesions are known to bleed when scraped. Infected patients may have a sore throat, barking cough, and hoarseness with a mild fever. Cervical lymphadenopathy and submandibular swelling is common for serious disease. The toxin circulates to various parts of the body, where it can cause major complications, including myocarditis, nephritis, polyneuropathy, and thrombocytopenia. C. diphtheriae is a gram-positive bacteria that is treated with penicillin or erythromycin. The toxin must be treated separately by a neutralizing antitoxin. Upon preliminary identification of culture, the Centers for Disease Control and Prevention should be notified of the case, and another culture sample should be sent to them.

A 60-year-old woman presents with neck and chest rash. Review of systems is significant for proximal weakness and myalgias of 3 months duration. Examination of the skin reveals a violet-hue on both eyelids, reddened macules on the neck, shoulders and chest and thick, scaly skin on the dorsal surfaces of the metacarpophalangeal and proximal interphalangeal joints. However, there are no tender subcutaneous nodules. You send the patient for electromyography and nerve conduction testing. You would expect findings most consistent with which of the following pathologies? Myopathy Neuropathy Retinopathy Vasculopathy

Myopathy Polymyositis is an idiopathic inflammatory myopathy. It causes proximal myalgias and weakness, similar to polyarteritis nodosa and polymyalgia rheumatica, which can present as pelvic and shoulder girdle functional impairments. It most often presents in females over age 50 years. In addition, pharyngolaryngeal weakness occurs, and manifests as dyspnea, dysphagia and dysphonia. Three classic skin findings commonly coexist with polymyositis. A heliotrope rash (symmetric violet-erythema, located periorbitally, especially about the eyelids), a cloak-like or cape rash (erythematous confluent macules on the neck, shoulders and chest), and Gottron's papules (scaly, violaceous thickened knuckle skin, sometimes confused with psoriasis). When these are present, the syndrome can be designated polymyositis-dermatomyositis (dermatomyositis). Lab testing usually reveals a positive ANA and high creatinine kinase. The classic EMG findings are those of myopathy: positive sharp waves, fibrillations, complex repetitive discharges and small amplitude, short duration, polyphasic motor units with an early recruitment pattern. Treatment options include prednisone, methotrexate and cyclophosphamide.

An 83-year-old woman with a history of hypothyroidism, diabetes mellitus and hypertension is brought to the emergency department with altered mental status. She is found to be hypothermic, hypotensive and bradycardic. Puffiness in the hands and face, a thickened nose and swollen lips are noted on physical exam. Pupils are normal size, equal and reactive to light. Which of the following is the most likely diagnosis? Adrenal insufficiency Diabetic ketoacidosis Myxedema coma Opiate overdose

Myxedema coma Myxedema coma is defined as severe hypothyroidism leading to decreased mental status, hypothermia and other symptoms related to slowing of function in multiple organs. Myxedema coma can occur as the culmination of severe, long-standing hypothyroidism or be precipitated by an acute event such as infection or myocardial infarction. It is a medical emergency with a high mortality rate. Fortunately, it is now a rare presentation of hypothyroidism, likely due to earlier diagnosis as a result of the widespread availability of thyrotropin assays. Patients usually present with decreased mental status and hypothermia, but hypotension, bradycardia, hyponatremia, hypoglycemia, and hypoventilation are often present as well. Puffiness of the hands and face, a thickened nose, swollen lips and an enlarged tongue may occur secondary to nonpitting edema with abnormal deposits of mucin in the skin and other tissues, referred to as myxedema.

A 68-year-old woman with a history of diabetes mellitus and hypertension presents to the emergency department for left knee pain. She denies history of trauma. On exam, there is no overlying erythema, wounds, or lesions on the skin. Range of motion is limited due to pain. A plain radiograph of the knee is notable for linear calcifications in the articular cartilage. Joint aspiration of both joints reveals positively birefringent crystals. Which of the following oral medications is the most appropriate treatment? Allopurinol Colchicine Naproxen Prednisone

Naproxen Naproxen is the most appropriate treatment for this woman with calcium pyrophosphate deposition disease, or pseudogout. Pseudogout is characterized by the deposition of calcium-containing salts resulting in an acute pain in one or more joints. Pseudogout is most commonly seen in older adults and is seen equally in both men and women. Pseudogout is frequently associated with metabolic disorders such as hemochromatosis, hyperparathyroidism, diabetes mellitus, hypothyroidism, Wilson disease, and gout. Patients with pseudogout typically present with severe pain in one or more joints. The knee is the most commonly involved joint. Other joints typically infected include wrists, ankles, and elbows. Laboratory findings are usually nonspecific, such as leukocytosis, and elevated sedimentation rate and serum C-reactive protein. Arthrocentesis is the gold-standard diagnostic study and is critical for ruling out infectious arthritis. Positively birefringent rhomboid-shaped crystals are the hallmark synovial fluid finding for pseudogout. Synovial fluid leukocyte concentration in acute pseudogout is usually 15,000 to 30,000 per mL, with the majority being neutrophils. Punctate and parallel linear radiodensities in the cartilage are the defining feature of pseudogout on plain film radiography. Treatment for pseudogout typically involves intra-articular steroid injection, if feasible, nonsteroidal anti-inflammatory drugs (NSAIDs), colchicine, and systemic corticosteroids. Nonselective NSAIDs are typically first-line management unless contraindicated. If NSAIDs are contraindicated, colchicine is typically recommended, if it can be initiated within 24 hours of attack onset. Systemic corticosteroids are used when NSAIDs and colchicine are contraindicated or ineffective.

A patient presents with complaints of edema, malaise and sudsy urine. A microscopic urinalysis shows oval fat bodies and a Maltese cross pattern under polarized light. These findings are most consistent with which of the following diagnoses? Glomerular nephritis Nephritic syndrome Nephrotic syndrome Polycystic kidney disease

Nephrotic syndrome Sudsy or foamy urine indicates high levels of protein in the urine. Oval fat bodies are renal tubular cells which have absorbed filtered lipids, indicating high levels of excess lipids in the urine (i.e. lipiduria). These fat bodies produce a Maltese cross pattern under polarized light. The high levels of protein, along with fat in the urine, point to nephrotic syndrome. Lipiduria is highly sensitive for nephrotic syndrome. Nephrotic syndrome is a constellation of edema, hypoalbuminemia and the urinary excretion of greater than 3 g of protein per day due to a glomerular disorder. Treatment includes ACE inhibitors, fluid and sodium restriction, diuretics for swelling and appropriate management of potential secondary causes.

A 65-year-old man with cirrhosis and ascites presents to the emergency room with diffuse abdominal discomfort, fever, and altered mental status. His home medications include furosemide and spironolactone. On physical exam he is febrile and his abdomen is mildly tender with positive shifting dullness and fluid wave. You perform a diagnostic paracentesis. Which of the following findings best supports your diagnosis? Bloody appearance of ascites Neutrophil count > 250 pH > 7.34 Polymicrobial growth on ascitic culture

Neutrophil count > 250 Spontaneous bacterial peritonitis (SBP) is an infection of ascitic fluid that cannot be attributed to any other intrabdominal conditon. It is one of the most common bacterial infections in patients with cirrhosis and should always be at the top of the differential when examining a tender abdomen in the presence of ascites in a febrile patient. A peritoneal fluid absolute neutrophil count greater than 250 cells is the accepted criterion for the diagnosis of spontaneous bacterial peritonitis. Gram-negative bacteria are the most common pathogens causing spontaneous bacterial peritonitis and most are difficult to culture, therefore a positive culture is not needed for the diagnosis. Treatment is directed primarily at early administration of empiric antibiotics for the duration of 7-10 days.

Which of the following is more consistent with the diagnosis of encephalitis rather than meningitis? Fever Headache Neck stiffness New psychiatric symptom

New psychiatric symptom The clinical distinction between encephalitis and meningitis is typically characterized by the presence of a distinct neurologic abnormality in encephalitis. Encephalitis should be considered in patients presenting with the following clinical features, alone or in combination: new psychiatric symptoms, cognitive deficits (aphasia, amnesia, acute confusional state), seizures, and movement disorders. Often, there are symptoms of meningeal involvement as well, such as a headache, photophobia, and fever. There is overlap between encephalitis and meningitis. When this occurs, the condition is referred to as meningoencephalitis.

A 43-year-old woman presents with episodic epigastric pain that frequently follows a fatty meal and can last anywhere from 15 minutes to approximately two hours. At times the pain radiates toward her right shoulder. She has associated nausea without vomiting. Which of the following is the most appropriate next step in managing this patient? Obtain serum amylase and lipase levels Order a contrast-enhanced computed tomography scan Order a right upper quadrant abdominal ultrasound Prescribe oral omeprazole

Order a right upper quadrant abdominal ultrasound Ordering a right upper quadrant abdominal ultrasound would be the next best step in managing this patient with suspected gallbladder pathology. Cholelithiasis, although frequently asymptomatic, can cause episodic pain or biliary colic. The pain can be described as constant, squeezing, or gnawing, located in the epigastrium or right upper quadrant of the abdomen, and radiating toward the right shoulder. This pain can typically follow a fatty meal, and often patients describe pain upon wakening or pain that awakens them in the middle of the night. Individuals with cholelithiasis, or gallstones, can develop acute cholecystitis or can never develop symptoms caused by the gallstones. An ultrasound will demonstrate gallstones. Patients with frequent biliary colic can undergo an elective cholecystectomy for definitive treatment.

A woman with systemic sclerosis complains of painful blue fingers. It only seems to affect her during the cold winter months, especially if she doesn't wear gloves. She doesn't complain of arthralgias, myalgias or pruritus. Her most recent chest radiograph shows bibasilar fibrosis. Her last four serum creatinine results were normal. Which of the following medications do you recommend? Lisinopril Nifedipine Prednisone UVA-1 phototherapy

Nifedipine Systemic sclerosis has a multitude of manifestations. One of the more common signs is Raynaud's phenomenon. This cold temperature-induced condition results in intermittent and recurrent digital vasoconstriction, which causes finger pallor, cyanosis or hyperemia, as well as pain, numbness, ischemia and possibly ulceration. Treatment is geared at avoiding cold temperatures, wearing gloves and thick socks and smoking cessation. Medications can also be used, and include aspirin, prostaglandins, vasodilators and calcium-channel blockers, like nifedipine, especially when there is concomitant pulmonary fibrosis.

A 65-year-old man with a history of glucose-6-phosphate dehydrogenase deficiency presents to the emergency department with yellow skin and dark urine for 2 days. The patient reports that he was prescribed an antibiotic for a urinary tract infection 5 days ago. Vital signs include a HR of 102 bpm, BP of 120/80 mm Hg, RR of 20/min, oxygen saturation of 98% on room air, and T of 98.6°F. Physical examination findings include scleral icterus, jaundiced skin, conjunctival pallor, and no abdominal tenderness or masses. Which of the following antibiotics is most likely to have caused this patient's symptoms? Amoxicillin-clavulanate Cephalexin Doxycycline Fosfomycin Nitrofurantoin

Nitrofurantoin Glucose-6-phosphate dehydrogenase (G6PD) deficiency is the most common red blood cell enzymatic defect. The physiological role of G6PD is to protect red blood cells against oxidative injury. Therefore, patients with G6PD deficiency are at increased risk of oxidative injury to red blood cells, which can manifest as acute hemolytic anemia. G6PD is more common in males since it has an X-linked recessive inheritance pattern. Heterozygous females are unaffected. Most patients with G6PD deficiency are asymptomatic most of the time. However, patients can experience intermittent acute hemolytic episodes triggered by medications, foods (fava beans), or infections. In addition, some patients with G6PD deficiency have chronic anemia. The common medication triggers include nitrofurantoin, phenazopyridine, trimethoprim, and primaquine. The symptoms of an acute hemolytic episode typically begin 2-4 days after an oxidant exposure. Classic symptoms include pallor, jaundice, and dark urine. The severity of episodes varies between mild and self-limited to severe and life-threatening. The typical laboratory findings of all types of hemolytic anemia include anemia, reticulocytosis, indirect hyperbilirubinemia, elevated lactate dehydrogenase, and decreased haptoglobin. The peripheral blood smear findings associated with G6PD deficiency include microspherocytosis, eccentrocytes (bite cells), blister cells with hemoglobin puddled to one side, and Heinz bodies (collections of denatured globin chains attached to the red blood cell membrane seen on stained peripheral blood smears). The bite cells are caused by macrophages in the spleen removing the denatured hemoglobin. The diagnosis of G6PD is confirmed by a low G6PD enzyme activity level. The management of acute hemolytic episodes is to remove the inciting agent. In addition, aggressive hydration or blood transfusions may be required in cases with severe anemia. Patients with chronic hemolysis (rare) due to G6PD require folic acid supplementation. Patients who are diagnosed with G6PD deficiency should be educated on the common triggers (medications and foods) to avoid.

A 21-year-old woman presents with urinary frequency and dysuria for 2 days. Vital signs are within normal limits. Examination reveals mild suprapubic tenderness with no costovertebral angle tenderness. What is appropriate management of this patient? Ceftriaxone Nitrofurantoin Noncontrast CT scan of the abdomen and pelvis Phenazopyridine

Nitrofurantoin This patient presents with an uncomplicated acute cystitis and can be treated with nitrofurantoin. Urinary tract infection (UTI) is an inflammatory response to microorganisms in the urinary tract. Patients will present with dysuria, frequency, urgency, hematuria, and suprapubic pain when infection is in the bladder. Cystitis represents infection of the lower urinary tract (pyelonephritis is infection of the upper urinary tract) and is common in women of all age groups but relatively uncommon among young men. Uncomplicated cystitis is defined as inflammation of the bladder in patients in the absence of pregnancy, medical comorbidities, and toxic appearance. The most common causative organisms are Escherichia coli and Staphylococcus saprophyticus. First-line antibiotic agents for uncomplicated cystitis include nitrofurantoin and trimethoprim-sulfamethoxazole (TMP-SMX). Ciprofloxacin is a common first-line drug in regions where resistance to TMP-SMX is greater than 10-20%. Pregnancy should be ruled out in all women of child-bearing age prior to choosing an antibiotic.

Which of the following is the leading cause of nonsolid organ cancer-related death? Acute myeloid leukemia Hodgkin lymphoma Multiple myeloma Non-Hodgkin lymphoma

Non-Hodgkin lymphoma Non-Hodgkin lymphoma (NHL) is the leading cause of non-solid organ cancer-related death. It has a significantly worse prognosis than that of Hodgkin lymphoma. Signs and symptoms of NHL include persistent, painless, peripheral lymphadenopathy. It is found in the epitrochlear nodes, mesenteric nodes, mediastinal nodes, retroperitoneal nodes, and pelvic nodes. Enlargement of oral and nasal lymphoid tissue is also consistent with NHL. Any lymph node that is > 1 cm in size and persistent for > 4 weeks without association with a documented infection should be considered for biopsy. NHL may mimic infectious mononucleosis. Patients may also present with chest discomfort or superior vena cava (SVC) syndrome. Diagnosis is typically made by lymph node biopsy or bone marrow aspiration. Management is with supportive care, chemotherapy, and radiation. Other potential treatments include interferon-α, monoclonal antibodies, and bone marrow transplantation.

A 56-year-old man presents to the emergency department with right upper quadrant pain that radiates to the right scapula. His pain increases with deep respiration, and he reports associated nausea and multiple episodes of vomiting. His medical history includes hypertension, atherosclerosis, and gout with medical treatment involving lisinopril, atorvastatin, aspirin, and allopurinol. Vital signs include a HR of 104 bpm, RR of 18/min, BP of 156/99 mm Hg, T of 101.9°F, and SpO2 of 98% on room air. The patient has a nondistended abdomen that is tender to palpation in the right upper quadrant. Abdominal ultrasound findings are equivocal, and cholescintigraphy is ordered. Which of the following findings from this diagnostic study are considered diagnostic of the suspected diagnosis? Gallbladder wall thickening and edema Nonvisualization of the gallbladder on delayed images Presence of pericholecystic stranding Small amount of fluid in the gallbladder fossa Visualization of contrast within the common bile duct and gallbladder

Nonvisualization of the gallbladder on delayed images Acute cholecystitis is an inflammatory condition involving the gallbladder. This inflammatory response is most often subsequent to an obstruction of the cystic duct by a gallstone, which leads to inflammation secondary to mechanical, chemical, or bacterial factors. The most commonly indicated bacterial organisms involved in acute cholecystitis include Escherichia coli, Klebsiella sp, Streptococcus sp, and Clostridium sp. Clinical signs and symptoms of acute cholecystitis include biliary pain in the right upper quadrant of the abdomen that progressively worsens. This pain may radiate to the interscapular area, right scapula, or shoulder and is exacerbated by peritoneal irritation to include jarring and deep respiration. Other signs and symptoms include fever, anorexia, nausea, vomiting, and, occasionally, jaundice. An enlarged, tense gallbladder may be palpable during physical exam with localized rebound tenderness in the right upper quadrant. Lab results commonly include leukocytosis with less frequent findings including hyperbilirubinemia and elevated liver enzymes. Ultrasound of the right upper quadrant may demonstrate gallstones, gallbladder wall thickening, and a sonographic Murphy sign. If ultrasound is negative but acute cholecystitis is still suspected a cholescintigraphy study can be ordered. Cholescintigraphy, commonly referred to as a HIDA scan, uses a radioisotope that is injected intravenously, taken up by hepatocytes, and excreted into the bile. This scan is most useful for evaluating the patency of the common bile duct and ampulla. Nonvisualization of the gallbladder on delayed images is considered diagnostic of acute cholecystitis. Additional diagnostic evaluation includes magnetic resonance cholangiopancreatography (MRCP) or abdominal computed tomography (CT). Results from MRCP include detection of stones within the cystic duct and gallbladder wall thickening while abdominal CT findings include gallbladder wall edema, pericholecystic stranding and fluid, and high-attenuation bile. Acute cholecystitis is treated initially with supportive care, including intravenous fluid therapy, correction of electrolyte disorders, and pain control. Empiric antibiotic therapy should be initiated with ertapenem or piperacillin-tazobactam in low-risk community-acquired infections. Alternative regimens may be necessary for patients with risk factors (e.g., advanced age, immunocompromised, major medical comorbidities) or those with health care-associated acute cholecystitis. Emergency cholecystectomy is warranted in patients with progressive clinical courses (e.g., hemodynamic instability, intractable pain) or suspicion of complicated acute cholecystitis. Stable, uncomplicated patients should receive cholecystectomy during their initial hospitalization, while patients with comorbidities or severe disease can be treated initially with antibiotics and gallbladder drainage. Complications include gangrenous cholecystitis, perforation, emphysematous cholecystitis, cholecystoenteric fistula, and gallstone ileus.

Which of the following agents is considered the vasopressor of choice for treatment of septic shock? Dopamine Epinephrine Norepinephrine Phenylephrine

Norepinephrine Norepinephrine is considered the vasopressor of choice for treatment of septic shock. Norepinephrine acts primarily as an α-adrenergic agonist, causing vasoconstriction that results in an increase in blood pressure. It also has β-adrenergic properties, which causes an increase in cardiac output and heart rate. The combination of α-adrenergic and β-adrenergic properties benefits patients who have septic shock. Norepinephrine also has a short duration of action, which allows for rapid adjustment of dosing in response to changes in a patient's hemodynamic status.

A 45-year-old woman presents to your office with a complaint of pain "all over." She tells you that over the past few years her pain has worsened and she also experiences daily fatigue, difficulty concentrating, and headaches. Physical exam reveals multiple points of tenderness to palpation. Which of the following lab results would be expected? Decreased vitamin B12 Elevated erythrocyte sedimentation rate Elevated thyroid stimulating hormone Normal complete blood count

Normal complete blood count Fibromyalgia is a syndrome that causes chronic, widespread pain and tenderness. The etiology of fibromyalgia is unknown and the pathogenesis has not been proven, however it is believed to be related to a dysfunction in central pain processing. In addition to complaints of pain, patients often present with fatigue, cognitive difficulties, and multiple somatic and psychiatric symptoms. Fibromyalgia is a diagnosis of exclusion and patients should be evaluated for other disorders before making the diagnosis. Physical exam findings are generally within normal limits other than tenderness to palpation of multiple soft tissue points. Laboratory test results, including complete blood count, are within normal limits. There is no cure for fibromyalgia and management involves a multi-disciplinary approach including education, lifestyle modifications and medication. Patients should be counseled regarding proper diet, physical exercise, stress reduction, and healthy sleep habits. An initial pharmacologic approach with monotherapy using amitriptyline, duloxetine, pregabalin or milnacipran is recommended. Prognosis for patients with fibromyalgia varies, with better outcomes seen when patients seek help, engage in more physical exercise, pace their activities and in those that have less guarding during physical exam.

A 40-year-old woman presents to her primary care provider's office with complaints of droopy eyelids and difficulty chewing. She has difficulty keeping her eyes open as the day goes on. Throughout a meal, she has progressive difficulty chewing. She holds her jaw up at the end of the meal. Which of the following physical exam findings would this woman most likely exhibit? Decreased sensation to light touch in extremities Diminished or absent deep tendon reflexes Hyperreflexive deep tendon reflexes Normal deep tendon reflexes

Normal deep tendon reflexes A patient with myasthenia gravis will most likely exhibit normal deep tendon reflexes. Myasthenia gravis is an autoimmune disorder characterized by skeletal muscle weakness and fatigability. Myasthenia gravis is the most common neuromuscular transmission disorder. The majority of patients who develop myasthenia gravis have autoantibodies that attack acetylcholine receptors. Myasthenia gravis has a bimodal age distribution with the first peak in the second to third decades and a later peak in the sixth to eighth decade. The first peak has a female predominance and the second peak has a male predominance. Neonatal myasthenia gravis can occur as a result of transplacental passage of maternal autoantibodies to neuromuscular junctions. The hallmark symptom of myasthenia gravis is fluctuating skeletal muscle weakness and fatigue. The weakness and fatigue are made worse after contractile force or activity of the muscle. More than half of patients with myasthenia gravis present with ocular symptoms of ptosis, diplopia, or both. Bulbar muscles are commonly involved, resulting in difficulty chewing, dysphagia, and voice changes. Respiratory muscle involvement may lead to respiratory insufficiency and pending respiratory failure. Certain drugs can exacerbate myasthenia gravis, some of these drugs include aminoglycosides, fluoroquinolones, beta blockers, magnesium, and neuromuscular blocking agents used during anesthesia. The diagnosis of myasthenia gravis is based on clinical history and exam, serologic tests for acetylcholine receptor autoantibodies, and with nerve conduction studies. A chest radiograph and computed tomography scan should be obtained to assess for the presence of a thymoma. Acetylcholinesterase inhibitors, such as pyridostigmine, are typically the first line treatment for symptomatic myasthenia gravis. Chronic immunosuppressive agents or corticosteroids are used in patients who are still significantly symptomatic on pyridostigmine. Plasmapheresis or intravenous immunoglobulin (IVIG) are used in patients with severe myasthenia gravis. A thymectomy is recommended for patients younger than 60 years of age.

A 78-year-old woman presents to the emergency department via ambulance after having a generalized seizure at home. Per her accompanying family member, the patient developed a fever and headache earlier today. She was acting strange before the seizure started. There is no recent head injury or illness, but she had a cold sore a few days ago. The patient's medical history is pertinent for hyperlipidemia and glaucoma, for which she takes atorvastatin, latanoprost eye drops, and timolol eye drops. Vital signs reveal a blood pressure of 108/72 mm Hg, heart rate of 112 bpm, respiratory rate of 24/min, oxygen saturation of 97%, and temperature of 101.7°F. The patient is obtunded and confused on examination. Deep tendon reflexes are 4+. A right-sided hemiparesis is noted. Kernig sign and Brudzinski sign are negative. An MRI of the brain is performed and is shown above. Given the suspected diagnosis, which of the following will CSF analysis reveal, in addition to CSF protein content? Low glucose, decreased lymphocyte count, decreased erythrocyte count Low glucose, increased neutrophil count Normal glucose, increased lymphocyte count, increased erythrocyte count Normal glucose, increased neutrophil count, increased erythrocyte count Normal glucose, normal cell counts

Normal glucose, increased lymphocyte count, increased erythrocyte count Encephalitis is an infection involving the tissues of the brain. It is most commonly caused by viral infections, such as herpes simplex virus (HSV) (most common cause), arboviruses, West Nile virus, varicella-zoster virus, and lyssaviruses (e.g., rabies). Protozoan, bacterial, rickettsial, helminth, and spirochetal infections can also be implicated. The infection is associated with a high degree of mortality if left untreated. Unlike meningitis, encephalitis causes abnormal brain function. Patients with encephalitis may have a prodrome of flu-like illness. Symptoms often include headache, fever, change in behavior, and speech abnormalities. Seizures can also develop. Symptoms develop rapidly. Meningismus signs, such as neck stiffness and photophobia, are uncommon. On physical examination, patients are febrile with altered levels of consciousness. A postictal state may be observed if the patient recently experienced a seizure. Focal neurologic deficits, such as hemiparesis or cranial nerve palsies, are common. Patients with suspected encephalitis must undergo neuroimaging with CT scan or MRI. HSV encephalitis has a predilection for the temporal lobe, which can be observed on imaging. Electroencephalography (EEG) is often performed in patients with encephalitis who have seizures. Lumbar puncture is performed once a space-occupying lesion has been ruled out with imaging to further evaluate the cerebrospinal fluid (CSF). CSF analysis in patients with encephalitis demonstrates an increased white blood cell count (predominantly lymphocytes), elevated protein count, and normal to low glucose level. Increased red blood cells suggests HSV encephalitis. CSF can be sent for additional testing, such as viral PCR testing, Gram stain, and culture. Empiric treatment for HSV encephalitis should be started in all patients presenting with encephalitis of unknown origin or suspected HSV encephalitis. This includes intravenous acyclovir. Antiseizure medications should be administered if seizure develops. Neurologic complications are prevalent in surviving patients, with sequelae such as cognitive impairment and speech disorders.

A 35-year-old meat cutter comes to your office with persistent symptoms of nausea, vomiting, and diarrhea, which began about 36 hours ago on the last day of a 5-day Caribbean cruise. His wife was sick during the first 2 days of the cruise with similar symptoms. Physical examination is negative, and a stool specimen is negative for white blood cells. Which one of the following is the most likely cause of his illness? Giardia Hepatitis A Norovirus Rotavirus

Norovirus Recent reports of epidemics of gastroenteritis on cruise ships are consistent with Norovirus infections due to waterborne or foodborne spread. In the United States these viruses are responsible for about 90% of all epidemics of nonbacterial gastroenteritis. The noroviruses are common causes of waterborne epidemics of gastroenteritis and have been shown to be responsible for outbreaks in nursing homes, on cruise ships, at summer camps, and in schools. Symptomatic norovirus gastroenteritis typically develops 24-48 hours after ingestion of contaminated food or water or after contact with an infected individual. The onset can be abrupt or gradual, but each episode is short-lived, lasting only 24-72 hours. The absence of fecal leucocytes and occult blood in stool is helpful in ruling out other enteroinvasive infectious diarrhea processes. Stool culture should be performed to exclude infection with bacterial organisms such as Yersinia, Shigella, Salmonella, and Campylobacter species. Oral fluid and electrolyte replacement is generally adequate for the treatment of norovirus infections. Symptomatic relief can be achieved using antiemetics for nausea and vomiting and analgesics for myalgias and headache.

A 55-year-old man with a history of alcohol use disorder and cirrhosis presents to the emergency department reporting abdominal distension and weight gain. He states that his symptoms have been slowly progressing over the past 3 weeks. He is not taking any medications and reports no orthopnea, abdominal pain, lethargy, or confusion. Vitals are temperature 37°C, blood pressure 120/80 mm Hg, pulse 89 bpm, and SpO2 98% on room air. Physical exam reveals a man with jaundice in no acute distress. His abdomen is markedly distended and nontender to palpation in all four quadrants with bulging flanks. There is dullness to percussion in the flank region. The remainder of the physical exam is unremarkable. Which of the following is the best next step in management? Obtain abdominal ultrasound Perform diagnostic paracentesis Prescribe oral diuretic with outpatient follow-up in 2 weeks Prescribe oral lactulose with outpatient follow-up in 2 weeks Recommend sodium restricted diet

Obtain abdominal ultrasound This patient is presenting with ascites, or the accumulation of fluid within the peritoneal cavity. The most common cause of ascites in the United States is cirrhosis, which accounts for about 80% of cases. Other common causes include infection, malignancy, and heart failure. The development of ascites in patients with cirrhosis is related to portal hypertension, which causes vasodilatory substances such as nitric oxide to accumulate, resulting in splanchnic vasodilation and renal hypoperfusion. Hypoperfusion of the kidneys activates the renin-angiotensin-aldosterone system (RAAS), causing increased fluid retention. Patients with ascites typically present with abdominal distension that may be accompanied by abdominal discomfort, weight gain, shortness of breath, and peripheral edema. The distension is usually progressive and develops over the course of several weeks. If the ascites is secondary to cirrhosis, patients may also have signs of hepatic decompensation, such as confusion (encephalopathy) or evidence of gastrointestinal bleeding. Patients with ascites commonly have bulging flanks with dullness to percussion (flank dullness) on examination. They may also have shifting dullness (change in location of dullness to percussion when the patient is repositioned). If there are indications of ascites based on history and physical exam, the first step in diagnosis should be abdominal ultrasound to confirm its presence and to look for evidence of cirrhosis or malignancy. Diagnostic paracentesis is most useful for determining the cause of ascites. Ascites fluid should be evaluated for appearance (e.g., clear, bloody, cloudy), serum-to-ascites albumin gradient, cell count and differential, and total protein concentration. Most occurrences of small amounts of cirrhosis-related ascites can be managed with dietary sodium restriction alone. For patients with moderate to large ascites, diuretic therapy, therapeutic paracentesis, or both may be necessary. Diuretic treatment is usually with spironolactone alone or in combination with furosemide. The prognosis for patients with ascites due to cirrhosis is poor, with a < 50% 2-year survival rate after the onset. Thus, liver transplantation should be considered for these patients.

A 38-year-old man with a history of heavy alcohol use presents with hematemesis. After obtaining a history and performing a physical exam, a Mallory-Weiss tear is suspected. The patient's vital signs are stable with a normal heart rate and blood pressure. Which of the following is the next best step in caring for this patient? Administer an antiemetic Obtain intravenous access Place a nasogastric tube Prepare the patient for emergent upper endoscopy

Obtain intravenous access Obtaining intravenous (IV) access is the most important first step in caring for a patient with gastrointestinal (GI) bleeding. Intravenous access allows blood to be drawn for a type and screen, coagulation studies, and determination of hematocrit and hemoglobin levels. It also allows for fluid resuscitation to maintain intravascular volume. Two large bore IV catheters should be used. A Mallory-Weiss tear is a tear in the esophageal mucosa often following forceful vomiting. Patients with a history of chronic alcohol use are at greater risk for developing a Mallory-Weiss tear. Once the patient is stabilized, an esophagoduodenoscopy (EGD) can confirm the diagnosis and allows for treatment options if the bleeding has not resolved spontaneously. Epinephrine injection, embolization, or coagulation therapy can be used to stop the bleeding although most cases resolve spontaneously as the mucosa heals over 48 to 72 hours.

A man who is pale and ill-appearing presents to the emergency department with reports of two episodes of bright red vomitus. A medical history review reveals alcohol use disorder, chronic hepatitis, and cirrhotic liver disease. Vitals include a HR of 105 bpm, RR of 18/min, BP of 96/55 mm Hg, T of 97.7°F, and SpO2 of 95% on room air. Laboratory testing reveals hemoglobin 10.1 g/dL, hematocrit 38.7%, INR 1.7, and platelet count 82,000/μL. Two large-bore intravenous access lines are placed with aggressive fluid repletion and prophylactic antibiotic administration initiated. What is the next step in treatment? Fresh frozen plasma transfusion Octreotide infusion Platelet transfusion Propranolol administration Transvenous intrahepatic portosystemic shunt

Octreotide infusion Esophageal varices are dilated submucosal veins that develop secondary to portal hypertension. This vascular abnormality is present in 50% of patients with cirrhosis of the liver and may result in significant upper gastrointestinal bleeding. Patients with actively bleeding esophageal varices present with signs and symptoms of acute gastrointestinal hemorrhage, including hematemesis and melena. Acute gastrointestinal hemorrhage results in hypovolemia, postural vital signs, or shock in patients with esophageal varices. Laboratory testing should include a complete blood count, platelet count, prothrombin time, INR, serum liver enzymes, serum creatinine, and blood urea nitrogen. Varices that are actively bleeding require blood type and crossmatch to prepare for potential transfusion. Any patient with bleeding esophageal varices requires stabilization with intravenous fluid replacement. Antibiotic prophylaxis with intravenous agents such as ceftriaxone reduces in-hospital mortality and the risk of serious infection in patients with esophageal varices. Vasoactive substances (octreotide, somatostatin, terlipressin) reduce splanchnic and hepatic blood flow and portal venous pressures and should be administered after patients are initially stabilized. Additionally,l band ligation via endoscopy can improve survival. Vitamin K should be administered to patients with cirrhosis and an abnormal prothrombin time and patients with hepatic encephalopathy and increased ammonia should additionally be treated with lactulose. Endoscopy is performed within 12-24 hours of hemodynamic stabilization to exclude other or associated causes of upper gastrointestinal bleeding. Balloon tamponade can also be used to control acute bleeding while more definitive treatment measures are being prepared. Portal pressure can be reduced long-term with the administration of nonselective beta-blockers (e.g., propranolol, nadolol), with a target heart rate of 55-60 bpm. A transvenous intrahepatic portosystemic shunt (TIPS) procedure is appropriate for patients who have had recurrent bleeding from gastric varices, who have portal hypertensive gastropathy, who have not responded to endoscopy or pharmacologic therapies, who are unwilling or unable to undergo other therapies, or who live in remote locations. Once the shunt is placed, it should be periodically monitored using Doppler ultrasonography or hepatic venography to evaluate for thrombosis or stenosis. Patients with severe cirrhosis and portal hypertension require eventual liver transplantation for definitive treatment. Without treatment, bleeding esophageal varices have a 50% chance of spontaneously stopping, but most patients have a recurrent variceal bleed within the first 6 weeks after surviving the prior bleeding episode. Factors that increase the risk of esophageal variceal bleed include size of the varices, presence of red wale marking on the varix during endoscopy, severity of liver disease (according to the Child-Pugh score), and active alcohol use. Fresh frozen plasma transfusion (A) is not appropriate in patients with cirrhosis and an elevated INR once stabilized as it does not have proven benefit but does have potential harms, such as increased portal pressures and risk of portal vein or deep vein thrombosis. Platelet transfusion (C) is recommended for a platelet count < 50,0000 per mcL in patients with decompensated cirrhosis and active severe upper gastrointestinal bleeding. Propranolol administration (D) is a treatment for long-term reduction of portal pressures, but it is not recommended for patients with actively bleeding esophageal varices. Transvenous intrahepatic portosystemic shunt (E) is not appropriate in a patient with actively bleeding esophageal varices and is instead used as a long-term treatment for patients with recurrent variceal bleeding, those who have not had success with prior therapies, those who cannot undergo less invasive treatment options, or those who live in remote locations and thus have a higher risk of mortality related to variceal bleeding.

A 73-year-old woman is being evaluated for visual disturbances. She describes several episodes that felt as if the 'curtains were coming down' on her. Soon after these episodes, her vision returns to normal. Physical examination shows a tortuous and dilated temporal artery. Which of the following is most likely to be associated with this patient's condition? Increased intracranial pressure Polymyalgia rheumatica Transient ischemic attack Trigeminal neuralgia

Polymyalgia rheumatica Amaurosis fugax (AF), a transient loss of vision in one or both eyes, is a classical ophthalmologic finding seen in patients presenting with giant cell arteritis. AF consists of episodes of temporary blindness that soon returns to normal vision following each episode. Patients typically describe a sensation of the "curtains coming down" as each episode occurs. Polymyalgia rheumatica is seen in about 50% of patients with giant cell arteritis.

A 29-year-old woman complains of mouth sores. On examination, you note white plaques covering the tongue and buccal mucosa. When scraped off using a tongue blade you note an erythematous base. Laboratory testing reveals a positive rapid HIV. Which of the following is the most likely diagnosis? Herpes simplex virus Kaposi sarcoma Oral candidiasis Oral hairy leukoplakia

Oral candidiasis The patient has oral candidiasis, also known as thrush. Candidiasis is a mucocutaneous fungal infection that affects up to 80% of AIDS patients. Characteristic white plaques cover the tongue and buccal mucosa, which can be easily scraped off using a tongue blade. Although similar in appearance, the plaques associated with oral hairy leukoplakia cannot be scraped off, distinguishing it from oral candidiasis. Candida can also affect the esophagus, resulting in painful swallowing and ultimately malnutrition. Oral candidiasis is associated with CD4 counts below 500 cells/mm3 and esophageal candidiasis with more advanced disease (CD4 counts below 100 cells/mm3). Oral candidiasis is managed with clotrimazole or nystatin suspension. Oral fluconazole and amphotericin B are reserved for refractory or recurrent disease.

A 35-year-old woman presents to your office to establish care as a new patient. She has a history of ulcerative colitis with multiple flare-ups. Which of the following is the most appropriate maintenance therapy? Hydrocortisone suppositories Oral mesalamine Oral metronidazole Oral prednisone

Oral mesalamine Oral mesalamine is a 5-aminosalicylic acid (5-ASA) derivative used in the maintenance treatment of ulcerative colitis. Ulcerative colitis is a disease that begins distally at the rectum and progresses proximally towards the cecum, with no skip lesions seen (a differentiating factor from Crohn's disease). Tenesmus and bloody diarrhea are the most common symptoms. Ulcerative colitis is a rare condition where smoking appears protective (again, a differentiator from Crohn's disease). Laboratory studies often show anemia, increased erythrocyte sedimentation rate (ESR), and decreased serum albumin. Abdominal X-ray shows colonic dilatation. Sigmoidoscopy or colonoscopy are used to confirm the diagnosis, but are not used during an acute flare due to the risk of perforation or toxic megacolon. Rectal or oral 5-ASA medications (e.g. mesalamine or sulfasalazine) are used in maintenance treatment. Acute flares are treated with rectal or oral glucocorticoids.

A 16-year-old boy presents with 3 days of crampy abdominal pain and bloody diarrhea that started after eating a chicken salad sandwich. He has decreased fluid and solid intake but no vomiting. His BP is 110/70 mm Hg, HR is 80 bpm, and RR is 12/min. What is the most appropriate treatment for this patient? Intravenous ceftriaxone Intravenous fluids Oral ciprofloxacin Oral rehydration

Oral rehydration Salmonellosis is a gastroenteritis caused by Salmonella enterica, a nontyphoidal serotype of Salmonella. It is caused by ingestion of contaminated water or food, particularly chicken and uncooked eggs. Symptoms appear about 8-48 hours after ingestion and present as fever, nausea, vomiting, crampy abdominal pain, and bloody or "pea soup" diarrhea. Diagnosis may be made with a stool culture but is not always necessary, as the illness is usually self-limiting. The best treatment for salmonellosis is supportive care and focus on fluid replacement to combat dehydration. For this patient, who has no vomiting and does not show altered vital signs, oral rehydration is most appropriate. Antibiotic treatment with ciprofloxacin or ceftriaxone may be considered for certain cases (e.g., severe illness, immunocompromised host such as a patient with sickle cell disease).

An 11-year-old boy with a medical history of asthma presents to the clinic with his parent. His current medications include inhaled fluticasone propionate. His parent states they noticed a thick, white coating on his tongue for the past week. The patient reports no difficulty swallowing or mouth pain. He is afebrile and has a heart rate of 110 bpm, respiratory rate of 20 bpm, blood pressure of 100/60 mm Hg, and SpO2 of 99% on room air. Physical examination reveals a well-appearing boy in no acute distress. His oral mucosa is pink and moist without ulceration, erythema, or edema. A thick, white plaque is noted on his tongue. It is easily scraped off with a tongue depressor but reveals an erythematous tongue beneath. The remainder of his physical exam findings are normal. Which of the following is the most effective measure to prevent the most likely diagnosis? Nystatin mouthwash Oral rinse after use of inhaler Periodic glucose monitoring Prophylactic antibiotics Surgical removal with close follow-up every 3 months

Oral rinse after use of inhaler The patient in the vignette presents with oral candidiasis, or thrush, secondary to inhaled glucocorticoid use. Candidiasis is a common fungal infection of the skin and oral mucosa, typically caused by an overgrowth of Candida albicans. C. albicans is considered normal flora in the gastrointestinal and genitourinary tracts, but in certain circumstances, it can invade and cause disease. In addition to oropharyngeal disease, candidal infection can also occur on the skin (most commonly in moist skinfolds) or vagina and in the blood or urinary tract. Risk factors for candidal infection include immunocompromised states (e.g., AIDS, malignancy, organ transplant, infancy), denture use, smoking, oral or inhaled corticosteroid use, and antibiotic use. Patients with oral candidiasis often report patchy white lesions on the tongue or buccal mucosa. This may lead to a cotton-mouth feeling, loss of taste, or pain with swallowing. Some patients may be asymptomatic. Diagnosis of oral candidiasis is usually made clinically in patients with characteristic findings and risk factors for infection. However, in patients where the diagnosis is not straightforward, a Gram stain or potassium hydroxide preparation can be performed from tongue scrapings, which will reveal budding yeasts with or without hyphae. Oral candidiasis can also be distinguished from other oral diseases because the plaques can easily be scraped off the tongue. Treatment of oral candidiasis involves topical antifungal agents (typically mouthwashes), such as nystatin or clotrimazole for mild disease, or systemic therapy with fluconazole for moderate to severe disease. Prevention of oral candidiasis is mainly centered around oral hygiene. For example, patients using inhaled steroids should consider oral rinses after medication use. Dentures should be cleaned every night, and patients should avoid prolonged antibiotic or steroid use if possible.

A 52-year-old man with a history of heavy alcohol use presents with hematemesis. He has had several episodes of bright red vomitus over the last 2 hours. His blood pressure is 136/84 mm Hg, and his heart rate is 86 bpm. A complete blood count reveals a hematocrit and hemoglobin that is within normal limits. What is the next best step in management? Admit the patient to the intensive care unit for further observation Insert a nasogastric tube to evacuate gastric contents Order a type and screen and begin fluid resuscitation Send the patient for endoscopy

Order a type and screen and begin fluid resuscitation Ordering a type and screen and beginning fluid resuscitation are important first steps in the management of hematemesis with suspected esophageal varices. In cases of acute blood loss, the hematocrit and hemoglobin may not decline immediately. A type and screen will be ready to help facilitate a blood transfusion in the event that the patient is actively hemorrhaging. This patient's history of heavy alcohol use puts him at risk for esophageal varices. Cirrhosis, either viral or alcohol induced, is the most common cause of esophageal varices in the United States. Other risk factors include portal hypertension, other liver diseases, and parasitic infections in resource-limited countries. Esophageal varices carry a mortality rate of 30 to 50 percent because of the amount of blood loss that can occur. Endoscopic verification is necessary for a definitive diagnosis and to develop a treatment plan. The amount of blood loss can be significant, and emergency measures include assessing the rate and volume of bleeding, assessing the need for airway management, obtaining intravenous access, beginning fluid resuscitation, and assessing for the need for blood products. Medical management may include using octreotide to decrease hepatic blood flow and variceal pressure. Treatment can include band ligation or sclerotherapy. Sclerotherapy involves injecting agents to promote clotting.

A 26-year-old gravida 3 para 2 at 12 weeks gestation presents with fever, myalgias, headache, and malaise. There have been multiple cases of influenza in the community and her influenza swab is positive. Which one of the following is recommended by the Centers for Disease Control and Prevention in this situation? Acyclovir Oseltamivir Rimantadine Supportive therapy only

Oseltamivir Influenza is primarily diagnosed clinically. Sudden onset of symptoms is a telltale sign of influenza. Common symptoms include high fever, headache, sore throat, myalgia, cough, rhinorrhea, and fatigue. The Centers for Disease Control and Prevention recommends treatment for persons at higher risk for complications from influenza because studies have shown that antiviral treatment may shorten the duration of symptoms and reduce the risk of complications, especially in these higher risk populations. Pregnant and postpartum women, including those who have had pregnancy loss, are at risk of severe influenza-related complications because of the changes in immune, respiratory, and cardiovascular systems that occur during pregnancy. The Center for Disease Control recommends that neuraminidase inhibitors be prescribed for pregnant women and for those up to two weeks postpartum who have suspected or confirmed influenza. Women can continue to breastfeed while being treated with antivirals. Treatment with antiviral agents, should be started within 48 hours of symptom onset. The recommended treatment of influenza in pregnancy is one of the neuraminidase inhibitors, such as oseltamivir. Neuraminidase inhibitors have modest effectiveness in reducing influenza-related symptoms in patients and they protect against both strains of influenza A and influenza B.

A 70-year-old man presents to the emergency department after his spouse called emergency medical services because he was less responsive than usual. His spouse reports that for the past 6 weeks he has had a cough, hemoptysis, and unexplained weight loss. He became very drowsy the past couple of days. Vital signs include a HR of 90 bpm, BP of 122/80 mm Hg, RR of 24/min, oxygen saturation of 94% on room air, and T of 98.6°F. Physical examination reveals a drowsy man who is arousable to painful stimuli. He has a regular rate and rhythm and rhonchi with auscultation of the mid-lung fields. The patient's laboratory findings are below: Glucose: 135 mg/dL Sodium: 116 mEq/L Blood urea nitrogen: 30 mg/dL Creatinine: 1.0 mg/dL Serum osmolality: 254 mOsm/kg Urine osmolality: 140 mOsm/kg Which of the following conditions is this patient at risk for with rapid correction of hyponatremia? Cerebral edema Congestive heart failure Diplopia Osmotic demyelination syndrome Ventricular dysrhythmias

Osmotic demyelination syndrome The patient in the vignette has symptoms of lung cancer and has hyponatremia. Small cell lung cancer can cause syndrome of inappropriate antidiuretic hormone secretion as a paraneoplastic syndrome, which can cause hyponatremia. Patients who have severe hyponatremia can present with decreased levels of consciousness, including a comatose state, and seizures. Patients with hyponatremia who either have severe hyponatremia (serum sodium < 120 mEq/L) or symptomatic hyponatremia are usually treated in the hospital for close monitoring while the hyponatremia is corrected. The treatment of severe hyponatremia must balance correcting the hyponatremia to avoid worsening symptoms but not correcting too quickly and causing osmotic demyelination syndrome. Osmotic demyelination syndrome occurs when individuals with chronic hyponatremia (hyponatremia present for at least 48 hours) have irreversible damage to the brain from overly rapid correction of sodium. The clinical manifestations typically occur 2-6 days after the overly rapid correction and include dysarthria, dysphagia, paraparesis or quadriparesis, behavioral disturbances, movement disorders, seizures, lethargy, confusion, disorientation, obtundation, and coma. The diagnosis should be suspected in patients with new neurologic symptoms following rapid correction of sodium. Osmotic demyelination syndrome can be prevented by not correcting sodium more than 6-8 mEq/L in any 24-hour period. Furthermore, desmopressin can sometimes be used to prevent osmotic demyelination syndrome. The recommended goal for correcting severe or symptomatic hyponatremia is to correct by 4-6 mEq/L in the first 24-hour period. This 4-6 mEq/L correction should be accomplished in the first 6 hours in patients with severe symptoms of hyponatremia, such as a coma or seizure. Hypertonic (3%) saline is often used in the treatment of hyponatremia. Fluid restriction is also recommended in patients with severe hyponatremia and one or more of the following: hypervolemic status (such as heart failure and cirrhosis), syndrome of inappropriate antidiuretic hormone secretion, advanced kidney disease, and primary polydipsia. Vasopressin antagonists may also be used in the treatment of refractory hyponatremia.

Which antiepileptic medication commonly causes hyponatremia at therapeutic doses? Lamotrigine Oxcarbazepine Phenytoin Topiramate

Oxcarbazepine Oxcarbazepine causes profound hyponatremia < 125 in 2.5 percent of patients, and some degree of hyponatremia in as many as 50 percent of patients. Most patients develop hyponatremia within the first three months of initiating oxcarbazepine treatment, though some patients can develop hyponatremia more than one year after initiating treatment. The mechanism is not well understood. The risk increases when patients are on other antiepileptic medications and thiazide diuretics. Neurologists frequently monitor serum sodium levels for the first three months after initiating treatment and advocate checking a serum sodium level on any patient who is on oxcarbazepine and presents with headache, somnolence, drowsiness, or increased seizure frequency.

A 22-year-old man presents to the ED with unilateral, throbbing headache. He reports no family history of headache, but he has had this type of headache before. Beginning around age 20 years, he started having four to six "flares" a day for 6 weeks at a time. Each flare would last 30 minutes, and they mainly occurred at night. Eventually they resolved but reappeared after his 21st birthday and lasted for about two months. He had been asymptomatic until 2 days ago when the headaches returned. Physical examination shows conjunctival injection and rhinorrhea. Which of the following treatments should most likely be initiated first in this patient? Acetaminophen, aspirin, and caffeine Oxygen Topiramate Verapamil

Oxygen Cluster headaches occur most often in middle-aged men. It is described as a recurrent, unilateral, excruciating periorbital headache that lasts from 15 minutes to three hours. Generally headaches are typically nonpulsatile and constant, frequently occurring at night. Cluster headaches on the other hand are associated with ipsilateral conjunctival injection, lacrimation, and nasal congestion. Cluster headaches classically take place in groups over days to weeks, occurring at the same time of day and in the same location. Acute attacks can be treated with 100% oxygen. Other treatments include triptans, ergotamine, intranasal lidocaine, and butorphanol.

A 19-year-old man presents to the Emergency Department for right knee pain. He has no significant past medical history but recently recovered from having diarrhea for several days. He has intermittent back pain and itchy eyes, but review of systems is otherwise negative. His vital signs are within normal limits for his age. His extremities appear atraumatic with no deformities. His knee is mildly painful with active range of motion without overlying skin changes. Which of the following is the most likely diagnosis? Avascular necrosis Gouty arthritis Reactive arthritis Rheumatoid arthritis

Reactive arthritis Reactive arthritis is an acute, nonpurulent seronegative spondyloarthropathy classically characterized by asymmetric oligoarthritis, urethritis and conjunctivitis. The latter two symptoms may or may not be present; only a minority of patients have the "classic triad" of symptoms at presentation. Vague constitutional symptoms such as fatigue, malaise and fever are common. Symptoms typically begin approximately one to six weeks after an antecedent gastrointestinal or genitourinary illness (e.g., dysentery, sexually transmitted illness). Typical organisms causing post-enteric reactive arthritis include Salmonella, Shigella, Campylobacter, Yersinia, Escherichia coli and Clostridium difficile. Common organisms implicated in post-venereal reactive arthritis include Chlamydia and Ureaplasma. The lower extremities, especially the knees, feet, ankles and toes are affected more than the upper extremities. Dactylitis, or "sausage digit", is a distinctive feature of peripheral spondyloarthropathies but is not specific for reactive arthritis. Patients may also complain of back pain or buttock pain. Synovial fluid analysis is nonspecific and shows generalized inflammatory findings. Arthritic symptoms tend to last for three to five months but can become chronic. Treatment mainly involves analgesia with high-dose nonsteroidal anti-inflammatory drugs (NSAIDs), such as indomethacin, but combination antibiotic therapy is indicated in cases of Chlamydia-induced reactive arthritis.

A 65-year-old woman presents with a scaling, itchy rash near her left nipple for the past month. She has tried over-the-counter cortisone and lotions without relief. On physical examination, there is an erythematous, scaling rash that involves the left nipple and a portion of the left areola. A hard mass approximately 1 cm in diameter is located beneath the areola. The area is nontender. Her last mammogram was 2 years ago and was negative for any concerning abnormalities. What is the most likely diagnosis? Candida mastitis Ductal ectasia Intraductal papilloma Nipple adenoma Paget disease

Paget disease Paget disease of the breast is a rare malignancy that presents clinically as a dermatitis in the nipple and areola areas. Usually, the presentation is unilateral and associated with itching or burning in the area. The rash associated with Paget disease of the breast appears eczematous and erythematous, although it may appear ulcerative or eroded. The rash typically begins on the nipple and spreads toward the areola. The peak incidence of occurrence is between 50 and 60 years of age. Diagnosis is made by finding intraepithelial adenocarcinoma cells on full-thickness punch or wedge biopsy of the nipple. Some cases of Paget disease of the breast express hormone receptors, which can help guide treatment options. Mammography is necessary to identify the mass and assess for associated cancers. Mammographically occult lesions require MRI for further evaluation. The treatment and prognosis of Paget disease of the breast are dependent on the extent of malignancy. The standard treatment is typically mastectomy, although surgical excision and radiation may be an option. Sentinel lymph node biopsy is important to evaluate for axillary lymph node involvement.

A 35-year-old man presents to a pulmonology clinic for follow-up on laboratory studies he had performed due to chronic cough and dyspnea. He reports that he has never smoked cigarettes, and his medical history consists of diabetes mellitus treated with metformin. He has a family history of emphysema. His vital signs today include a heart rate of 85 bpm, blood pressure of 132/76 mm Hg, respiratory rate of 30 breaths per minute, pulse oxygenation of 97% on room air, and temperature of 98.8°F. Physical examination reveals a well-appearing man. He has a regular rate and rhythm and mild intermittent lower lobe wheezing. You are reviewing the genotyping that you ordered for his suspected condition and his results are ZZ. What would you most expect the pathology of this patient's lungs to demonstrate? Distal acinar emphysema Mucous gland hyperplasia Panacinar emphysema Proximal acinar emphysema Subpleural apical blebs

Panacinar emphysema Emphysema is a pathological term that describes structural changes seen in chronic obstructive pulmonary disease (COPD). These changes include abnormal and permanent enlargement of airspaces distal to the terminal bronchioles and destruction of the airspace walls. The acinus refers to the structures distal to the terminal bronchioles, and it includes the respiratory bronchioles, alveolar ducts, alveolar sacs, and alveoli. There are subtypes of emphysema that damage varying parts of the acinus. Panacinar emphysema is defined by enlargement or destruction of all parts of the acinus, and it is most often associated with alpha-1 antitrypsin deficiency. Alpha-1 antitrypsin is a protein produced in the liver that helps to inhibit elastase. Elastase is a protease enzyme released from immune cells that helps to fight off infection in the lungs. While elastase is beneficial in its immunology role, it causes damage to healthy tissue if present in excessive amounts. Individuals with alpha-1 antitrypsin deficiency do not have sufficient alpha-1 antitrypsin to inhibit elastase, which leaves the elastase unchecked and leads to the damage of healthy tissue. Emphysema due to alpha-1 antitrypsin deficiency manifests clinically with cough, dyspnea, and wheezing. Alpha-1 antitrypsin deficiency is an inherited condition that may be present in varying degrees of severity based on the genes inherited. It is appropriate to screen all patients with COPD for alpha-1 antitrypsin deficiency, but patients diagnosed with COPD at ≤ 45 years of age, patients with COPD who are nonsmokers, patients with a family history of emphysema, and patients with predominantly basilar chest radiograph changes are particularly at high risk. The recommended screening laboratory studies are a serum level of alpha-1 antitrypsin and alpha-1 antitrypsin genotyping. The treatment of alpha-1 antitrypsin deficiency is very similar to the standard care for patients with COPD. This may include beta-agonist and muscarinic antagonist bronchodilators, inhaled or oral glucocorticoids, pulmonary rehabilitation, supplemental oxygen, preventive vaccinations, and prompt treatment of lower respiratory tract infections. Patients with alpha-1 antitrypsin deficiency should be advised to avoid active or passive cigarette smoke exposure. In addition, patients with alpha-1 antitrypsin deficiency with spirometry evidence of airflow limitation should be treated with weekly intravenous augmentation therapy of pooled human alpha-1 antitrypsin.

Which of the following statements best explains the use for penicillin in patients with a history of acute rheumatic fever? Penicillin is only recommended for patients with carditis during their acute rheumatic fever Penicillin is recommended to prevent Group A Strep colonization, which is more common in patients with acute rheumatic fever Penicillin is used for its anti-inflammatory effect on the affected heart valves Penicillin is used to prevent future episodes of acute rheumatic fever, which can lead to worsening heart disease

Penicillin is used to prevent future episodes of acute rheumatic fever, which can lead to worsening heart disease Penicillin is recommended as prophylaxis for patients with rheumatic heart disease to prevent future Group A Strep infections, which can lead to recurrence of acute rheumatic fever and worsening heart disease. Prior to the availability of penicillin prophylaxis, up to 75% of patients with rheumatic heart disease had a recurrence of acute rheumatic fever. Such recurrences can cause worsening of heart disease in patients who had carditis during their initial episode of acute rheumatic fever. The risk of recurrent acute rheumatic fever decreases with time from the initial episode of acute rheumatic fever. Penicillin prophylaxis is recommended for patients with both carditis (A) and chorea during acute rheumatic fever. Although patients who presented with chorea may not have concomitant carditis, they have a 20% chance of developing carditis in the future. Colonization with Group A Strep (B) is not known to be more common in patients with acute rheumatic fever. However, decreasing Group A Strep colonization is recommended, as colonization may increase risk of recurrence of acute rheumatic fever. Penicillin is not used for its anti-inflammatory effect on the affected heart valves (C), as anti-inflammatory properties of penicillin in the absence of infection have not been described. Instead, salicylates are used to decrease inflammation.

What is the most common cause of upper gastrointestinal bleeding? Aortoenteric fistula Boerhaave syndrome Esophageal varices Peptic ulcer disease

Peptic ulcer disease Peptic ulcer disease (PUD) is responsible for almost half of all upper gastrointestinal bleeding. PUD involves both gastric and duodenal ulcers. H. pylori bacteria is thought to be the cause of 70-80% of duodenal ulcers and 60-70% of gastric ulcers. The bacteria causes inflammation of the stomach and duodenal mucosa and disrupts natural defense mechanisms leading to ulceration. NSAIDs are the second leading cause of PUD. Patients typically present with epigastric pain that is burning or gnawing in nature. Pain may also be described in the chest and back. The pain with duodenal ulcers usually occurs a couple of hours after meals and at night and is typically improved or relieved by eating.

A 25-year-old man presents to the clinic reporting sudden shortness of breath and pain with breathing. He reports no preexisting medical conditions or current medication use but is a current smoker. Vital signs include HR 143 bpm, RR 27 breaths per minute, BP 102/71 mm Hg, T 98.1°F, and SpO2 87% on room air. Physical examination demonstrates a tall, thin man with diminished breath sounds and decreased tactile fremitus on the right side. Chest radiograph results are shown above. Which of the following is the best initial treatment? Administer anticoagulants Administer diuretics Perform chest tube thoracostomy Perform needle decompression Provide supplemental oxygen and observation

Perform chest tube thoracostomy Pneumothorax is defined as gas in the pleural space, with spontaneous pneumothorax occurring in the absence of an external event. This condition is more common in tall, thin male patients between ages 10 and 30 years. Risk factors for primary spontaneous pneumothorax include subpleural blebs, smoking (cigarettes and cannabis), and genetic predisposition. Secondary spontaneous pneumothorax presents as a complication of underlying lung disease and tends predominantly to affect men older than 55 years. While nearly every lung disease can cause spontaneous pneumothorax, the most commonly associated diseases are chronic obstructive pulmonary disease (COPD) and tuberculosis (TB). Laboratory and ECG findings are generally nonspecific, while arterial blood gas results classically indicate hypoxemia and respiratory alkalosis. A patient who is stable is defined as one who can speak in full sentences and has a respiratory rate < 24 breaths per minute, heart rate between 60 and 120 beats per minute, normal blood pressure, and oxygen saturation > 90% on room air. For patients who are unstable or those with severe respiratory distress, rapid bedside pleural ultrasonography is the recommended diagnostic modality to diagnose a pneumothorax. Patients who are stable should undergo chest radiography in the upright position, with a pneumothorax being identified by a white visceral pleural line that is convex toward the chest wall. If a diagnosis cannot be made with radiography, patients should undergo chest computed tomography, which is the most accurate diagnostic modality for detecting a pneumothorax. A large pneumothorax is defined as more than 3 cm between the pleural line and chest wall at the apex on a chest radiograph, while a small pneumothorax is any that is ≤ 3 cm from the chest wall at the apex or ≤ 2 cm from the chest wall at the hilum. In patients who are stable with a first episode of spontaneous pneumothorax, a small pneumothorax should be treated by observation for 4 to 6 hours with or without supplemental oxygen. A large pneumothorax in this patient population requires drainage with aspiration via needle or catheter with subsequent placement of a chest tube or catheter if aspiration fails to re-expand the lung. Any patient who is unstable should undergo immediate chest tube thoracostomy with needle decompression implemented in those with delayed chest tube insertion. Patients with underlying pleural disease should additionally be treated for their underlying disease following more immediate treatment of the current pneumothorax.

A patient is being evaluated for dyspnea and lower extremity swelling. On physical exam the patient has jugular venous distention with inspiration and 2 + pedal edema. Hepatojugular reflex is present. The patient has clear lung fields and no murmur is appreciated. Which of the following findings would support the diagnosis of constrictive pericarditis over restrictive cardiomyopathy? Cardiomegaly Left bundle branch block Pericardial knock S3 heart sound

Pericardial knock A Pericardial knock is associated with constrictive pericarditis, which is the most likely diagnosis in this patient. Approximately 50% of patients with constrictive pericarditis present with a pericardial knock, which is an accentuated heart sound occurring slightly earlier than a third heart sound. This would not be expected in restrictive cardiomyopathy. Conversely, an audible S3 is frequently present in persons with restrictive cardiomyopathy because of the abrupt cessation of the rapid ventricular filling. Constrictive pericarditis is a rare but disabling condition characterized by impaired filling resulting from restraint of ventricular diastolic expansion by a stiff pericardium. Any cause of pericarditis can lead to the condition. Some common causes include viruses, cardiac surgery, mediastinal irradiation and connective tissue disease. Patients present with dyspnea, fatigue and peripheral edema. Examination may show evidence of right sided heart failure including ascites, pedal edema, hepatojugular reflux and jugular vein engorgement with inspiration, also known as a Kussmaul sign. Pulmonary congestion is absent. Both constrictive pericarditis and restrictive cardiomyopathy can present with signs and symptoms of right sided heart failure making it difficult to distinguish one from the other. Further diagnostic testing and specific physical exam findings can aid in making the correct diagnosis.

A 76-year-old man with colorectal cancer presents to the ED with dyspnea and fatigue. He is hypotensive, tachycardic, tachypneic and afebrile. The jugular venous pulse rides high on lateral neck inspection. Cardiac sounds, but not breath sounds, are distant. There is no discernable friction rub or murmur, however, his blood pressure decreases during inspiration. An ECG reveals normal rhythm, increased rate and decreased voltages. Which of the following treatments is most appropriate in this patient's plan of care? Cardioversion Endarterectomy Hemodialysis Pericardiocentesis

Pericardiocentesis This scenario most likely represents cardiac tamponade. Pericardial tamponade refers to the dampening effect of rapidly accumulating pericardial effusion. An increase in intrapericaridal pressure compresses the heart chambers, decreases venous return and ultimately decreases cardiac output. As this occurs, it becomes ever more difficult for blood to flow from chamber to chamber. Causes include pericarditis, traumatic aortic dissection and myocardial rupture. Patients usually present with severe dyspnea, fatigue and hypotension. Typical exam findings include Beck's triad of hypotension, distant heart sounds and increased jugular venous pressure. Tachycardia and clear-sounding tachypnea are common. Pulsus paradoxus, a decrease in systolic blood pressure more than 10 mm Hg during inspiration, is also commonly present. However, pulsus paradoxus also accompanies constrictive pericarditis, congestive heart failure, pulmonary embolism, and end-stage obstructive pulmonary disease. Distant heart sounds and friction rubs may be present. Chest radiographs show large cardiac silhouettes, and ECGs may reveal a widespread decrease in voltage with an effusion and electrical alternans in tamponade. Classic echocardiographic findings are effusion, interventricular septal shift during inspiration, diastolic collapse of the right atrium and respiration-timed alterations in transvalvular flow. This medical emergency is treated with cardiopulmonary stabilization, pericardiocentesis (percutaneous drainage of pericardial fluid), cautious volume replacement and inotropic medications such as dobutamine.

A 44-year-old woman presents to the clinic reporting shortness of breath and palpitations. She has a history of hypertension and generalized anxiety disorder, for which she takes metoprolol and fluoxetine. Vital signs demonstrate HR 115 bpm, RR 18 breaths per minute, BP 147/92 mm Hg, T 98.8°F, and SpO2 99% on room air. Physical exam demonstrates vesicular breath sounds in all lobes bilaterally and resonance to percussion on chest examination. The rest of the exam is unremarkable. The patient reports no recent surgery, trauma, immobilization, hemoptysis, or malignancy. She has had a superficial venous thrombosis in the past but reports no history of deep venous thrombosis or pulmonary embolism. Relevant laboratory results include a D-dimer value of 430 ng/mL. ECG reveals sinus tachycardia. Which of the following is the best next step in management of this patient? Chest radiograph CT pulmonary angiography Initiation of anticoagulant therapy Reassurance and follow-up Ventilation-perfusion scan

Reassurance and follow-up Pulmonary embolism (PE) is caused by blockage of the pulmonary venous system due to a thrombus. It necessitates immediate evaluation and appropriate treatment as it can prove fatal, with most deaths being recognized antemortem. Sudden-onset dyspnea, pain on inspiration, tachypnea, and tachycardia constitute the predominant signs and symptoms of pulmonary embolism. Pain with forced dorsiflexion of the foot, called Homan sign, is a classic but nonspecific sign of distal venous thrombosis and pulmonary embolism. Primary risk factors for developing pulmonary embolism are synonymous with those for distal venous thromboembolism (i.e., venous stasis, vessel wall injury, and hypercoagulable state [Virchow triad]). Wells criteria include clinical signs and symptoms of deep venous thrombosis, pulmonary embolism as the most likely diagnosis, tachycardia > 100 beats per minute, immobilization of > 3 days, surgery in the previous 4 weeks, hemoptysis, or malignancy. A score > 4.0 suggests a PE is likely, while scores ≤ 4.0 indicate a low likelihood for PE. Laboratory tests may include a D-dimer, but elevation of this value is caused by a variety of mechanisms, and thus this test is best used to rule out pulmonary embolism in patients with a low pretest probability. Values < 500 ng/mL (0.5 µg/mL) are considered normal. The patient in the vignette above, for example, has a Wells score of 1.5 and a D-dimer < 500 ng/mL, which decreases the clinical suspicion for a pulmonary embolism and requires only reassurance and follow-up at this time. Pulmonary angiography remains the gold standard test for diagnosing pulmonary embolism, but helical CT pulmonary angiography has become the preferred clinical test as it is noninvasive and widely available. A ventilation-perfusion scan (V/Q scan) can be performed in patients unable to undergo computed tomography. Due to obstruction of the pulmonary circulation, patients can develop systemic circulatory congestion and subsequent right heart failure. This complication manifests as peripheral edema, jugular venous distention, and gastrointestinal and hepatic congestion, resulting in anorexia, nausea, and vomiting. Anticoagulation should be started during admission for secondary prevention and is continued after discharge for 3 months to life-long, depending on the suspected pulmonary embolism etiology. Chest radiograph (A) is not a reliable diagnostic imaging modality in diagnosing or excluding pulmonary embolism. Additionally, this patient has a low probability of a pulmonary embolism due to her low Wells score and normal serum D-dimer level. The signs and symptoms she is experiencing are better explained by her generalized anxiety disorder or an acute panic attack. CT pulmonary angiography (B) is the preferred diagnostic imaging for pulmonary embolism. It is not appropriate in this patient as she has a low likelihood of pulmonary embolism and her symptoms are most likely related to her generalized anxiety disorder. Initiation of anticoagulant therapy (C) is not appropriate in a patient with a Wells score of 1.5 and a normal serum D-dimer level. Patients with a high clinical suspicion for pulmonary embolism should be started on anticoagulants. Ventilation-perfusion scan (E) is a less commonly used modality for diagnosing pulmonary embolism and is best implemented for those unable or unwilling to undergo CT pulmonary angiography. This modality is not indicated for the patient in the vignette above as she has a low probability of pulmonary embolism based on her risk factors and laboratory results.

A 13-year-old boy presents to clinic with a worsening rash. The rash started on his right forearm and has progressed to involve the right upper arm and the left arm. It is not pruritic or painful. The rash has been present for about three weeks. He also complains of headache and fatigue. The patient returned from a camping trip in Pennsylvania one month ago. On exam, he has several erythematous macules with central clearing on the upper extremities and trunk. Which of the following associated findings is this patient most likely to have? Chronic encephalitis Complete heart block Monoarticular arthritis Peripheral facial nerve palsy

Peripheral facial nerve palsy This patient has early disseminated Lyme disease as evidenced by the secondary erythema migrans lesions. Lyme disease is caused by transmission of Borrelia burgdorferi, a spirochete, via the bite of the tick Ixodes. Transmission is most common in the northeast United States. Lyme disease occurs in multiple stages. The initial stage is early localized disease occurring within the first month of exposure, characterized by erythema migrans at the site of the tick bite. Next, early disseminated disease may develop from release of the spirochetes into the bloodstream, three to twelve weeks following exposure. The classic findings in early disseminated disease are secondary erythema migrans lesions, constitutional symptoms, cranial neuropathy, meningitis, and carditis. The most common cranial neuropathy is facial nerve (CN VII) palsy. In endemic areas, Lyme disease is the leading cause of facial nerve palsy.

A 4-year-old girl presents to the clinic with her parents reporting weakness in both legs that started in her feet and has progressively ascended to now reach the level of her hips. She reports a deep aching pain in her legs and a new-onset inability to walk. Medical history reveals a period of 4 days of bloody diarrhea, fever, and abdominal pains that resolved without treatment about 10 days ago. The patient is taking no medications. Vital signs reveal a BP of 90/50 mm Hg, HR of 110 bpm, RR of 18/min, and T of 98.6°F. Physical exam reveals absent reflexes at bilateral ankles and knees with symmetric muscle weakness of bilateral feet, calves, and thighs. The patient has no sensory deficits of either limb, and upper extremity reflexes and movement are normal. A magnetic resonance image shows gadolinium enhancement of the lumbosacral nerve roots. Laboratory values are included below. CBC: WBC: 8.8 × 103/µL RBC: 4.01 × 106/µL Hemoglobin: 12.2 g/dL Hematocrit: 37.1% Mean corpuscular volume: 93 fL Mean corpuscular hemoglobin: 30.4 pg Mean corpuscular hemoglobin concentration: 32.9 g/dL Red blood cell distribution width: 12.8% Platelets: 263 103/µL Neutrophils: 67% Lymphocytes: 22% Monocytes: 7% Eosinophils: 3% Basophils: 1% Cerebrospinal fluid (lumbar puncture): Appearance: clear Opening pressure: 12 cm H2O WBC: 0 cells/µL Glucose level: 100 mg/dL Protein level: 65 mg/dL Which of the following structures is primarily impaired in this patient's condition? Dorsal spinal columns Motor cortex Neuromuscular junctions Peripheral nerve fibers Ventral spinal columns

Peripheral nerve fibers The patient in the above vignette is demonstrating classic signs and symptoms of Guillain-Barré syndrome. Guillain-Barré syndrome is an uncommon neuropathic condition manifested by bilateral, progressive ascending muscle weakness with minimal to absent reflexes. Guillain-Barré syndrome occurs when the body produces antiganglioside antibodies, which attack peripheral nerve fibers and destroy their myelin sheath. Campylobacter jejuni infections are the provoking factor in over 30% of patients with Guillain-Barré syndrome worldwide. Other triggers include infections with cytomegalovirus, Mycoplasma pneumoniae, Haemophilus influenzae, Epstein-Barr virus, human immunodeficiency virus, lymphoma, Hodgkin lymphoma, systemic lupus erythematosus, stressful events, surgery, and immunization. For diagnosis of Guillain-Barré syndrome, a patient must demonstrate diminished or absent reflexes and progressive muscle weakness. Sensory deficits are minimal in most patients with Guillain-Barré syndrome, except in one subtype (acute motor-sensory axonal neuropathy). Patients may also have cranial nerve involvement, autonomic dysfunction, elevated protein in the cerebrospinal fluid, slowing of nerve conduction velocity on electromyography, and gadolinium enhancement of the affected nerves on magnetic resonance imaging. Guillain-Barré syndrome can be deadly if the muscles of respiration are affected. The course of Guillain-Barré syndrome is self-limited, and most patients recover fully within 1 year of symptom onset. Treatment is supportive, with special attention to the airway and respiratory system. Intravenous immunoglobulin treatment and plasma exchange have proven somewhat beneficial in severe cases.

A 19-year-old woman presents to the ED with pain and dyschromatopsia in her right eye. She also describes varying degrees of intermittent paresthesias over the previous month and occasional transient gait disturbance. An MRI shows white matter pathology. Which of the following lumbar puncture findings is associated with her condition? IgM and IgG antibodies to Borrelia burgdorferi Pleocytosis and oligoclonal bands of immunoglobulin G Positive VDRL test Xanthochromia

Pleocytosis and oligoclonal bands of immunoglobulin G This patient has multiple sclerosis, which is a neurologic disorder that causes variable motor, sensory, visual, and cerebellar dysfunction as a result of multiple focal areas of CNS demyelination. The patient's orbital pain is likely due to optic neuritis. Dyschromatopsia is a change in color perception and may be more prominent than visual disturbance. Although MRI is the gold standard, lumbar puncture can aid in the diagnosis. The lumbar puncture in approximately 50% of the cases will show pleocytosis, which is an increased number of lymphocytes. In 85-95% of cases, there will be oligoclonal bands of immunoglobulin G.

A previously healthy 6-year-old girl presents to your office with a complaint of an itchy scalp. She has been scratching her head and neck constantly and has not slept well due to severe itching. Physical exam reveals excoriations on the head and neck and white nits attached to the hair shafts. Which of the following is the most appropriate treatment? Lindane Permethrin Petroleum jelly Trimethoprim-sulfamethoxazole

Permethrin Pediculosis capitis is a condition caused by infestation of the scalp and hair by the head louse, Pediculus humanus capitis. Also referred to as lice or lice infestation, pediculosis capitis is very common worldwide and most frequently affects children. Lice are parasites that live on the human body and feed on human blood. The injection of saliva after biting causes an allergic reaction with resulting pruritus. Lice spread through person-to-person contact or through contact with fomites (e.g. brushes or clothing). Diagnosis is through physical exam revealing the presence of nits, immature lice, or adult lice. First-line treatment is with topical pediculicides, such as permethrin. A second treatment should be applied on day nine after the first treatment to ensure eradication.

A 32-year old woman and her 2 children present to your office with a 4-day history of intense itching that is worse at night. They specifically complain of pruritus under their arms and in between their fingers and toes. They deny any changes in household cleaners, new carpets or any allergies to pets. On physical exam you notice small papules and vesicles on the extremities and the axilla. You also notice burrows in between the fingers and toes. Skin scrapings confirm your suspected diagnosis. Which of the following is the first-line treatment for this disease? Corticosteroid cream Malathion lotion Permethrin cream Topical mupirocin

Permethrin cream Based on the history and physical exam, this patient and her children most likely have scabies. Human scabies is an intensely pruritic skin infestation caused by the host-specific mite Sarcoptes scabiei hominis. Bite distribution and intractable pruritus that is worse at night, as well as scabies symptoms in close household contacts should immediately rank scabies at the top of the clinical differential diagnosis. Small papules, vesicles and burrows in the webbed spaces of the fingers and toes, axilla, elbow and belt line are the classical distribution of scabies lesions. The treatment of choice for primary scabies infection is the application of topical scabicidal agents, with repeat application in 7 days. The treatment of choice is permethrin 5% lotion. Individuals affected by scabies should avoid skin-to-skin contact with others. Patients with typical scabies may return to school or work 24 hours after the first treatment.

A 19-year-old woman presents to the emergency department with severe flushing, tremors, and vision changes. Vital signs are BP 240/110 mm Hg, HR 104 bpm, RR 20/min, and T 100.4°F (38°C). She was recently found to have a mass on her kidney. Which of the following is the most appropriate course of treatment for this patient? Esmolol followed by phenoxybenzamine Labetalol followed by clonidine Phentolamine followed by labetalol Tamsulosin followed by hydralazine

Phentolamine followed by labetalol This patient has signs and symptoms consistent with pheochromocytoma. A pheochromocytoma is a catecholamine-secreting tumor. It is typically located in the adrenal glands as is seen in 90% of patients but may be extra-adrenal. Management of hypertensive emergency, as is seen in this patient with evidence of end-organ damage on fundoscopic exam, in patients with pheochromocytoma is a two-step process. You must obtain alpha-blockade prior to beta-blockade. This is classically done by administering phenoxybenzamine, which is an alpha-1 and alpha- 2 blocker, or phentolamine, a nonspecific alpha-blocker, followed by a beta blocker, such as labetalol or propranolol. Without appropriate alpha-blockade prior to beta-blockade, unopposed alpha activity can precipitate a malignant hypertensive crisis. Definitive management is by surgical resection of the tumor. Signs and symptoms of pheochromocytoma include episodic signs of excess catecholamine activity. This includes palpitations, sweating, tremor, weight loss, fever, and significant hypertension. Diagnosis is multimodal and includes plasma metanephrine levels and a 24-hour urine collection with levels of urine creatinine, catecholamines, vanillylmandelic acid, and urine metanephrines obtained.

A 57-year-old man presents for several weeks of fatigue, night sweats, and episodic chills. His exam is remarkable for splenomegaly. A complete blood count shows leukocytosis at 175,000 and mild thrombocytosis. The peripheral blood smear shows a left-shifted myeloid series; blasts are less than 5%. Which of the following additional findings is most consistent with your suspected diagnosis? Auer rods Elevated hematocrit Philadelphia chromosome Rouleaux formation

Philadelphia chromosome The patient has chronic myeloid leukemia (CML) which is confirmed by the presence of the Philadelphia chromosome, a reciprocal translocation between the long arms of chromosomes 9 and 22. CML is a myeloproliferative disorder that occurs most often in middle-aged patients, causing fatigue, night sweats, and low-grade fevers. The physical exam may show splenomegaly and sternal tenderness as marrow overexpansion progresses. In addition to presence of the Philadelphia chromosome, laboratory findings include a median white blood count of 150,000 and peripheral blood showing a left-shifted myeloid series. The red blood cell morphology is normal, and platelets may be normal or slightly elevated. The bone marrow is hypercellular with marrow cells showing less than 5% myeloblasts. The initial treatment of choice for chronic-phase CML is imatinib mesylate (Gleevec®) , a tyrosine kinase inhibitor that is able to normalize hematologic abnormalities and suppress the abnormal Philadelphia chromosome gene in 98 percent of cases. Untreated CML is an unstable disease that can progress to an accelerated blast crisis similar to acute leukemia. An allogeneic stem cell transplant may be needed if medical therapies cannot control the disease, or if the patient has accelerated to a blast crisis.

You have recently diagnosed a reduced ejection fraction in a 50-year-old man with congestive heart failure. He is currently stable and adherent to his medications. Which of the following vaccinations is indicated for this patient at this time to prevent further exacerbations? Hepatitis A Meningococcal Pneumococcal Tdap Varicella

Pneumococcal Congestive heart failure is a chronic, clinical syndrome characterized by a reduction in cardiac output and fluid retention. It can be caused by any condition that interferes with ventricular filling or ejection of blood. Management of patients with heart failure is complex and requires a multidisciplinary team to address the root cause, improve symptoms, and prevent further decompensation. As part of preventative care, either the 20-valent pneumococcal conjugate vaccine (PCV20) or 15-valent pneumococcal conjugate vaccine (PCV15) should be administered to patients between the ages of 19 and 64 who have been diagnosed with heart failure and who have never received pneumococcal vaccination in the past. If PCV15 is administered, a dose of 23-valent pneumococcal polysaccharide vaccine (PPSV23) should be administered, typically at least 1 year later (although a minimum interval of 8 weeks between vaccines can be considered in patients with immunocompromising conditions, cerebrospinal fluid leaks, or cochlear implants). Patients with heart failure should receive an annual influenza vaccination as well. There are many other nonpharmacologic interventions for patients with heart failure that will improve quality of life and decrease symptoms. First, patients should limit their dietary sodium intake to 2-3 grams per day. The reduction in cardiac output that occurs in heart failure results in the stimulation of the renin-angiotensin-aldosterone system, which leads to the abnormal retention of sodium and water (which causes the peripheral edema and pulmonary congestion seen in heart failure). Not only does limiting sodium consumption prevent exacerbations of heart failure, but it also may lower the necessary diuretic dose required to achieve euvolemia in these patients. For patients who have persistent fluid retention despite sodium restriction and use of diuretic medications, restricting fluid intake to less than 2 L daily may be appropriate. Patients with heart failure may experience unintentional weight loss, called cardiac cachexia. Patients with low body mass index are at a higher risk of mortality, therefore, it is imperative that the nutritional needs of these patients are addressed to prevent risk of infection and hospitalization. Depression is unfortunately common in patients with heart failure and should be managed with psychotherapy and selective serotonin reuptake inhibitors. All patients with heart failure should be counseled to stop smoking and limit alcohol consumption. Exercise testing should be conducted in all patients with heart failure to determine the level of function and to assist in implementing a personalized exercise program to maintain function. Polysomnography is recommended for patients who have sleep disturbances. Continuous positive airway pressure should be provided for all patients with comorbid sleep apnea. Supplemental oxygen is not recommended for patients with heart failure unless there is a comorbid pulmonary disorder. Patients with heart failure should refrain from taking nonsteroidal anti-inflammatory drugs due to the increased risk of renal failure, especially when taken with angiotensin-converting enzyme inhibitor therapy.

A 66-year-old man with a medical history significant for hypertension and chronic kidney disease presents for a routine examination. Laboratory tests are available and include an estimated glomerular filtration rate of 31 mL/min/1.73 m2, potassium of 4.1 mmol/L, magnesium of 2 mEq/L, calcium of 10 mg/dL, and parathyroid hormone level of 160 pg/mL. A physical exam is completed and is unremarkable. Vital signs are a BP of 132/82 mm Hg, HR of 87 bpm, SpO2 of 97% on room air, and T of 98.4°F. Decreasing the intestinal absorption of which of the following will benefit the patient the most? Calcium Magnesium Phosphate Potassium Sodium

Phosphate Chronic kidney disease leads to hyperphosphatemia through phosphate retention secondary to the reduced filtering of phosphate in the kidneys. Phosphate is inversely proportional to levels of circulating calcium and phosphate retention causes secondary hyperparathyroidism. The hypersecretion of parathyroid hormone (PTH) is typically able to maintain calcium homeostasis despite increased phosphate retention. Progressive chronic kidney disease leads to changes in bone structure, including osteitis fibrosa, osteomalacia, and adynamic bone disease. PTH levels should be monitored in patients with chronic kidney disease. When PTH levels begin to rise, dietary phosphate restrictions, oral phosphate binders, and the administration of calcitriol are used to suppress PTH secretion and prevent the development or worsening of osteitis fibrosa.

A 12-year-old girl presents to the office with anal itching that seems to be worse at night. She has no issues with constipation or any other changes in her bowel habits. On physical exam, you see some excoriations around the anus but no tear or palpable hemorrhoid. This first occurred a few days after returning home from a summer camp in northern Michigan, where she was in a cabin with 15 other girls for one month. Which of the following is the most likely diagnosis? Internal hemorrhoid Lyme disease Pinworms Scabies

Pinworms Pinworm is the most common helminthic parasite encountered by primary care providers in developed nations. It is acquired by ingesting parasite eggs, and most people remain asymptomatic after being colonized. Pinworm (E. vermicularis) is a quintessential intestinal parasite with no geographic constraints. It is transmissible by close contact with colonized persons. People have had pinworm for thousands of years, and before modern sanitation, colonization by pinworm probably was universal. E. vermicularis has a simple life cycle with a "hand to mouth" existence. The worm is acquired by ingesting parasite eggs. Most often these eggs are on the hands of the host. However, the small eggs also may become airborne, inhaled, and then swallowed. During the night, egg-laden females migrate out of the anal canal and onto the perianal skin. Each female deposits up to 17,000 eggs, which mature rapidly, becoming infective within 6 hours and causing extreme pruritus. Infestation typically causes perianal itching and then scratching gathers eggs onto the hands, promoting reinfection and transmission to others. Diagnosis is made by the cellophane tape test, and all members of the household should be treated. Pinworm infection is readily treated with a single 100-mg dose of mebendazole or a 400-mg dose of albendazole, both doses repeated in two weeks.

A 45-year-old man presents to the office complaining of a "racing heart", palpitations, increased sweating, and headaches for the past three months. He denies any new life stressors and history of anxiety or panic attacks. He is not currently on any medications. The patient reports his father experienced similar symptoms when he was around the same age and was ultimately diagnosed with a tumor on his adrenal gland. Patient's blood pressure in the office is 164/98 mm Hg and his pulse rate is 88 beats per minute with a regular rhythm. Physical examination reveals a diaphoretic, well developed man without focal neurological deficits. TSH and T3/T4 levels were all reported within normal limits. Which of the following diagnostic tests would be the most appropriate next step for this patient? 24-hour urine fractionated metanephrines and catecholamines CT scan Genetic testing Plasma fractionated metanephrines

Plasma fractionated metanephrines Plasma fractionated metanephrines would be the initial biochemical test performed for a patient who is considered high risk for a pheochromocytoma (family history, familial tumor syndrome, history of previously resected pheochromocytoma, or presence of adrenal mass found incidentally). Pheochromocytoma is a neuroendocrine catecholamine-secreting tumor typically found on the adrenal gland. These types of tumors are often associated with familial tumor disorders, such as multiple endocrine neoplasia type 2 (MEN2) and von Hippel-Lindau (VHL) syndrome. The triad associated with pheochromocytoma include palpitations or tachycardia, episodic headaches, and diaphoresis. Patients have a history of hypertension. The majority of tumors are located on the adrenal gland. Patients in whom a pheochromocytoma is suspected, who have a positive family history, or who have an adrenal mass found incidentally, should be tested for the disease. Prior to diagnostic studies, any possible offending drug should be discontinued. In patients who are low risk for a pheochromocytoma, 24-hour urine fractionated metanephrines and catecholamines should be the first biochemical test. After a positive biochemical test, patients should be evaluated via imaging (either CT scan or MRI) to locate the catecholamine-secreting tumor. Genetic testing may also be indicated if a familial disorder is suspected. Treatment includes surgical resection of the pheochromocytoma preceded by a preoperative α-adrenergic blockade using phenoxybenzamine and β-adrenergic blockade using a β-blocker such as propranolol to maintain cardiovascular and hemodynamic stability during surgery.

You are treating a 50-year-old coal-miner's hypertension with lisinopril. He has been complaining of 3-months of progressive dyspnea. You order a chest radiograph which shows bilateral upper lobe honeycombing. A high resolution computed tomogram shows multiple small, round opacities only in the upper lobes. The lower lobes appear normal. Which of the following is the most likely diagnosis? Goodpasture syndrome Granulomatosis with polyangiitis (GPA) Pneumoconiosis Scleroderma

Pneumoconiosis Pneumoconiosis is an occupational respiratory disease due to inhalation of inorganic dusts. It is common in miners. In 2010, it resulted in 125,000 deaths in the US. History is key in determining the probable agent. A common form is Coalworker's pneumoconiosis, also known as miner's lung or black lung, which is due to chronic inhalation of coal and carbon particles. Machinists, especially grinders, and pottery makers, can develop the silicon particle form called silicosis. Construction workers, especially boat builders, can suffer from asbestosis. Clinically, the diagnosis is made in a patient with dyspnea and the classic occupational exposures as above. Radiography may reveal small cystic radiolucencies described as honeycombing. High-resolution computed tomography offers better evaluation, and typically shows small, round opacities which denote inflammatory areas of dust-laden macrophages and fibrosis, as is quite common in the carbon deposits of coalworker's pneumoconiosis. Interestingly, the upper lobes are more affected in miner's lung and silicosis, while the lower lobes are more affected in asbestosis. Progressive pulmonary fibrosis, with resultant restrictive lung disease pathology and pulmonary function test results, typically occurs. Other than removing oneself from the offending agent and dusty environment, treatment is mainly supportive.

A 30-year-old man presents with asymmetric myalgias and arthralgias. He also complains of difficulty climbing stairs. You note fever, hip and shoulder muscle weakness and tender palpable purpura, without associated atrophy on examination. However, there is no facial or truncal rash. Laboratory testing reveals a low hematocrit, a high creatine kinase, a negative antinuclear antibody titer, and an elevated erythrocyte sedimentation rate. Which of the following is the most likely diagnosis? Polyarteritis nodosa Polymyalgia rheumatica Polymyositis Pseudogout

Polyarteritis nodosa Polyarteritis nodosa is a systemic vasculitis of small to medium vessels. It represents an autoimmune inflammatory disorder of unknown origin. It results in transmural fibrinoid necrosis. It typically affects younger males. Symptoms include myalgia and weakness, arthralgias, tender subcutaneous palpable nodules, abdominal pain with diarrhea or GI bleeding and glomerular ischemia with hypertension, renal failure, and hematuria. Compared to other vasculitides, polyarteritis nodosa does not affect the pulmonary vasculature, however it is accompanied by systemic inflammatory signs such as low grade fever, malaise, unintentional weight loss, and night sweats. Treatment includes prednisone and cyclophosphamide.

A 45-year-old man presents complaining of three days of fever and myalgias, as well as sores on both of his shins. Physical exam reveals tender, nodular lesions surrounded by a starburst pattern of erythema. What is his most likely diagnosis? Granulomatosis with polyangiitis Henoch-Schoenlein purpura Nodular vasculitis Polyarteritis nodosa

Polyarteritis nodosa Polyarteritis nodosa is an autoimmune disease that causes necrotizing vasculitis of small and medium-sized muscular arteries as well as of renal and visceral arteries. It often presents with cutaneous manifestations as well as constitutional symptoms such as fever, asthma, and myalgias. If left untreated it progresses from cutaneous involvement to cause cardiovascular complications, bowel infarction and perforation, as well as renal failure. These complications lead to a high morbidity and mortality if systemic steroids are not used to delay the progression of the disease. Diagnosis is made via a deep wedge biopsy of suspicious nodules on the skin. The nodules associated with polyarteritis nodosa follow the course of the involved arteries, forming painful violaceous plaques that are surrounded by livedo reticularis. This is often described as "starburst" livedo, and is pathognomonic for the disease. This, in addition to the classic location of the nodules on the lower legs, makes polyarteritis nodosa the most likely diagnosis for this patient. Nodular vasculitis (C) is form of panniculitis that leads to lipocyte injury, necrosis, inflammation, and granulation. It often presents as bluish nodules on the posterior aspect of the lower legs but is not accompanied by constitutional symptoms or a starburst pattern. Henoch-Schonleinpurpura (B) is a hypersensitivity vasculitis that often affects children following an upper respiratory infection with group A streptococci. Like polyarteritis nodosa, it is also accompanied by renal involvement, but it's cutaneous symptoms are more often described as palpable purpura making it an unlikely diagnosis in this patient. Granulomatosis with polyangitis (A) is a systemic vasculitis causing palpable purpura as well as involvement of the upper airways, lungs, and kidneys; all of which are not seen in this patient.

35-year-old man presents to the Emergency Department with gross hematuria and worsening right flank pain over the past couple of days. The patient has a history of multiple urinary tract infections and hypertension. In the ED, the patient is found to be hypertensive but afebrile. Urinalysis shows for 2+ protein, WBC 3/hpf, RBC > 100/hpf. A kidney ultrasound is seen above. What is most likely the diagnosis? Polycystic kidney disease Renal abscesses Renal cell carcinoma Simple renal cysts with urinary tract infection

Polycystic kidney disease Polycystic kidney disease (PKD) is among the most common hereditary diseases in the USA. Fifty percent of patients will have end-stage renal disease (ESRD) by age 60 years. Individuals between 30-59 years of age with two or more cysts, seen on ultrasonography or CT scan, in each kidney are highly likely to have polycystic kidney disease. Genetic testing confirms the diagnosis of PKD. Common clinical findings of polycystic kidney disease include abdominal and flank pain, history of hematuria due to ruptured cyst, renal infection, nephrolithiasis and hypertension. The condition is also associated with a higher risk for cerebral aneurysm, mitral valve prolapse, aortic aneurysms, aortic valve abnormalities, and colonic diverticula. Echocardiogram may be completed to evaluate for valvular abnormalities. Cerebral angiogram not recommended unless the patient has family history of aneurysm. While there is no cure for PKD, aggressive hypertension management and low-protein diet is beneficial to prolong time to ESRD. Palliative measures may be taken to relieve pain by cyst decompression and antibiotic treatment for renal infections. Ultimately, these individuals will need to be placed on dialysis or undergo kidney transplantation.

A 2-month-old boy is sent to the emergency department by his pediatrician for a cough and an abnormal CBC with lymphocytosis. He is up-to-date with immunizations. His older sibling, who is 4 years old, is not up-to-date since the pediatrician suspended his immunizations due to a developing neurologic condition. The sibling has also had a febrile illness and has been coughing for more than 3 weeks. You observe the 2-month-old coughing and see a period of perioral cyanosis. What method of confirmatory testing has the best combined sensitivity and specificity for the diagnosis? Blood cultures Direct fluorescent antibody on nasal swabs Polymerase chain reaction of nasopharyngeal secretions Sputum cultures

Polymerase chain reaction of nasopharyngeal secretions The patient has pertussis. Pertussis is a highly contagious respiratory infection caused by Bordetella pertussis. Pertussis is commonly called whooping cough, secondary to the sound that is made on inspiration immediately following a paroxysm of coughing. There has been a recent resurgence of pertussis in the United States. This resulted in a revision of the booster schedule to require patients older than 10 years to obtain Tdap rather than tetanus (Td) alone. Transmission is via respiratory droplets and has an incubation of seven to 10 days before symptoms begin. Pertussis is a three-phase illness with each phase lasting approximately one week to multiple months. First, there is the catarrhal phase with symptoms similar to an upper respiratory infection. Second, is the paroxysmal phase. This includes harsh coughing paroxysms lasting minutes. Finally, the third phase is the convalescent phase. This last phase demonstrates clinical recovery but, unfortunately, the patient remains infectious. Symptoms include persistent cough, posttussive emesis, apnea, cyanosis, fever, and inspiratory whoop. Lab work may reveal an absolute lymphocytosis. The most sensitive and specific testing is via polymerase chain reaction (PCR) of nasopharyngeal secretions.

Which of the following laboratory findings is most commonly associated with scleroderma? Elevated erythrocyte sedimentation rate Positive anti-Smith antibodies Positive antinuclear antibodies Positive citrullinated peptide antibodies

Positive antinuclear antibodies A patient with scleroderma would most likely have positive antinuclear antibodies. Scleroderma is an uncommon, chronic disorder characterized by widespread vascular dysfunction and progressive fibrosis of the skin and internal organs. Scleroderma is more common in women than men. Scleroderma can be divided into localized scleroderma and diffuse scleroderma, or systemic sclerosis. Limited scleroderma is the more common variant. The clinical and pathologic manifestations seen in scleroderma are the result of severe fibroproliferative vascular lesions of small arteries and arterioles and excessive deposition of collagen. Patients with scleroderma typically present with swollen, puffy fingers, skin thickening and tightening, Raynaud phenomenon, or perioral skin tightening. Other complaints due to specific organ involvement may include gastroesophageal reflux, dyspnea, persistent dry cough, arthralgia, hypertension, chronic renal insufficiency, sicca syndrome, and paresthesias. On physical exam, the affected skin appears tight, shiny, and hairless. Telangiectasias, calcinosis, flexion contractures, and dry rales may also be noted. Laboratory studies should include a complete blood count, serum creatinine, creatinine kinase, urinalysis, antinuclear antibody (ANA) test, and other serologic tests. ANA test is positive in the majority of patients. Anti-topoisomerase antibody (Anti-Scl-70) and anti-RNA-polymerase III are highly specific. Pulmonary function testing should be performed in patients with systemic sclerosis. Treatment is based on managing complications and symptomatic relief. Raynaud phenomenon is commonly treated with calcium channel blockers. Esophageal reflux can be managed with proton-pump inhibitors. Renal dysfunction and hypertension are can be managed with angiotensin-converting enzyme (ACE) inhibitors. Congestive heart failure and chronic kidney disease are common causes of mortality.

Which finding on synovial fluid analysis is most consistent with the diagnosis of pseudogout? Elevated synovial leukocyte count Multiple gram-negative cocci Negatively birefringent urate crystals Positively birefringent calcium pyrophosphate dihydrate crystals

Positively birefringent calcium pyrophosphate dihydrate crystals Calcium pyrophosphate crystal deposition disease, also referred to as pseudogout, is a common crystal-induced arthropathy that generally affects the large joints. Pseudogout has a similar clinical presentation to gout, but the etiology is different. Pseudogout may be idiopathic, especially in the elderly. It may also be caused by trauma, hyperparathyroidism, hemochromatosis, and medications that cause hypomagnesemia such as loop diuretics or proton pump inhibitors used in peptic ulcer disease. Patients present with acute onset of severe pain, inflammation and edema in the knees, ankles, elbows or wrists. Pseudogout is generally monoarticular, but may present in multiple joints as well. Diagnosis is by synovial fluid analysis, so arthrocentesis is necessary for patients with monoarticular arthritis. Synovial fluid analysis will show positively birefringent calcium pyrophosphate dihydrate crystals, which confirms the diagnosis. Initial treatment for pseudogout is with nonsteroidal anti-inflammatory drugs (NSAIDs) or colchicine.

Which of the following can be used to distinguish a seizure from a syncopal episode? Loss of consciousness Loss of urinary continence Post-ictal period Tongue biting

Post-ictal period Syncope and seizure are often confused and difficult to differentiate. However, patients with syncope should not experience a post-ictal period or prolonged state of confusion. Syncope is defined as a sudden, transient loss of consciousness along with a loss of postural tone. The majority of cases are benign but determining which cases are potentially life-threatening or are harbingers of bad outcomes is difficult. All syncopal episodes result from the same pathophysiology; dysfunction of both cerebral hemispheres or dysfunction of the reticular activating system in the brainstem. After a syncopal episode, patients may have brief (seconds) episodes of confusion but will not experience a true post-ictal period.

A 12-year-old boy presents with three days of fever, malaise, and sore throat. Which favors a diagnosis of infectious mononucleosis over streptococcal pharyngitis? Fever Pharyngeal exudates Posterior cervical adenopathy Tender bilateral anterior cervical adenopathy

Posterior cervical adenopathy Mononucleosis typically occurs secondary to infection with Epstein-Barr virus. Streptococcal pharyngitis occurs secondary to infection with Group A Streptococcus. The clinical presentations of mononucleosis and streptococcal pharyngitis share significant overlap. Both cause fever, tonsillar enlargement and exudates, cervical lymphadenopathy, and fatigue. Certain features may help to differentiate between the two entities. For instance, mononucleosis classically causes symmetric posterior cervical adenopathy. The lymphadenopathy may be tender. Lymphadenopathy may also may occur in other areas of highly concentrated lymph nodes such as the axillary or inguinal region, or it may be generalized throughout the body. On the other hand, streptococcal pharyngitis classically causes bilateral tender anterior cervical adenopathy and does not cause generalized lymphadenopathy. In addition, children and adolescents with mononucleosis may develop splenomegaly. Splenomegaly is not a feature of streptococcal pharyngitis. Of note, patients with Epstein-Barr viral infections may develop a characteristic maculopapular rash if they receive amoxicillin for presumed streptococcal pharyngitis.

A 28-year-old man presents to the emergency department with lethargy and vomiting. He has a history of type I diabetes mellitus. Vital signs include heart rate 112 bpm and blood pressure 110/80 mm Hg. Laboratory analysis reveals sodium 135 mEq/L, potassium 4.0 mEq/L, chloride 100 mEq/L, bicarbonate 10 mEq/L, creatinine 1.4 mg/dL, glucose 558 mg/dL, pH 7.2. Normal saline and insulin have been ordered. Which of the following should be administered next? 5% dextrose with 0.45% saline Magnesium Potassium chloride Sodium bicarbonate

Potassium chloride The diagnosis of diabetic ketoacidosis can be established with the presence of hyperglycemia, ketosis and acidemia. The goals of treatment are aimed at insulin therapy, fluid resuscitation and electrolyte replacement. The exact amount of insulin administered varies, however many start with a bolus of 0.1 U/kg or 10 U of regular insulin IV followed by a maintenance of 0.1 U/kg/hr regular insulin IV. Fluid resuscitation should start with a one to two liter bolus of normal saline in adults or a 20 mL/kg bolus in a child. Potassium levels are often initially high or normal due to severe acidemia. The potassium levels often decrease significantly as the acidemia is corrected along with the administration of insulin. Potassium should be administered with intravenous fluids when the potassium is less than or equal to 5.0 mEq/mL. In patients where the initial potassium is low (< 3.3 mEq/L), levels could become life-threatening following the administration of insulin, therefore potassium should be vigorously repleted in concentrations of 20 to 40 mEq/L prior to initiating insulin therapy.

A 58-year-old woman presents with progressively worsening muscle weakness. Physical exam reveals weakness of the bilateral hip flexors. Creatine kinase and aldolase are elevated. A muscle biopsy shows primary inflammation with the CD8/MHC-I complex and no vacuoles. Which of the following is the best initial treatment? Methotrexate Plasmapheresis Prednisone Tacrolimus

Prednisone Prednisone is first-line treatment for polymyositis (PM); an autoimmune myopathy that typically affects individuals over the age of 45. Proximal weakness of the hip flexors and the deltoids bilaterally is typically the presenting symptom, and patients will commonly complain of progressively worsening weakness in the proximal muscles over a period of weeks or months. Lab tests will reveal an elevated aldolase and creatine kinase. Around 30 percent of patients will have Anti-Jo-1 autoantibodies. A small percentage of patients may also have a positive rheumatoid factor. The mainstay of therapy is reducing inflammation through immunosuppression and increasing quality of life through physical therapy and exercise. First-line therapy usually consists of prednisone.

A 6-year-old boy presents to the Emergency Department with periorbital swelling. His mother notes that for the past three days the periorbital swelling would appear in the morning and gradually decrease throughout the day. The boy also presents with loss of appetite, abdominal pain, and loose bowel movements. The physical examination reveals an fussy child with blood pressure of 100/70 mmHg and temperature of 37oC. He has periorbital edema, anicteric sclerae, pink palpebral conjunctiva, clear breath sounds, a nontender abdomen, no bipedal edema, and capillary return less than 2 seconds. Urinalysis is positive for 3+ proteinuria and red blood cell count of 10 cells/hpf. Serum cholesterol and triglyceride levels are elevated with serum albumin less than 2.5 g/dL. What is the initial drug of choice for the most likely diagnosis? Cyclophosphamide Cyclosporine Levamisole Prednisone

Prednisone The child presents with manifestations of nephrotic syndrome, the most common type of which is minimal change nephrotic syndrome. Other types include mesangial proliferation, focal segmental glomerulosclerosis, membranous nephropathy, and membranoproliferative glomerulonephritis. It is more common in boys than in girls and occurs between 2 to 6 years of age. Children commonly present with periorbital edema which is worse in the morning and gradually decreases throughout the day. Hypertension and gross hematuria, while more common in glomerulonephritis, rarely occur in idiopathic nephrotic syndrome. Laboratory results are distinctive and include 3+ or 4+ proteinuria and microscopic hematuria seen on the urinalysis. A spot urine protein:creatinine ratio exceeds 2, and urinary protein excretion exceeds 50 mg/kg/day. There is also hypoalbuminemia with serum albumin level of less than 3 g/dL and hyperlipidemia with elevated serum cholesterol and triglycerides. Prednisone is given at a dose of 60 mg/m2/day in a single daily dose for 4 to 6 consecutive weeks and is gradually tapered for the next two months. Eighty to ninety percent of children with minimal change nephrotic syndrome respond well to steroid therapy within the first five weeks of treatment.

A 12-year-old boy presents with facial weakness for 2 hours. He states he woke up this morning and noticed the right side of his face did not move the right way. Examination reveals a right facial droop of the upper and lower face. He is unable to fully close his right eye. The examination is otherwise unremarkable. What management should be pursued? CT scan of the head Intravenous thrombolytics Lumbar puncture Prednisone

Prednisone This patient presents with a peripheral facial nerve (CN VII) palsy or Bell palsy, which should be managed with oral steroids (prednisone). Bell palsy is an idiopathic, peripheral facial nerve palsy. The facial nerve innervates the muscles of the scalp, external ear, and muscles of facial expression. Additionally, the sensory portion supplies the anterior two-thirds of the tongue. Patients will typically present with complaints of weakness of the facial muscles, including the mouth, eyelid, and forehead. One of the critical issues is to ensure the paralysis is, in fact, peripheral and not central. In central facial nerve palsy, the muscles of the upper part of the face (forehead in particular) are spared. This is due to the fact that the upper third of the face has dual innervation centrally from bilateral cerebral cortices. Thus, in a patient who has sparing of the upper part of the face, further diagnostic workup should be performed to determine the cause of central facial nerve paralysis. Peripheral facial nerve palsy is treated with prednisone 1 mg/kg/day for 7-10 days. The role of antivirals (e.g., acyclovir) is controversial, as it is unclear if herpes simplex virus 1 is a causative organism, but may be included in the treatment of severe to complete facial weakness. Additionally, artificial tears and taping the affected eye closed at night are recommended for those with inadequate eye closure to prevent drying out of the cornea and subsequent abrasions or ulcerations.

A 56-year-old woman with a history of diabetes mellitus type 2 presents to your office with a complaint of bilateral foot pain. She describes the pain as a burning sensation that occurs with rest and improves with activity. Her most recent hemoglobin A1C was 11.3%. Which of the following is the most appropriate initial therapy? Lamotrigine Oxcarbazepine Oxycodone Pregabalin

Pregabalin Peripheral neuropathy is a major cause of morbidity in patients with diabetes mellitus. Risk factors for the development of diabetic neuropathy include duration and severity of hyperglycemia, hyperlipidemia, smoking and hypertension. Diabetic neuropathy is a symmetrical sensory polyneuropathy that generally affects the distal lower extremities first. Clinical manifestations of diabetic neuropathy include pain in the feet described as burning or tingling that occurs at rest and improves with activity. Symptoms are sometimes preceded by a recent change in glycemic control. There are three aspects to the treatment of diabetic neuropathy: glycemic control, foot care and pain management. The American Diabetes Association recommends a step-wise management strategy in patients with diabetic neuropathy that begins with excluding other neurologic etiologies and stabilizing glycemic control. Pharmacologic treatment options include anticonvulsants such as pregabalin, the dual serotonin and norepinephrine reuptake inhibitor duloxetine, or tricyclic antidepressants such as amitriptyline.

A 40-year-old man presents to the emergency department reporting sudden onset of intense right flank and right lower abdominal pain that comes and goes in waves and is now radiating to his right testicle. The patient also reports nausea and vomiting but has no fever or dysuria. His medical history is positive for a cholecystectomy 3 years ago and hypertension treated with lisinopril daily. Vital signs are a HR of 100 bpm, BP of 150/90 mm Hg, T of 98.8°F, and RR of 16/min. Physical exam reveals no costovertebral angle tenderness, a soft abdomen with no rebound tenderness, and normal genitalia. Intravenous fluids and intramuscular ketorolac are administered and result in improvement of symptoms. Results of additional studies are shown below: Urinalysis: Leukocytes: negative Nitrites: negative Urobilinogen: negative Protein: 0 mg/dL pH: 6.0 Blood: negative Specific gravity: 1.010 Ketones: negative Bilirubin: negative Glucose: negative Serum creatinine: 0.80 mg/dL Computed tomography of abdomen/pelvis: 6 mm right mid-ureteral stone with no hydronephrosis What is the best next step in management of this patient's condition? Admit and consult urology for stone removal Admit and continue intravenous fluid administration until stone passage Encourage caffeine consumption to stimulate urination and discharge home Prescribe oral ciprofloxacin 500 mg twice daily for 10 days and discharge home Prescribe oral tamsulosin 0.4 mg once daily and discharge home

Prescribe oral tamsulosin 0.4 mg once daily and discharge home The patient in the above vignette has signs and symptoms of a ureteral stone (calculus). Calculi in the urinary tract often form in the kidney and descend down the ureter to exit through the urethra. As the stones pass the ureteropelvic junction and the ureterovesical junction, they may lodge and produce pain, although stones may lodge in any portion of the urinary tract. The pain of urinary tract stones is colicky, coming in waves that may last 30 minutes to an hour and may be so intense that they induce nausea and vomiting. A stone lodged in the kidney may produce costovertebral angle tenderness, whereas a stone lodged in the mid-ureter or lower tract may produce abdominal pain that radiates to the groin. Fever is uncommon in patients with uncomplicated urinary tract stones. Urinalysis may be normal or may show blood. Complete blood count and complete metabolic panel are generally normal, although a stone causing hydronephrosis may lead to increased creatinine levels. Definitive diagnosis of urinary tract calculi is with computed tomography of the abdomen and pelvis. Treatment of urinary tract calculi depends on the size of the calculus and the presence or absence of hydronephrosis. If no hydronephrosis is noted and the stone is smaller than 10 mm, conservative outpatient management with analgesics is appropriate. Alpha-blocking agents, such as tamsulosin, increase the rate of expulsion of ureteral stones and should be prescribed for patients with ureteral calculi that measure > 5 mm and < 10 mm. Prevention of further formation of urinary stones includes increased water intake, decreased carbonated beverage intake, eating a balanced diet, and supplementation with citrate.

A 75-year-old woman was hospitalized 2 days ago after experiencing an ST segment elevation myocardial infarction. She had no prior cardiac history. She underwent percutaneous coronary intervention and had antifibrinolytic therapy started in appropriate time frames. A heart echocardiogram showed an ejection fraction of 35% after the infarct. Several medications may be used in the postmyocardial infarction treatment for this patient. What is the survival benefit of angiotensin-converting enzyme inhibitors in patients who have suffered reduced ejection fraction? Decreases preload Improves myocardial oxygen supply-demand Interferes with platelet activation Prevents postmyocardial infarction cardiac dysrhythmias Prevents ventricular remodeling

Prevents ventricular remodeling After a patient undergoes an ST segment elevation myocardial infarction (STEMI), the left ventricle undergoes several changes known as ventricular remodeling. The infarct spreads and causes disruption of myocardial cells and tissue loss leading to progressive dilation of the left ventricle. Progressive dilation can lead to hemodynamic instability, heart failure, and poor prognosis. Angiotensin-converting enzyme (ACE) inhibitors are shown to reduce mortality in patients after a STEMI by preventing ventricular remodeling. In addition, they have been shown to increase the left ventricular ejection fraction at 1 month and 1 year. All patients who experience a STEMI with known left ventricle dysfunction, are older, or who have prior STEMI should be started on an ACE inhibitor. These medications have not shown much benefit in patients with normal left ventricular function.

Which of the following is an absolute contraindication to the diphtheria vaccine? Family history of sudden infant death syndrome Moderate or severe illness Previous anaphylaxis to DTaP Previous seizure within three days of last DTaP dose

Previous anaphylaxis to DTaP The routine childhood immunization schedule in the United States includes vaccination against diphtheria, tetanus and pertussis. There has been a dramatic decrease in the incidence of diphtheria since the initiation of the vaccine in the 1940's. Corynebacterium diphtheriae causes acute cutaneous or respiratory illness and can be fatal. Mortality is higher in children under the age of five. The recommended schedule is five doses of the diphtheria, tetanus and pertussis (DTaP) vaccine at ages 2 months, 4 months, 6 months, 15-18 months and 4-6 years. The tetanus toxoid, reduced diphtheria toxoid, and acellular pertussis vaccine (TDaP) is used as a single booster dose for adolescents or adults. Absolute contraindications to the DTaP vaccine include previous anaphylaxis to DTaP, anaphylactic reactions to latex, and progressive neurologic disorder and encephalopathy within seven days of a previous dose of DTaP without identifiable cause. A family history of sudden infant death syndrome (A) is not an absolute contraindication to administration of DTaP. A number of factors merit precautions in administration of the DTaP vaccine, but are not contraindications. These include moderate or severe illness (B), previous seizure within three days of last dose (D), a temperature of greater than 104°F within 48 hours of previous dose, and a history of Guillain-Barré syndrome.

Which of the following is an independent cause of secondary hypertension? Addison's disease Atrial fibrillation Hypercholesterolemia Primary aldosteronism

Primary aldosteronism Primary aldosteronism is a potential cause of secondary hypertension. Hypertension can be divided into essential or secondary hypertension. Approximately 95% of patients with elevated blood pressure have essential hypertension. Secondary cause of elevated blood pressure should be suspected in patients with severe or resistant hypertension, in patients younger than 30 years without risk factors for hypertension, in patients with malignant hypertension, or hypertension onset before the age of puberty. Potential causes of secondary hypertension include renovascular disease, primary kidney disease, primary aldosteronism, obstructive sleep apnea, long-term corticosteroid use, coarctation of the aorta, thyroid disease, drugs, or pheochromocytoma. Renovascular disease is the most common potentially correctable cause of secondary hypertension. Signs, symptoms, and laboratory findings are dependent upon the etiology. Patients with secondary hypertension due to renovascular disease may present with an abdominal bruit or decreased kidney function after initiating antihypertensive therapy. Primary aldosteronism typically causes hypokalemia, mild hypernatremia, or drug-resistant hypertension. Patients with sleep apnea are usually obese and may complain of daytime somnolence, fatigue, headache, or depression. Drug-induced hypertension can be caused by oral contraceptives, decongestants, nonsteroidal anti-inflammatory drugs (NSAIDs), or cocaine. The treatment of secondary hypertension is based upon the underlying etiology. Complications of untreated hypertension include heart failure, cerebrovascular disease, renal insufficiency, and aortic dissection.

A 42-year-old woman with no significant past medical history presents to the family practice office for evaluation of new onset thrombocytopenia. She denies any recent illness and does not take any medications or supplements. She denies the development of skin lesions, neurologic or constitutional symptoms. CBC with differential, peripheral smear, BMP, PT/INR and aPTT are otherwise within normal limits. What is the most likely diagnosis? Disseminated intravascular coagulation Hemolytic uremic syndrome Primary immune thrombocytopenia Thrombotic thrombocytopenic purpura

Primary immune thrombocytopenia Primary immune thrombocytopenia, also known as idiopathic thrombocytopenic purpura (ITP), is an acquired cause of isolated thrombocytopenia caused by the development of antibodies directed at platelets. It is a diagnosis of exclusion and can present in a range of severity from an asymptomatic incidental lab abnormality to acute onset of petechial rash, wet and dry purpura or retinal hemorrhages. ITP rarely presents with symptoms other than mucocutaneous bleeding of varying severity. Lab testing is focused on excluding alternative causes of thrombocytopenia. Typically, thrombocytopenia is the only abnormality detected. Coagulation testing with PT/INR and aPTT are normal. Bone marrow evaluation is reserved for those over the age of 60 who present with idiopathic thrombocytopenic purpura as they are more likely to have thrombocytopenia occurring as a result of a myelodysplastic syndrome than someone under the age of 60. Management of ITP is focused on maintaining a safe platelet count rather than a normal platelet count. Platelet counts greater than > 50,000 are acceptable. Glucocorticoids, IVIG and immunosuppressive therapy is used to help patients achieve and maintain safe platelet counts.

A 6-month-old boy presents to the emergency department with fever and poor feeding for the past day. The patient's parent reports he has had several episodes of vomiting and diarrhea. The patient is up-to-date on vaccinations. Vital signs include a HR of 200 bpm, BP of 65/38 mm Hg, RR of 40/min, oxygen saturation of 95% on room air, and T of 103.2°F. Physical examination reveals a listless infant with dry mucous membranes and a nontender abdomen. Laboratory studies include the following: White blood cell count: 25,000 cells/microL Urinalysis: negative for nitrites and leukocyte esterase Cerebrospinal fluid white blood cell count: 1,500 cells/microL Cerebrospinal fluid glucose: 35 mg/dL Cerebrospinal fluid protein: 250 mg/dL Cerebrospinal fluid Gram stain: gram-negative cocci in pairs Which of the following is the most appropriate care for the patient's close household contacts? Observation only Prophylactic azithromycin Prophylactic penicillin Prophylactic rifampin Prophylactic vaccination

Prophylactic rifampin Meningococcal disease, which is caused by Neisseria meningitidis, is transmitted by exchanging respiratory and throat secretions during close, lengthy contact. It is one of the most common causes of bacterial meningitis in all age groups except neonates. The methods used to prevent meningococcal infection include antimicrobial chemoprophylaxis, use of droplet precautions, vaccination prior to exposure, and avoidance of exposure. Antimicrobial chemoprophylaxis is recommended for individuals who have had close contact with someone diagnosed with meningococcal meningitis. Close contact is defined as more than 8 hours of contact while in close proximity (< 3 feet) or direct exposure to the patient's oral secretions during the 7 days before the onset of the patient's symptoms. Common examples of close contacts include household members, roommates, intimate contacts, young adults exposed in dormitories, and travelers who were seated directly next to the patient on a flight lasting more than 8 hours. Chemoprophylaxis is not indicated for most health care workers who care for a patient with meningococcal meningitis because of the brief exposures. Exceptions include more direct exposures to respiratory secretions, such as health care workers involved in intubation or suctioning. Antimicrobial chemoprophylaxis should be administered as early as possible after the index patient has been identified. Ideally, this is within 24 hours, and chemoprophylaxis is not recommended if more than 14 days have passed since the exposure. The preferred chemoprophylactic regimens for protection against meningococcal disease include rifampin, ciprofloxacin, or ceftriaxone. Rifampin is not recommended for pregnant patients because it has teratogenic effects in laboratory animals. Ciprofloxacin should not be used in pregnant patients or in situations in which fluoroquinolone-resistant strains of Neisseria meningitidis have been identified. Due to concerns for resistance, the Centers for Disease Control and Prevention recommends considering antimicrobial susceptibility testing when deciding which chemoprophylactic regimen to use.

A previously healthy, 60-year-old man presents to your office with a complaint of hand tremor. He tells you that the tremor started in his right hand and over the past few months began affecting both hands. He denies any other symptoms. Which of the following is the most appropriate therapy? Clonidine Lorazepam Nifedipine Propranolol

Propranolol Tremor is defined as a rhythmic, wavelike movement of a part of the body that occurs consistently and varies in severity. It is caused by muscle contractions and is the most common movement disorder. The most common type of action or postural tremor is an essential tremor. The cause of essential tremor is unknown and there are no pathological findings associated with the condition; however, there is a familial connection. Essential tremor affects men and women equally, however men often have more symptoms with the hands and women have more symptoms with the head. Patients generally present with a complaint of tremor as the only symptom. It usually starts in one upper extremity before progressing to involvement of both upper extremities. The tremor symptom is often intermittent at first, then becomes more persistent. In most cases the tremor will be exacerbated by voluntary activity like drinking from a glass. There are no specific laboratory or diagnostic markers for essential tremor. If the personal and family history and physical exam point to essential tremor, no additional workup is needed. The most common treatment and first-line therapy for essential tremor is with the beta-blocker propranolol. Primidone, an anticonvulsant agent, is also indicated for use in the treatment of essential tremor.

An obese 34-year-old woman is brought to the emergency department with respiratory distress. Two months ago, she was in the hospital for knee surgery. Paramedics report an acute onset of dyspnea and pleuritic chest pain. She also complains of a tender thigh on the same side of her knee surgery. She is tachycardic and tachypneic, and mildly hypotensive. Examination reveals decreased breath sounds but no hyperresonance. An emergent chest radiograph is relatively normal except for some mild atelectasis. Which of the following is the most likely diagnosis? Acute bronchitis Pleural effusion Pneumothorax Pulmonary embolism

Pulmonary embolism Pulmonary embolism refers to the obstruction of a pulmonary artery by thrombus, tumor, air, or fat that originated elsewhere in the body, mostly from the deep veins of the lower extremities. It is classified as acute or chronic, and massive or submassive. It is often a fatal disease, leading to a mortality rate of 30% without treatment. Risk factors include immobilization, surgery, or central venous instrumentation within the last three months, stroke/paresis/paralysis, cancer, chronic cardiac disease, autoimmune disease, obesity, >1 pack per day tobacco use, hypertension, and a history of deep vein thrombosis. Most patients experience dyspnea with or without wheezing, cough, pleuritic chest pain, orthopnea, lower extremity pain or swelling, tachypnea, tachycardia, jugular venous distension, decreased breath sounds, and an accentuated pulmonic component of S2. These signs and symptoms are variable and nonspecific. Furthermore, up to 32% of patients present asymptomatically. As such, the diagnosis can be difficult. The mainstay of treatment is anticoagulation.

A 33-year-old woman with no significant medical history presents to the primary care clinic for jaw weakness while chewing. She states that, for the past 6 months, she has noticed her jaw muscles feel weak after prolonged chewing. Recently, she has also noticed occasional drooping of her eyelids that varies throughout the day and can affect both eyes. The patient does not take any medications on a regular basis. Her heart rate is 82 beats/minute, blood pressure is 124/76 mm Hg, respiratory rate is 14 breaths/minute, oxygen saturation is 100%, and temperature is 97.1°F. Her physical exam reveals pupils that are equal, round, and reactive to light. You note mild ptosis of her left eyelid, and she notes subjective weakness of her jaw with repeated chewing on exam. You perform an ice pack test of her left eye, which is positive. Findings on the rest of her neurologic exam are normal. You obtain serologic testing for acetylcholine receptor antibodies and muscle-specific tyrosine kinase antibodies, which are positive. This confirms your suspected diagnosis. Which of the following is the best next step in management based on this information? Ciprofloxacin 500 mg PO BID for 7 days Intravenous immune globulin Plasmapheresis Prednisone 60 mg PO daily for 14 days Pyridostigmine 180 mg PO BID

Pyridostigmine 180 mg PO BID This patient is presenting with transient, progressive muscle weakness of the ocular and bulbar muscles, which is consistent with myasthenia gravis, so pyridostigmine daily would be the most appropriate first-line therapy. She describes muscle fatigability, or muscle weakness, that worsens with prolonged muscle use. Fluctuating muscle weakness and fatigability are characteristic of myasthenia gravis, which is an autoimmune neuromuscular disorder that affects the postsynaptic neuromuscular junction. Myasthenia gravis can be limited to the ocular muscle group or can be generalized if there is involvement of the limbs, bulbar, or respiratory muscles. Myasthenia gravis presents in a bimodal distribution between the ages of 10 and 30 more commonly in the female population and between the ages of 50 and 70 more commonly in the male population. Patients often report fluctuating muscle weakness and fatigability that is worsened later in the day. The fluctuating nature of muscle weakness in myasthenia gravis helps to differentiate it from other neuromuscular conditions, such as amyotrophic lateral sclerosis or Lambert-Eaton syndrome. The most common initial presentation is ocular muscle weakness with diplopia, ptosis, or both. Bulbar muscle weakness can present as jaw fatigability, dysphagia, or voice changes. Myasthenia gravis can also affect the respiratory muscles, which can cause a severe, life-threatening respiratory insufficiency known as myasthenic crisis. The diagnosis of myasthenia gravis can be made clinically based on the patient's history and physical exam. One test that can support the diagnosis is the ice pack test. In this test, an ice pack is placed over the affected extraocular muscles. Neuromuscular transmission increases at colder temperatures, so improvement of ptosis after ice application supports the diagnosis of myasthenia gravis. Serologic testing for antibodies to the acetylcholine receptor and muscle-specific tyrosine kinase, in addition to other autoantibodies, can be used to confirm a diagnosis of myasthenia gravis. Electromyography can also be used for diagnostic confirmation. Acetylcholinesterase inhibitors such as pyridostigmine are the best initial treatment for myasthenia gravis. This class of medications delays the breakdown of acetylcholine, thereby increasing the amount of acetylcholine at the neuromuscular junction. The onset of action of pyridostigmine is within 15 minutes, and the time to maximal benefit is 2 hours after administration. Dosing is variable and must be tailored to each individual patient's needs. Pyridostigmine can cause cholinergic side effects, including abdominal pain, diarrhea, increased salivation, and diaphoresis. Plasma exchange and Intravenous immune globulin (IVIG) can be given for patients with acute exacerbations and myasthenic crises.

A 20-year-old woman presents with weakness in her left wrist. She states that she fell asleep in a chair after a night of heavy drinking. On physical examination, she is unable to extend her wrist. What nerve is compromised? Axillary nerve Median nerve Radial nerve Ulnar nerve

Radial nerve This patient presents with a wrist drop caused by radial neuropathy also called "Saturday night palsy." The radial nerve arises from the C5-T1 roots. It controls extension of the fingers, thumb, wrist, and elbow. Symptoms of radial neuropathy depend on the location of compression. Compression in the axilla (typically from improper use of crutches) causes weakness of extension at the elbow, wrist, and fingers. More typically, the nerve is compressed between the humeral shaft and another hard surface and results in weakness with extension at the wrist and fingers. This typically occurs from deep sleep (often secondary to inebriation). About 90% of radial nerve mononeuropathies that occur during sleep, coma, or anesthesia recover within 6-8 weeks. Patients should be placed in a wrist splint with 60 degrees of dorsiflexion to prevent atrophy and contractures.

A 29-year-old woman with no significant medical history presents to her primary care office to further discuss a chest X-ray that was done during a recent emergency department visit. The patient reports she was seen in the emergency department 2 weeks ago for cold-like symptoms, which have since resolved. At this time, the patient states she is back to her baseline and reports none of the following symptoms: cough, congestion, heartburn, or chest pain. The chest X-ray from the emergency department shows an incidental finding of a retrocardiac air-fluid level. Her vitals during today's visit are heart rate of 89 bpm, respiratory rate of 17/minute, oxygen saturation on room air of 99%, and blood pressure of 125/86 mm Hg. On physical exam, her heart rate is a regular rate and rhythm, abdomen is soft and nontender throughout, there is no hepatosplenomegaly, and lung sounds are clear. Which of the following is the best treatment of choice for this patient? Begin histamine H2 receptor antagonist daily High-dose proton pump inhibitor daily Nissen fundoplication Reassurance and observation Upper gastrointestinal endoscopy

Reassurance and observation This patient has a sliding hiatal hernia, which occurs when the lower esophageal sphincter moves above the diaphragm and leads to dysfunction of the gastroesophageal junction reflux barrier. Most sliding hiatal hernias are asymptomatic, such as in the patient above. However, if patients do develop symptoms, they are similar to gastroesophageal reflux disease (GERD), such as heartburn, belching, and regurgitation. For this patient, the diagnosis was made by a chest radiograph, which showed a retrocardiac air-fluid level. However, diagnosis can be confirmed via endoscopy, esophageal manometry, or barium swallow, which is the most sensitive test. For all asymptomatic sliding hiatal hernias, the treatment of choice is reassurance and observation. For patients with sliding hiatal hernias who have symptoms, the treatment of choice is to treat the GERD-like symptoms with diet and lifestyle modifications, antacids, histamine H2 receptor antagonists, or if symptoms are severe, proton pump inhibitors. Patients who are symptomatic and have a paraesophageal hernia may require surgical correction.

A 45-year-old man with a history of hypertension and gout presents to the clinic. He reports significant right-sided flank pain that has waxed and waned over the past several days, along with some mild nausea. He reports no dysuria, urinary frequency, and urgency. His current medications include lisinopril, hydrochlorothiazide, and allopurinol. Vitals include a temperature of 37°C, blood pressure of 165/90 mm Hg, heart rate of 95 bpm, respiratory rate of 14/min, and SpO2 of 98% on room air. On physical exam, cardiac and pulmonary exams are unrevealing except for the tachycardia previously noted. The abdomen is soft, nondistended, and nontender to palpation. He has moderate right costovertebral angle tenderness. An ultrasound reveals hydronephrosis and proximal ureteral dilation. Which of the following is most likely to be seen on urinalysis? Muddy brown casts Red blood cell casts Red blood cells White blood cell casts White blood cells

Red blood cells The patient presents with nephrolithiasis, or kidney stones. Nephrolithiasis is a common condition, affecting more than 1 in 10 adults annually worldwide. Most kidney stones (approximately 80%) are composed of calcium oxalate. Other less common stones may be composed of calcium phosphate, uric acid, struvite, or cystine. Stones are formed when these elements become oversaturated in the urine and cannot be dissolved appropriately. Risk factors for nephrolithiasis include male sex, age (most common between ages 40 and 60), reduced fluid intake, dietary factors, primary hyperparathyroidism, hypertension, gout, diabetes mellitus, and obesity. Patients with nephrolithiasis typically present with acute renal colic, which is intermittent cramping and pain in the flank or abdomen. This may be accompanied by nausea, vomiting, or hematuria. Pain can range in severity from mild to debilitating, and its location is dependent on the site of stone obstruction. For example, upper ureteral or kidney pelvis obstruction typically presents as flank pain, while obstruction in the lower ureter will present as groin or abdominal pain. However, if the stones are small or still present in the renal pelvis, the patient may be completely asymptomatic. Evaluation of patients with suspected nephrolithiasis should include a detailed history and physical exam, urinalysis, and diagnostic imaging. Urinalysis should reveal the presence of red blood cells (either microscopic or gross hematuria). Crystals may also be visualized in the urine. CT of the abdomen and pelvis without contrast is the preferred modality for the diagnosis, but ultrasound or abdominal X-ray can also be used. Treatment includes pain control with NSAIDs or opioids, depending on severity, as well as straining the urine to confirm stone passage. However, in patients with stones > 5 mm, the probability of unassisted passage is low. In patients with stones > 5 mm but < 10 mm, medical expulsive therapy with alpha-blockers can hasten or improve the chance of stone passage. For stones > 10 mm or smaller stones that fail to pass within 4 to 6 weeks, a urology referral for mechanical intervention is warranted. Current options for mechanical removal of kidney stones include shock wave lithotripsy, ureteroscopic lithotripsy, percutaneous nephrolithotomy, and laparoscopic stone removal. Patients with a history of kidney stones are more likely to have subsequent kidney stones than stone-naïve patients. Patients should be advised to increase daily fluid intake and make dietary modifications as needed to prevent a recurrence

A 65-year-old man with obesity presents to the clinic with intermittent, painless hematochezia for the past week. He has no history of inflammatory bowel disease or hemorrhoids. He smokes cigarettes and reports an overall poor diet with frequent consumption of red meat. Blood pressure is 124/80 mm Hg, pulse is 80 beats per minute, and temperature is 98.8°F. No hemorrhoids are noted on exam and there is no evidence of active bleeding. Initial blood analysis reveals a hemoglobin of 13.6 g/dL and a white blood cell count of 9,600/mcL. What is the best next step in management of this patient? Admit for intravenous antibiotics Order a plain abdominal radiograph Prescribe oral antibiotics Refer for colonoscopy

Refer for colonoscopy Diverticular bleeding is the most common cause of significant lower gastrointestinal bleeding. The incidence of diverticulosis and diverticulitis increases with age. Risk factors for diverticular disease include high dietary consumption of red meat, low dietary fiber, sedentary lifestyle, BMI > 25 kg/m2, and cigarette smoking. Diverticular bleeding usually presents as painless hematochezia, while diverticulitis usually presents as left lower quadrant abdominal pain and tenderness with leukocytosis. It is uncommon to present with symptoms of both diverticular bleeding and diverticulitis. Colonoscopy is the test of choice when a patient presents with painless hematochezia, to identify the source of the bleeding. The majority of cases of bleeding from diverticulosis resolve spontaneously, however, patients are at risk for recurrent bleeding.

A 42-year-old man presents to your office with complaints of uncontrollable fidgeting, irritability and difficulty with short-term memory. He tells you that his father had similar symptoms and died at a young age. Which of the following is the most appropriate next step in management? Discussion of end-of-life issues Magnetic resonance imaging Positron emission tomography scan Referral for genetic testing

Referral for genetic testing Huntington disease (HD) is an autosomal dominant inherited, neurologic disorder characterized by choreiform movements, dementia and psychiatric disturbances. Patients with early symptoms of HD present with mild chorea which may be mistaken for fidgeting. Cognitive decline occurs and the behavioral aspect of the disorder often presents with symptoms of depression, mania, psychosis or changes in personality. Diagnosis is with genetic testing and patients who are suspected to have HD should be referred as soon as possible. Asymptomatic patients with a known family history of HD may also request genetic testing prior to the onset of symptoms. Currently there is no cure and no treatment that will modify the disease process. Treatment focuses on supportive care for the patient and family along with management of symptoms. Prognosis is poor, as HD is a progressive disorder that leads to disability and death.

A 13-year-old boy with no significant medical history presents to urgent care with a headache 3 days after a closed head injury. The patient states that he stood up from a kneeling position and hit the top of his head on a wood cabinet. There was no loss of consciousness or evidence of seizure activity. In addition to the headache, he reports difficulty concentrating at school and dizziness. His physical examination is unremarkable. What management is indicated? CT scan of the head with contrast CT scan of the head without contrast MRI of the brain Referral to primary care physician

Referral to primary care physician The patient presents with minor head trauma and concerns consistent with a concussion and should have follow-up arranged with their primary care provider or concussion specialist. A concussion is a minor traumatic brain injury (TBI) that is often seen in MVCs and collision sports (e.g., football, hockey). It is typically caused by a rotational or acceleration-deceleration injury. Patients will present with a number of non-specific symptoms, including headaches, dizziness, confusion, amnesia, difficulty concentrating, and blurry vision but do not have focal neurologic findings. Despite the absence of severe intracranial injury, patients can have chronic and debilitating symptoms from concussions. Neurology referral is recommended, as patients should have functional testing and tracking of their symptoms to resolution. It is vital to counsel patients to avoid contact sports or activities that increase the risk of recurrent injury as these patients are at risk for more severe injury with a second impact.

A 70-year-old man with a history of cigarette smoking and BMI of 31.2 kg/m² presents to the clinic with gross hematuria for the past week. He reports unintentional weight loss of 15 pounds during the past 6 weeks. He has had no difficulty urinating, painful urination, or changes in urinary frequency. Vital signs include a HR of 80 bpm, BP of 120/80 mm Hg, RR of 20/min, oxygen saturation of 98% on room air, and T of 98.6°F. Physical examination reveals a regular rate and rhythm with lungs clear to auscultation. His abdomen is soft and nontender without palpable masses. His prostate is enlarged but symmetric, and firm on a digital rectal exam. His urinalysis is negative for leukocyte esterase, nitrites, pyuria, and bacteriuria but shows 100 RBC/hpf. His CT is shown above. Which of the following is the most likely diagnosis? Acute bacterial prostatitis Bladder cancer Polycystic kidney disease Prostate cancer Renal cell carcinoma

Renal cell carcinoma Renal cell carcinoma is the most common type of kidney cancer. It occurs most often between 50 and 70 years of age and is more common in men than women. The most important risk factor for renal cell carcinoma is a history of cigarette smoking. There are several possible presentations of renal cell carcinoma. These include asymptomatic patients who have the tumor detected incidentally on diagnostic tests performed for other reasons, patients with local symptoms due to the renal cell carcinoma, and patients with symptoms of distant metastasis. The most common symptoms are flank pain, palpable renal mass (more common in patients with a BMI < 25 kg/m2), hematuria, and weight loss. However, patients rarely have each of these clinical findings. The recommended diagnostic imaging modality is a CT scan of the abdomen with and without contrast, which would show kidney mass. The diagnosis is confirmed with biopsy, but this is often obtained at the same time as surgical treatment. The primary treatment is surgical removal of the tumor with a radical or partial nephrectomy. Patients who have locoregionally advanced disease are at higher risk of recurrence following surgical treatment and are often given adjuvant immunotherapy with pembrolizumab. Immunotherapy is also often recommended for patients with metastatic renal cell carcinoma.

A 60-year-old man with a history of hypertension and diabetes mellitus type 2 presents to your office for his annual exam. He reports noncompliance with his routine medications and has been taking ibuprofen daily for the past six months due to headaches. Which of the following is the most useful initial imaging study to evaluate this patient for chronic kidney disease? CT scan with contrast CT scan without contrast MRI Renal ultrasound

Renal ultrasound Risk factors for chronic kidney disease include a history of hypertension, African ancestry, diabetes mellitus, and the use of NSAIDs. Ultrasound is the most useful imaging study to confirm that both kidneys are present, symmetric, and to estimate their size. It can also determine if a renal mass or obstruction is present. Bilateral small-sized kidneys indicate a diagnosis of chronic kidney disease that is long-standing. Kidneys may be of normal size with either acute or subacute renal disease. Evaluation of serum creatinine is the most important initial step in evaluating a patient with abnormal kidney function. Comparison of current and past measurements of serum creatinine can help determine if renal dysfunction is acute and therefore reversible. Serum creatinine concentration is used with age, sex, race and body weight to determine the estimated glomerular filtration rate. CT scan without contrast (B) and MRI (C) are used to diagnose renovascular disease. CT scan with contrast (A) should be avoided in patients who are suspected to have chronic kidney disease because radiographic contrast dye can induce renal failure.

A 74-year-old man presents to the clinic with worsening nocturia and urinary hesitancy. The symptoms have been present for 3 years but have worsened over the past month. The patient reports no abdominal pain, fever, or weight loss. Vital signs include a HR of 80 bpm, BP of 120/80 mm Hg, RR of 20/min, oxygen saturation of 98% on room air, and T of 98.6°F. Physical examination reveals a soft and nontender abdomen and a diffusely enlarged and nontender prostate on digital rectal examination. The patient's creatinine today is 2.8 mg/dL, and it was 1.4 mg/dL 6 months ago. His prostate-specific antigen is 3.0 ng/mL. Postvoid residual volume is 500 mL. Which of the following is the most appropriate next step in his evaluation? CT abdomen and pelvis without contrast Cystoscopy Kidney, ureter, and bladder radiograph Prostate biopsy Renal ultrasound

Renal ultrasound The patient in this vignette has an acute kidney injury and an elevated postvoid residual urine. Renal ultrasound is the most appropriate next step in evaluation because it can confirm the presence of urinary tract obstruction and hydronephrosis. Urinary tract obstruction must be treated immediately to avoid irreversible damage to the kidney. Patients who have urinary tract obstruction and hydronephrosis due to bladder outlet obstruction from benign prostatic hyperplasia are often temporarily treated with urinary catheterization. These patients should also be started on medical therapy, such as an alpha-blocker, if this has not already been attempted. Patients with benign prostatic hyperplasia who have an elevated postvoid residual volume despite medical therapy require surgical treatment. The surgical treatment should be delayed until the patient's medical condition, including any sequelae from the obstruction, has been optimized. There are a variety of surgical procedures that can be used to treat benign prostatic hyperplasia, including transurethral resection of the prostate. CT abdomen and pelvis without contrast (A) is not the recommended initial diagnostic test since it exposes the patient to radiation and the patient does not have symptoms or signs of an obstructing kidney stone (severe flank pain, nausea, vomiting, or gross hematuria). Cystoscopy (B) is a procedure that allows for visual examination of the lining of the bladder. It allows for biopsies to be taken and is the procedure used to diagnose bladder cancer. However, renal ultrasound is the preferred procedure to rule out urinary tract obstruction, which is the next step for this patient. Kidney, ureter, and bladder radiograph (C) can detect some radiodense calcified kidney stones, but it cannot detect hydronephrosis. Prostate biopsy (D) is incorrect because prostate cancer is not the most likely diagnosis. The patient has a normal prostate-specific antigen and does not have asymmetric nodules on digital rectal exam.

A 36-year-old woman presents with numbness to her arms and legs for 2 hours. She had a total thyroidectomy 3 days prior to presentation. Vital signs are normal, and the patients exam is significant for a clean, well-healing surgical incision on her neck and twitching of the corners of the mouth with tapping at the angle of the jaw. What of the following is appropriate management? Obtain a CT scan of the head Replenish calcium Replenish potassium Stop thyroid hormone

Replenish calcium Hypoparathyroidism is a common cause of hypocalcemia. During thyroidectomy, parathyroid glands are often removed, and this may lead to hypocalcemia that presents as paresthesias. This patient also presents with a positive Chvostek sign (twitching of the facial muscles in response to stimulation of the facial nerve), indicating hypocalcemia. Hypocalcemia can present with numerous clinical manifestations, including muscle cramping, paresthesias, shortness of breath from bronchospasm, hypotension, dysrhythmias, and cardiovascular collapse in severe forms. QTc prolongation is common in moderate to severe hypocalcemia. Chvostek and Trousseau (carpal spasm with inflation of a blood pressure cuff) signs may be seen and indicate nerve irritability as a result of hypocalcemia. A serum calcium level is diagnostic, and the patient should be treated with calcium repletion.

A 17-year-old girl presents to the emergency department with nausea and dizziness after striking her head on the ground during a fall yesterday while playing basketball. The patient's parent who witnessed the fall states that her daughter did not lose consciousness but was transiently confused for a couple of minutes after the fall. Vital signs include a heart rate of 92 bpm, blood pressure of 120/80 mm Hg, respiratory rate of 20/min, oxygen saturation of 98% on room air, and temperature of 98.6°F. Physical examination findings include pupils that are equal, round, and reactive to light, intact extraocular movements, intact facial and extremity sensation and motor function, and a stable gait. Which of the following is the best recommendation for this patient's return to activities? Avoid sporting activity participation indefinitely Immediately return to normal activities Increase cognitively stimulating activities for the next 48-hours Rest for 24-48 hours followed by a gradual return to normal activity Strict bed rest until symptoms completely resolve

Rest for 24-48 hours followed by a gradual return to normal activity Concussions (mild traumatic brain injuries) are diagnosed in patients who have traumatic brain injuries with a Glasgow Coma Scale (GCS) score of 13-15 and at least one of the following features: any period of loss of consciousness, memory loss of the traumatic event or events before or after, or any alteration in mental status at the time of the injury. Patients with a GCS score of 9-12 are considered to have a moderate traumatic brain injury. People who have experienced a possible concussion should be evaluated by a medical professional, which may occur initially on the sideline of a sporting game, in a clinic, or at the emergency department. Patients having a period of unconsciousness > 1 minute associated with a head injury, persistent mental status alterations, or abnormalities on neurologic examination require urgent evaluation in an emergency department. CT of the head without contrast is the recommended initial imaging study for individuals with acute head trauma who require imaging. Indications for a head CT scan following mild head trauma include GCS score < 15, suspected open or depressed skull fracture (scalp laceration, hematoma, or bony step-off of the skull), signs of basilar skull fracture (hemotympanum, periorbital bruising, and retroauricular bruising), at least two episodes of vomiting, new neurologic deficit, presence of bleeding diathesis or use of an anticoagulant, seizure, age of at least 60 years, retrograde amnesia of 30 minutes or longer before the traumatic episode, high impact head injury (pedestrian struck by motor vehicle, occupant ejected from motor vehicle, and fall from 3 feet or more), and intoxication. Patients diagnosed with a concussion require close observation either by a caretaker at home or in the hospital. Each of the following is generally considered an indication for inpatient observation in a patient with an acute concussion: GCS score < 15, seizure, bleeding diathesis or anticoagulation, and no responsible caregiver at home. Patients who do not meet at least one of these criteria can be monitored closely at home. Patients who experience clinical deterioration during monitoring should have a noncontrast CT scan of the head. Athletes who experience a concussion should not return to play the same day and should not return at all until symptoms have resolved off medication. Patients with uncomplicated concussion (patients without an abnormal head CT scan if indicated) should undergo a period of physical and cognitive rest for at least 24 hours. They can gradually return to normal activities as symptoms improve.

A 59-year-old woman has furosemide-resistant peripheral edema and ascites. Her medical history is significant for sarcoidosis. You order an echocardiogram which reveals increased wall thickness and decreased cavity size of the right ventricle, enlarged atria and a normal appearing left ventricle. Cardiac MRI shows no fatty deposition in the ventricular walls. Which of the following is the most likely diagnosis? Arrhythmogenic right ventricular cardiomyopathy Dilated cardiomyopathy Hypertrophic cardiomyopathy Restrictive cardiomyopathy

Restrictive cardiomyopathy Restrictive cardiomyopathy (RCM) is caused by decreased myometrial compliance in the absence of pericardial disease. Several etiologies exist including infiltrative disorders (e.g. amyloidosis, sarcoidosis and hemochromatosis), autoimmune disorders (e.g. scleroderma and polymyositis), storage disorders (e.g. Gaucher's and Fabry's), toxin exposure (e.g. anthracyclines and radiation), esosinophilic disorders (e.g. Loffler's disease), metastatic cancer and diabetes mellitus. In RCM, myocardial compliance is decreased, ventricular cavity size is decreased and wall thickness is normal or increased in a symmetric fashion. Right-sided heart failure is more common than left sided disease, and as such, patients present more with peripheral edema than pulmonary edema. Common associated findings include tachyarrhythmias, thromboembolism (mural thrombi may be seen on echocardiogram), hepatomegaly and ascites, increased jugular venous pulsation, regurgitant murmurs and S3 or S4. It is also common for patients with RCM to fail diuretic management of peripheral edema.

A 14-year-old boy presents to the Emergency Department for a rash. He reports the abrupt onset of fever, headache, and myalgias three days ago. This morning, he developed a blanching, red, macular rash on his wrists and palms that now involves his extremities and trunk. What is the most likely cause of his symptoms? Measles virus Neisseria meningitidis Rickettsia rickettsii Staphylococcus aureus

Rickettsia rickettsii Rocky Mountain spotted fever (RMSF) is a febrile, tick-borne illness caused by Rickettsia rickettsii. Patients present with abrupt onset of fever, headache, myalgias, and nausea followed three to five days later with a blanching, macular rash that initially is found on the wrists and ankles before spreading centripetally. The rash later becomes petechial. Despite its name, RMSF is relatively rare in the Rocky Mountain states and is found primarily in the southeastern United States. Carried mostly by the American dog tick (Dermacentor variabilis) and the Mountain wood tick (Dermacentor andersoni), R. rickettsii is an obligate intracellular bacteria that damages endothelial cells. This in turn starts a cascade of reactions that result in widespread vascular lesions that manifest as the clinical features of the disease. Without treatment, mortality is near 25%. Management includes supportive care and doxycycline.

You are seeing an elderly man with new onset of peripheral edema, head fullness and neck venous engorgement. Initial testing shows normal left heart function. You suspect cor pulmonale. Which of the following tests is the most accurate in confirming this diagnosis? Cardiac magnetic resonance imaging Electrocardiography Right heart catheterization Ultrafast ECG-gated computed tomography

Right heart catheterization Cor pulmonale is defined as an alteration in the structure and function of the right ventricle caused by a primary disorder of the respiratory system. Although the most common cause of right heart failure is left-sided heart disease, cor pulmonale is right heart dysfunction due to a lung, and not heart, problem. Just like systemic hypertension causes changes in left ventricular function, pulmonary hypertension causes changes in right ventricular function. This major underlying pathology is largely due to some kind of pulmonary vascular bed compromise, which can be primary pulmonary hypertension or thromboembolic disease, but more commonly anatomic compromise (COPD, interstitial lung disease and rheumatologic or connective tissue/collagen vascular disorders) or vasoconstrictive compromise (chronic hypoxic states and acidemia). When evaluating a patient with right heart failure, cor pulmonale is considered if pulmonary pathology is causative. However, if the etiologic evaluation is void of a pulmonary source, then the diagnosis of cor pulmonale cannot be made. In this situation, the clinician then tries to pinpoint a cardiac or blood disorder. Even when a pulmonary source is found to be a cause of right side heart failure, it is equally important to determine if there is such a coexisting non-pulmonary cause, such as increased blood viscosity, atrial and ventricular defects, congenital heart disease, cardiomyopathies and constrictive pericarditis. The general approach to evaluating a patient with suspected cor pulmonale begins with routine lab testing, chest radiography and electrocardiography (ECG). Further investigation of underlying pulmonary pathology is then accomplished via pulmonary function testing, ventilation/perfusion (V/Q) scanning and chest computed tomography. Right heart catheterization is the most accurate but invasive test to confirm the diagnosis of cor pulmonale.

A woman presents with dyspnea on exertion. Cardiac examination reveals an apical mid diastolic murmur. You also notice pitting edema in both her legs. She undergoes echocardiographic testing. Which of the following abnormalities would you most expect to see on the echocardiogram? Decreased trans-mitral mean pressure gradient Increased opening of the mitral valve leaflets Right atrial hypertrophy Right ventricular hypertrophy

Right ventricular hypertrophy One of the diastolic murmurs is mitral stenosis, which is associated with rheumatic heart disease and endocarditis. This murmur is best appreciated at the apex with the bell, especially when the patient is placed in the left lateral decubitus position. It is characterized as a late diastolic, low frequency rumble which does not radiate. It is commonly associated with a left sternal border thrill. The intracardiac narrowing of this condition hampers outward flow from the left atrium, causing a "pressure back-up" into the pulmonary circulation and ultimately the right heart. Subsequently, right ventricular hypertrophy, and possibly right heart failure, are frequent associated findings. Symptoms of mitral stenosis include fatigue, weakness, dyspnea on exertion, orthopnea, palpitations and chest pain. If the condition is chronic, and right heart failure is present, peripheral edema, ascites and hepatomegaly may be present. Pregnancy and exercise can exacerbate the fatigue and weakness encountered in someone with mitral valve stenosis.

A 17-year-old girl presents with a fever, myalgias, and headache. She noted a rash that began 4 days after she noted her fever. The macular rash began on her wrists and ankles and spread toward her chest. She recently returned from hiking the Appalachian Trail in North Carolina about 1 week ago and is concerned about her symptoms. Which of the following is the most likely diagnosis? Babesiosis Colorado tick fever Lyme disease Rocky Mountain spotted fever

Rocky Mountain spotted fever Rocky Mountain spotted fever (RMSF) is a life-threatening infection caused by Rickettsia rickettsii transmitted by dog ticks. Its name derives from its original description in Montana and Idaho in the late 19th century and from the typical petechial rash occurring initially on the wrists and ankles and spreading centripetally (toward the center). It may also involve the palms and soles. Despite its name, most cases are reported from the southeastern and south-central United States. A little more than 50% of the cases present with the classic triad of rash, fever, and tick exposure, although the rash is rarely present during the first 3 days of the illness and usually appears on day 4. Abrupt onset of fever, severe headache, and myalgias are the most common presenting symptoms 5-7 days after the tick bite.

A 34-year-old man presents to the emergency department with complaints of worsening chest pain, fever, and malaise. The pain is pleuritic, worsens when he lies down and improves when he leans forward. On exam, he appears unwell, but is not in acute distress. Auscultation over the precordium reveals a scratchy, grating sound with a normal S1 and S2. Which of the following is the most likely electrocardiogram finding in this patient? Shortened PR interval and slurring of the QRS complex ST segment depression in leads V2-V4 ST segment diffuse elevation with reciprocal ST depression in leads aVR and V1 U waves and flat T waves

ST segment diffuse elevation with reciprocal ST depression in leads aVR and V1 This patient with pericarditis will most likely have diffuse ST segment elevation (except for aVR and V1 which will show reciprocal ST depression and PR elevation). Pericarditis is an inflammation of the pericardial sac. The pericardium is composed of a visceral layer, which forms the epicardium, and the parietal layer. These layers are separated by a space that normally contains 15-50 mL of serous fluid. During acute pericarditis, changes in the ECG reflect inflammation of the epicardium, since the parietal layer of the pericardium is electrically inert. The ECG evolves through as many as four stages during acute pericarditis. Lasting hours to days, stage 1 involves generalized ST segment elevations and PR segment depression. ST and PR segments normalize during stage 2. Stage 3 development is variable in some patients. Stage 3 is characterized by diffuse T wave inversions. Stage 4 represents normalization of the ECG. However, not all causes of pericarditis cause alterations in the ECG. Uremic pericarditis rarely causes epicardium inflammation and therefore, the ECG will show none of the changes described above. Chest pain is the most consistent finding associated with pericarditis. The pain is typically described as sharp and pleuritic. The tendency for the pain to decrease in intensity when the patient leans forward is a distinctive characteristic. Sitting up and leaning forward reduces the pressure on the parietal pericardium and allows for diaphragm splinting. In addition to an ECG, echocardiography should be performed to evaluate for pericardial effusions and to rule out cardiac tamponade. Unless contraindicated, nonsteroidal anti-inflammatory drugs (NSAIDs) are considered first line treatment for acute pericarditis. NSAIDs should not be used to treat pericarditis after myocardial infarction or uremic pericarditis.

Twenty-four hours after eating a salad containing bean sprouts, a 25-year-old man became ill with fever, abdominal pain and diarrhea. Which of the following organisms is the most likely cause of his symptoms? Clostridium perfringens Cryptosporidium parvum Enterotoxigenic Escherichia coli Salmonella enterica

Salmonella enterica Foodborne illness should always be on the differential when a patient presents with gastrointestinal symptoms such as nausea, vomiting, diarrhea, abdominal pain and fever. A thorough history provides important information in diagnosing foodborne illness and should include description of clinical symptoms, exposure to high-risk types of food and the duration of time between consumption of the infected food and onset of symptoms. Salmonella enterica is a gram-negative bacterium that causes inflammatory diarrhea. Signs and symptoms of inflammatory diarrhea may include the presence of blood or mucus in the stool, severe abdominal pain and fever. Salmonellosis is typically associated with consumption of raw meat or poultry, but can also be associated with fresh produce such as bean sprouts, tomatoes, lettuce, and melons. The illness is self-limited and treatment generally is with fluid replacement, although antibiotics are used in certain cases (i.e. immunocompromised patients and children). A consult should be done with the local health department to report the illness and help determine next steps.

A 38-year-old man presents with fever, fatigue, cough, and increasing dyspnea. The patient has no chills or night sweats. On physical exam, lungs are clear to auscultation bilaterally. Hepatosplenomegaly is noted on abdominal exam. Chest X-ray findings include bilateral hilar adenopathy and diffuse reticular infiltrates. Labs reveal leukocytopenia, hypercalcemia, and elevated erythrocyte sedimentation rate. What is the most likely diagnosis in this patient? Histoplasmosis Mycoplasma pneumoniae Pulmonary tuberculosis Sarcoidosis

Sarcoidosis Sarcoidosis is a multi-organ disease, usually of idiopathic origin. It is more common in Black and White European patients. Patients commonly present with respiratory symptoms, including cough, dyspnea, and chest discomfort. Patients may also have malaise, fever, and multiple nonpulmonary symptoms. Sarcoidosis is characterized by noncaseating granulomatous inflammation in multiple affected organs. Organs involved include lungs, lymph nodes, liver, spleen, heart, central nervous system, and salivary glands. Physical exam findings include enlarged parotid glands, lymphadenitis, erythema nodosum, and enlargement of the liver or spleen. Lab tests show eosinophilia, leukopenia, hypercalcemia, and elevated erythrocyte sedimentation rate. Chest X-ray findings include bilateral hilar adenopathy, with diffuse reticular infiltrates. Diagnosis is confirmed by fine-needle lymph node biopsy or transbronchial lung biopsy, showing noncaseating granulomas. Treatment of sarcoidosis is with corticosteroids.

An 88-year-old woman with osteoporosis sustained a hip fracture after falling. The fracture was surgically treated, and she was discharged to the inpatient rehabilitation unit. Her main complaint now is leg pain and numbness. On examination, you find hamstring and ankle weakness, an absent ankle reflex, and numbness on the posterolateral leg. Hip flexion, hip abduction, and knee extension strength is intact. Injury to which of the following nerves is most likely causative? Femoral Obturator Sciatic Superior gluteal

Sciatic The hip is a very common site for osteoporotic fractures, especially in an older woman. Hip dislocation or fracture is a common cause of compressive sciatic neuropathy. Other causes include prolonged bed rest, piriformis syndrome, and pelvic masses. Proximal sciatic neuropathy results in a wide array of symptoms, including sensory changes in the peroneal, tibial, and sural distributions (medial lower leg and arch of foot are likely spared due to saphenous innervation); absent ankle, decreased hamstring, and preserved knee reflexes; local pain that radiates through the posterior thigh and leg; and weakness in most of the leg EXCEPT for hip flexion and extension (femoral and gluteal nerve innervation), hip abduction and adduction (superior gluteal (D) and obturator (B) nerve innervation), and knee extension (femoral nerve (A) innervation). Electrodiagnostics (EMG and nerve conduction studies) are helpful in distinguishing sciatic neuropathy from L5 or S1 radiculopathy.

A 45-year-old man presents to your office with questions about prostate cancer screening. He does not have a family history of prostate cancer and wants to know at what age he should begin screening. According to the U.S. Preventive Services Task Force, which of the following is the most appropriate next step in management? Begin screening at age 50 Order serum prostate-specific antigen now, then begin screening at age 50 Perform an initial digital rectal exam now, then begin screening at age 50 Screening is not recommended

Screening is not recommended In the United States, prostate cancer is the most frequently diagnosed type of cancer in men after skin cancer. Prostate cancer is seen more commonly in Black men, and the likelihood of developing this type of cancer increases with age. Risk factors include a family history of prostate cancer, cigarette smoking, and a diet high in animal fat. Routine screening for prostate cancer has been a controversial subject. Current United States Preventive Services Task Force guidelines recommend against prostate-specific antigen (PSA) screening for prostate cancer. Some men will request screening, and a discussion about the risks and benefits of screening should occur prior to initiating PSA testing or digital rectal exam (DRE). Most patients diagnosed with prostate cancer are asymptomatic, and the cancer is found on DRE or because of an elevated serum PSA. Diagnosis is made with biopsy. Treatment decisions are determined after discussion with the patient about the severity of disease and quality of life related to treatment side effects. Options include active surveillance, prostatectomy, radiation therapy, and hormonal therapy.

A 16-year-old boy comes to the clinic complaining of a change in mental status. He had a 2 cm painless genital ulcer that resolved on its own about one year ago. Screening tests reveal a positive rapid plasma reagin. Which of the following represents the appropriate treatment regimen? Send a cerebrospinal fluid VDRL and start penicillin G 4 million units IV every 4 hours Send a cerebrospinal fluid VDRL and start penicillin G benzathine 2.4 million units IM once weekly Send an FTA-ABS and start ceftriaxone 2 g IV daily Send an FTA-ABS and start penicillin G 4 million units IV every 4 hours

Send a cerebrospinal fluid VDRL and start penicillin G 4 million units IV every 4 hours The various manifestations of syphilis are time dependent. Syphilis is classified as primary, secondary, and tertiary. Tertiary syphilis comprises three types: neurosyphilis, cardiovascular syphilis, and late benign syphilis (gummatous). The patient in this scenario is suspected of having late-stage neurosyphilis (based on his history of primary syphilis), given his positive RPR and altered mental status. Other symptoms of neurosyphilis include personality changes, bladder incontinence, headache, hearing loss, and blurred vision. While this patient has tertiary neurosyphilis, neurosyphilis can occur in any stage of syphilis infection. RPR is a screening test. A cerebrospinal fluid (CSF) VDRL should be sent to confirm the diagnosis. CSF pleocytosis is common prior to initiation of treatment, and serial LPs should be performed after treatment is completed to confirm cure. Adequate treatment of neurosyphilis is based largely on achieving treponemicidal levels of penicillin in the CSF. Treponema pallidum is highly susceptible to penicillin, which is the drug of choice for all stages of syphilis. Serum levels of penicillin should be maintained for many days because treponemes divide slowly in early syphilis and penicillin acts only on dividing cells. Therefore, treatment is with intravenous penicillin G 3-4 million units every 4 hours continuously for 10-14 days.

A 50-year-old woman presents to her surgeon for a routine follow-up after her thyroidectomy. She reports no symptoms in the office today. Her thyroid was removed 6 months ago due to medullary thyroid cancer, and no complications were noted during the surgery. She has no other significant medical history and is on no medications. Her vitals include a BP of 122/78 mm Hg, HR of 85 bpm, RR of 12/min, T of 98.7°F, and SpO2 of 100% on room air. On physical examination, her incision is well healed, and no lymphadenopathy is present. Which of the following should be monitored to evaluate for medullary thyroid cancer recurrence? Anti-thyroid peroxidase antibodies Serum calcitonin Serum thyroxine Thyroid-stimulating hormone Thyrotropin-releasing hormone

Serum calcitonin Medullary thyroid cancer accounts for around 1-2% of all thyroid cancers, and risk factors include prior head and neck radiation, family history, or (multiple endocrine neoplasia type 2) MEN2. It arises from the thyroid parafollicular cells, which produce hormones such as adrenocorticotropic hormone (ACTH), calcitonin, and serotonin. Most patients with thyroid cancer are asymptomatic. Less common symptoms include dysphagia or hoarseness from upper digestive tract compression and systemic symptoms secondary to hormone secretion from the tumor, such as facial flushing and diarrhea. On physical exam, the most common finding is a painless thyroid nodule, with or without lymphadenopathy. Calcitonin levels are almost always elevated in these patients, while thyroid-stimulating hormone (TSH) levels tend to be within the normal range. A cold nodule on radioactive iodine uptake scan, indicating no uptake and a nonfunctioning nodule, is suspicious for thyroid cancer and must be investigated further. The diagnosis of thyroid cancer is usually made after fine-needle aspiration in a patient presenting with a painless nodule. Treatment options include surgical removal of the thyroid. Follow-up after thyroidectomy for medullary thyroid cancer usually involves yearly ultrasounds and serum calcitonin levels, as calcitonin is an important tumor marker in this disease. TSH levels tend to be normal in patients with medullary cancer and thus are not helpful to assess for disease recurrence but can be used to monitor post-treatment thyroid levels. Post surgery, levothyroxine should be started and thyroid levels closely monitored to maintain euthyroid status. Similarly, adjuvant therapy with radioiodine is not indicated because the tumor cells do not concentrate iodine.

A 28-year-old man presents to the emergency department with severe upper abdominal pain, nausea, and vomiting for 8 hours. Vital signs today include a heart rate of 112 bpm, blood pressure of 125/85 mm Hg, respiratory rate of 20/minute, oxygen saturation of 98% on room air, and temperature of 99.2°F. Physical examination reveals a man in acute distress with epigastric tenderness but no abdominal rigidity, rebound tenderness, or bruising. Laboratory findings include a white blood cell count of 7,000/μL, lipase of 2,200 U/L, aspartate aminotransferase of 35 U/L, alanine aminotransferase of 42 U/L, alkaline phosphatase of 85 U/L, and total bilirubin of 0.8 mg/dL. Gallbladder ultrasound reveals a diffusely enlarged and hypoechoic pancreas but no gallstones or gallbladder wall thickening. Which of the following additional studies may reveal the cause of this condition? CFTR genetic testing Serum ceruloplasmin Serum gamma-glutamyl transpeptidase Serum lipid profile Urine drug screen

Serum lipid profile Acute pancreatitis is a disease process marked by pancreatic inflammation caused by pancreatic enzymes that have leaked from damaged pancreatic cells autodigesting the pancreas. The most common causes of acute pancreatitis are gallstones and alcohol use. However, other causes include hypertriglyceridemia, endoscopic retrograde cholangiopancreatography, medication adverse effects, autoimmune disease, and idiopathic disease. A serum lipid profile should be obtained in patients with acute pancreatitis to assess for hypertriglyceridemia. The classic presentation is an abrupt onset of severe upper abdominal pain with nausea and vomiting. The pain often radiates to the back. Exam findings include epigastric abdominal tenderness. Hemorrhagic pancreatitis can cause Cullen sign (periumbilical bruising) or Grey Turner sign (bruising in bilateral flanks) in a minority of cases. Acute pancreatitis requires two of the three criteria for diagnosis: characteristic symptoms, lipase elevation to three times the upper limit of normal, and characteristic imaging findings. Serum lipase levels rise within 4-8 hours, peak at 24 hours, and return to baseline by 8-14 days. The characteristic CT findings of acute pancreatitis include focal or diffuse enlargement of the pancreas with heterogeneous enhancement. Patients with acute pancreatitis are generally admitted to the hospital. The recommended treatment is supportive care, including intravenous fluids and pain medication. Intravenous opioids are usually required for pain control. The diet is slowly advanced as tolerated.

A 67-year-old man presents to the family medicine clinic for worsening low back pain for the past 6 weeks. The patient reports no recent trauma or heavy lifting. He currently takes lisinopril 20 mg daily for hypertension and has no other relevant medical history. Current vital signs are temperature of 99.6°F, BP of 110/78 mm Hg, HR of 94 bpm, RR of 18/minute, and oxygen saturation of 97% on room air. A physical exam reveals moderate tenderness along the lumbar spinous processes. The patient appears fatigued and has a generalized pallor. Laboratory studies reveal hemoglobin of 9.8 g/dL, calcium of 12.5 mg/dL, and creatinine of 1.4 mg/dL. Which of the following is the most appropriate next step in evaluation? Antinuclear antibody assay Bone marrow biopsy Kidney ultrasound MRI of the lumbar spine Serum protein electrophoresis

Serum protein electrophoresis Multiple myeloma is a malignancy of the hematopoietic stem cells in bone marrow, resulting in neoplastic proliferation of plasma cells and producing a monoclonal immunoglobulin. The infiltration of these plasma cells into the bone marrow leads to lytic lesions, bone pain, pathologic fracture, and end-organ damage. It is the second most common hematologic malignancy and occurs most commonly in older adults, with a median age of diagnosis of 65 to 74. The most common presenting symptom is anemia, occurring in 73% of patients at the time of initial presentation. Anemia occurs due to infiltration of plasma cells into bone marrow and the distal tubules of the kidneys, leading to decreased production of RBCs in bone marrow and kidney injury, which also contributes to decreased erythropoietin from the kidneys. Hypercalcemia is often present due to destruction of bone. Bone pain, elevated creatinine, fatigue, weight loss, and increased rate of infections are also commonly seen. A diagnosis of multiple myeloma is made by the presence of monoclonal immunoglobulin on serum protein electrophoresis and bone marrow biopsy. Bone marrow biopsy reveals the presence of clonal plasma cells in the marrow, and a result ≥ 10% is diagnostic of multiple myeloma. Anemia is nearly universal in patients with multiple myeloma. A peripheral smear may reveal Rouleaux formation, consisting of red blood cells in a stack-of-coins appearance. Additional studies include the presence of free monoclonal kappa or lambda light chains in the urine, known as Bence Jones proteinuria. A 24-hour collection of urine may also show elevated total protein with urinary M-protein spike. Bone radiographs, CT, or MRI typically shows lytic lesions in the spine and can help evaluate the extent of bone disease. Treatment may consist of immunomodulating medications, chemotherapy, and plasma exchange, as well as steroids and bisphosphonates.

A 70-year-old man with a history of chronic kidney disease, hypertension, and type 2 diabetes mellitus presents to his nephrologist's office for follow-up. His vital signs are HR 84 bpm, RR 18/min, BP 124/82 mm Hg, T 98.3°F, SpO2 98%, and BMI 30 kg/m2. The patient is currently on hemodialysis 3 days per week. His medications include insulin aspart, insulin glargine, linagliptin, metoprolol, and lisinopril. Laboratory results show an estimated glomerular filtration rate of 20 mg/mmol, blood urea nitrogen of 25 mg/dL, creatinine of 1.9 mg/dL, serum calcium of 8.6 mg/dL, serum phosphate of 6.0 mg/dL, and magnesium of 2.4 mg/dL. His serum phosphate levels remain elevated despite implementing phosphate restrictions in his diet. He is asymptomatic and reports no symptoms of muscle cramps or weakness, paresthesias, or circumoral numbness. Which of the following is the best next step in managing his condition? Calcium citrate Continue dietary reduction of phosphate Hemodialysis Intravenous calcium Sevelamer

Sevelamer Phosphate binder (sevelamer) is the appropriate treatment of choice in asymptomatic patients with chronic hyperphosphatemia. The most common causes of hyperphosphatemia are transcellular shift of phosphate and decreased renal clearance due to chronic kidney disease (CKD). Other causes include tumor lysis syndrome, rhabdomyolysis, and hypoparathyroidism. Patients are generally asymptomatic but may present with symptoms of hypocalcemia such as muscle cramps or spasms, paresthesias, circumoral numbness, or QTC prolongation. Laboratory findings show hyperphosphatemia and are consistent with those of the underlying disease. In patients undergoing dialysis, the mainstay of treatment is achieving a serum phosphate concentration of 3.5-5.5 mg/dL. If serum phosphate is > 5.5 mg/dL, treatment to lower serum phosphate levels is indicated. The treatment of choice for hyperphosphatemia is phosphate binders, dietary restrictions, or both. A dietary restriction of 900 mg/day of phosphate should be implemented under the care of a nutritionist to prevent additional malnutrition, which is commonly seen in dialysis patients. Phosphate binders are divided into two groups: calcium-containing binders, such as calcium carbonate and calcium acetate, and non-calcium-containing binders, such as sevelamer and lanthanum. Non-calcium-containing binders are preferred over calcium-containing binders, which may be associated with increased serum calcium levels, vascular calcification, and bone disease. Studies have shown that increasing the dose of dialysis above the recommended dose does not improve clinical outcomes.

An 8-year-old boy presents to the emergency department via EMS after having a seizure. His parents are both at the bedside and report that the patient has had diarrhea with dark red mucus for 2 days. The diarrhea started watery and then progressed to dark red with mucus, and it also hurts when he has bowel movements. His vital signs are a HR of 90 bpm, RR of 19/min, oxygen saturation on room air of 99%, BP of 98/60 mm Hg, and T of 102.8°F. On physical exam, the patient has diffuse abdominal tenderness without guarding or rebound. Which of the following is the most common cause of this patient's suspected diagnosis? ntussusception Rotavirus Salmonella Shigella sonnei Vibrio cholerae

Shigella sonnei The patient in the above vignette has Shigelle sonnei, which is a highly virulent fecal-oral infection. The patient presents with abrupt onset of lower abdominal pain, high fever, explosive bloody mucoid diarrhea, and dysentery. The patient can have significantly high fevers which may lead to febrile seizures. Stool studies show leukocytes and red blood cells. Diagnosis is made by a stool culture which would be positive for Shigella. A Sigmoidoscopic exam would show punctate areas of ulceration. Complications include bacteriemia, sepsis reactive arthritis, hemolytic-uremic syndrome, and seizures. Treatment includes supportive care with fluids and a fluoroquinolone such as ciprofloxacin or levofloxacin. It is important not to give antimotility drugs in cases of shigellosis because it can lead to toxicity.

A 54-year-old woman with a past medical history significant for Sjögren syndrome presents with unilateral facial pain that began yesterday. Physical exam reveals right parotid gland swelling with overlying erythema and warmth, trismus, and purulent ductal discharge. Which of the following is the most likely diagnosis? Kimura disease Lymphoepithelial lesion Sialadenitis Sialolithiasis

Sialadenitis Sialadenitis is an infection of the submandibular or salivary glands, including the parotid gland. Patients typically present with acute glandular swelling, skin erythema and warmth, tenderness, trismus, purulent ductal discharge, fever, and malaise. The most common cause is salivary stasis from an underlying stricture or obstruction of the duct, which allows bacterial overgrowth and infection. Individuals with Sjögren syndrome are at greater risk of developing this infection because of the salivary stasis associated with the syndrome. Complications from untreated infections include cellulitis, abscess formation and Ludwig angina. Intravenous antibiotics (e.g., nafcillin in addition to metronidazole and ceftriaxone) is first-line treatment. Additional treatments include warm compresses and analgesics, massage of the gland, sialogogues, improved oral hygiene, and rehydration. Most patients recover completely; however, higher mortality rates are seen in immunocompromised and debilitated patients.

A 39-year-old stonemason presents with dyspnea that has been present for several months. A chest X-ray shows an eggshell calcification of the hilar lymph nodes and multiple small 1-2 mm nodules throughout the lung. Which of the following is the most likely diagnosis? Asbestosis Byssinosis Siderosis Silicosis

Silicosis Silicosis is one of many forms of pneumoconiosis, a chronic fibrotic lung disease caused by the inhalation of inorganic materials. Silicosis is due to the prolonged inhalation of silica (silicon dioxide) particles. It is most likely in sandblasting, rock mining, and stone cutting. Silicosis is usually asymptomatic, but prolonged-exposure cases result in dyspnea and obstructive, restrictive pulmonary dysfunction. Small, round nodules are formed through the lung. Chest radiography showing calcification of the hilar lymph nodes ("eggshell" calcification) is highly suggestive of silicosis. Treatment is supportive and smoking cessation is essential. Corticosteroids can be used to relieve any chronic alveolitis. Patients with silicosis are at increased risk of pulmonary tuberculosis. If active or inactive tuberculosis is suspected, multi-drug treatment should be initiated.

A 24-year-old woman presents to the office reporting progressive fatigue for the past several months. She is otherwise healthy and not on any medications. Her menstrual cycles are regular, lasting between 3 and 5 days, and she uses about five or six regular tampons per day. Her temperature is 98.6°F, blood pressure is 112/75 mm Hg, heart rate is 85 bpm, oxygen saturation is 98% on room air, and respiratory rate is 22/min. A physical exam is unremarkable. Her CBC reveals a hemoglobin of 10.2 g/dL and a mean corpuscular volume of 72 fL. The patient is prescribed daily oral ferrous sulfate and returns for a follow-up in 6 weeks. A repeat CBC is unchanged. Which of the following is the most likely diagnosis? Aplastic anemia Hereditary spherocytosis Iron deficiency anemia Sideroblastic anemia Thalassemia

Thalassemia The thalassemias are a group of hereditary disorders characterized by decreased production of either the alpha- or beta-globin chain of hemoglobin, which results in reduced hemoglobin synthesis. Normal adult hemoglobin is hemoglobin A, which makes up most of the circulating hemoglobin in the body. Two alpha-globin chains and two beta-globin chains make up a tetramer that forms hemoglobin A. The thalassemias can be divided into different types depending on the type of chain (alpha or beta) affected. Alpha-thalassemias are characterized by a decrease in alpha-globin chain synthesis, and their severity depends on the number of deleted or mutated alleles. Silent carriers (mutation or deletion of only one alpha-globin locus) are asymptomatic with normal hemoglobin and hematocrit levels and do not require treatment. Those with alpha-thalassemia minor (mutation or deletion of two alpha-globin loci) will have mild anemia with microcytic, hypochromic red blood cells (RBCs) and will not require treatment. Hemoglobin H disease (mutation or deletion of three alpha-globin loci) is a more severe form characterized by microcytic, hypochromic anemia and may require periodic transfusions. The most severe form of alpha-thalassemia occurs when all four alpha-globin loci are affected, leading to stillbirth or death shortly after birth. Beta-thalassemias are classified as major, minor, and intermedia. Beta-thalassemia major is a severe form characterized by severe anemia, jaundice, hepatosplenomegaly, growth restriction, and failure to thrive. Treatment includes frequent blood transfusions to sustain life. Those with beta-thalassemia minor are typically asymptomatic or have mild anemia, and treatment is usually unnecessary. Beta-thalassemia intermedia is characterized by varying degrees of anemia, and while most patients do not require transfusions in childhood, most will become transfusion-dependent in adulthood. The most common type of thalassemia is thalassemia minor, with the beta form being more common than the alpha form. Patients with this condition will typically have hypochromic, microcytic anemia. A CBC will reveal anemia with red blood cells that are microcytic (low mean corpuscular volume [MCV]) and hypochromic (due to low mean corpuscular hemoglobin [MCH]). Additionally, there may be an elevated reticulocyte count and a normal to elevated red blood cell count due to an increased number of small and pale red blood cells being produced. The mild forms of thalassemia are commonly mistaken for iron deficiency anemia. Iron deficiency anemia can be ruled out with an unsuccessful trial of iron therapy, like this patient, or with further iron studies, which will be normal or reveal an elevated transferrin saturation or ferritin. Mild thalassemia does not require treatment. Those with severe thalassemia usually require regular blood transfusions, folic acid supplementation, and in some cases, splenectomy or allogeneic stem cell transplantation.

A 46-year-old woman presents complaining of dry mouth, fatigue, and lymphadenopathy. On review of systems, she also mentions being bothered by dry eyes. Serologic testing is positive for antinuclear antibodies. What is her most likely diagnosis? Sarcoidosis Scleroderma Sjögren syndrome Systemic lupus erythematosus

Sjögren syndrome Sjögren syndrome is an autoimmune disease that causes inflammation and dysfunction of the exocrine glands. The two main glands that are affected by this disease are the lacrimal and salivary glands, which leads to xerophthalmia and xerostomia. Sjögren syndrome may also present with numerous extraglandular manifestations, such as hepatomegaly and splenomegaly, but the two most common are Raynaud's phenomenon and small-vessel vasculitis. Additionally, patients with Sjögren syndrome may present with systemic features such as fatigue, fever, weight loss, and lymphadenopathy. Diagnosis is made based on the presence of xerophthalmia and xerostomia as well as evidence of at least one autoantibody (i.e. antinuclear antibodies, rheumatoid factor, anti-Ro antibodies, and anti-La antibodies). Treatment may be symptomatic for symptoms of exocrine deficiencies or systemic for extraglandular involvement. Often, symptomatic management consists of artificial tears or saliva. Systemic treatment may consist of antimalarial agents or immunosuppressive agents such as chloroquine or methotrexate, respectively. A patient with dry mouth, dry eyes, systemic symptoms, and an autoantibody is most likely to have Sjögren syndrome.

A previously healthy 40-year-old woman who has been staying at a homeless shelter presents to your office with a pruritic rash. She tells you that the itching is worse at night and that the rash is located in between her fingers and at her waistline. Which of the following diagnostic methods can be used to help confirm the diagnosis? Skin biopsy Skin scraping Tzanck testing Wood lamp

Skin scraping Scabies is a pruritic skin infestation caused by the mite Sarcoptes scabiei hominis. It is transmitted by person-to-person contact with the female mite causing the classic skin manifestations, including burrows dug into the epidermis where the eggs are laid. Scabies occurs more commonly in children and sexually active adults. Exposure to crowded conditions is a risk factor. Scabies epidemics are often seen in homeless shelters, nursing homes, or other institutional settings. Diagnosis is generally a clinical one based on history and location of lesions, but diagnostic testing, such as skin scraping, can help to confirm the diagnosis. To perform the skin scraping, a drop of mineral oil is placed over the burrow on the skin and then scraped superficially with a scalpel. The specimen is placed on a slide and viewed under the microscope where mites may be seen, confirming the diagnosis. Treatment involves eradicating the mites along with controlling transmission and decontaminating the living environment. First-line therapy is with the topical scabicidal medication permethrin with repeat treatment in 7 days. Other topical agents that are sometimes used include ivermectin, lindane, or benzyl benzoate. Pruritus may be treated with an oral antihistamine such as diphenhydramine or hydroxyzine. Secondary skin infections may require oral antimicrobial treatment with the choice of agent determined by culture and sensitivity information. Skin biopsy (A) is a procedure that is used to evaluate skin growths to rule out malignancy, to confirm the type of malignancy in a known malignant growth, or when the diagnosis is unknown and multiple etiologies are being considered. Tzanck testing (C), also known as Tzanck smear, is used to help diagnose herpes infection. It involves unroofing a vesicle to excise the base of the lesion and placing the specimen on a microscopic slide. It is then stained and viewed. A Wood lamp (D) is used to help diagnose tinea infections. The affected area is viewed in a darkened room with the Wood lamp and certain specimens will fluoresce under the light, confirming the diagnosis.

A 32-year-old G3P2 woman who is 9 weeks pregnant and has a penicillin allergy presents to her OB/GYN office to discuss recent lab results. On today's visit, her vital signs are a HR of 90 bpm, RR of 19/min, oxygen saturation on room air of 99%, BP of 118/82 mm Hg, T of 98.8°F, and BMI of 27.6 kg/m2. On physical exam, there is no rash or any concerning skin lesions. Her initial labs for her first prenatal visit showed positive rapid plasma reagin. She then had a confirmatory fluorescent treponemal antibody absorption drawn, which was also positive. Which of the following is the most appropriate treatment for this patient? Azithromycin Doxycycline No antibiotic treatment until delivery Skin testing and penicillin desensitization Tetracycline

Skin testing and penicillin desensitization The patient in the above vignette tested positive for syphilis at her prenatal visits. Syphilis is caused by Treponema pallidum. It is routinely checked at the first prenatal visit for all patients and then again repeated at 28-32 weeks and at delivery for all patients who are at high risk of infection. Diagnosis is made through serologic testing. Initial screening is completed with a nontreponemal test such as a rapid plasma reagin (RPR) or a toluidine red unheated serum test (TRUST). There is a high rate of false-positive nontreponemal tests in pregnancy. Therefore, if either of these screening tests are positive, then it is confirmed with a treponemal test such as fluorescent treponemal antibody absorption (FTA-ABS). Syphilis presents differently depending on the stages. Primary syphilis presents with a single painless ulcer. Secondary syphilis presents with a rash that begins in the trunk and spreads outward toward the palms and soles, fever, and malaise. Latent syphilis is typically asymptomatic. Lastly, tertiary syphilis includes infiltrative tumors of the skin, bones, or liver along with cardiovascular disease. Penicillin G is the standard treatment for both pregnant and nonpregnant patients. In patients who are pregnant with a known penicillin allergy, such as the patient in the above vignette, a skin test should be performed to confirm the allergy and then she should be desensitized prior to treating with penicillin G.

A 61-year-old man presents to the clinic for urinary urgency and frequency, nocturia, and dribbling urine consistently after he voids. He endorses a weak stream and a feeling that his bladder won't empty fully. You plan to start treatment for benign prostatic hyperplasia if his physical exam confirms it. Which of the following physical exam findings is most indicative of benign prostatic hyperplasia? A warm, boggy, prostate gland that is tender on palpation Asymmetric enlargement of the prostate gland with unilateral induration Normal size and texture of the prostate gland Smooth, firm, elastic enlargement of the prostate gland

Smooth, firm, elastic enlargement of the prostate gland Smooth, firm enlargement of the prostate gland is characteristic of benign prostatic hyperplasia (BPH). BPH is the most common benign tumor in men, with the incidence estimated at 50% of men by 60 years. Between ages 55-75, the likelihood of developing obstructive voiding symptoms rises significantly. Symptoms include hesitancy and straining to urinate, decreased force and caliber of the urine stream, and the need to urinate twice within 2 hours due to incomplete emptying. Post void dribbling can occur due to the residual urine in the bladder after voiding. Men may also develop irritative bladder symptoms including urgency, frequency, and nocturia. Though prostate size does not correlate with symptoms, a rectal exam will usually show some degree of smooth, firm enlargement. Induration should not be present, as this would be more concerning for malignancy. A distended bladder may be palpated. The decision to get a serum prostate-specific antigen level is controversial, as it is difficult to use this value to determine whether a patient has BPH as opposed to cancer. The American Urological Association symptoms index should be used to gauge severity of BPH and guide treatment. Recurrent urinary tract infections, gross hematuria, bladder stones, or kidney disease merit surgical resection and require referral to urology. Otherwise, mild symptoms can be managed with observation. Moderate to severe symptoms merit medical therapy, for which many options exist. Alpha-blockers will reduce symptoms by improving urine flow at the bladder neck. The 5-alpha reductase inhibitors, interrupts the processing of testosterone and can actually decrease the size of the prostate and improve symptoms. Six months are required to see improvement. Phosphodiesterase-5 inhibitors, like tadalafil, can offer improvement as well. Combination therapy is usually beneficial and tolerated.

You are seeing a patient who presented with microcytosis, microcytic anemia, and abnormal red blood cell morphology with acanthocytes and target cells. You suspect alpha-thalassemia. Which of the following family histories is most commonly associated with this diagnosis? Ashkenazi Jewish descent Black descent Italian descent Southeast Asian descent

Southeast Asian descent A family history of Southeast Asian or Chinese descent is more commonly associated with the alpha-thalassemia syndromes. Four copies of the alpha-globin chain are present in healthy adults. When three copies are present, the patient is said to be a silent carrier of alpha-thalassemia. When two copies are present, the patient has alpha-thalassemia trait, which is a minor form of thalassemia. When only one of globin chain is present, the patient has hemoglobin H disease. These patients can undergo hemolytic crisis during periods of stress or infection. Lab findings will show a mild anemia with an extremely low mean corpuscular volume (MCV). Microcytes, hypochromia, target cells, and acanthocytes can be found on the peripheral blood smear. Hemoglobin electrophoresis and reticulocyte counts will be normal. Patients with alpha-thalassemia trait typically do not require treatment. They are often misdiagnosed as being iron deficient, thus they should be identified as having alpha-thalassemia to avoid unnecessary iron treatment.

A 9-year-old boy with no significant medical history presents to the primary care clinic for a dry cough that he has had for the past 5 weeks. His cough is intermittent without clear triggers. At today's visit, his heart rate is 70 beats/minute, blood pressure is 106/64 mm Hg, respiratory rate is 16 breaths/minute, oxygen saturation is 99%, and temperature is 98.5°F. On his physical exam, his heart sounds are regular in rate and rhythm, with no murmurs. His lungs are clear to auscultation bilaterally, without wheezing, rhonchi, or rales. He is not using accessory muscles of respiration. What is the best next step in evaluating his cough? Computed tomography scan of the chest Culture for Bordetella pertussis Referral for rigid bronchoscopy Spirometry Sweat chloride level

Spirometry Chronic cough is defined as a cough lasting ≥ 4 weeks. The most common causes of chronic cough in children include asthma, gastroesophageal disease, and respiratory tract infections. For pediatric patients presenting with chronic cough, clinicians should seek to rule out underlying pulmonary disease. Initial evaluation of a chronic cough typically begins with a detailed history and physical exam. Next, patients should undergo spirometry to evaluate for asthma or restrictive lung disease. Spirometry should only be obtained if patients are old enough to participate in testing and is usually limited to patients ≥ 6 years of age. If spirometry is unrevealing, a chest X-ray can be ordered to evaluate for possible etiologies, including pneumonia, bronchiectasis, and foreign bodies.

A 57-year-old woman with cirrhosis complains of worsening distension of the abdomen and edema up to her lower legs despite compliance with eating less than 2 grams daily of sodium. She denies abdominal pain, constipation, dyspnea or fevers. On exam, her abdomen is distended and has a notable fluid wave. She has 2+ pitting edema to the level of her ankles. Which medication is the most appropriate next step in the management of this patient? Hydrochlorothiazide Lactulose Omeprazole Spironolactone

Spironolactone This patient has worsened ascites secondary to portal hypertension as a result of cirrhosis. Portal hypertension leads to ascites through a series of hemodynamic changes resulting in the movement of fluid into the peritoneum. The renin-angiotension-aldosterone system is activated leading to sodium retention and additional fluid accumulation. Spironolactone is an aldosterone antagonist, helping to reverse sodium retention, conserve potassium and achieve diuresis. The starting dose is 100 mg daily, with titration up to 400 mg as needed and as tolerated. Side effects of spironolactone are many and may include decreased libido, impotence and gynecomastia. Some patients with ascites secondary to cirrhosis may be managed without diuretic therapy by initiation of a diet including less than 2 grams of sodium per day. Patients with ascites refractory to sodium restricted diets are often placed on combination therapy with both spironolactone and furosemide. Ascites that does not respond to dietary sodium restriction and dual therapy with spironolactone and furosemide is treated with therapeutic paracentesis. Hydrochlorothiazide (A) is a thiazide diuretic, used as first-line therapy in the management of benign essential hypertension. Though this patient may also have hypertension for which hydrochlorothiazide might be effective, it is not the first choice for the management of ascites secondary to portal hypertension as a result of cirrhosis. Lactulose (B), is a synthetic disaccharide containing glucose and fructose, used for the management of hepatic encephalopathy, a complication of cirrhosis. It works by reducing the level of ammonia and other cerebral toxins in the circulation. It does not have a role in reducing ascites. Omeprazole (C) is a proton-pump inhibitor most commonly used to treat gastro-esophageal reflux disease. It may be used in patients with cirrhosis who have developed the complication of peptic ulcer disease, or for such patients who have undergone sclerotherapy for esophageal varices which have developed as a complication of portal hypertension. It is not considered first-line therapy in the management of ascites.

Which of the following is the correct recommended treatment for the severity of heart failure according to New York Heart Association (NYHA) functional class? Anticoagulation for NYHA class I-IV heart failure regardless of ejection fraction Calcium channel blockers for NYHA class III-IV heart failure and ejection fraction < 40% Hydralazine plus nitrates for white patients with NYHA class III-IV heart failure and ejection fraction < 40% Spironolactone for NYHA class III-IV heart failure and ejection fraction < 35%

Spironolactone for NYHA class III-IV heart failure and ejection fraction < 35% Spironolactone for class III-IV heart failure and an ejection fraction less than 35% is recommended by the New York Heart Association (NYHA). Indications for specific medications for heart failure with reduced ejection fraction (HFrEF) are generally based on the patient's functional status as measured by the NYHA functional class. Treatment with angiotensin-converting enzyme inhibitors and beta-blockers are standard medical therapy for all patients with HFrEF regardless of functional stage. Spironolactone, in addition to standard medical therapy, is indicated for more severe heart failure, specifically NYHA class III-IV. It should also be used in class II patients if the ejection fraction is less than or equal to 30%.

A 46-year-old woman with a history of HIV infection presents to the emergency department for 3 weeks of fevers, cough, and fatigue. During the interview, she notes that she has recently returned from travel in India. She is currently taking dolutegravir 50 mg PO once daily for HIV. At today's visit, her heart rate is 78 beats/minute, blood pressure is 128/84 mm Hg, respiratory rate is 16 breaths/minute, oxygen saturation is 98%, and temperature is 100.5°F. On her physical exam, the heart sounds are regular in rate and rhythm, with no murmurs, lungs are clear to auscultation bilaterally, without wheezing, rhonchi, or rales. She is not using accessory muscles of respiration. You obtain a chest X-ray as seen above. Which of the following tests is the most appropriate next step to confirm the suspected diagnosis for this patient? Interferon-gamma release assay Nasopharyngeal swab reverse transcription polymerase chain reaction testing Sputum acid-fast bacilli smear and sputum culture Tuberculin skin test Urine antigen test

Sputum acid-fast bacilli smear and sputum culture Tuberculosis (TB) is an infection caused by the bacterium Mycobacterium tuberculosis, and it is seen worldwide. Risk factors for tuberculosis infection include living in a region with endemic TB, immunosuppression, substance use, malnutrition, and living or working in congregate settings such as hospitals, correctional facilities, nursing homes, and homeless shelters. Patients who are being evaluated for possible TB should be asked about these risk factors. Many patients who are infected can suppress this infectious agent, leading to latent tuberculosis, but some patients may progress to active tuberculosis. Active TB may be asymptomatic or can cause symptoms including fever, sweats, cough, fatigue, or pleuritic chest pain. Less common symptoms include weight loss and hemoptysis. Hilar lymphadenopathy and upper lobe infiltrates are common chest X-ray findings in TB. The manifestations of tuberculosis may be more pronounced in patients who are immunosuppressed. If tuberculosis is suspected, a chest X-ray can provide useful information. To test for active TB, sputum cultures should be obtained for acid-fast bacilli smear and sputum culture. Sputum may also be sent for nucleic acid amplification testing (NAAT). Treatment regimens include a combination of isoniazid, rifampin, pyrazinamide, and ethambutol.

A 74-year-old woman presents with complaints of fever, productive cough with bloody sputum, shortness of breath, and headache. These symptoms developed and worsened drastically over the past 3 days. She recently recovered from an influenza infection 1 week ago. Her medical history otherwise includes only well-controlled hypertension. Vital signs on presentation are as follows: T 39°C, HR 106, BP 110/75, RR 30, oxygen sat 95% RA. A chest radiograph is obtained and a subsequent CT scan of the chest demonstrates multiple cavitary lung lesions. Which of the following organisms is most likely responsible for this patient's presentation? Clostridum perfringens Escherichia coli Mycobacterium tuberculosis Staphylococcus aureus

Staphylococcus aureus This patient's presentation of pneumonia with multiple cavitary lesions on imaging is consistent with a post-viral secondary necrotizing pneumonia. The most common organism in necrotizing pneumonia, particularly after a viral upper respiratory infection, is S. aureus. Necrotizing pneumonia is known to be caused by a specific S. aureus strain that produces Panton-Valentine Leukocidin (PVL). Often, this infection and the ensuing pneumonia that develops, is preceded by an influenza infection. Typically this S. aureus strain is also methicillin resistant. A CT of the chest with contrast is useful in diagnosis, and empiric therapy should be initiated promptly (vancomycin or linezolid, piperacillin/tazobactam). Surgical intervention may be necessary if complications develop - such as septic shock, gross hemoptysis and empyema. The following should be considered in the differential diagnosis of pulmonary cavitation: necrotizing pneumonia, lung abscess, septic pulmonary embolism, fungal/mycobacterial infection, vasculitis, primary/metastatic tumor, rheumatoid nodules, congenital cysts. Defining characteristics of necrotizing pneumonia include: preceding influenza infection, rapid onset and progressive symptom worsening, decreased WBC count, airway hemorrhages, respiratory failure, necrotic destruction of lung parenchyma, high mortality rate. A preceding viral infection brings a large number of immune cells to the lung tissue, such that when secondary bacterial infection strikes, there is a catastrophic activation and destruction of immune mediators that damage lung tissue and lead to necrotizing pneumonia.

A 65-year-old man presents to his clinician requesting a letter of medical optimization for an elective left total knee arthroplasty scheduled in 4 weeks. He reports no symptoms today. His medical history includes hypertension, obesity, and severe osteoarthritis of his bilateral knees. His current medications include hydrochlorothiazide and metoprolol succinate. His blood pressure is 142/86 mm Hg, heart rate is 64 bpm, respirations are 18/min, SpO2 is 100% on room air, and temperature is 98.6°F. On cardiac exam, rate and rhythm are regular, and no new murmurs are heard. Breath sounds are normal, without rales, rhonchi, or wheezing. Pulses of the bilateral upper and lower extremities are 2+ and equal. No lower extremity edema is noted. ECG shows a normal sinus rhythm, ventricular rate of 64 bpm, left ventricular hypertrophy with repolarization abnormality, PR interval of 146 ms, and QRS duration of 90 ms. An echocardiogram reveals left ventricular systolic dysfunction and a left ventricular ejection fraction of 40%. Which of the following is the most appropriate next step in the management of this patient's condition? Reassure and follow up in 3 months Refer for implantable cardioverter-defibrillator placement Start furosemide Start lisinopril Start mineralocorticoid receptor antagonist

Start lisinopril This man has asymptomatic left ventricular systolic dysfunction, which presents as signs and symptoms of depressed left ventricular systolic function without heart failure, such as lower extremity edema, dyspnea, orthopnea, fatigue, and exercise intolerance. It is important to identify and treat these patients as they are at an increased risk for developing clinically significant symptomatic heart failure and death. Causes may include cardiomyopathy, coronary heart disease, valvular disease, myocarditis, peripartum cardiomyopathy, hypertension, and cardiotoxic drugs. Echocardiography is the main diagnostic tool when evaluating for left ventricular systolic dysfunction. Other studies are not required for diagnosis. However, an ECG may show nonspecific abnormalities or left ventricular hypertrophy prompting further workup. Chest X-ray will yield useful information about cardiac size and shape and the pulmonary vasculature. Laboratory studies have no place in determining this diagnosis. B-type natriuretic peptide is not sensitive but may be used to monitor the course of illness. Treatment is dependent on multiple factors, including a history of myocardial infarction (MI), left ventricular ejection fraction (LVEF), and other comorbidities. In this patient's case, starting an ACE inhibitor such as lisinopril would be appropriate. Angiotensin-converting enzyme (ACE) inhibitors are beneficial in the prevention and treatment of heart failure because they reduce afterload and preload, prevent angiotensin II from triggering harmful cardiac remodeling, and reduce sympathetic activation. Reductions in afterload increase the ejection fraction by enhancing the ventricular stroke volume. In patients without a history of MI, an ACE inhibitor or angiotensin II receptor blocker (ARB) is recommended as initial treatment. If the patient has had a prior MI, initial treatment with a beta-blocker is recommended prior to starting ACE inhibitor or ARB therapy. Reassuring the patient with follow-up scheduled in 3 months (A) would be appropriate for patients without prior history of MI and a current LVEF > 40%. This patient has an LVEF < 40%, and thus treatment is recommended. Referral for implantable cardioverter-defibrillator (ICD) placement (B) is appropriate for patients who have a history of MI and are taking recommended medication while still meeting one of the following criteria: left ventricular ejection fraction ≤ 35%, 40 days out from MI, or the patient has nonsustained or inducible ventricular tachycardia and an LVEF < 40%. Furosemide (C) is a loop diuretic used to treat symptoms of heart failure such as swelling of the lower extremities or pulmonary edema. Since this patient is asymptomatic, furosemide is not indicated. Mineralocorticoid receptor antagonists (E) may be appropriate for a patient post-MI who has been titrated on a beta-blocker and an ACE inhibitor or ARB. Other criteria for adding a mineralocorticoid receptor antagonist include normal serum potassium level, normal serum creatinine level, LVEF < 40%, or history of diabetes mellitus. The patient must also be able to be monitored for changes in kidney function and serum potassium.

A 35-year-old woman presents to the clinic to discuss her laboratory results. The patient initially presented with amenorrhea for the past 4 months, galactorrhea, and headaches. Vital signs include a heart rate of 80 bpm, blood pressure of 120/80 mm Hg, respiratory rate of 20/minute, oxygen saturation of 98% on room air, and temperature of 98.6°F. Physical examination reveals milky nipple discharge bilaterally without palpable breast masses. Her urine pregnancy test is negative. The patient's prolactin level is 756 ng/mL. You decide to start the patient on medication to treat the suspected condition. Which of the following is the mechanism of action of the first-line medication used to treat the suspected condition? Inhibition of gonadotropin-releasing hormone Inhibition of hypothalamic dopaminergic neurons Stimulation of adrenal mineralocorticoid receptors Stimulation of gonadotropin-releasing hormone Stimulation of pituitary dopamine receptors

Stimulation of pituitary dopamine receptors Pituitary adenomas are benign tumors of the anterior pituitary gland. They are further classified based on the size of the tumor and the underlying cell type. Lactotroph (prolactin-secreting) adenomas are the most common cell type, making up 50% of pituitary adenomas. Microadenomas are < 1 cm in length, while macroadenomas are > 1 cm in length. The clinical manifestations of pituitary adenomas may include the symptoms related to the sellar mass and symptoms related to pituitary hormone abnormalities. Patients can experience headaches due to the space-occupying nature of the tumor. The clinical manifestations of lactotroph (prolactin-secreting) adenomas include symptoms of hyperprolactinemia and hypogonadism. Hyperprolactinemia causes galactorrhea (abnormal milky nipple discharge) and results in hypogonadism. The clinical manifestations of hypogonadism in women include amenorrhea, infertility, vaginal dryness, and decreased bone mineral density. Hypogonadism in men results in infertility and symptoms of low testosterone. Lactotroph pituitary adenomas may initially be suspected based on symptoms of hyperprolactinemia or hypogonadism or when a pituitary adenoma is incidentally identified on magnetic resonance imaging of the brain performed for other reasons. An elevated prolactin serum concentration confirms hyperprolactinemia and is supportive of a lactotroph adenoma. Magnetic resonance imaging with contrast of the pituitary is the best imaging modality to evaluate for the presence of a pituitary adenoma. Patients diagnosed with pituitary adenomas should undergo an evaluation of the hormones produced by the anterior pituitary gland. The initial treatment of lactotroph pituitary adenomas, including microadenomas and macroadenomas, consists of dopamine agonists, such as cabergoline or bromocriptine. These medications stimulate pituitary dopamine receptors. Cabergoline is the preferred dopamine agonist because it is more efficacious and better tolerated than bromocriptine. Patients who do not improve with dopamine agonists despite maximizing the dosage can consider transsphenoidal surgery, ovulation induction with clomiphene citrate (for women wishing to become pregnant), or hormonal replacement (estradiol and progesterone for women or testosterone for men).

A 47-year-old man with chronic low back pain presents to the emergency department with epigastric pain for 3 weeks. The pain is burning, without radiation, and occurs 2-3 hours after eating. Vital signs are normal. He takes ibuprofen and naproxen for his back pain. What management is indicated? Admit for endoscopy Start esomeprazole and refer for outpatient evaluation Start ranitidine and refer for outpatient evaluation Stop ibuprofen and naproxen and refer for outpatient evaluation

Stop ibuprofen and naproxen and refer for outpatient evaluation The patient presents with clinical gastritis, likely secondary to the use of nonsteroidal anti-inflammatory drugs (NSAIDs). Technically, gastritis is a histologic diagnosis indicating inflammation of the gastric mucosa and can only be definitively made after endoscopy and biopsy. Clinicians, however, refer to the symptoms of dyspepsia (pain in the upper abdomen that presents as bloating or heartburn) as gastritis. The most common cause of gastritis is Helicobacter pylori infection, but it also commonly results from medications, including salicylates and NSAIDs (ibuprofen, naproxen). Acute gastritis typically presents with abdominal pain located in the epigastric area that is burning in nature and can also have bloating or nausea associated with it. Treatment should begin with removal of any possible inciting agents including alcohol, smoking, steroids, or NSAIDs.

A 35-year-old woman comes to the urgent care clinic complaining of a 3-week history of difficulty seeing, blurred vision, eye pain, and cough. She describes the cough as being dry and nonproductive. She has no past medical history and takes no medications. Her blood pressure is 140/85 mm Hg and her temperature is 38.1°C (100.5°F). Ophthalmologic examination shows uveitis. Chest X-ray shows bilateral hilar adenopathy. Which of the following additional findings would also most likely be found in this patient? Ferruginous bodies Increased sweat chloride Positive PPD skin test Subcutaneous nodules

Subcutaneous nodules Uveitis, bilateral hilar adenopathy, and a dry cough most likely indicate sarcoidosis. Sarcoidosis is granulomatous disease that can form nodules in multiple organs. When these nodules form in the skin, they are referred to as subcutaneous nodules (ie, erythema nodosum). Patients with sarcoidosis typically present with fatigue, weight loss, arthritis, dry eyes, blurry vision, and respiratory symptoms (eg, cough, dyspnea). Management usually involves the use of corticosteroids (eg, prednisone).

Which of the following medications should be used in the treatment of cluster headache if high-flow oxygen is ineffective? Aspirin Caffeine Sumatriptan Verapamil

Sumatriptan Sumatriptan is a selective serotonin receptor agonist used as first-line pharmacotherapy to treat migraine headaches with or without aura and cluster headaches in addition to oxygen therapy or when oxygen treatment is ineffective. Cluster headache is characterized by severe orbital, supraorbital, or temporal head pain accompanied by unilateral sweating, conjunctival injection, lacrimation, miosis, ptosis, rhinorrhea, agitation, or restlessness. It most commonly affects men and symptoms may be short-lived, but can attack up to several times a day. The mainstay of treatment is the deliverance of oxygen via a nonrebreathing facial mask. In addition to this, using pharmacotherapy such as a triptan, particularly sumatriptan, has been shown to increase the rate of recovery. Sumatriptan can be used orally, intranasally, or through an injection. In the case of a cluster headache, a subcutaneous injection has been found to be the most effective.

A 15-year-old boy with severe asthma is seen in the clinic for a follow-up. He has been hospitalized three times during the past three months due to asthma exacerbations. He required prolonged steroid use to control his asthma. You note abnormal findings on examination that make you suspect Cushing's syndrome. Which exam finding is associated with Cushing's syndrome? Distal atrophy and weakness Dry skin Hypotension Supraclavicular fat pads

Supraclavicular fat pads The patient has received pharmacologic doses of steroids to control his asthma, which makes him at risk for iatrogenic Cushing's syndrome. The signs and symptoms of Cushing's syndrome result from chronic exposure to excess glucocorticoid. There is a large spectrum of manifestations, depending on duration and intensity of excess steroid production. The most common feature of patients with Cushing's syndrome is progressive central obesity that usually involves the face, neck, trunk, and abdomen. The extremities are usually spared and may be wasted. Enlarged fat pads that fill the supraclavicular fossae and obscure the clavicles are one of the most specific signs. The fat pads make the neck appear thick and shortened.

A 59-year-old man with a history of hypertension, dyslipidemia, and chronic low back pain presents for evaluation of sudden-onset abdominal pain that radiates to his right shoulder. His current medication regimen includes enalapril, propranolol, aspirin, ibuprofen, and atorvastatin. He describes intense, diffuse abdominal pain that began 2 hours ago and has since decreased in severity. Vital signs are HR 133 bpm, RR 20/min, BP 102/82 mm Hg, T 95.7°F, and SpO2 95% on room air. Physical exam reveals an ill-appearing man with a weak radial pulse and cool extremities. Abdominal palpation demonstrates marked abdominal rigidity and right lower quadrant tenderness. Cardiac biomarkers are within normal limits, and initial ECG monitoring reveals left ventricular hypertrophy without signs of ischemia or infarct. Abdominal radiograph images are shown above. Which of the following is the best next step in intervention following initial stabilization? Barium swallow study Maintenance intravenous fluids and bowel rest Nasogastric tube placement and serial abdominal exams Surgical consultation Upper endoscopy

Surgical consultation Peptic ulcer disease is a gastrointestinal disorder most commonly caused by nonsteroidal anti-inflammatory drug use and Helicobacter pylori infection that results in ulcerative lesions within the stomach and duodenum. Free perforation may occur when a lesion ulcerates through the gastrointestinal wall. Other complications include gastric outlet obstruction, penetration, and bleeding. Patients with comorbid disease, advanced age, and more severe physiologic insult (e.g., hypotension, metabolic acidosis, acute kidney injury, hypoalbuminemia) are more likely to have poor outcomes from complicated peptic ulcer disease. The clinical manifestations of ulcer perforation can be described in three phases. The initial phase occurs within 2 hours of onset and is characterized by sudden, severe, diffuse abdominal pain that may radiate to the right shoulder or bilateral shoulders. Physical examination in this phase reveals tachycardia, a weak pulse, cool extremities, a low temperature, and progressive abdominal rigidity. The second phase presents between 2 and 12 hours after onset with regression of abdominal pain and marked increasing abdominal rigidity. Normal dullness to percussion over the liver may be obscured by peritoneal free air, and palpation may demonstrate tenderness of the pelvic peritoneum and right lower quadrant. Abdominal distention develops in the third phase of clinical manifestations more than 12 hours after initial onset. Hypovolemia and hyperthermia are found on clinical examination during this phase. A three-way abdominal film series, including an upright chest X-ray and anterior-posterior and lateral abdominal X-rays, with free air in the abdomen strongly indicates a perforated viscus. Abdominal ultrasound or CT imaging may be used to detect small amounts of free air or fluid if plain film radiographic findings are equivalent. Any patient with a suspected perforated ulcer should be stabilized urgently with supportive care, including a nothing by mouth diet, fluid resuscitation, and a proton pump inhibitor and antibiotics given intravenously. Continued abdominal contamination and hemorrhage from a perforated peptic ulcer necessitates immediate surgical consultation for urgent surgical intervention. The prognosis for patients with perforated peptic ulcers worsens after 6 hours from symptom onset, and thus timely evaluation and treatment are essential.

A 30-year-old woman with a history of Crohn disease presents to the emergency department with a report of rectal pain for about 2 weeks. She states she has pain with sitting and bowel movements and has noticed some discharge from her rectal area. She reports no fevers. Her vital signs show a heart rate of 80 bpm, blood pressure 119/75 mm Hg, respiratory rate 17 breaths/min, and SpO2 of 98% on room air. On physical exam, her abdomen is soft with mild tenderness to palpation in the suprapubic region. On rectal exam, there is scant purulent drainage with a small pustule noted in the perianal region. There is a palpable cord felt from the external surface to the anal canal. Imaging and labs are pending. What is the most appropriate management for the suspected diagnosis? Empiric antibiotics Incision and drainage Sitz baths Stool test for ova and parasites Surgical referral for evaluation

Surgical referral for evaluation Perianal disease can affect up to one-third of patients with Crohn disease and may include hemorrhoidal tags, anal stricture, perirectal abscess, and anorectal fistula. Anorectal fistula develops after an anal abscess ruptures or is drained. It forms via a tract connecting the abscess to the perirectal skin. Patients will often present with an anal abscess that is not healing and reports of intermittent rectal pain, especially with defecation, as well as with sitting or other physical activity. On physical exam, there may be purulent drainage or a pustule-type lesion in the perianal area, and excoriation and inflammation of the external skin may be present. A cord from the external opening to the anal canal may also be palpable. The fistula can often be probed to determine its depth, but care should be taken to avoid perforation. Evaluation includes MRI, endosonography, or both to determine the exact anatomy of the fistula, followed by surgical referral for evaluation and further treatment as necessary next steps. As initial treatment, a silk tie may be placed through the fistula tract to allow for inflammation to subside while still allowing drainage. After the inflammation has subsided, a fistulotomy can be performed in simple fistulas or an anorectal flap with drainage catheter in more complex tracts or those located above the anal sphincter.

A 3-year-old boy presents with a nontender left scrotal mass that has been present since birth. Scrotal mass often increases in size when he strains or sneezes. On physical exam, a soft, nontender mass that transilluminates is noted. Doppler ultrasonography shows a cystic fluid collection in the left scrotum. Which of the following is the most effective management for this patient? Antibiotic therapy Incision and drainage Manual detorsion Supportive treatment Surgical repair

Surgical repair A hydrocele is a benign fluid-filled mass that typically forms between the parietal and visceral layers of the tunica vaginalis. It can be classified as a communicating or noncommunicating hydrocele. A communicating hydrocele typically results from a patent processus vaginalis that communicates with the parietal peritoneum, thereby allowing peritoneal fluid to pass into the scrotum. The fluid around the testis in a noncommunicating hydrocele is from the mesothelial lining of the tunica vaginalis, as there is no communication between the scrotum and the peritoneum. Hydroceles are commonly seen in newborns, with a spontaneous resolution by the first or second year of life. Noncommunicating hydroceles that are seen in older children and adolescents may be idiopathic or secondary to epididymitis, orchitis, testicular torsion, torsion of the appendix testis, trauma, or tumor. As a result, these conditions must be excluded in the presence of a hydrocele in older children and adolescents. Patients with hydroceles present with a scrotal mass that may increase in size during the day or by Valsalva (as often seen in communicating hydroceles). A noncommunicating hydrocele does not change in size. A physical exam may reveal a soft, nontender scrotal fullness that transilluminates. Diagnosis of a hydrocele can be made clinically. Doppler ultrasonography is used to exclude other causes of hydroceles but may also show a cystic fluid collection in the affected scrotum. Supportive treatment of a communicating hydrocele is indicated for neonates and children younger than 2 years of age. If communicating hydroceles do not resolve by age 2 years, or if noncommunicating hydroceles become symptomatic, a surgical repair must be performed to decrease the risk of developing incarcerated inguinal hernias.

A 65-year-old man with significant tobacco use presents with general malaise, unexplained weight loss, and occasional flank pain. Computed tomography of the abdomen shows a localized renal mass in the left kidney. What is the only curative treatment for this condition? Chemotherapy Radiofrequency ablation Stem cell transplantation Surgical resection

Surgical resection Renal cell carcinoma is the most common type of kidney cancer in adults. Cigarette smoking doubles the risk of developing renal cell carcinoma. Other risk factors include obesity, occupational exposure to certain chemicals and hypertension. Diagnosis is commonly delayed as symptoms are nonspecific and insidious in development. Few patients present with the classic triad of flank pain, hematuria, and flank mass. Other more common signs and symptoms include weight loss, fever, hypertension, hypercalcemia, night sweats, malaise and left-sided varicocele in men. Helpful laboratory tests include urinalysis with urine cytology to evaluate for abnormal cells, complete blood count to assess for the presence of anemia or infection, and an electrolyte panel with renal profile to assess kidney function. Imaging studies used to diagnose and stage renal masses include urography, computed tomography, positron emission tomography scan, ultrasonography, and magnetic resonance imaging. Surgical resection remains the only known curative treatment for localized renal cell carcinoma, but can also be used to improve outcomes or for palliative care. Therapy using immunomodulatory agents is standard in patients with metastatic disease. Chemotherapy is only used occasionally and is only effective for certain types of tumors.

A 45-year-old woman presents for evaluation of right upper quadrant pain. The pain was initially intermittent but has become more constant without radiation. She does not have fever. Her examination is notable for tenderness in the right upper quadrant without a Murphy sign. She undergoes a right upper quadrant ultrasound as seen above. Which of the following is the most likely diagnosis? Cholangitis Cholecystitis Choledocholithiasis Symptomatic cholelithiasis

Symptomatic cholelithiasis The ultrasound image displays a gallstone within the gallbladder without signs of pericholecystic fluid or wall thickening consistent with biliary colic (symptomatic cholelithiasis). Although the name includes colic, the pain may be a steady constant pain. The physical examination reveals tenderness in the right upper quadrant classically without rebound, guarding or a Murphy's sign

A 32-year-old woman presents to your office with a complaint of productive cough that started 6 days ago. She reports occasional wheezing, chest wall tenderness, and has been afebrile since the onset of symptoms. Her husband had similar symptoms and was prescribed azithromycin by his primary care provider. Which of the following is the most appropriate next step in management? Order a chest X-ray Prescribe a course of azithromycin Send a sputum sample for culture Symptomatic treatment

Symptomatic treatment Acute bronchitis is an inflammation of the bronchi caused by upper respiratory infection. It is a self-limiting condition that is most commonly of viral etiology. Patients with acute bronchitis generally have few systemic symptoms. The most common clinical presentation is a cough lasting up to 10-20 days. The cough may be productive and patients often mistakenly attribute purulent sputum in acute bronchitis with a bacterial infection. The symptoms of acute bronchitis are less severe than those of influenza or pneumonia and help determine diagnosis and course of treatment. Symptomatic treatment with acetaminophen or NSAIDs and other supportive care measures are recommended as initial management of patients with acute bronchitis. Chest X-ray (A) is used to rule out pneumonia and should be ordered when a patient has abnormal vital signs including temperature greater than 38°C, pulse greater than 100/min, respirations greater than 24/min, or age greater than 75 years with more severe clinical symptoms. Antibiotics (B) are usually not indicated due to the viral nature of acute bronchitis, but are often prescribed. Acute bronchitis is one of the most common reasons for antibiotic abuse. Sending a sputum sample for culture (C) is not recommended for patients with acute bronchitis. Culture may be indicated in cases where pertussis is suspected.

A 29-year-old woman is concerned with a rash. On inspection, you appreciate a central facial rash. You notice that the nasolabial folds are not affected. You also detect alopecia and lip crusting. The neck, chest and trunk are void of cutaneous abnormalities. The dorsal digits, and other extensor surfaces, are not affected. Which of the following is the most likely diagnosis? Dermatomyositis Psoriasis Rosacea Systemic lupus erythematosus

Systemic lupus erythematosus Rashes can be difficult to evaluate, especially when considering dermatologic versus rheumatologic conditions. A crossover occurs in four main conditions: rosacea, systemic lupus erythematosus (SLE), psoriasis and dermatomyositis. SLE is associated with a red, butterfly-shaped, malar rash, which is located on the central face but not in the nasolabial folds. In comparison, rosacea's malar rash affects the central face and nasolabial folds. Furthermore, systemic symptoms do not occur in rosacea. Facial rash can also present with dermatomyositis, however, this central facial rash is not termed malar, but rather heliotrope, and appears as a waxing-waning violet-colored (violaceous) rash that mainly affects the eyelids but also affects the nasolabial folds. To add to this difficulty, SLE is associated with another rash called discoid rash. This is described as raised chronic inflammatory sores with scarring and scaling, which occur on the face, ears and scalp, and can easily be confused with psoriasis. Patients with SLE may have a malar rash, or discoid rash, or both.

One of your patients has just been diagnosed with a mixed connective tissue disorder, but she does not know which one. However, she remembers hearing the rheumatologist talk about "too much collagen". Which one of the following autoimmune diseases does this patient most likely have? Polymyositis Sjögren syndrome Systemic lupus erythematosus Systemic sclerosis

Systemic sclerosis Progressive systemic sclerosis (SS), sometimes referred to as scleroderma, is a chronic progressive connective tissue disease, marked by excessive skin and organ-epithelial collagen production and deposition. In other words, multisystem fibrosis is present and the body becomes "scarred". It is an autoimmune disease of unknown origin, but is associated more with certain factors, such as exposure to silica, organic solvents, aliphatic hydrocarbons, epoxies, pesticides, vibratory tools, certain cosmetics, cocaine and drugs such as bleomycin and carbidopa. It is a rare disease. Signs and symptoms include fatigue, weakness, inflammatory myopathy, myalgias, arthralgias, dependent edema, joint contracture, pruritic thick skin, dysphagia, vomiting, abdominal cramping, diarrhea, dyspnea, nonproductive cough, atypical chest pain and palpitations. The main presenting manifestations are myalgias, arthralgias and Raynaud's phenomenon, a cold-induced, distal digital arteriolar contraction which leads to pallor, ischemia and ulceration. Skin findings predominate on physical exam, and include face, neck, trunk and proximal arm taut thick skin, skin atrophy, areas of hyper- and hypopigmentation, telangiectasias and Raynaud's findings. Lab abnormalities include increased ESR and CRP, thrombocytopenia and microangiopathic hemolytic anemia, hypergammaglobulinemia and increased creatinine phosphokinase. Abnormal renal indices are present if there is kidney involvement. Antinuclear antibodies are positive and anticentromere antibodies are negative in most patients.

A 55-year-old man presents to the office interested in obtaining medication for his erectile dysfunction. He states he has trouble maintaining and keeping erections 75% of the time and has no other medical problems. His main concern is that he is single and sometimes not sure when the opportunity will present to have sex, so he wants the medication with the longest duration. Which of the following medications should you recommend? Sildenafil Tadalafil Vardenafil Yohimbine

Tadalafil Tadalafil has a duration of 24-72 hours. Phosphodiesterase type 5 inhibitors are first-line therapy for erectile dysfunction (ED) and are effective in most cases, including antidepressant-induced and diabetes-related ED. In 1998, sildenafil was the first PDE5 inhibitor to be approved by the FDA, followed by vardenafil and tadalafil in 2003. The medications differ in absorption, potential effective time interval, and side effects. Sildenafil (A) and vardenafil (C) have a short duration of action (approximately 3 hours). Yohimbine (D) antagonizes alpha-2 adrenergic receptors. In one systematic review, yohimbine improved self-reported sexual function and penile rigidity compared with placebo. It has not been clinically researched for ED in the last decade. Yohimbine has significant adverse effects, including elevation of blood pressure and heart rate, increased motor activity, nervousness, irritability, and tremor. It has a half-life of 36 minutes.

A 53-year-old man is brought into the ED confused and disoriented. His partner reports he was feeling weak and nauseous and had multiple episodes of vomiting. On further questioning, he also reports an increase in hiccuping, a metallic taste in his mouth, and general pruritus. His blood pressure is 160/90, and he is tachypneic at 30 breaths per minute. Heart rate is 94 beats per minute. He is afebrile. On exam, dependent edema is noted. Multiple screening tests are sent, and they are notable for an elevated serum creatinine level of 2.1 mg/dL with an estimated glomerular filtration rate of 50 mL/min and blood glucose of 280 mg/dL on metabolic panel. His urinalysis is positive for protein. What is the most likely underlying cause for his presenting symptoms? Diabetes mellitus Nephrolithiasis Nephrotic syndrome Polycystic kidney disease

The most common causes of chronic kidney disease are diabetes mellitus (type 2) and hypertension. In addition to being the leading cause of chronic kidney disease, diabetes is also the most common cause of end-stage kidney disease. The complicated pathophysiology of diabetes leads to cellular injury and a change in tubular function, which stresses glomeruli, leading to sclerosis and kidney insufficiency due to the chronic reduction in glomerular filtration. Patients with diagnosed diabetes mellitus should be routinely monitored for kidney function because of the risk for progression of kidney disease. However, many individuals with type 2 diabetes are asymptomatic and, without routine primary care and annual monitoring, may have diabetes and diabetic kidney disease for years before it is diagnosed. Even prediabetes and hyperglycemia can be associated with albuminuria and decreased glomerular filtration rate (GFR). Therefore, while chronic kidney disease is defined as > 3 months of GFR < 60 mL/min or kidney damage, many patients with type 2 diabetes can meet the time duration of the definition at the time of their diabetes diagnosis. Patients with kidney insufficiency leading to chronic kidney disease may be asymptomatic but, with worsening disease, will experience signs of uremia such as pruritus, nausea, vomiting, hiccups, and a metallic taste in the mouth. Uremic pericarditis, delirium, seizures, and coma are signs of more severe disease. Patients may also present with uremic frost, a manifestation of severe azotemia in which evaporating sweat leaves behind tiny yellow-white urea crystals on the skin, giving it a frosted appearance. Diagnosis is typically clinical in the setting of diabetes and demonstrated decrease of kidney function. Management of diabetic kidney disease is multipronged, focusing on blood pressure control through angiotensin inhibition (angiotensin-converting enzyme inhibitors or angiotensin II receptor blockers), glycemic control, lipid control, and lifestyle modifications, including healthy eating, regular exercise and weight loss, and smoking cessation if applicable. Unfortunately, many individuals with chronic diabetic kidney disease will ultimately develop end-stage kidney disease and are at increased risk of death from cardiovascular events. However, a comprehensive management approach can significantly stabilize kidney function and decrease these risks.

A 34-year-old man presents with a deep laceration to the left leg after falling off a motorcycle. The wound is contaminated with rocks and dirt. He states that he has not received a tetanus shot since completing vaccinations as a child. After irrigation and repair, which of the following should be administered? No tetanus prophylaxis needed Tdap Tdap and tetanus immune globulin Tetanus immune globulin

Tdap This patient presents with a dirty wound and no tetanus booster in the last 5 years and thus requires a tetanus booster during this presentation. Tetanus is a toxin-mediated disease that is characterized by uncontrollable skeletal muscle spasms. It can cause hypoventilation, hypoxia, and death if the toxin affects the muscles of respiration. Tetanus is a relatively rare disease particularly in resource-rich countries where vaccination programs have been successful. Primary immunization confers protective antibodies to nearly 100% of patients. Immunity wanes between 5 and 10 years after completion of the initial vaccination series and so patients should have a booster shot every 10 years. It typically affects patients who have sustained a deep, penetrating wound. In adult patients with a history of a primary series in the past, any wound that is not a clean/minor wound should be given a tetanus shot (Tdap) if their last booster was > 5 years ago. If the patient has a history of not completing a primary series, they should be given a Tdap regardless of the wound depth or size in addition to tetanus immune globulin if the wound is high risk.

A 35-year-old man presents to the ED with sudden-onset dyspnea and right-sided chest pain after being involved in a motor vehicle collision. He has no significant medical history and takes no medications. His blood pressure is 90/65 mm Hg, and the pulse rate is 105 bpm. On the physical exam, he displays evidence of labored breathing and accessory muscle use. Diminished breath sounds and absent tactile fremitus with hyperresonance to percussion on the right side are noted. His chest X-ray is shown above. What is the most likely diagnosis? Cardiac tamponade Esophageal rupture Pleural effusion Pulmonary embolism Tension pneumothorax

Tension pneumothorax The patient above is most likely experiencing a tension pneumothorax. Pneumothorax should be suspected in all patients who present with acute dyspnea and chest pain. Risk factors for a tension pneumothorax include trauma (e.g., rib fractures), mechanical ventilation, and underlying lung disease. Patients often report pain on one side of their chest, which correlates with the location of the pneumothorax. A tension pneumothorax arises when air builds up in the pleural space and compromises venous return. This can lead to hypotension, tachycardia, and shortness of breath. Characteristic physical findings include diminished breath sounds, decreased or absent tactile fremitus, and hyperresonance to percussion. Evidence of labored breathing, accessory muscle use, or both may also be seen. Hemodynamic instability (tachycardia, hypotension) suggests a tension pneumothorax and impending cardiopulmonary collapse. The diagnosis of pneumothorax is established by demonstrating a white visceral pleural line on the chest radiograph (as shown in the chest X-ray above). Treatment depends on severity and cause. Patients with a tension pneumothorax often need a chest tube placed, which uses suction to help the lung expand.

A young woman presents with a bilateral headache. She describes it as a band-like pressure from her forehead to neck, feeling as if her head is in a vice. She reports no prodromal symptoms, nausea, or photophobia. Cervical spine range-of-motion and skull palpation do not reproduce the headache. Which of the following is the most likely diagnosis? Cluster headache Migraine without aura Occipital headache Tension-type headache

Tension-type headache Tension-type headache (TTHA) is the most common type of recurring headache other than migraine and the most common type overall. Tension headaches present as a bilateral headache with pain in the frontal and occipital regions in a bandlike distribution. Headaches are exacerbated by stress, fatigue, glare, or noise. It also often involves contraction of the scalp and posterior neck muscles, with a normal neurologic exam. Treatment is with aspirin, NSAIDs, and ergotamines.

A 55-year-old woman presents to your office with complaints of fatigue, dry skin, constipation and weight gain. Physical exam results include a heart rate of 58 bpm and diminished deep tendon reflexes. Laboratory testing reveals an elevated TSH. Which of the following is the most appropriate next step in management? Begin treating patient with a beta-blocker Begin treating patient with levothyroxine Test serum free T3 Test serum free T4

Test serum free T4 When a patient presents with symptoms of hypothyroidism, serum TSH is the initial test indicated. An elevated TSH alone does not provide enough information to diagnose hypothyroidism. Other pathologies may cause elevated TSH such as resistance to TSH or thyroid hormone, a TSH-secreting pituitary adenoma, or primary adrenal insufficiency. When TSH is elevated, serum free T4 should then be tested. High serum TSH with low serum free T4 indicates primary hypothyroidism. Clinical signs and symptoms of hypothyroidism can vary greatly, and diagnosis is reliant on laboratory testing. Treatment for primary hypothyroidism is replacement therapy with T4.

A 36-year-old man presents to the emergency department after accidentally puncturing his palm with a rusty nail. He has received two prior doses of a tetanus toxoid-containing vaccinations. Which of the following regimens should be given to this patient? No prophylaxis needed Tetanus immune globulin only Tetanus toxoid and tetanus immune globulin Tetanus toxoid only

Tetanus toxoid and tetanus immune globulin Tetanus toxoid and tetanus immune globulin should both be given to a patient who sustains a dirty puncture wound and has received less than three doses of the tetanus vaccination in the past. Tetanus is a nervous system disorder caused by Clostridium tetani, an obligate anaerobe found in soil. When spores enter the body through openings of the skin, toxin is released causing muscle spasms and muscle rigidity. More than half of patients present with symptoms of trismus, or lockjaw. When tetanus is suspected, the patient's tetanus vaccination history must be revealed. If a patient presents with a clean and minor wound, a tetanus toxoid-containing vaccination such as the tetanus-diphtheria-acellular pertussis vaccine (Tdap) or tetanus-diphtheria vaccine (Td) is given only if a patient has gotten less than three doses of the vaccine in their lifetime or if their last dose of the vaccine was given over 10 years ago. For wounds contaminated with dirt, soil, feces, or saliva; puncture wounds; avulsions; or are a result of missiles, crushing, burns, or frostbite, they are considered to be at high risk for tetanus and therefore prophylaxis is warranted immediately after injury if indicated (> 5 years since last vaccine). In a patient with an unknown vaccination history or if they have had fewer than three doses of the vaccination in their past, they must receive the tetanus toxoid in either the Tdap or Td form and the tetanus immune globulin. Giving both ensures adequate protective levels against the toxin.

A 40-year-old woman presents to the emergency department for confusion and headache. She has a past medical history of seasonal allergies and hypothyroidism. Her pulse is 78 beats/minute, respiratory rate is 18 breaths/minute, blood pressure is 141/89 mm Hg, and the temperature is 101.1°F. Physical exam is remarkable for scattered petechiae. Laboratory findings include hemoglobin of 10 g/dL, platelet count of 25,000/microL, creatinine of 1.2 mg/dL, and normal fibrinogen. A peripheral blood smear is notable for multiple schistocytes. Which of the following is the most likely diagnosis? Disseminated intravascular coagulation Hemolytic uremic syndrome Idiopathic thrombocytopenic purpura Thrombotic thrombocytopenic purpura

Thrombotic thrombocytopenic purpura The woman in this case most likely has thrombotic thrombocytopenic purpura. Thrombotic thrombocytopenic purpura (TTP) is a thrombotic microangiopathic disorder caused by severely decreased von Willebrand factor-cleaving protease ADAMTS13 activity. The decreased activity results in ultra-large von Willebrand factors that cause platelet bridging and clumping. TTP can be acquired due to autoantibodies or hereditary due to mutations in ADAMTS13. TTP is more common in women and in African Americans. Patients with acquired TTP typically presents with severe microangiopathic hemolytic anemia and thrombocytopenia. Common symptoms include neurologic complaints, such as confusion, difficulty speaking, headache, and seizure, and gastrointestinal problems, such as pain, nausea, vomiting, and diarrhea. Fever is also a common finding. Initial laboratory studies should include complete blood count, peripheral blood smear, serum creatinine, serum lactate dehydrogenase (LDH), serum bilirubin, and coagulation studies. Markedly elevated LDH, elevated indirect bilirubin, normal coagulation studies, and thrombocytopenia are findings consistent with acquired TTP. Renal insufficiency is typically absent to mild, and acute renal failure is rare. Additional testing may include Coombs test and ADAMTS13 activity. Computed tomography scan of the head may be required for patients with neurological deficits. Imaging studies are normal in TTP. TTP is a medical emergency and is associated with high mortality if not treated promptly. Plasma exchange therapy is the mainstay of treatment, along with intravenous corticosteroids. With appropriate treatment, patient survival rate is approximately 90%.

A 30-year-old woman with no significant medical history presents to the clinic with a report of double vision for the past 2 months. The double vision seems to worsen throughout the day. The patient also reports that her jaw becomes fatigued while chewing. Vital signs include a heart rate of 92 bpm, blood pressure of 120/80 mm Hg, respiratory rate of 20/min, oxygen saturation of 98% on room air, and temperature of 98.6°F. You notice drooping of the bilateral eyelids on physical exam. Which of the following conditions is the patient's suspected diagnosis associated with? Lung adenocarcinoma Multiple endocrine neoplasia Pituitary adenoma Small cell lung cancer Thymoma

Thymoma Myasthenia gravis is an autoimmune disorder affecting the neuromuscular junctions. Autoantibodies bind to acetylcholine receptors at the neuromuscular junction, which blocks acetylcholine from binding to these receptors and causes muscle weakness that is worse with use and improves with rest. Myasthenia gravis occurs more often in women than men. The muscle weakness fluctuates in severity and can occur in ocular, bulbar, extremity, and respiratory muscles as a focal weakness affecting specific muscle groups. It tends to be worse later in the day. Common clinical manifestations include diplopia, ptosis, fatigue with chewing, and dysarthria. Myasthenic crisis is a life-threatening presentation of myasthenia gravis that is marked by neuromuscular respiratory failure. The diagnosis is often confirmed using serologic testing in patients with clinical findings suggestive of myasthenia gravis. Patients with serum autoantibodies against acetylcholine receptors or a receptor-associated protein muscle-specific tyrosine kinase (MuSK) are considered to have seropositive myasthenia gravis. However, 10% of patients with myasthenia gravis do not have these autoantibodies and are considered to have seronegative myasthenia gravis. The diagnosis of myasthenia gravis in seronegative patients can be supported by electrophysiologic testing, including repetitive nerve stimulation studies and single-fiber electromyography. Patients with ocular or bulbar manifestations should have an MRI of the brain to assess for alternative etiologies. Furthermore, a chest CT scan should be obtained in patients with myasthenia gravis to assess for a thymoma. Approximately 10-15% of these patients have an associated thymoma. There are several modalities used in the treatment of this disease. Acetylcholinesterase inhibitors, such as pyridostigmine, can lead to symptomatic improvement in some patients. Chronic immunosuppressive therapies, such as glucocorticoids, are indicated in patients with persistent severe symptoms despite taking pyridostigmine. The treatment of myasthenic crisis includes mechanical ventilation (if needed) and transient immunomodulating therapies, such as plasmapheresis or intravenous immune globulins. Thymectomy is recommended in all patients with myasthenia gravis who have a thymoma. Furthermore, thymectomy is recommended for most patients with nonthymomatous myasthenia gravis who are younger than 65 years of age.

A 34-year-old woman presents to the emergency department with tingling in her hands. On physical examination, she has twitching at the corner of her mouth upon tapping the side of her face. What is the most common etiology of this abnormality? Hydrofluoric acid exposure Massive blood transfusion Necrotizing pancreatitis Thyroidectomy

Thyroidectomy This patient is exhibiting signs and symptoms of hypocalcemia, most commonly caused by hypoparathyroidism. Partial or total thyroidectomy is the most common etiology of hypoparathyroidism as the parathyroid is located immediately behind and is often partially or totally attached to the thyroid gland. Other causes of hypoparathyroidism include a congenital deficiency or infiltrate of the parathyroid glands from metastatic carcinoma, hemochromatosis, or Wilson disease. Signs and symptoms are exclusively related to hypocalcemia, and include reduced myocardial contractility, perioral and distal extremity paresthesias, and tetany. There are two hallmark physical exam findings with significant hypocalcemia: Chvostek sign and Trousseau sign. Chvostek sign consists of facial twitching caused by tapping the facial nerve anterior to the ear. Trousseau sign consists of carpal spasm when a blood pressure cuff is inflated on the upper arm above the systolic pressure for greater than three minutes. Diagnosis of hypocalcemia is often made on physical exam; however, laboratory testing should be obtained including both a total and ionized calcium level, serum 25-(OH) and 1,25-(OH) vitamin D, and parathyroid hormone levels. An ECG should also be performed to evaluate for prolongation of the QT interval. Management includes calcium and vitamin D supplementation. In severe, symptomatic hypocalcemia, calcium gluconate or calcium chloride should be administered intravenously, depending on severity of symptoms. Calcium chloride has three times more calcium per equal volume, but has the risk of tissue ischemia and necrosis if infused peripherally. Thus, calcium gluconate is preferred. Hydrofluoric acid exposure (A) is an uncommon potential toxicologic cause of hypocalcemia. Massive blood transfusion (B) may cause hypocalcemia due to citrate toxicity; however, the hypocalcemia is typically transient and does not require intravenous supplementation. Necrotizing pancreatitis (C) causes hypocalcemia by saponification as calcium is used up in this process; however, this is an uncommon phenomenon.

A 40-year-old woman with a history of hypertension and anxiety presents to the clinic. Her medications include lisinopril and escitalopram. During her visit, she reports worsening anxiety for the past 2 months. She states her escitalopram dose, which previously controlled her anxiety symptoms, no longer seems effective. When describing her anxiety symptoms, she reports new palpitations, bilateral hand tremor, heat intolerance, and diarrhea. She also reports weight loss despite increased appetite. Vital signs are obtained, which reveal HR of 115 bpm, BP of 120/80 mm Hg, RR of 14 bpm, and SpO2 of 98% on room air. Physical exam reveals a thin woman in no acute distress. Periorbital edema, lid retraction, and exophthalmos are present on ophthalmic exam. There is a palpable goiter. Cardiac exam reveals regular rhythm with tachycardia, normal S1 and S2, and no murmurs, rubs, or gallops. Muscle strength is normal in all extremities, but a mild essential tremor is noted in her hands. Her bilateral lower extremities are shown above. The remainder of the physical exam findings are normal. Based on her presentation and physical exam findings, antibodies directed against which of the following is most specific for the presumed diagnosis? Thyroglobulin Thyroid peroxidase Thyroid-blocking hormone receptor Thyrotropin receptor Thyroxine

Thyrotropin receptor The patient in the vignette presents with primary hyperthyroidism. Hyperthyroidism is characterized by an overactive thyroid gland, in which excess thyroid hormones are produced. There are many etiologies of hyperthyroidism, the most common being Graves disease, an autoimmune condition in which autoantibodies to the thyrotropin receptor cause increased thyroid hormone synthesis and secretion and diffuse thyroid growth. Risk factors for hyperthyroidism include female sex, pregnancy, stress, medications (including iodine and iodine-containing drugs such as amiodarone), recent infection (most commonly viral), genetic susceptibility, and tobacco use. Patients with hyperthyroidism may present with reports of hyperactivity, irritability, anxiety, heat intolerance, palpitations, diarrhea, oligomenorrhea or amenorrhea, and weight loss despite normal or increased appetite. Common exam findings include tachycardia, new-onset atrial fibrillation (especially in older patients), warm or moist skin, tremor, proximal muscle weakness, enlarged thyroid gland (goiter), gynecomastia, and eyelid retraction or lag. Other signs, such as exophthalmos (abnormal protrusion of the eyes), periorbital and conjunctival edema, and infiltrative dermopathy (pretibial myxedema) are specific for Graves disease. Diagnosis of hyperthyroidism is made based on laboratory findings. All patients with primary hyperthyroidism will have low levels of thyroid-stimulating hormone and high levels of triiodothyronine and thyroxine. Other nonspecific laboratory findings may be seen as well, including low cholesterol levels and evidence of a normochromic, normocytic anemia. Once the diagnosis of hyperthyroidism is made, etiology must be determined. The presence of thyrotropin receptor antibodies is highly sensitive and specific for Graves disease. Determination of the radioactive iodine uptake can also be useful in determining the etiology of hyperthyroidism. High radioactive iodine uptake in the thyroid indicates de novo synthesis of the hormone, indicating Graves disease, toxic multinodular goiter, or toxic adenoma. On the other hand, low iodine uptake indicates inflammation or destruction of thyroid tissue, leading to increased secretion of thyroid hormones. Treatment of hyperthyroidism includes etiology-directed treatment and symptomatic treatment. Beta-adrenergic blockade is recommended in patients with symptomatic palpitations, tachycardia, or new-onset atrial fibrillation. For patients with Graves disease, hyperthyroidism should be treated with either radioactive iodine, antithyroid medication (thionamides) such as methimazole and propylthiouracil, or thyroidectomy. In the United States, radioactive iodine is the most common form of treatment, but there is no evidence any of the three treatment options are superior to another. Comorbidities and individual risk factors should be considered to select the best treatment option for each patient.

A 57-year-old man diagnosed with Parkinson's disease two years ago presents to a routine clinic visit with complaints of increased tremors and slowed muscle movements despite being on amantadine for one year. The nurse practitioner discontinues the amantadine and prescribes carbidopa-levodopa. What is the function of carbidopa when combined with levodopa? To act as a dopamine receptor agonist To inhibit decarboxylase from converting levodopa to dopamine in the systemic circulation To inhibit monoamine oxidase B from metabolizing dopamine To prevent dyskinesia that is a side effect of chronic levodopa therapy

To inhibit decarboxylase from converting levodopa to dopamine in the systemic circulation Carbidopa is usually combined with levodopa to inhibit decarboxylase from converting levodopa to dopamine in the systemic circulation. Parkinson's disease is a progressive neurodegenerative disease characterized by low dopamine levels. Dopamine is an inhibitory neurotransmitter in the substantia nigra of the midbrain and works with acetylcholine (excitatory neurotransmitter) to create normal motor movements. When there is a decrease in the inhibitory neurotransmitter, motor movements are less controlled and result in the characteristic tremors, rigidity, bradykinesia, and postural instability of Parkinson's disease. Levodopa is a central nervous system agent that converts to dopamine in the body and improves all symptoms of Parkinson's disease, including bradykinesia and tremors. Common side effects of levodopa include nausea, vomiting, hypotension, and cardiac dysrhythmias. When prescribed alone, levodopa requires high dosages because much of the dose is converted to dopamine in the systemic circulation before crossing the blood-brain barrier, where it can be take effect as a neurotransmitter. Carbidopa reduces this extracerebral breakdown of levodopa by inhibiting the enzyme that is responsible for metabolizing levodopa, decarboxylase. When combined with carbidopa, reduced amounts of levodopa are required since more is able to cross the blood-brain barrier and take effect. This lower dose of levodopa decreases side effects while still improving symptoms of Parkinson's disease. Current therapies for Parkinson's disease do not alter the underlying pathology of the disease but improve symptoms. Carbidopa-levodopa is the most commonly prescribed medication for Parkinson's disease, but other first-line agents include amantadine and dopamine agonists.

A 72-year-old man presents with progressive dysphagia. He initially had difficulty swallowing solid foods but is now having difficulty swallowing liquids. An esophagogastroduodenoscopy reveals a fungating mass on the middle one-third of the esophagus. Biopsy of the lesion is positive for squamous cell carcinoma. Which of the following historical features does this man most likely have? Exposure to industrial dyes Gastroesophageal reflux disease Lynch syndrome Tobacco use

Tobacco use Tobacco use is one of the most common risk factors for the development of squamous cell carcinoma (SCC) of the esophagus. The most common types of esophageal cancer are SCC and adenocarcinoma. Barrett metaplasia, from chronic, untreated gastroesophageal reflux disease (GERD), is the most common cause of esophageal adenocarcinoma. Tobacco smoking and alcohol consumption are the most common causes of esophageal SCC. Adenocarcinoma is most commonly seen in White men, while SCC is most commonly seen in Black and Asian individuals. Adenocarcinoma and SCC have a similar clinical presentation of progressive dysphagia. Difficulty swallowing solid food, such as meat, bread, or fruit, typically precedes dysphagia to liquids. Weight loss is commonly noted. Esophagogastroduodenoscopy with biopsy is the most commonly used diagnostic tool for establishing the diagnosis and determining staging. Most esophageal SCC is located in the middle thoracic esophagus. Adenocarcinomas are most commonly found in the distal esophagus and the gastroesophageal junction. CT and PET may be used for determining lymph node involvement. Lymph node involvement or metastases is associated with poor prognosis. Treatment varies by disease stage and may include surgical resection, chemotherapy, and radiation therapy.

A 3-day-old male neonate develops bilateral purulent eye discharge. He also exhibits a runny-nose as well as eyelid edema and redness. No other abnormalities on physical examination are noted. Which of the following could have been given as standard prophylaxis to prevent the development of this infant's condition? Intravenous acyclovir Intravenous ceftriaxone Oral erythromycin Topical erythromycin

Topical erythromycin This patient's presentation of bilateral eye discharge on day 3 of life is consistent with gonococcal ophthalmia neonatorum. Topical 0.5% erythromycin applied immediately after birth is effective prevention against development of neonatal gonococcal conjunctivitis. There are 3 common types of neonatal conjunctivitis encountered on board exams: chemical, gonococcal, and chlamydial. They may be easily differentiated based on the time of presentation. Chemical conjunctivitis often occurs within the first 24 hours of life and is typically secondary to the use of certain prophylactic topical ointments applied to newborns to prevent gonococcal conjunctivitis. Gonococcal conjunctivitis occurs 3-5 days after birth and chlamydial conjunctivitis occurs 5-10 days after birth. The US Preventive Services Task Force (USPSTF) issued a recommendation in 2012 stating that all newborns should receive prophylactic ocular topical medication in order to prevent development of gonococcal ophthalmia neonatorum. Acceptable agents include topical erythromycin 0.5%, topical tetracycline 1.0%, silver nitrate, or povidone-iodine 2.5%. However, erythromycin 0.5% ointment is the only drug approved by the FDA for this indication.

A 37-year-old man with a history of obesity and epilepsy presents to the primary care office. He takes phenytoin 300 mg twice daily. His vital signs are BP 108/69 mm Hg, HR 67 bpm, RR 15 breaths/min, T 98.2°F, and oxygen saturation 98% on room air. The patient states he has experienced migraine headaches for years. He says he has migraines an average of nine times per month. He describes his migraines as a unilateral, pounding sensation behind one of his eyes that is typically associated with nausea and sensitivity to light. At his last visit, you discussed lifestyle modifications, and the patient states he followed the modifications for months without improvement of his headaches. The patient would like to be started on prophylactic medication at this time. Which of the following is the best medication for this patient? Botulinum toxin Ergotamine Metoprolol Sertraline Topiramate

Topiramate Migraine headache is a disorder of recurrent attacks of unilateral, throbbing head pain that may or may not be associated with other neurological symptoms. Nausea and photophobia are commonly associated with migraine headaches. Migraine is a syndromic disorder of the brain that is, in most instances, inherited. The most common triggers include emotional stress, hormones in women, hunger, weather, sleep disturbances, odors, and neck pain. Neuroimaging is not necessary in most patients with migraine. Diagnosis is made clinically based on the patient's symptoms and history. Migraine headache attacks last anywhere between 4 and 72 hours, are unilateral, have a pulsatile quality, and are aggravated by physical activity. Either nausea or vomiting (or both) and photophobia or phonophobia are common during attacks, as well. More than four migraines per month or migraines that last longer than 12 hours and diminish a patient's quality of life despite acute treatment qualify patients for migraine prophylactic therapy. While several classes of medication can be used for migraine prophylaxis, the choice should be individualized according to patient-specific comorbid conditions and values. Topiramate is an anticonvulsant and is also indicated as a prophylactic medication for episodic migraines in patients with a history of seizure, thus is the best option for this patient.

Which of the following statements is most correct regarding total parenteral nutrition in a patient with Crohn disease? Patients tolerate enteral feedings well after total parenteral nutrition is discontinued Total parenteral nutrition carries the risk of deep vein thrombosis and cardiomyopathy Total parenteral nutrition is an alternative to chronic low dose steroids Total parenteral nutrition is not recommended in those with short bowel syndrome

Total parenteral nutrition is an alternative to chronic low dose steroids Crohn's disease is a chronic, inflammatory disease of the bowel involving any part of the gastrointestinal tract (mouth to anus), but with a predilection for the distal ileum and colon. The mucosal inflammation of Crohn's disease is transmural, leading to the formation of fistulas, abscesses, and stricture formation. Affected segments of bowel are often separated by segments of non-diseased bowel and are sometimes referred to as "skip lesions". Symptoms of Crohn's disease can include chronic diarrhea, fever, weight loss, and abdominal pain. Diagnosis is with colonoscopy, upper endoscopy, CT enterography or magnetic resonance enterography. Crohn's disease can be divided into four disease states, with each state necessitating differing treatment strategies. The most mild state is asymptomatic remission, where patients have no symptoms either after successful medical or surgical management, or spontaneous remission. These patients can be maintained with daily dosing of a drug from the 5-ASA class, such as mesalamine. The drug choice will be dependent on the disease site. Patients with mild-to-moderate Crohn's disease will manifest varying degrees of symptoms but will be able to tolerate oral nutrition without dehydration, significant (>10 percent) weight loss, abdominal tenderness, mass, obstruction, or toxicity. Many treatment options exist for these patients, including 5-ASA drugs, corticosteroids, antibiotics, probiotics, antidiarrheal medications and dietary changes. Patients who have failed treatment for mild-to-moderate disease or who manifest severe abdominal pain, significant weight loss, fever, nausea and vomiting, or fever are considered to have moderate-to-severe Crohn's disease. These patients often require hospitalization for intravenous glucocorticoids and often require treatment with biologic agents to obtain remission. Fulminant Crohn's disease is marked by such symptoms as high fevers, persistent vomiting, signs of abscess formation, peritoneal signs, intestinal obstruction, or patients whose symptoms persist despite glucocorticoid therapy or despite therapy with a biologic agent. Some of these patients will require surgical management. Many of these patients will require chronic, low-dose steroid therapy. Total parenteral nutrition can be considered as an alternative to chronic low-dose glucocorticoids. Many patients who receive total parenteral nutrition achieve remission, although often their symptoms will return once enteral feedings are resumed.

A 65-year-old man presents to the ED due to new-onset seizures. He reports he has been feeling unwell over the past few weeks with symptoms including fever, headache, and weakness. His daughter is with him and reports that he has seemed "out of it" and "easily confused" over the past few months. Medical history is notable for HIV with a last CD4 count of 45 cells/mcL. His medications include dolutegravir, emtricitabine, and tenofovir disoproxil fumarate. He reports he does not always take his HIV medications because it is hard to remember three different pills. Vital signs include a BP of 112/72 mm Hg, HR of 105 bpm, RR of 18/min, and T of 102.5°F Physical examination reveals left-sided global decreased motor strength and a slight resting tremor. Advanced imaging of the brain shows multiple ring-shaped contrast-enhancing cerebral lesions that involve the right basal ganglia. Which of the following pathogens is most likely responsible? Cryptococcus neoformans Herpes simplex virus Rhabdoviridae Toxoplasma gondii Treponema pallidum

Toxoplasma gondii Toxoplasmosis is an infection caused by Toxoplasma gondii, which is an intracellular protozoa found in mammals, including humans, and birds, with the definite host being cats. Toxoplasmosis is the most common CNS space-occupying lesion in patients with HIV. The most common presentation in patients with AIDS is toxoplasmosis encephalitis, which typically occurs with multiple necrotizing brain lesions. It typically occurs in patients with CD4 counts of < 50 cells/mcL. The encephalitis typically presents subacutely, with fever, headache, altered mental status, seizures, focal neurological findings, and other evidence of brain lesions. Concern for toxoplasmosis encephalitis warrants advanced brain imaging. CT and MRI of the brain typically show multiple ring-shaped contrast-enhancing cerebral lesions, typically involving the basal ganglia. Definitive diagnosis requires brain biopsy with search for organisms and histology evaluation. Therapy is generally not necessary in immunocompetent patients due to the self limiting nature of the primary illness. However, immunocompromised patients with evidence of active infection must be treated. Medications for toxoplasmosis are only active against tachyzoites and thus will not eradicate infection. Standard therapy is a combination of pyrimethamine, sulfadiazine, and folinic acid. After initial treatment, chemoprophylaxis to prevent reactivated infection is necessary unless antiretroviral therapy leads to immune reconstitution.

A woman presents with 30 minutes of double vision, vertigo, difficulty swallowing, and difficulty speaking. During her initial evaluation, these symptoms resolve and her neurologic exam returns to normal. Which of the following is the most appropriate diagnosis and the most likely affected artery? Embolic ischemia - anterior cerebral artery Thrombotic stroke - left anterior descending artery Transient ischemic attack - middle cerebral artery Transient ischemic attack - vertebrobasilar arteries

Transient ischemic attack - vertebrobasilar arteries Transient ischemic attacks (TIAs) are characterized by a transient episode of neurologic dysfunction caused by local brain, spinal cord, or retinal ischemia, without acute infarction. TIAs suggest impending thrombotic-ischemic stroke. Carotid pathology leads to TIAs demonstrated by hemiparesis, hemisensory, aphasia, confusion, and transient monocular blindness. Vertebrobasilar insufficiency leads to TIAs marked by hemiplegia or quadriplegia, varying sensory changes, blindness, hemianopsia, diplopia, vertigo, dysarthria, dysphagia, and facial, motor, and sensory change. Investigation includes Doppler ultrasonography of the carotids and vertebrobasilar system. Head CT scan will not show any acute changes. MR angiography may also be indicated. Selected cases may require endarterectomy, angioplasty, or lifelong anticoagulation.

A 19-year-old man with a previous history of abdominal pain and diarrhea presents to your office with complaints of fatigue, weight loss, sweats and malaise. He tells you that his mother has experienced similar symptoms, but doesn't like to go to the doctor so has never been evaluated. Which of the following is most suggestive of Crohn's disease rather than ulcerative colitis? Genetic predisposition Lesions affecting the colon Symptoms of diarrhea and abdominal pain Transmural inflammation

Transmural inflammation Inflammatory bowel disease (IBD) is caused by an abnormal immune response to normal intestinal flora. There are two types of IBD, Crohn's disease and ulcerative colitis. Ulcerative colitis is limited to the colon. Crohn's disease can affect any part of the gastrointestinal (GI) tract from the mouth to the anus with "skip lesions." The inflammation of Crohn's disease is transmural, compared to affecting only the mucosa and submucosa in ulcerative colitis. Patients with IBD present with systemic symptoms such as fatigue, weight loss, sweats and malaise. GI manifestations occur based on the area of the GI tract affected and can include abdominal pain, cramping, irregular bowel habits, and the passing of mucus without pus or blood in Chron disease or with blood in ulcerative colitis. Laboratory testing is not specific enough to provide the diagnosis of IBD, although can provide supporting information and help with management. Endoscopy may be used in evaluating Crohn's disease and colonoscopy is a valuable tool for both types of IBD.

A 26-year-old woman with a known history of AIDS presents to the ED for strange behavior, according to her boyfriend. Reportedly, she complained of a headache for a few days prior and then began acting bizarrely. In the ED, she has a temperature of 38.5°C. Neurological examination is remarkable for word-finding difficulties accompanied by episodes of clanging and echolalia, along with decreased attention span, recall, and consolidation. A contrast CT scan of the brain reveals multiple ring-enhancing lesions without evidence of midline shift. Which of the following is the most appropriate next step in management? Consult neurosurgery for a brain biopsy Obtain an MRI Treat with dexamethasone Treat with pyrimethamine and sulfadiazine Treat with trimethoprim-sulfamethoxazole

Treat with pyrimethamine and sulfadiazine This patient with AIDS and altered mental status most likely has cerebral toxoplasmosis, the most common cause of focal encephalitis in patients with AIDS. It is often accompanied by fever, headache, altered mentation, focal neurological deficits, and seizures. It is caused by the protozoa Toxoplasma gondii. The initial diagnosis is based on history, physical, and head CT scan. The appearance of multiple ring-enhancing lesions on contrast-enhanced head CT scan is pathognomonic. Treatment should be initiated with pyrimethamine and sulfadiazine. Some regimens also include folinic acid.

Which of the following is most correct regarding treatment of Group A Strep pharyngitis? Treatment prevents acute rheumatic fever but not post-streptococcal glomerulonephritis Treatment prevents both acute rheumatic fever and post-streptococcal glomerulonephritis Treatment prevents colonization with Group A Strep in the future Treatment prevents post-streptococcal glomerulonephritis but not acute rheumatic fever

Treatment prevents acute rheumatic fever but not post-streptococcal glomerulonephritis Treatment of Group A Strep pharyngitis prevents acute rheumatic fever, a non-suppurative, post-infectious complication of streptococcal pharyngitis. The diagnosis of acute rheumatic fever requires evidence of recent Group A Strep infection and the presence of either 2 major or 1 major and 2 minor Jones criteria. Acceptable evidence of recent Group A Strep infection includes a positive throat culture or rapid streptococcal antigen test or an elevated or rising anti-streptococcal antibody titer. Major Jones criteria include subcutaneous nodules, migratory polyarthritis, erythema marginatum, carditis, and chorea. Minor criteria include arthralgia, fever, elevated acute phase reactants (erythrocyte sedimentation rate or C-reactive protein), or a prolonged PR interval.

A 32-year-old man presents to the county health department with complaints of a painless sore on his penis. He reports having unprotected sexual intercourse 2 weeks ago. Physical exam reveals a single 2 cm nontender ulcer with a raised, indurated margin on the shaft of his penis. Which of the following is the most likely causative organism? Haemophilus ducreyi Herpes simplex virus Neisseria gonorrhoeae Treponema pallidum

Treponema pallidum The patient most likely has syphilis, a sexually transmitted infection caused by Treponema pallidum. The majority of syphilis cases are sexually acquired, however vertical transmission can occur. Patients who are at high risk for syphilis include men who have sex with men, inmates, patients who have multiple sexual partners, and patients with other sexually transmitted infections. Syphilis can be classified into four stages: primary, secondary, latent, and tertiary. Primary syphilis is characterized by a round, painless ulcer, known as a chancre. The chancre typically appears 2-3 weeks after exposure and heals within a few weeks. Approximately 25% of patients with untreated primary syphilis develop secondary syphilis. Secondary syphilis presents with a rash, fever, headache, malaise, anorexia, and diffuse lymphadenopathy. Like primary syphilis, secondary syphilis usually resolves spontaneously. Latent syphilis indicates the patient is still infected with T. pallidum, but is asymptomatic. Tertiary syphilis develops 1-30 years after initial infection. The most common complications of tertiary syphilis include neurosyphilis, cardiovascular syphilis (aortitis), and gummatous syphilis. Diagnostic testing can be performed through non-treponemal tests, treponemal tests, and direct visualization. Nontreponemal tests, such as Venereal Disease Research Laboratory (VDRL) or rapid plasma reagin (RPR), are commonly used as screening methods due to cost and ease of use. Positive nontreponemal tests require confirmation with treponemal tests, which are based on antibodies against specific treponemal antigens. Darkfield microscopy, although quickest and most direct, requires specialized equipment and prompt examination of the specimen. Under dark field microscopy, T. pallidum appear as thin, delicate corkscrews with tightly round spirals. Cerebrospinal fluid examination is the only method for diagnosing asymptomatic neurosyphilis. A one time single dose of intramuscular penicillin G is recommended treatment for early syphilis. Late stages of syphilis require three doses. All cases of syphilis should be reported to the Centers for Disease Control and Prevention.

A man is sent home with a prescription for an antibiotic after being diagnosed with a urinary tract infection. He returns because his eyes are yellow 2 days later. Which of the following antibiotics was most likely initially prescribed? Amoxicillin Cephalexin Doxycycline Trimethoprim-sulfamethoxazol

Trimethoprim-sulfamethoxazol Glucose-6-phosphate dehydrogenase (G6PD) deficiency is an X-linked recessive hereditary disease that results in characteristically low levels of G6PD. Individuals with this deficiency may exhibit nonimmune hemolytic anemia in response to a variety of oxidant stressors such as exposure to particular medications or, more commonly, infection. Trimethoprim-sulfamethoxazole can cause oxidative stress that leads to hemolytic anemia.

A 3-year-old boy presents to the Emergency Department after four episodes of emesis and three episodes of diarrhea starting this evening. He has had a few sips of water at home. His vital signs are reassuring except for mild tachycardia and he appears mildly dehydrated on examination. He is given a dose of ondansetron for nausea in the waiting room. What is the next best step in management? Administer a bolus of 0.9% sodium chloride Admit for maintenance intravenous fluids Discharge home with lactobacillus Trial of oral rehydration solution

Trial of oral rehydration solution The most likely diagnosis for a child with acute onset of vomiting and diarrhea is viral gastroenteritis. Food poisoning is also a consideration. Both are self-limited conditions and most affected children can be safely managed in the outpatient setting. The most important supportive intervention is maintenance of adequate intravascular volume. In cases of mild to moderate dehydration, oral rehydration therapy is preferred to intravenous hydration. In children with persistent emesis with attempted oral rehydration, anti-nausea medication may be effective in facilitating tolerance of oral intake. Ondansetron, a selective serotonin antagonist, has been shown to decrease the rate of hospitalization in children who receive a single dose in the emergency department. After taking the ondansetron, the boy can then proceed with a trial of oral rehydration solution, the mainstay of treatment for acute gastroenteritis. He should then be monitored for further emesis. If emesis persists despite anti-nausea medication, intravenous hydration may be required until adequate oral hydration can be assured.

A 15-year-old girl is in your clinic with her mother for a sports physical examination. She denies any symptoms and is doing well in school. There is no family history of sudden death or heart disease. She has been a vegan for the past year and takes food supplements. She has regular menstrual periods. Physical examination is normal. You perform a screening complete blood count (CBC) that reveals hemoglobin of 10 g/dL, mean corpuscular volume (MCV) of 68, with ferritin of 11 ng/mL (normal for females 12-150 ng/mL). Which of the following is the next best step in management? Increase intake of meat, grains, fruits, and vegetables Recheck CBC in one month Start parenteral iron Trial treatment with oral iron supplements

Trial treatment with oral iron supplements The adolescent in the vignette has iron deficiency anemia (IDA) that her vegan diet most likely causes. Adolescence is a time of increased iron needs because of the expansion of blood volume and increases in muscle mass. The incidence of iron deficiency among adolescents appears to be rising and those at particular risk are adolescents who limit their intake of meat products. The primary laboratory values obtained when evaluating an adolescent for iron deficiency include a complete blood count with red blood cell indices and serum ferritin. In most cases, iron deficiency is identified by a low serum ferritin concentration and iron deficiency anemia by a hemoglobin concentration below 11.0 g/dL combined with a serum ferritin less than 12 ng/mL. The anemia typically is microcytic and hypochromic. The treatment of iron deficiency for patients with mild or moderate degrees of iron deficiency (hemoglobin > 9 g/dL) involves a trial of treatment with oral iron supplements (3 to 6 mg of elemental iron/kg/day). Follow up monitoring should be performed to ensure response to the supplementation, which also helps to confirm the diagnosis of iron deficiency.

A 21-year-old woman presents to the emergency department with fever, malaise, and chest pain. Her vital signs in triage are T 39.0°C, HR 122, BP 110/60, RR 30. Physical examination reveals cellulitis along the antecubital area of the left arm, jugular venous distention, a holosystolic murmur best heard at the left sternal border, right upper quadrant tenderness, and 2+ bilateral lower extremity edema. Which of the following is the most likely cause of the patient's symptoms? Aortic regurgitation Mitral stenosis Pulmonic stenosis Tricuspid regurgitation

Tricuspid regurgitation Tricuspid regurgitation refers to incompetence of the tricuspid valve leaflets such that some blood flows backwards from the right ventricle into the right atrium during systole. Tricuspid regurgitation is most commonly functional but pathologic tricuspid regurgitation can occur due to elevated right heart pressure, or, more rarely, from endocarditis. Tricuspid endocarditis is rare and most commonly seen in intravenous drug users. S. aureus is the most commonly implicated organism in tricuspid endocarditis in intravenous drug users. Physical examination findings include a blowing holosystolic murmur best heard at the left sternal border which is enhanced by inspiration. Patients with acute tricuspid regurgitation will show symptoms of right heart failure, including jugular venous distention, a hepatojugular reflux, right upper quadrant tenderness from hepatic congestion, and peripheral edema.

You are concerned with hearing a new diastolic, rumbling murmur in one of your patients. This murmur is best heard with the bell over the left sternal border at the fourth intercostal space and is louder during inspiration. Which of the following is the most likely diagnosis? Aortic regurgitation Aortic stenosis Tricuspid regurgitation Tricuspid stenosis

Tricuspid stenosis Tricuspid stenosis is a murmur that is heard best with the bell over the left sternal border at the fourth intercostal space. It can be caused by myxomatous degeneration, rheumatic heart disease, congenital malformations and endocardial fibroelastosis. It rarely occurs alone, and almost always occurs with mitral stenosis. It is usually described as a diastolic rumble, which is louder than mitral stenosis during inspiration. There may be an associated thrill. Overall, it is a relatively rare murmur, and when it is an isolated finding, it usually does not require treatment. However, as it commonly occurs with mitral stenosis, surgical valve replacement or balloon valvuloplasty may be necessary.

A 35-year-old man with a history of human immunodeficiency virus presents to your office with complaints of urinary symptoms, chills and muscle aches. Physical exam findings include a temperature of 102.1F, pain to palpation of the suprapubic area, and a warm, firm, exquisitely tender prostate on rectal exam. Which of the following is the most appropriate therapy? Abacavir Nitrofurantoin Tamsulosin Trimethoprim-sulfamethoxazole

Trimethoprim-sulfamethoxazole Acute bacterial prostatitis occurs when microorganisms enter the prostate gland through the urethra. Prostatitis often presents in the primary care setting among young and middle-aged men, however bacterial prostatitis is relatively uncommon. Risk factors include conditions that predispose men to urogenital infections such as anatomical anomalies and urogenital instrumentation. Lower urinary tract symptoms including prostatitis occur more frequently in men with human immunodeficiency virus (HIV). It is unclear why this occurs. Men with acute bacterial prostatitis present as acutely ill with spiking fever, malaise, chills, urinary symptoms and myalgias. Diagnosis is made clinically with digital rectal exam. Urine gram stain and culture establishes the microbial etiology. Pathogens causing infection are generally gram-negative organisms, therefore empiric treatment with antibiotics that treat gram-negative infections should be administered immediately. First-line treatment is with trimethoprim-sulfamethoxazole or a fluoroquinolone such as ciprofloxacin or levofloxacin. Patients who cannot tolerate oral medication, are exhibiting signs of sepsis or have bacteremia should be hospitalized. Abacavir (A) is a nucleoside/nucleotide reverse transcriptase inhibitor used in the treatment of human immunodeficiency virus (HIV). Nitrofurantoin (B) is commonly used to treat urinary tract infections in women, but has poor tissue penetration and is not recommended for treatment of prostatitis. Tamsulosin (C) is an alpha-one blocker used to treat benign prostatic hypertrophy, symptoms of bladder outlet obstruction and ureteral calculi expulsion. Acute bacterial prostatitis occurs when microorganisms enter the prostate gland through the urethra. Prostatitis often presents in the primary care setting among young and middle-aged men, however bacterial prostatitis is relatively uncommon. Risk factors include conditions that predispose men to urogenital infections such as anatomical anomalies and urogenital instrumentation. Lower urinary tract symptoms including prostatitis occur more frequently in men with human immunodeficiency virus (HIV). It is unclear why this occurs. Men with acute bacterial prostatitis present as acutely ill with spiking fever, malaise, chills, urinary symptoms and myalgias. Diagnosis is made clinically with digital rectal exam. Urine gram stain and culture establishes the microbial etiology. Pathogens causing infection are generally gram-negative organisms, therefore empiric treatment with antibiotics that treat gram-negative infections should be administered immediately. First-line treatment is with trimethoprim-sulfamethoxazole or a fluoroquinolone such as ciprofloxacin or levofloxacin. Patients who cannot tolerate oral medication, are exhibiting signs of sepsis or have bacteremia should be hospitalized.

A 33-year-old woman presents to primary care for a follow-up for her UTI. She was diagnosed with a UTI 5 days ago in the office and was placed on trimethoprim 800 mg/sulfamethoxazole 160 mg twice per day for 7 days. This morning, she awoke with a fever, rash, and bilateral knee pain. She reports no cough, sore throat, or vaginal discharge but still has some dysuria with urination. She has no other significant medical history but had a progestin-containing intrauterine device placed last month for long-term birth control. Today, her vitals are a T of 100.9°F, BP of 164/78 mm Hg, RR of 13/min, HR of 88 bpm, and oxygen saturation of 99% on room air. She has a maculopapular rash over her trunk and extremities, no abdominal tenderness, positive bowel sounds in all four quadrants, and no costovertebral angle tenderness. Bilateral knees are without warmth or erythema, and she maintains full active range of motion. Laboratory evaluation of urine reveals white cells, red cells, and white cell casts. Her creatinine is 3.3 mg/dL and absolute blood eosinophil count is 600/microL. What is the most likely cause of her symptoms? Disseminated osteomyelitis caused by her UTI Intrauterine device Nephrolithiasis Dyelonephritis Trimethoprim-sulfamethoxazole

Trimethoprim-sulfamethoxazole Acute interstitial nephritis (AIN) accounts for around 10% of all cases of intrinsic acute kidney injury. The most common cause of interstitial nephritis is medications, with penicillins, cephalosporins, sulfonamides (such as trimethoprim-sulfamethoxazole), and NSAIDs being among the top offenders. Common symptoms reported include rash, fever, and arthralgias. Increased plasma creatinine is noted in all patients. Eosinophilia may be found in some patients. Urinalysis often reveals red cells, pyuria, white cell casts, proteinuria, and eosinophiluria. A kidney biopsy may be performed if the diagnosis is unclear, and treatment is aimed at stopping the causal agents and providing supportive treatment. Steroids may be indicated in more severe cases.

A new mother presents to the pediatrician's office for a regularly scheduled visit for her newborn son. She reports she has noticed scrotal swelling. She reports he does not seem to be in pain and his urinary and bowel habits appear similar to her previous sons. Physical examination of the infant boy reveals a soft and full hemiscrotum that transilluminates. Which of the following is the most likely location of the swelling in this newborn condition? Cremasteric fascia Dartos fascia Pampiniform plexus Tunica albuginea Tunica vaginalis

Tunica vaginalis Hydroceles are the most common cause of painless scrotal swelling. They are typically found in infants due to congenital abnormalities caused by an anatomical anomaly. Hydroceles are an abnormal accumulation of serous fluid between the two layers of the tunica vaginalis, which is the pouch of serous membrane that covers the testes. On exam, the testis will be palpable at the posterior aspect of the fluid collection with normal testicular sensation. Transillumination will reveal a homogeneous glow without shadows. Asymptomatic isolated noncommunicating hydroceles are typically observed as complications are rare and the condition often resolves spontaneously.

A 63-year-old man presents to your office complaining of episodic diarrhea and wheezing. His wife also mentions that his skin will occasionally look flushed. You suspect carcinoid syndrome. What initial diagnostic study is most appropriate to confirm this condition? Abdominal computed tomography scan Abdominal magnetic resonance imaging Erythrocyte sedimentation rate Twenty four hour urine excretion of 5-hydroxyindoleacetic acid

Twenty four hour urine excretion of 5-hydroxyindoleacetic acid Carcinoid tumors are rare neuroendocrine tumors of the digestive tract, lungs, and less commonly of the kidneys and ovaries. These tumors secrete vasoactive material such as serotonin, histamine, catecholamine, prostaglandins, and peptides resulting in carcinoid syndrome, which is characterized by episodic skin flushing, wheezing and diarrhea. Concerningly, these tumors may also lead to plaque-like fibrosis of the heart valves, eventually leading to carcinoid heart disease. Treatment of carcinoid tumors depends on their location and staging, but often involves localized resection of the tumors and symptomatic control. The diagnosis of carcinoid syndrome is best confirmed by measuring the 24-hour excretion of 5-hydroxyindoleacetic acid (5-HIAA) in the patient's urine. This will be elevated in those with carcinoid syndrome, as 5-HIAA is excreted in the urine after metabolizing the elevated levels of serotonin produced by carcinoid tumors.

A man who presents with syncope is placed on the cardiac monitor. On the monitor, you note a repeating trend of 6 P waves, 5 of which are followed by a narrow QRS complex and 1 of which is not followed by a QRS complex. The PR interval during this trend progressively increases. Which of the following is the most likely diagnosis? First-degree AV block Third-degree AV block Type I second-degree AV block Type II second-degree AV block

Type I second-degree AV block A key distinction between first-degree and second-degree heart block is that in first-degree block the P wave is always followed by a QRS complex. In other words, the ratio of P waves to QRS complexes is 1:1 (the electrical signal from the atria always passes to the ventricles). In second-degree AV block, the electrical impulse sometimes gets to the ventricles. There are two main types of second-degree AV block. In Mobitz type I (or Wenckebach) second-degree block, there is a progressive beat-to-beat lengthening of the PR interval until a P wave does not conduct through the AV node. The absent conduction and resultant "missing" QRS complex is called a "dropped" QRS, which represents an absent beat of ventricular contraction.

A 62-year-old man with a history of hypertension on amlodipine and type 2 diabetes mellitus on metformin presents to the emergency department with dull chest pain that started 8 hours ago. The patient appears mildly diaphoretic. Vital signs include a heart rate of 104 bpm, blood pressure of 135/92 mm Hg, and respiratory rate of 22 breaths per minute. He has a regular rate and rhythm, and his lungs are clear to auscultation bilaterally. An ECG was performed and shows "absence of new ST elevation or pathologic Q waves in contiguous leads on an electrocardiogram". The initial cardiac troponin I is 9.8 ng/mL. You discuss with the cardiologist on call who plans to perform cardiac catheterization with coronary angiography in 24 hours. Which of the following is an appropriate treatment to administer now? Alteplase Bivalirudin Enoxaparin Fondaparinux Unfractionated heparin

Unfractionated heparin The patient in the vignette meets the diagnostic criteria for a non-ST elevation myocardial infarction (NSTEMI). Non-ST elevation myocardial infarction is marked by an elevated troponin due to myocardial ischemia in the absence of new ST elevation or pathologic Q waves in contiguous leads on an electrocardiogram. Patients often present with chest pain or other anginal equivalents, such as dyspnea with exertion. The chest pain often radiates to one or both extremities, and patients frequently experience diaphoresis and nausea and vomiting with the chest pain. Anginal chest pain is classically worse with exertion and improved with rest. According to the Fourth Universal Definition of myocardial infarction, the typical ECG findings for non-ST elevation myocardial infarction are new horizontal or downsloping ST depression of at least 0.5 mm in two contiguous leads, T wave inversion > 1 mm in two contiguous leads with prominent R wave or R/S ratio, or both. Troponin is the cardiac biomarker that is most sensitive and specific for myocardial injury. The clinical setting must be considered to determine if myocardial infarction is the cause of the troponin elevation since there are several other causes of troponin elevation. In patients who present with suspected acute coronary syndrome, the initial troponin may be negative. In these cases, a troponin measurement should be repeated 2-6 hours later. The treatment of non-ST elevation myocardial infarction consists of dual antiplatelet therapy, anticoagulation, and, in most patients, early coronary angiography with revascularization. The recommended dual antiplatelet regimen is aspirin and a P2Y12 receptor antagonist, such as ticagrelor or prasugrel. Unfractionated heparin is the preferred anticoagulant in patients with non-ST elevation myocardial infarction in whom coronary angiography and possibly revascularization are going to be performed within 48 hours. Coronary angiography with revascularization is performed within 24-48 hours in most patients. It is indicated immediately in patients with one or more of the following features: hemodynamic instability, cardiogenic shock, severe left ventricular dysfunction or heart failure, recurrent or persistent rest angina despite intensive medical therapy, new or worsening mitral regurgitation, new ventricular septal defect, or sustained ventricular dysrhythmias. Furthermore, it is indicated within 24-48 hours in patients with a Thrombolysis in Myocardial Infarction (TIMI) score of at least 3. The preferred revascularization strategy, such as percutaneous coronary intervention or coronary artery bypass graft, varies based on the coronary disease on coronary angiography. Patients who have a TIMI score of 0-2 may receive intensive medical therapy. Patients with a non-ST elevation myocardial infarction should also be started on a beta-blocker (if there is no contraindication) and a high-intensity statin, such as atorvastatin or rosuvastatin, within 24 hours. Alteplase (A) is a fibrinolytic therapy that is often used in the treatment of ischemic strokes and in patients with ST elevation myocardial infarction in whom percutaneous coronary intervention is not available. However, trials have demonstrated that fibrinolytic therapy is not beneficial in the treatment of non-ST elevation myocardial infarction. Bivalirudin (B) is a direct thrombin inhibitor anticoagulant. However, heparin is the preferred anticoagulant because there is a possible higher risk of stent thrombosis with bivalirudin in patients who are undergoing early coronary angiography with possible revascularization within 48 hours of presentation. Enoxaparin (C) is a low-molecular-weight heparin anticoagulant. Unfractionated heparin is preferred over enoxaparin in patients who are undergoing early coronary angiography with possible revascularization because using enoxaparin increases the risk of pathologic bleeds. Fondaparinux (D) is a direct factor Xa inhibitor anticoagulant. However, unfractionated heparin is the preferred anticoagulant because there is a potential higher risk of stent thrombosis with fondaparinux in patients who are undergoing early coronary angiography with possible revascularization within 48 hours of presentation.

A 65-year-old man presents to the primary care clinic with trouble swallowing. His symptoms initially began about 2 months ago when he started having trouble swallowing solids and he had to chew longer and eat slower. In the last 2 weeks, he has noticed he is having trouble swallowing liquids, too. He also has odynophagia and has lost about 10 lbs in the last 2 months. He smokes and has a 40-year pack history, along with a history of hypertension and high cholesterol. He takes lisinopril 10 mg and atorvastatin 20 mg daily. Vital signs are BP 126/80 mm Hg, temperature 98.9°F, pulse 98 bpm, RR 14 breaths per minute, and BMI 29.6 kg/m2, and his physical exam is unremarkable today. What is the best next step in the management of this patient's condition? Chest radiographs Esophageal manometry Trial of a proton pump inhibitor Upper gastrointestinal barium swallow Upper gastrointestinal endoscopy

Upper gastrointestinal endoscopy The above scenario is suspicious for esophageal cancer. There are two main types of esophageal cancer (squamous cell carcinoma and adenocarcinoma), and both are typically seen in patients over the age of 50 who have a history of chronic tobacco or alcohol use. Intestinal metaplasia (secondary to GERD) is a risk factor for adenocarcinoma of the esophagus. Male sex and obesity are also risk factors for esophageal cancer. Patients with esophageal cancer often report progressive dysphagia to solid food, weight loss, and painful swallowing, though symptoms can be subtle at first. Physical exam is usually normal in patients with esophageal cancer. An upper gastrointestinal endoscopy with biopsy is the best test to evaluate for esophageal malignancy in this patient. If this patient reported a history of radiation, previous surgery due to esophageal or laryngeal malignancy, or a complex stricture, then a barium swallow should be considered. However, there is some controversy about whether to perform this test before an upper gastrointestinal endoscopy.

A 75-year-old man presents to the Emergency Department with acute lower abdominal pain and urinary retention. He was well until he woke up this morning to urinate and was unable to void. He reports a steady pressure in the suprapubic region and appears uncomfortable. His vital signs are within normal limits for his age. He has suprapubic tenderness on examination without costovertebral angle tenderness. A bedside ultrasound is obtained and shown above. Which of the following is the most appropriate next best step in management? Abdominopelvic computed tomography Bilateral renal ultrasound Suprapubic catheter placement Urethral catheter placement

Urethral catheter placement This elderly man has acute urinary retention likely secondary to benign prostatic hyperplasia (BPH), which is the most common cause of acute urinary retention in men. Urethral catheter placement will relieve acute urinary retention, alleviate pain, and prevent hydronephrosis. As the central (urethral) portion of the prostate gland enlarges with age, it creates a bladder outlet obstruction resulting in decrease in the strength and caliber of the urine stream despite detrusor muscle contraction of the distended bladder. This creates lower abdominal distension, pain, fullness and urinary urgency. Other obstructive causes of acute urinary retention in men include phimosis, paraphimosis, urethral strangulation, meatal stenosis or stricture, penile trauma and prostate cancer. Causes in women include cystoceles and ovarian or uterine tumors. After obtaining a complete history and physical examination as to the cause of the patient's urinary retention, assess bladder volume using ultrasonography. Bedside ultrasound is a rapid means of evaluating for urinary retention. If the diagnosis is confirmed, placement of a urethral catheter should be performed. For patients who have recently undergone urethral or urologic instrumentation, urologic consultation should be sought prior to catheterization.

A 33-year-old man arrives at the urgent care with a fever and dysuria for several days. An abdominal exam reveals suprapubic tenderness. A gentle digital rectal exams reveals a warm, exquisitely tender prostate. Which of the following interventions is contraindicated in the diagnosis and treatment of his most likely condition? Aminoglycoside antibiotics Percutaneous suprapubic tube Quinolone antibiotics Urethral catheterization

Urethral catheterization Urethral catheterization is contraindicated in the management of acute bacterial prostatitis, the most likely diagnosis in this patient. Acute bacterial prostatitis usually results from the ascent of gram-negative organisms up the urethra as urine refluxes into the prostatic ducts. Symptoms will include fever, dysuria, urinary urgency or frequency, and suprapubic, perineal, or sacral pain. Urinary obstruction may result as the infection progresses. A digital rectal exam typically shows a warm, boggy prostate. However, great care should be used to perform a gentle rectal exam, as vigorous prostatic massage in a patient with acute bacterial prostatitis can lead to septicemia. Non-specific laboratory findings may include leukocytosis with a left shift. A urinalysis will show bacteria and pyuria. Urine should be cultured to distinguish the offending organism. Septic or extremely ill patients may require hospitalization with intravenous aminoglycoside antibiotics until the fever and symptoms resolve. Urinary obstruction should be managed by placement of a percutaneous suprapubic tube in lieu of a urinary catheter. Acute bacterial prostatitis should be treated for 4-6 weeks with a quinolone antibiotic with a follow-up culture on urine and prostatic secretions to ensure resolution of the infection. Appropriate treatment usually prevents the development of chronic bacterial prostatitis. Aminoglycoside antibiotics (A) and quinolone antibiotics (C) are both appropriate for management of acute bacterial prostatitis. The selection of antibiotic for bacterial prostatitis is largely dependent on the severity of symptoms and patient allergies. A percutaneous suprapubic tube (B) is a safe alternative to urethral catheterization in patients with acute bacterial prostatitis in the event that they develop urinary obstruction during treatment.

A 75-year-old woman with a medical history of dementia, hypertension controlled by lisinopril, and type 2 diabetes mellitus controlled by metformin presents to the emergency department. She is brought in by her daughter, who states that her mother "isn't acting right." The symptoms started 12 hours ago. Her daughter states that her mother is not listening when spoken to and is having a hard time paying attention to what is going on around her. The daughter also states that her mother is telling her that there are shadows in the room that are talking to her and frightening her. Per her daughter, the patient has not had any recent falls or head trauma. Vital signs are a BP of 122/82 mm Hg, HR of 90 bpm, RR of 16/min, oxygen saturation of 98%, and a T of 99°F. A fingerstick blood glucose is performed and is 98 mg/dL. Upon physical exam, the patient is notably agitated and does not respond to questions directed at her. Heart and lung sounds are normal, cranial nerves are intact bilaterally, and deep tendon reflexes are brisk. No bruising of the skin or obvious injuries are noted. The rest of the physical exam is unremarkable. What is the best next step in the assessment of this patient's condition? CT head Electroencephalogram Lumbar puncture Urinalysis Urine toxicology

Urinalysis Delirium is commonly encountered in older adults with medical illnesses. Delirium is defined as a disturbance in cognition and attention that occurs over a short period of time (hours to days). This change represents a change from baseline, and there is clinical evidence that the symptoms are caused by a medical condition, substance intoxication or withdrawal, or medication side effects. Symptoms are not explained by a pre-existing neurological disorder and do not occur during reduced arousal. Risk factors for delirium include underlying brain disease such as stroke or dementia, older age, and sensory impairment. Polypharmacy is also an important risk factor to consider. Disturbance in consciousness is one of the principal symptoms of delirium. A patient with delirium will have trouble maintaining focus and attention in most cases. In advanced cases of delirium, the patient will appear lethargic and drowsy. Patients with delirium will also experience disorientation, disturbances in speech, and perceptual disturbances. The Confusion Assessment Method (CAM) is a common tool for the assessment of delirium. The CAM method requires acute onset and fluctuating course of symptoms, inattention, and either disorganized thinking or altered level of consciousness. A thorough review of the patient's current medications is also important to determine if symptoms are being caused by the medicines that the patient is taking. Anticholinergic medications in particular are known to cause delirium in older adults. Laboratory evaluation, including urinalysis, is the first step in determining the cause of delirium. Urinary tract infection is a common cause of delirium in older adults and is treated with antibiotics once diagnosed. Treatment of delirium is dependent on the cause of the symptoms.

A man presents to the office for a wellness visit for the first time in 14 years. He has recently turned 65 years old and now has medical insurance. He reports no symptoms today. He has no significant medical history, and his medications include acetaminophen and ibuprofen as needed for occasional headaches. He reports smoking 10 cigarettes per day for the last 40 years. Home blood pressure measurements range from 146/88 mm Hg to 168/96 mm Hg. On exam, his BMI is 35 kg/m2. His blood pressure is 162/94 mm Hg, heart rate is 88 bpm, respirations are 18 breaths/min, SpO2 is 96% on room air, and temperature is 98.6°F. Cardiac, pulmonary, and abdominal exams are within normal limits. Bilateral pedal pulses are 2+ and equal bilaterally. ECG shows normal sinus rhythm. CBC and blood chemistries are unremarkable. Which of the following is the best next step in the management of this patient's condition? Abdominal ultrasound Brain natriuretic peptide Renal arteriogram Urinalysis and lipid profile Urine metanephrines

Urinalysis and lipid profile This patient is newly diagnosed with essential hypertension, which is defined by the American College of Cardiology (ACC) and American Heart Association (AHA) as systolic blood pressure > 130 mm Hg or diastolic blood pressure > 80 mm Hg on two separate readings on two separate days. According to the AHA/ACC 2017 guideline, stage 1 hypertension is systolic blood pressure 130-139 mm Hg or diastolic blood pressure 80-89 mm Hg. Stage 2 hypertension is systolic blood pressure ≥ 140 mm Hg or diastolic blood pressure ≥ 90 mm Hg. In patients without other comorbidities, initial agents include thiazide diuretics, angiotensin-converting enzyme (ACE) inhibitors, angiotensin receptor blockers (ARBs), and long-acting calcium channel blockers. Workup for a new diagnosis of hypertension should include a CBC, basic metabolic panel, lipid panel, thyroid-stimulating hormone, urinalysis, and an electrocardiogram. A lipid profile is important for determining cardiovascular risk for patients with hypertension. Many patients with hypertension will have coexisting hyperlipidemia, and statin therapy may be indicated to reduce the risk of cardiovascular events. The American College of Cardiology's atherosclerotic cardiovascular disease (ASCVD) risk calculator is a helpful tool for determining a patient's cardiovascular risk and planning the next steps in treatment, including statin therapy. Urinalysis should be collected in patients with newly diagnosed hypertension to detect hematuria and estimate albumin excretion. Serum creatinine, which is included in the basic metabolic panel, will give a more specific look at kidney function and should also be included in the workup at diagnosis.

A 23-year-old man presents to the emergency department reporting his first-ever seizure. He is drowsy, but his roommate accompanies him and was a witness to the seizure. The roommate reports the patient cried out, fell to the floor, and began to shake his entire body. His eyes were open during the episode, but he did not talk or respond, and the duration of the episode was < 3 min. Afterward, the patient went immediately to sleep. The patient has no history of illness, reports no head trauma, has no family history of epilepsy, takes no medications, and reports no substance use. Vital signs are within normal limits. Physical exam reveals contusions to the right elbow and right hip and a normal neurologic exam. Laboratory studies are listed below. CBC: WBC: 5.9 × 103/µL RBC: 4.83 × 106/µL Hemoglobin: 14.4 g/dL Hematocrit: 43.2% Mean corpuscular volume: 89 fL Mean corpuscular hemoglobin: 29.8 pg Mean corpuscular hemoglobin concentration: 33.3 g/dL Red blood cell distribution width: 12.7% Platelets: 368 × 103/µL Neutrophils: 58% Lymphocytes: 29% Monocytes: 10% Eosinophils: 2% Basophils: 1% BMP: Glucose: 100 mg/dL Blood urea nitrogen: 13 mg/dL Creatinine: 0.76 mg/dL Estimated glomerular filtration rate: 106 mL/min/1.73m2 Blood urea nitrogen/creatinine ratio: 17 Sodium: 137 mmol/L Potassium: 4.3 mmol/L Chloride: 98 mmol/L Carbon dioxide: 24 mmol/L Calcium: 10.1 mg/dL Liver function test: Albumin: 4.9 g/dL Total bilirubin: 0.3 mg/dL Alanine aminotransferase: 20 IU/L Aspartate aminotransferase: 18 IU/L Alkaline phosphatase: 78 IU/L Protein: 8.4 g/dL Which of the following is the best next step in evaluation of this patient? Computed tomography of brain Electroencephalogram Lumbar puncture Magnetic resonance imaging of brain Urine toxicology screen

Urine toxicology screen First nonfebrile seizures may indicate new-onset epilepsy but may also be indicative of other, systemic diseases or toxins. Seizures may be provoked by drug use, drug withdrawal, fever, intracranial surgery, head trauma, stroke, infection, hyperglycemia, hypoglycemia, porphyria, electrolyte imbalance, or tumors. These are called acute symptomatic seizures. The evaluation of a patient with a first-ever nonfebrile seizure should begin with a thorough history of what events led to the seizure and a thorough physical exam. Urine toxicology should always be evaluated, along with serum analysis for electrolyte or glucose imbalance and for signs of infection. Many patients may report no substance use, but this should not preclude urine toxicology testing in the setting of a new-onset seizure. If toxicology and laboratory values are negative, further testing is warranted. Computed tomography of the head will rule out tumors, bleeding, or anatomic abnormalities. Patients who appear to have an unprovoked seizure after the above workup will warrant further testing to hone the diagnosis of epilepsy and tailor treatment. The risk of having a second seizure is greatest in the first 2 years after a primary, unprovoked seizure. The risk of further seizures after an acute symptomatic seizure depends on the underlying cause.

A 52-year-old woman presents to her primary care clinic with fatigue and pruritus for the past 2 months. Vital signs today include a HR of 82 bpm, BP of 130/80 mm Hg, RR of 20/min, oxygen saturation of 99% on room air, and T of 98.6°F. Physical examination reveals scleral icterus and scattered excoriations. Her abdomen is soft and nontender. Laboratory findings include a white blood cell count of 7,000/μL, lipase of 85 U/L, aspartate aminotransferase of 52 U/L, alanine aminotransferase of 65 U/L, alkaline phosphatase of 290 U/L, total bilirubin of 3.5 mg/dL, and antimitochondrial antibody titers of 1:100. Which of the following is the recommended initial pharmacologic treatment of the suspected diagnosis? Bezafibrate Glecaprevir-pibrentasvir Obeticholic acid Prednisone Ursodeoxycholic acid

Ursodeoxycholic acid Primary biliary cholangitis is a chronic progressive autoimmune disease that affects the intrahepatic bile ducts. It causes chronic cholestasis (decreased flow of bile) and can result in cirrhosis. It is much more common in women (approximately 90%) and is most often diagnosed between 30 and 65 years of age. The diagnosis of primary biliary cholangitis may be initially suspected due to abnormal liver biochemical tests in asymptomatic patients or patients may have clinical manifestations from cholestasis, such as pruritus, fatigue, right upper quadrant discomfort, and jaundice. Exam findings can include excoriations, jaundice, and hepatosplenomegaly. It is common for patients with primary biliary cholangitis to have other autoimmune diseases, such as Sjögren syndrome. Primary biliary cholangitis should be suspected in adults who have an elevated alkaline phosphatase without extrahepatic biliary obstruction. Other supportive laboratory findings include elevated antimitochondrial antibody, elevated antinuclear antibody, elevated high-density lipoprotein, elevated gamma-glutamyl transpeptidase, and elevated 5-nucleotidase. The diagnosis of primary biliary cholangitis is established in patients who have at least two of the following in the absence of extrahepatic biliary dilation or comorbid liver disease: an alkaline phosphatase elevated to at least 1.5 times the upper limit of normal, antimitochondrial antibody titers elevated to at least 1:40, and histologic evidence of primary biliary cholangitis (nonsuppurative destructive cholangitis and destruction of interlobular bile ducts). Patients can be evaluated for extrahepatic biliary dilation with a right upper quadrant abdominal ultrasound. Liver biopsy is often unnecessary to make the diagnosis. All patients diagnosed with primary biliary cholangitis should be referred to a hepatologist. It is important for patients who are diagnosed with primary biliary cholangitis to be vaccinated against hepatitis A virus and hepatitis B virus. They should also have the pneumococcal vaccine and other routinely recommended vaccines, such as the annual influenza vaccine. Patients with primary biliary cholangitis should abstain from alcohol. Ursodeoxycholic acid is recommended for all patients with primary biliary cholangitis because it improves long-term survival, slows disease progression, and improves liver biochemical tests. Patients with primary biliary cholangitis require routine monitoring of the following laboratory studies: aspartate aminotransferase, alanine aminotransferase, alkaline phosphatase, total bilirubin, platelet count, prothrombin time, thyroid-stimulating hormone, vitamin A, and vitamin D. In some cases, patients with primary biliary cholangitis require a liver transplant. Indications for referral for evaluation for liver transplant include total bilirubin > 5 mg/dL, albumin < 2.8 g/dL, clinical manifestations of liver decompensation (ascites, variceal bleeding, and hepatic encephalopathy), intractable pruritus, and recurrent nontraumatic bone fractures.

A 67-year-old man is brought to the emergency department via ambulance. He lives at home with his wife and reports persistent fevers for the past 36 hours. He is experiencing photosensitivity, severe headache, and neck stiffness. The patient is otherwise healthy, with a past medical history of hypertension controlled with chlorthalidone, and hyperlipidemia controlled with atorvastatin. A CT scan is ordered and is normal. A lumbar puncture is performed, which shows an elevated opening pressure, elevated protein and decreased glucose. The initial Gram stain from the microbiology laboratory is read as "many gram-positive diplococci," with final speciation and susceptibility testing pending. Which one of the following should be administered empirically to this patient? Ampicillin Ampicillin and cefotaxime Vancomycin Vancomycin and ceftriaxone

Vancomycin and ceftriaxone Bacterial meningitis is an infection of the lining around the brain and spinal cord. This patient has been found to have Streptococcus pneumoniae, a common cause of bacterial meningitis in this age group. Out of possible etiologies for meningitis, the Gram stain result, gram-positive diplococci, is consistent with only this organism. While awaiting susceptibilities, this patient should be treated empirically with vancomycin and ceftriaxone. Streptococcus pneumoniae is typically highly susceptible to ceftriaxone, however, more than 5% of isolates are considered non-susceptible (termed penicillin-resistant Streptococcus pneumoniae). The prevalence of resistance varies by geographical region. Because of the severity of the infection and the potential for ceftriaxone resistance, vancomycin should be added initially for patients infected with Streptococcus pneumoniae until susceptibilities are known. Both vancomycin and ceftriaxone penetrate the meninges to provide adequate cerebrospinal fluid concentrations for the eradication of susceptible bacteria. Ceftriaxone is a third-generation cephalosporin with bactericidal activity against Streptococcus pneumoniae. It binds to and inhibits the penicillin-binding protein responsible for cross-linking the bacterial peptidoglycan cell wall. Vancomycin is also bactericidal and binds to the d-alanyl-d-alanine subunit of cell wall precursor molecules preventing cross-linking.

A 46-year-old woman presents to the emergency department with fever, cough, and hemoptysis. She has a history of intravenous opioid use. Vital signs are BP 110/65 mm Hg, HR 120 beats per minute, RR 20 breaths per minute, and T 103.4°F. On auscultation of the chest, you hear a faint systolic ejection murmur. Which of the following is the most appropriate initial therapy? Ampicillin and gentamicin Ceftriaxone Oxacillin and rifampin Vancomycin and ceftriaxone

Vancomycin and ceftriaxone This patient is presenting with signs and symptoms of infectious endocarditis. Risk factors for infectious endocarditis include rheumatic heart disease, congenital or acquired valvular disease, and intravenous drug use. Right-sided endocarditis involves either the pulmonic or tricuspid valve. It is classically seen in intravenous drug users. Organisms implicated in right-sided endocarditis include Staphylococcus aureus, Streptococcus pneumoniae, and gram negative bacteria. Presenting symptoms often include fever, cough, hemoptysis, chest pain, and dyspnea. Right-sided endocarditis is frequently misdiagnosed initially as pneumonia. Management includes antibiotics for the suspected organism based on the clinical situation. In an IV drug user, coverage should include methicillin-resistant Staphylococcus aureus, staphylococci, streptococci and enterocci. The most appropriate antibiotic choice for this patient would be vancomycin and ceftriaxone.

A 63-year-old man with a history of alcohol use disorder presents with fever, headache, and nuchal rigidity. What antibiotics should be administered? Ceftriaxone, ampicillin Vancomycin, cefazolin, ampicillin Vancomycin, ceftriaxone Vancomycin, ceftriaxone, ampicillin

Vancomycin, ceftriaxone, ampicillin This patient presents with symptoms concerning for meningitis and should immediately be started on antibiotics covering the most likely pathogens. Bacterial meningitis typically presents with a rapidly progressive course. There are a number of causative organisms, but the most common are Streptococcus pneumoniae, Neisseria meningitidis, and Listeria monocytogenes. S. pneumoniae ​is the most common organism in all adults, while N. meningitidis is more likely to be encountered in younger adults. L. monocytogenes is more common at the extremes of age (neonates and ≥ 65 years) and in patients with immunosuppression and alcohol use disorder. Patients may experience fever, headache, photophobia, neck pain, vomiting, altered mental status, lethargy, and seizures. Kernig and Brudzinski signs (physical findings consistent with nuchal rigidity) are classic findings but are seen in less than 50% of adults. A noncontrast head CT should be performed in patients with suspected meningitis if they have signs of increased intracranial pressure or mass effect (focal neurologic findings, altered mental status, or papilledema) and in those with significant immunocompromise (e.g., HIV or AIDS). The diagnosis of meningitis is made by analysis of the cerebrospinal fluid after lumbar puncture. Antibiotics should not be withheld from patients while waiting for CT scan or lumbar puncture. While awaiting the results of cerebrospinal fluid and blood cultures, broad-spectrum antibiotics covering the most likely pathogens should be started. Vancomycin is administered to cover resistant S. pneumoniae and a third-generation cephalosporin (ceftriaxone) for N. meningitidis. Ampicillin should be given for patients who are at risk for L. monocytogenes.

A 7-year-old girl presents to the clinic complaining of intermittent fever and generalized skin lesions. The fever started one to two days before the appearance of skin lesions, which began as tiny papules and progressed rapidly to clear vesicles followed by pustules and scabs. These lesions are widely distributed, but are noted predominantly on the torso. The child's mother does not believe in the benefits of immunization therefore the girl has not received any vaccinations. One of the girl's classmates also had the same manifestations two weeks ago and has been absent from class since then. Which of the following is the most likely etiologic agent? Poison ivy Solenopsis invicta Staphylococcus aureus Varicella zoster virus

Varicella zoster virus Varicella zoster virus is a neurotropic human herpes virus causing primary, latent and recurrent infections. The primary infection is manifested as varicella, or chickenpox, such as with this particular patient. Varicella is an acute febrile rash illness that has variable severity but is usually self-limited. It usually begins 14-16 days after exposure, although the incubation period can range from 10-21 days. Prodromal symptoms such as fever, malaise, anorexia, headache and abdominal pain may occur one to two days before the appearance of the rash and often resolve within two to four days after the onset of rash. Skin lesions appear first on the scalp, face and trunk. Initially, rashes appear as intensely pruritic erythematous macules evolving through the papular stage to form clear, fluid-filled vesicles. Clouding and umbilication of lesions begins in 24-48 hours. Crusting may be visible among the initial lesions, but new crops may still form on the trunk and eventually, the extremities. Simultaneous presence of lesions in various stages of evolution is characteristic of varicella. The distribution of lesions is usually central or centripetal, with the greatest number found on the trunk and proximal extremities. Secondary bacterial infections of the skin, usually due to group A Streptococcus and S. aureus, may occur. These may range from impetigo to cellulitis, lymphadenitis and subcutaneous abscess. Recrudescence of fever three to four days after the initial exanthem may herald a secondary bacterial infection. More invasive complications include gangrenosa, sepsis, pneumonia, arthritis, osteomyelitis, and necrotizing fasciitis. Often manifest as nuchal rigidity, seizures and altered consciousness, encephalitis and acute cerebellar ataxia are highest among patients younger than five years old and those older than 20 years of age. Diagnosis is primarily clinical. Leukopenia may be present during the first 72 hours from onset of rash, followed by lymphocytosis. Antiviral treatment modifies the course of varicella. Acyclovir has demonstrated efficacy and a good safety profile in the treatment of all children with varicella. However, acyclovir is not recommended routinely by the American Academy of Pediatrics for the treatment of varicella in otherwise healthy child because of marginal benefit, the cost of the drug and the low risk of complications of varicella.

A 25-year-old man presents to the clinic with intermittent painless swelling in his scrotum for 3 days. The patient initially noticed the swelling while taking a shower. Vital signs include HR of 90 bpm, BP of 120/80 mm Hg, RR of 20/minute, oxygen saturation of 98% on room air, and T of 98.6°F. Physical examination reveals a soft nontender scrotal mass that is present when the patient stands and disappears when the patient lies down. Light does not pass through the mass when a penlight is held behind it. Which of the following is the most likely diagnosis? Epididymitis Hydrocele Spermatocele Testicular cancer Varicocele

Varicocele Varicoceles are a common scrotal condition defined by dilation of the pampiniform plexus of spermatic veins. They are more common on the left side because the left gonadal vein drains into the left renal vein at a perpendicular angle. The classic presentation is a dull and aching left testicular pain that is present when the patient is standing but resolves when the patient is in a supine position. In some cases, varicoceles can be associated with decreased fertility. The diagnosis of a varicocele can be made clinically in most cases, but it can be confirmed with a scrotal ultrasound when necessary. Asymptomatic varicoceles do not require treatment. However, if the varicocele is painful, causes reduced testicular size, or is associated with infertility, it can be treated with ligation or percutaneous venous embolization. Epididymitis (A) is an infectious inflammatory condition of the epididymis, which is a tube-like structure on the posterior aspect of the testicle. Epididymitis is a painful condition. A hydrocele (B) is a collection of peritoneal fluid between the parietal and visceral layers of the tunica vaginalis. Hydroceles can be idiopathic or reactive to acute conditions, such as torsion or epididymitis. Small hydroceles are often painful, but large hydroceles cause more pain. Hydroceles will transilluminate (allow light to pass through), which distinguishes them from hernias, masses, and varicoceles. A spermatocele (C) refers to an epididymal cyst > 2 cm. An epididymal cyst presents as a soft and round mass in the head of the epididymis. They generally do not require treatment. Spermatoceles are located focally within the epididymal head and are not worse when standing or improved with recumbency. Testicular cancer (D) usually presents with a painless and solid mass in a testicle. Testicular cancer would not be present when standing but resolve when recumbent.

A 75-year-old woman is brought to your office by her daughter, who tells you that her mother has recently been exhibiting short-term memory loss and confusion over the recent months. The patient's medical problems include type 2 diabetes mellitus, hypertension, hypercholesterolemia, and osteoarthritis. She had a stroke last year and has residual mild hemiparesis. On cognitive testing she is able to recall only one of three words, and all the numbers are on one side on the clock-drawing test. Which of the following is the most likely diagnosis? Alzheimer's disease Dementia with Lewy bodies Frontotemporal dementia Vascular dementia

Vascular dementia Vascular dementia is a common type of dementia that is seen in patients with risk factors such as hyperlipidemia, hypertension and diabetes. Patients with vascular dementia have patchy cognitive impairment, often with focal neurologic signs and symptoms. Onset may be abrupt, with a stepwise decline. Patients who have had a stroke are at increased risk for vascular dementia. The 3 most common mechanisms of vascular dementia are multiple cortical infarcts, a strategic single infarct, and small vessel disease. MRI findings that are suggestive of vascular dementia are bilateral multiple infarcts located in the dominant hemisphere and limbic structures, multiple lacunar strokes. The mainstay of management of vascular dementia is the prevention of new strokes. This includes administering antiplatelet drugs and controlling major vascular risk factors. Aspirin has also been found to slow the progression of vascular dementia.

A 20-year-old woman fainted while standing in line at the grocery store. The patient says she felt nauseus and diaphoretic before the episode. She reports no bowel or bladder incontinence and did not experience a postictal state. The woman in line behind her observed jerking motions of her face and fingers. She has no past medical history and does not take any medications. What is the most likely diagnosis? Hypertrophic cardiomyopathy Orthostatic hypotension Seizure Vasovagal syncope

Vasovagal syncope Vasovagal, also called neurocardiogenic syncope, is often called the "common faint" and is the most common cause of syncope. Vasovagal syncope is a transient loss of consciousness caused by systemic hypotension and cerebral hypoperfusion. It is a neurally mediated reflex response characterized by bradycardia or peripheral vasodilation. Patients with vasovagal syncope are usually young and healthy. The clinical presentation of "classic" vasovagal refers to syncope triggered by provoking factors such as noxious stimuli, pain, blood draw, intense emotion, fear of bodily injury, prolonged standing, or heat exposure. Prodromal symptoms include feeling warm, sweating, nausea, and pallor. In addition, some patients present with myoclonic or other involuntary movements that are suggestive of a seizure but are actually due to cerebral hypoxia secondary to hypotension. This seizure like activity, historically, has been referred to as a "fainting fit." Vasovagal syncope is not dangerous; however, it is important that other more serious causes of syncope be ruled out.

A 50-year-old man with a history of hypertension, diabetes and stage IV chronic kidney disease with a GFR of 25, presents to the emergency department complaining of shortness of breath and stabbing chest pain for the past hour. Vital signs are BP 145/70, RR 36, HR 115, and pulse oximetry 89% on room air. An ECG reveals sinus tachycardia. Two days ago he returned to Los Angeles on a business trip from China. Which of the following is the most appropriate diagnostic test for this patient? Chest CT pulmonary angiography Chest radiography Pulmonary angiography Ventilation/perfusion scan

Ventilation/perfusion scan Ventilation/perfusion scan is the most appropriate diagnostic imaging modality for this patient who likely has a pulmonary embolism. For most patients with suspected pulmonary embolism, chest CT pulmonary angiography is the first-choice diagnostic imaging modality because it is sensitive and specific for the diagnosis. However, this patient has stage IV chronic kidney disease and chest CT pulmonary angiography is contraindicated. Ventilation/perfusion scanning is reserved for those with suspected pulmonary embolism in whom chest CT pulmonary angiography is contraindicated such as renal insufficiency with an estimated GFR < 30, contrast allergy, pregnancy or morbid obesity. In general,ventilation/perfusion scanning is a sensitive test for the diagnosis of pulmonary embolism, but is poorly specific due to the high number of false-positive test results. Chest CT pulmonary angiography (A) is usually the first choice study for diagnosis of a pulmonary embolism, but it is inappropriate in this patient with advanced chronic kidney disease. Certain findings on chest radiography (B) may be suggestive of a pulmonary embolism, but are not considered diagnostic. Pulmonary angiography (C) is the most specific examination available and is considered the gold standard; however it is more invasive and rarely used.

You refer a 45 year old man with a movement disorder to a neurologist. She obtains a brain MRI which shows significant atrophy of the basal ganglia. She also comments on finding significant choreiform movements and ballism, as well as dementia. She initiates drug therapy with the specific goal of addressing the chorea since the patient states it is interfering with his function, but asks you to follow-up for medication monitoring. Which of the following class of medications will you most likely be titrating? Dopamine-agonist Ergotamine-derivative Tricyclic antidepressant Vesicular monoamine transporter type 2 (VMAT2) inhibitor

Vesicular monoamine transporter type 2 (VMAT2) inhibitor Huntington's disease is an autosomal dominant genetic disorder with gradual onset and progression of chorea and dementia, usually starting at 35-50 years of age. The major pathology exists as neuronal loss, astrogliosis and eventual atrophy of the caudate nucleus and putamen of the basal ganglia, and the cerebral cortex. Three main symptoms exist: movement disorder (characterized by chorea, ballism, dystonia and parkinsonian features), cognitive disorder (mostly dementia) and mood or behavior disorder (depression, bipolar disorder, psychosis, personality disorders, sexual and sleep disturbance). Chorea is defined as excessive, spontaneous and abrupt movements or irregular frequency and random distribution. Examples include restlessness, fidgeting, gesture and facial expression alterations and dancelike gait. A milder form, choreathetosis, involves slow distal writhing movements, whereas more severe forms, ballism, is marked by gross, proximal, flinging movements of the extremities. Athetosis refers to contorted, twisting movements. Patients sometimes try to mask these choreiform movements as purposeful movements (parakinesia). If the abnormal movements interfere with daily function, consider treating Huntington's with vesicular monoamine transporter type 2 (VMAT2) inhibitors, such as tetrabenazine and deutetrabenazine.

A 16-year-old girl presents with her mother for a sports physical examination. She plays volleyball and reports no chest pain, difficulty of breathing, or family history of heart disease. She has been vegan since the age of 13. She is a straight-A student. Physical exam is normal. You obtain a complete blood count, which reveals hemoglobin of 10 g/dL and mean corpuscular volume of 110 fL. Which of the following is the most likely diagnosis? Iron deficiency anemia Lead toxicity Vitamin B12 deficiency Zinc deficiency

Vitamin B12 deficiency The patient above has macrocytic anemia, which is defined by a decreased hemoglobin and MCV > 100 fL. The macrocytic anemia is most likely secondary to a vitamin B12 deficiency. Vitamin B12 deficiency can result from inadequate dietary intake in certain settings, such as in patients who are strict vegans and avoid all animal products. Animal products (meat and dairy products) provide the only dietary source of cobalamin for humans. The usual diet for those living in resource-rich countries contains 5-7 mcg of cobalamin per day, while the minimum daily requirement is 6-9 mcg per day. Malabsorption of vitamin B12 also occurs in pernicious anemia due to intrinsic factor deficiency and in ileal resections and Crohn disease. Breastfeeding infants of vitamin B12-deficient mothers are also at risk for significant deficiency. Vitamin B12 deficiency should be suspected in patients with one or more of the following clinical or laboratory findings: oval macrocytic red blood cells (MCV > 100 fL) on the peripheral blood smear, the presence of hypersegmented neutrophils on the peripheral blood smear, and unexplained neurologic signs and symptoms. The common neurologic symptoms in children include irritability, hypotonia, developmental delay, developmental regression, and involuntary movements. On the other hand, sensory deficits, paresthesias, and peripheral neuritis are seen in adults

A 67-year-old woman presents to the clinic for her annual wellness exam. She is accompanied by her daughter who expresses concern about her mother's cognition. Over the past 6 months, the daughter has noticed that her mother seems to be more forgetful. She states that her mother has been independent up until this point but now is having a harder time completing tasks at home such as paying her bills. The patient has also gotten lost on more than one occasion while driving in areas that she is familiar with. Her vital signs are a BP of 120/78 bpm, HR of 90 bpm, RR of 16/min, oxygen saturation of 96%, and T of 98.6°F. The physical examination reveals normal heart and lung sounds, regular deep tendon reflexes, a normal cranial nerve exam, and strength testing that is within normal limits. The rest of the physical exam is otherwise unremarkable. The patient scores a 21 out of 30 on the Montreal Cognitive Assessment exam. What is the best next step in the evaluation of this patient? Electroencephalogram HIV testing Rapid plasma antigen Vitamin B12 level Vitamin D level

Vitamin B12 level Dementia is a disorder that causes cognitive decline over time that affects a person's independence and their ability to perform normal activities of daily living. Alzheimer disease is the most common form of dementia. Other causes of dementia include dementia with Lewy bodies, frontotemporal dementia, and Parkinson disease dementia. In all forms of dementia, cognitive impairment changes from baseline over time, and the patient might not notice these changes. Often, a family member will express concern before the patient does. Short-term memory impairment is the most common symptom of dementia and usually presents the earliest. Additional symptoms include deficits in executive function, language, and visuospatial function. Cognitive testing is indicated in any patient who is experiencing symptoms of cognitive decline. The most common tests for cognition are the Montreal Cognitive Assessment (MoCA) and the Folstein Mini-Mental State Exam (MMSE). A vitamin B12 level should be obtained in any patient with cognitive symptoms along with a thyroid-stimulating hormone and a free T4 level. A drug history and depression screen should also be completed. Nonpharmacologic treatment for dementia includes aerobic exercise of at least 30 minutes per day several days per week. Exercise may decrease the rate of decline in addition to decreasing the amount of caregiving a patient may need. Vitamin E may also reduce the rate of functional decline in patients with Alzheimer disease. Pharmacologic treatment of dementia includes cholinesterase inhibitors such as donepezil, rivastigmine, and galantamine. Additionally, patients with dementia should discontinue driving.

Which of the following cerebrospinal fluid analyses is most consistent with viral meningitis? WBC 2,900 cells/µL, glucose 20 mg/dL, protein 200 mg/dL WBC 350 cells/µL, glucose 15 mg/dL, protein 190 mg/dL WBC 400 cells/µL, glucose 20 mg/dL, protein 270 mg/dL WBC 425 cells/µL, glucose 75 mg/dL, protein 90 mg/dL WBC 5 cells/µL, glucose 50 mg/dL, protein 225 mg/dL

WBC 425 cells/µL, glucose 75 mg/dL, protein 90 mg/dL Normal adult cerebrospinal fluid (CSF) contains no more than 5 leukocytes/µL. WBCs > 5 cells/µL may be suggestive of a central nervous system infection. Below is a table representing the typical spinal fluid results for meningeal processes. Although the diagnosis of viral meningitis is often straightforward, there can be an overlap of CSF findings in patients with early and partially treated bacterial meningitis, making the distinction somewhat difficult. Moreover, while neutrophils tend to predominate in bacterial meningitis, they may be present in the CSF for the first 24 hours with viral meningitis. Enteroviruses are the most common cause of viral meningitis infections; other causes include herpes simplex virus, varicella-zoster virus, mumps, HIV, lymphocytic choriomeningitis virus, and arboviruses (eg, West Nile virus).

Which one of the following groups of lab results is most consistent with a complication that commonly occurs 6-12 weeks after acute hepatitis? WBC decreased, RBC decreased, platelets decreased WBC decreased, RBC increased, platelets increased WBC increased, RBC decreased, platelets decreased WBC increased, RBC increased, platelets increased

WBC decreased, RBC decreased, platelets decreased Aplastic anemia is a known complication of acute hepatitis. This affects up to 2% of all patients after their initial illness. Hepatitis-associated aplastic anemia most often affects adolescent boys and young men, is most commonly seen 6-12 weeks after hepatitis, and can be fatal if untreated. Its etiology is unclear. There is no known association with blood transfusions, drugs, or toxins. Most patients have been seronegative for hepatitis A, B, and C, but it is believed to be the result of autoimmune bone marrow failure. Lab findings show a decrease in the WBC, RBC, and platelet counts.

Which of the following represents the best treatment plan for correction of systemic acidosis in status epilepticus? Fomepizole 15 mg/kg IV infusion over 30 minutes Intravenous insulin infusion at 0.1 units/kg/hour Sodium bicarbonate 2-5 mEq/kg IV infusion over four to eight hours Watchful waiting for auto-correction of the acidosis once seizure activity is controlled

Watchful waiting for auto-correction of the acidosis once seizure activity is controlled Status epilepticus refers to a prolonged epileptic crisis (> or equal to 5 minutes of continuous seizure activity) or more than one seizure without recovery from the postictal state in between episodes. Patients may present with a long history of seizure disorder or may be experiencing their first seizure event. In patients with a history of seizure disorder, a change to their medication regimen is the most common precipitating factor in status epilepticus. Other causes of the disorder are stroke, hemorrhage, tumor, infection, electrolyte abnormalities and toxins. Patients in status epilepticus may present with a sustained tonic-clonic seizure or may suffer sustained simple partial or complex partial seizures. Complications resulting from status epilepticus can be life-threatening and include respiratory failure, hypotension, acidosis, hyperthermia, rhabdomyolysis, and aspiration. Treatment of status epilepticus involves stabilizing the patient and rapid administration of a benzodiazepine such as midazolam. This is followed by administration of an intravenous anticonvulsant such as phenytoin. If the seizure activity does not cease after these two measures, then a barbiturate or a general anesthetic such a propofol are used. Correction of the systemic acidosis found in status epilepticus is not necessary since the acidosis is thought to have anticonvulsant effects. Also, the acidosis corrects itself quickly once seizure activity has ceased, and treatment with alkalinizing agents often leads to a rebound systemic alkalosis. Treatment of status epilepticus with benzodiazepines and anticonvulsants should not be delayed, due to the serious nature of the disorder. However, after the patient is stabilized and no longer actively seizing, measures must be taken to investigate the cause of the seizure activity. These measures are symptom-specific but can include complete blood count, complete metabolic panel, toxicology screening, electroencephalography, computed tomography or magnetic resonance imaging of the brain, chest radiograph, or lumbar puncture.

A 27-year-old woman presents to the clinic reporting urinary urgency, frequency, and dysuria for the past 2 days. She has associated symptoms of nausea and chills. She states she has never experienced anything like this before. She notes vaginal intercourse with her monogamous partner earlier this week. She reports no history of sexually transmitted infections and no significant medical or surgical history. She takes a multivitamin daily but reports no prescription medication. Vital signs include a HR of 112 bpm, BP of 120/80 mm Hg, RR of 16/min, oxygen saturation of 98% on room air, and T of 100.6°F. Physical examination is positive for costovertebral angle tenderness. Beta-human chorionic gonadotropin is negative. A diagnosis of pyelonephritis is made. Which of the following findings will best differentiate simple cystitis from pyelonephritis? Bacteriuria Leukocytosis Microscopic hematuria Pyuria White blood cell casts

White blood cell casts Acute pyelonephritis is an infectious disease of the kidney, most often caused by gram-negative bacteria such as Escherichia coli, Proteus, Klebsiella, Enterobacter, or Pseudomonas. The infection usually begins as a lower urinary tract infection and ascends to the kidney. Symptoms of acute pyelonephritis include fever, flank pain, and irritative voiding symptoms. Urinalysis reveals pyuria, bacteriuria, and on occasion, hematuria. A positive urine culture will also be seen. To differentiate pyelonephritis from acute cystitis, white blood cell casts may be seen in urinalysis. These white cell casts indicate a renal origin for the pyruria. Treatment of acute pyelonephritis includes identifying the offending microorganism and adjusting antibiotics according to sensitivity.

A 23-year-old woman presents with symptoms of recurrent nosebleeds, bleeding of the gums, and menorrhagia. Significant laboratory results include a normal platelet count, abnormal platelet function analysis, normal prothrombin time, and abnormal activated partial thromboplastin time. Which of the following is the most likely diagnosis? Disseminated intravascular coagulation Hemophilia A Thrombotic thrombocytopenic purpura von Willebrand's disease

von Willebrand's disease Von Willebrand's disease commonly presents with mucosal bleeding such as epistaxis, gingival bleeding, gastrointestinal mucous membrane bleeding, and menorrhagia. vWD is an autosomal dominant, congenital bleeding disorder. It is the most common genetic coagulopathy, found in 1% of the population. Most cases are mild, but can be exacerbated by the use of aspirin or NSAIDs. The absence or low levels of von Willebrand's factor (vWF), which binds to factor VIII, results in the inability to form a primary platelet plug (e.g. platelet function disorder). Laboratory tests confirm the diagnosis and show normal PT, prolonged aPTT in 50% of patients with vWD (due to decreased factor VIII activity), normal platelet count with abnormal platelet function, and low vWF. Desmopressin acetate (DDAVP), available as a nasal spray, is commonly used for treatment. DDAVP is a synthetic form of antidiuretic hormone (ADH), which induces the release of vWF. Factor VIII concentrates can also be administered if necessary.


संबंधित स्टडी सेट्स

CH 57 Drugs for Attention Deficit Hyperactivity and Narcolepsy (E4)

View Set

Batson et al. (1981): Empathy-altruism theory

View Set